Contract Law Notes-1

You might also like

Download as pdf or txt
Download as pdf or txt
You are on page 1of 316

Allen Contract Law

2023

1
TABLE OF CONTENTS

CHAPTER 1: FORMATION OF CONTRACT 15

Objective ascertainment of offer and acceptance 15

Agreement 16

Offer 18
What is an offer? 18

Offer and requests for information 18

Offers and invitations to treat 19

Presumptive rules or conventions to particular common situations 20

Displays and advertisements 20

Auctions 23

Tenders 24

Automated machines 25

Online shops 26

Unsolicited goods or services 27

How and when is an offer be terminated? 27

Revocation 27

Rejection by the offeree 29

Lapse of the offer 30

Death of offeror or offeree 30

Acceptance 30
What is an acceptance 30

Communication 31

Generally must be communicated to the offeror 31

Postal acceptance rule 32

One-way instantaneous 33

Method of acceptance 33

Prescribed acceptance 33

Acceptance by silence 34

2
Acceptance with knowledge of the offer 35

Consideration 36
Basic idea and justification 36

Nexus 37

Consideration must move from the claimant 38

Consideration must be requested by the promisor 38

Past consideration is not good consideration and implied assumpsit 38

Value 40

Consideration must be sufficient but need not be adequate. 40

Intangibles and moral obligations 41

Illusory consideration 41

Motive 42

Specific context: Compromise and forbearance to sue 42

Specific context: Pre-existing duties imposed by public law 44

Specific context: Pre-existing duties owed to third parties 45

Specific Context: Pre-existing duties owed to the other party 46

Outstanding obligations 46

More for the same 46

Less for the same 47

Reform of Consideration 50

Promissory Estoppel 52
Legal rule 52

Clear Promise 52

Reliance 53

Inequitable to go back on the promise 55

Effect of promissory estoppel: Suspensory or Extinctive 57

Shield or sword 58

Intention to create legal relations 60


Social agreements 60

Commercial agreements 62

3
CHAPTER 2: TERMS 64
Introduction 64

Express Terms 64
Ascertaining the terms 64

Representations vs Terms 64

Written contracts and the parol evidence rule 67

Collateral contracts (oral) 68

Incorporation of Terms 69

Incorporation by Signature 69

Incorporation by notice in unsigned documents 71

Incorporation by previous course of dealing or trade usage 73

Interpretation of Terms 75

Is the evidence admissible to aid interpretation? 75

General rules and parol evidence of subjective intent 75

Evidence of prior negotiations and subsequent conduct 77

Added requirements 80

Principles of interpretation 81

Illustrative cases 82

Y.E.S F&B Group v Soup Restaurant 82

Ngee Ann Development v Takashimaya Singapore 83

Yap Son On v Ding Pei Zhen 84

CISG v Ong Puay Koon 85

Implied terms 86
Terms implied in fact 86

Illustrative case 87

Terms implied in law 88

Terms implied in statute 89

Invalidation of exception clauses 90


Exception (limiting or excluding contractual liability) vs defining contracting parties’ legal obligations? 90

Fundamental breach as a rule of construction 90

4
Contra proferentem rule 91

Unfair Contract Terms Act 93

What kind of liability and contracts does UCTA apply to? 93

What terms does UCTA apply to? 93

Reasonableness test 94

CPFTA 96

CHAPTER 3: FRUSTRATION 98
Historical development of the law 98

Juridical basis 98

Test for frustration 101

Illustrations in specific factual situations 102


Supervening impossibility 102

Destruction of subject matter 102

Unavailability of subject matter 102

Failure of a particular source 103

Impossibility of method of performance 105

Death/incapacity of contracting party in personal contracts 105

Supervening illegality 106

Delay and hardship 106

Increased costs/lower profits 106

Failure of underlying purpose of the contract 108

Temporary unavailability/delay 108

Exceptions and limits of frustration 109


Self-induced frustration and power to elect 109

Construction of the contract and foresight 111

Express provision: Force majeure and hardship clauses. 111

Implied provision and foresight 112

Effects of Frustration 114

5
At common law 114

Frustrated contracts act 115

Restitution of money (S2(2),(3) FCA) 116

Restitution of benefits (S2(4) of the FCA) 116

CHAPTER 4: BREACH OF CONTRACT AND TERMINATION FOR BREACH 119


What is a breach of contract 119
Breach and fault 119

Order of performance and breach 120

Right to terminate for breach 120


Why would innocent party want to terminate the contract? 120

When does the innocent party have a right to terminate for a breach? 121

Situation 1: Express termination clause 121

Situation 2: Renunciation 124

Situation 3(a): Condition-warranty 125

Condition and factors 125

Definition of a warranty 130

Situation 3(b): Hongkong Fir 130

Anticipatory breaches 132

Election and limits 134


Exercise of right and timing 134

Loss of right to terminate 135

Consequences of election 135

Termination 135

Affirmation 136

Restriction on affirmation – White & Carter 137

Cooperation 137

Legitimate interest 137

Defaulting party rights and order of performance 138


Independent obligations 138

6
Dependent obligations 139

Divisible obligations 139

Entire obligations 140

CHAPTER 5: REMEDIES FOR BREACH OF CONTRACT 142


Introduction 142

Damages 142
Compensatory principle 142

Non-pecuniary losses 143

Proof of loss 145

Date of assessment 146

Measure of damages and interests protected: Expectation, reliance and restitution 148

Expectation interest 148

3 typical measures: Cost of cure, diminution in value, loss of amenity 148

Loss of chance 149

Reliance interest 151

Loss of chance to bargain, restitution interest 153

Limiting damages 155

Causation, contributory negligence 155

Remoteness of damage 156

Mitigation 160

Agreed remedies 161


Liquidated damages 161

Deposits 166

Deposits and advanced payment 166

Pre-contract deposits 167

Specific Remedies 168


Specific performance 168

Injunctions 169

Action on agreed sum 170

7
CHAPTER 6: PRIVITY OF CONTRACT 172
Privity rule and consideration rule 172

Contracts conferring benefits to third parties 173


Promisee rights 174

Damages 174

Albazero Exception (Narrow ground) 175

Broad Ground 178

Specific Performance 179

Injunctive relief 180

Third party enforcement 181

Changing the status of the third party 181

CRTPA 184

Right of third party to enforce contractual term 184

Limits to variation and cancellation of contract (common law position and CRTPA) 185

Defences available to the promisor 186

Double liability 187

Exceptions 187

Contracts imposing burdens on third parties 187

CHAPTER 7: MISREPRESENTATION 189


General conditions of liability 189
Misrepresentation 189

Understanding the representation 189

Objective meaning 189

Express and implied representations 190

Representations by conduct (including concealment, silence) 190

Self-induced misrepresentation 192

Actionable representations – statements as to fact and law 192

Statements of fact 192

8
Statement of opinion (recharacterized as statement that opinion maker holds the opinion, statement that

opinion maker had reasonable grounds for making statement) 192

Statement of intention/future conduct (recharacterized as representor holding that intention

represented) 193

Statements of the law 195

Puffs 195

The representation must be false 196

Continuing representations and change of circumstances 196

The representation must be made to the party misled 196

The misrepresentation must have induced the misled party to enter into contract 197

Inducement and reliance 197

Representee could have discovered the truth – not a bar 199

Representee knows the truth – quare if this is a bar? 200

Representee relied on third party/due diligence not a bar 200

Misrepresentation was corrected - bar 201

Inducement, materiality and fraud 201

Types of misrepresentation 202


Fraudulent misrepresentation 202

Negligent misrepresentation 203

Innocent misrepresentation 204

Remedies 204
Primary remedy: Rescission 204

Bars to rescission – affirmation 204

Bars to rescission – lapse of time 205

Bar to rescission – third parties 206

Bars to rescission – restitutio in integrum impossible 207

Remedies: Damages 207

Fraudulent misrepresentation 207

Negligent misrepresentation 208

Innocent misrepresentation prior to the Misrepresentation Act 208

9
Damages under the Misrepresentation Act 209

Damages under the fiction of fraud 209

Damages in lieu of recission 212

Indemnity on rescission 213

Exclusion clauses/basis clauses/non-reliance and no representation clauses 214


Agency situation 215

Representations made by the representor 216

CHAPTER 8: MISTAKE 218


Introduction 218

Communication/formation mistakes 218


Mutual mistake/latent ambiguity 218

Unilateral mistake 220

As to identity 220

Importance of the identity/Existence or non-existence 221

Written contracts 222

Face to face contracts 223

As to terms 225

At common law 225

In equity 227

Expectation/Common Mistake 228


Mistaken assumptions at law 228

Contractual allocation of risk 228

Fault 229

Shared fundamental mistake 229

Mistaken assumptions in equity 232

Rectification 233
Rectification of contractual documents for mistakes as to terms 233

Common mistake as to terms 234

Unilateral mistake as to terms 237

10
Non Est Factum 239

CHAPTER 9: ILLEGALITY 241


General illegality 241
Denying enforcement 241

Statutory illegality: Express prohibition 241

Statutory illegality: Implied prohibition 243

Common law illegality: Established head of public policy 245

Common law illegality: contracts tainted by illegality 247

Recovery of benefits conferred 250

Not in pari delicto 250

Locus poenitentiae 253

Independent cause of action 254

Unjust enrichment 254

Independent tort/trust claims 255

Other claims? 255

Restraint of trade 256


What is a restraint of trade and when does it apply? 256

General approach 257

Legitimate (proprietary) interests 257

Reasonable in the interests of the parties 258

Reasonable in the public interest 259

Case studies 259

Sale of business context 259

Nordenfelt v Maxim Nordenfelt Guns 259

CLAAS Medical v Ng Boon Ching 259

Employment context 261

Man Financial v Wong Bark Chuan David 261

Smile Dental v Lui Andrew 263

Lek Gwee Noi v Humming Flowers 264

Exclusive dealing agreements 266

11
Esso Petroleum Co v Harper's Garage 266

National Aerated Water v Monarch Co 266

Exclusive promotion agreements 267

Nuanced employment provisions 269

Forfeiture of benefit clauses – Mano Vikrant Singh v Cargill 269

Prohibition on owning shares in a competing business – Tillman v Egon 270

Trade association rules 270

Pilkadaris v Asian Tour 270

Severance 271
Term severance 271

Textual severance 272

Illustrative cases: 272

Notional severance 272

CHAPTER 10: DURESS 274


Judicial basis of duress 274
Inadequacy of the overborne will theory 274

Illegitimate pressure theory 275

Elements of duress 276

Classes of duress and illegitimate pressure 277


Duress to person – unlawful acts 277

Duress to goods – unlawful acts 277

Economic duress – threats to breach contracts – special unlawful act 278

Statement of inevitable 278

Good faith/Bad faith 279

Reasonableness of demand 280

Statutory policy 280

Illustrative cases 281

The Atlantic Baron – Duress made out 281

B&S Contracts v Victor Green – Duress made out 281

12
DSND Subsea v Petroleum Geo-Services – Duress not made out 282

Huyton SA v Peter Cremer – Duress not made out 282

Sharon Global v LG – Duress not made out 283

Lawful act duress 284

Illustrative cases 284

Refusal to contract – CTN Cash and Carry – Duress not made out 284

Exercise of a legal right or discretion – R v AG for England and Wales – Duress not made out 285

Exercise of legal right to sue – Lee Kuan Yew v Chee Soon Juan – Duress not made out 285

Exercise of legal right to sue – Tam Tak Chuen – Duress made out 286

Taking advantage of weaker bargaining power – EC Ridout SGHC – Duress not made out 286

Threat to sue – Goh Bee Lan v Yap Soon Guan – Duress not made out 287

Classes of duress and causation 287


Duress to persons 287

Duress to goods, economic duress, lawful act duress 288

General factors in ascertaining causation 289

Remedies for duress 290


Recission of contract 290

Restitution for unjust enrichment 290

Damages 291

CHAPTER 11: UNDUE INFLUENCE 292


General rule 292
Class 1: Actual undue influence 292

Class 2: Presumed undue influence 296

Class 2A: Irrebuttable presumption that one party had influence over the other 296

Class 2B: Factually established relationships of trust and confidence 298

Transaction which calls for an explanation 300

Rebutting the evidential presumption of undue influence 301

Third party undue influence 302


Put on inquiry 302

13
Curing the infection 303

CHAPTER 12: UNCONSCIONABILITY 306


Pre-BOM v BOK 306

BOM v BOK 311


Reversal of burden of proof – is the transaction fair, just and reasonable? 312

14
Chapter 1: Formation of Contract

Objective ascertainment of offer and acceptance

1. How does contract law determine the intention of the parties? In considering whether
there is a contract formed (in the song and dance of forming a contract), does the law
care about what the parties actually think?

2. As a starting point, the Courts take an objective approach, which refers to how a
reasonable person would interpret a party’s intention from her conduct in all the
circumstances (Smith v Hughes).1

3. In whose shoes does the reasonable person stand in ascertaining the intentions of the
parties? The approach taken under contract law is the “promisee objectivity” approach
– it is from the perspective of the person to whom the promise/acts are made to (i.e. the
recipient) which the reasonable person stands (Tribune Investment Trust Inc v Soosan
Trading).2

a. The difference between subjective and objective ascertainment and promisee


objectivity is illustrated with a simple (Korean drama) example. A makes small
handmade gifts to B frequently. A treats B with special surprises on special
occasions. A frequently messages B and tells her how lovely she is. From B’s
perspective, what conclusion would a reasonable person infer about A’s
intentions? (i.e. A probably has a love interest in B). However, there is nothing
to stop A from being a silly donkey, thinking that such acts are socially

1
Smith v Hughes [1871] LR 6 QB 597 (EWCA) ("Smith v Hughes") per Blackburn J, “If, whatever a
man’s real intention may be, he so conducts himself that a reasonable man would believe that he was
assenting to the terms proposed by the other party, and that other party upon that belief enters into the
contract with him, the man thus conducting himself would be equally bound as if he had intended to
agree to the other party’s terms.”
2
Tribune Investment Trust Inv v Soosan Trading Co Ltd [2000] 2 SLR(R) 407 (SGCA) ("Tribune
Investment Trust Inv v Soosan Trading Co Ltd") at [40], “Nevertheless, the function of the court is to try
as far as practical experience allows, to ensure that the reasonable expectations of honest men are not
disappointed. To this end, it is also trite law that the test of agreement or of inferring consensus ad idem
is objective. Thus, the language used by one party, whatever his real intention may be, is to be construed
in the sense in which it would reasonably understood by the other.”

15
acceptable or normal when A is only subjectively caring for B as a younger
sister.

4. Justification – Why do we take an objective approach? The primary reason is that there
is a need to protect the reasonable expectations of contracting parties. Contract law
facilitates the security of transactions and it would be impossible to ensure this security
if people could simply say “when I said ‘white’ I meant ‘black’”. 3 The law also cuts
back on expectations by limiting them to the reasonable ones of the recipient.

a. Using the dating example again from above, assuming A now merely interacts
with B normally. However, B, being the hopeless romantic, subjectively
believes that A is madly in love with her. Most of her friends would, without a
doubt, tell her that she is a hopeless romantic and ask her to wake her idea up.

Agreement

5. In general, the starting point for determining whether there is an agreement is to


consider whether there is an offer by one party which is met by an acceptance by the
other. This is the legal convention (i.e. well established song and dance) by which an
agreement in a contract is established.

6. The problem with contract law is whether this is the only method in which an
agreement between the parties have been found. Facts are often, in several cases, force-
fit into this paradigm.

a. In loan agreements, the drafting of the contract is handled by the lead counsel,
which takes instructions from his clients as well as the comments from the
borrower and other parties. Can it be said that there is an offer by any of the
party which was accepted by the rest? In these cases, after all the negotiating
points have been settled, counsel will put forward a “execution version” of the
contract which all parties simply sign off.

3
Mindy Chen-Wishart, Contract Law (OUP, 6th Ed, 2018) at [2.1.1]. Note: Mindy’s other two
justifications are less persuasive – while we cannot determine a party’s actual intention, there is nothing
to stop us from inferring the actual intentions. In practice, reference is still made to the objective standard
as a basis to infer the subjective intentions.

16
b. A competition organiser creates rules for a race. The competitors participate in
the race. Can it be said that as between the competitors, they have offered the
rules to each other and accepted the same?

c. Similar observations has been made for example in Gibson v Manchester CC


where Diplock LJ observed that “there may be certain types of contract, though
I think they are exceptional, which do not fit easily into the normal analysis of
a contract as being constituted by offer and acceptance”. 4

7. Perhaps the better view is that the offer-acceptance model is one of the many methods
in which an agreement could be found. It is a sufficient but not necessary approach.
Examples would be the test in Butler v Ex-Cell-O.

a. In Butler v Ex-Cell-O, the manufacturer quoted a price for manufacturing a


machine for the buyer on its standard terms with a price variation clause. The
buyer placed an order on its standard form without any price variation. The
buyer’s form contained a tear-off slip which invited the manufacturer’s
signature acknowledging the order to be on the buyer’s terms. The manufacturer
signed and returned the slip, but also attached a letter stating that they were
supplying the machine on their original terms. The manufacturer relied on its
own terms to claim an additional sum on delivery of the machine. The EWCA
held that the manufacturer’s original offer was met by the buyer’s counter offer,
which the manufacturer accepted when it signed and returned the
acknowledgement slip in the buyer’s terms. The accompanying letter was
construed not to be a counter-offer but only a means of identifying the order.

b. Denning MR put forward an alternative approach in that the Courts should


determine whether the parties have (1) reached agreement on all material points,
even though there may be differences between the forms and conditions, and
(2) if the terms can be reconciled so as to give a harmonious result, all is well
and good. If they are irreconcilable, the conflicting terms may have to be
scrapped and replaced by reasonable implication.

4
Gibson v Manchester CC [1979] 1 WLR 294 (UKHL) ("Gibson v Manchester CC")

17
c. Although Denning’s approach is often criticised for uncertainty, it bears to note
that his approach is mirrored in many international standards such as the US
Uniform Commercial Code (§ 2-207), Vienna Convention on Contracts for the
International Sale of Goods (Art 19), the UNIDROIT Principles of International
Commercial Contracts (Art 2.11); and the European Draft Common Frame of
Reference (Arts II.-4:208-II.-4:209). The general principle is that only material
inconsistencies will prevent contract formation (see the list of material terms in
Vienna Convention – price, payment, quality and quantity of goods, place and
time of delivery, extent of one party’s liability to the other or the settlement of
disputes).

d. Even if it was found that there was no contract (an outcome which the Courts
did not want to avoid), it may be possible to place a claim in the law of unjust
enrichment as well (unjust factor being failure of condition – being that there is
a contract on their terms).

Offer

What is an offer?

8. The test for whether the conduct or statements made by one party is an offer is whether
that party evinces an intention to be bound, objectively ascertained. 5

Offer and requests for information

9. As an illustration of this test is in Harvey v Facey. In this case, H sent a telegram to F


asking, “Will you sell us Bumper Hall Pen? Telegraph lowest cash price.” F replied,
“Lowest price for Bumper Hall Pen, £900.” H purported to accept this “offer”. The
Privy Council found no binding contract because F merely supplied information in
response to H’s request. F evinced no intention to be bound by H’s acceptance. 6

5
See Aircharter World Pte Ltd v Kotena Nasional Bhd [1999] 2 SLR(R) 440 (SGCA) ("Aircharter World
Pte Ltd v Kotena Nasional Bhd") at [30], “An offer, simply defined, is an expression of willingness to
contract made with the intention (actual or apparent) that it shall become binding on the person making
it as soon as it is accepted by the person to whom it is addressed.”
6
Harvey v Facey [1893] AC 552 (UKPC) ("Harvey v Facey")

18
a. Consider in the context of Carousell listings which specifies “$0” in the listing
price for a Hello Kitty soft-toy. Would a response of “$20” to a query asking
how much the item is going for amount it is being sold be considered to be an
offer?

b. Changing the facts a little, would a response of “I am willing to sell this at $20
to you” amount to an offer?

c. Interestingly, Carousell seems to treat all listings as invitations to treat – there


is a button for the buyer to “make offer” and it is for the seller to accept.

Offers and invitations to treat

10. A party’s communication may not be an offer (lacks an intention to be bound), but is
merely an intention to treat (an expression of willingness to negotiate with another
party). This is illustrated by the cases concerning the sale of council houses in UK:

a. Storer v Manchester CC – offer – The Manchester City Council sent a brochure


advertising a scheme for tenants to buy their council houses. S submitted a
formal application to buy, which the council responded by letter saying “I
understand you wish to purchase your council house and enclose the agreement
for sale. If you will sign the agreement and return it to me, I will send you the
agreement signed on behalf of the corporation in exchange.” S signed and
returned the agreement, but control of the council changed (from Conservative
to Labour). This was a test case to consider whether there was a contract
concluded. The EWCA held that a contract was concluded – the council’s letter
objectively evinced an intention to be bound by the terms of the agreement as

19
soon as S accepted it. The subsequent signing by the council was merely to
return a counterpart (i.e. gets a copy of what he has signed). 7

b. Gibson v Manchester CC – invitation to treat – G inquired the price for the


council house. In response, the council informed G of the price which the
“council may be prepared to sell the house” and giving details of the mortgage
proposal. The council expressly stated that its letter should not be “regarded as
a firm offer of a mortgage” and that G is required to complete a further form if
he wished to make a “formal application”. There was a subsequent change in
control of the council, and the issue was whether there was a contract concluded
(as a test case). It was held that the council’s letter was not an offer but merely
an invitation to treat, which was clear from the statements in the letter regarding
the mortgage and the formal application.8

Presumptive rules or conventions to particular common situations

11. The Courts have, in specific situations, enacted certain presumptive rules and
conventions. These rules reflect what a reasonable person would generally view, in
those circumstances, certain communications as an offer or not. These may be displaced
by evidence of contrary intention.

Displays and advertisements

12. The general rule is that displays or advertisements of goods for sale are invitations to
treat. The customer is regarded as making the offer when she presents the goods at the
cashier. The shop-keeper has the power to accept or reject the customer’s offer.

a. Mindy argues that the “reasonable consumer or reader is likely to believe that
the storekeeper or advertiser intends to offer the items for sale”, is this view
defensible (statistically do we have information on this point?)9

b. Should we allow the shop-keeper the power to change his mind as to the terms?
The policy consideration here is whether there is a risk of misleading

7
Storer v Manchester CC [1974] 1 WLR 1403 (EWCA) ("Storer v Manchester CC")
8
Gibson v Manchester CC [1979] 1 WLR 294 (UKHL) ("Gibson v Manchester CC")
9
Mindy Chen-Wishart, Contract Law (OUP, 6th Ed, 2018) at [2.2.1.2].

20
advertising or discriminative sales. Mindy takes the view that perhaps contract
law should recognise this interest and relook at the presumptions or the
application of the rules.

c. This is illustrated in the case of R v Warwickshire CC. The case concerned an


employee which placed a notice outside his employer’s electric retail store
offering to “beat any TV, Hi-Fi and Video price by £20”. A customer saw a TV
set in the store advertised elsewhere at £159.95, but sought to purchase the
television for £20 lower. The employee refused. It was held that, under the
consumer protection legislation, refusing a consumer goods and services on the
terms set out in a notice may be misleading price indication, which was an
offence. 10 For a local example, see the Huawei $54 promotion saga where
Huawei advertised that handphones were sold at $54 dollars instead of the usual
price, but had sold out so quickly (very little stocks).11

d. At the end of the day, it is still the test of “intention to be bound” that is of
concern. The presumptions play very little in reality, save for a quick rule of
thumb.

13. Illustrations:

a. Pharmaceutical Society of GB v Boots Cash Chemists – Priced goods on


display – Invitation to treat – In this case, a legislation required that certain
drugs are to be sold under the supervision of a registered pharmacist. Boots
introduced self-service shopping and the pharmacist was positioned to supervise
the sale at the cashier. Whether Boots contravened the legislation depended on
when offer and acceptance took place to form the sale. If the displayed goods
was an offer and the customer’s placing of the item in the basket or presenting
for purchase was acceptance, then Boots would have contravened the legislation
as the supervision came after the sale. It was held that Boots did not contravene

10
R v Warwickshire CC [1993] AC 583 (UKHL) ("R v Warwickshire CC")
11
https://www.channelnewsasia.com/news/singapore/huawei-says-clearly-underestimated-demand-in-
s-54-phone-11766302

21
the legislation as displays and advertisements are generally only invitations to
treat.12 Mindy has criticised the reasons for the conclusions however.13

b. Patridge v Cittdenden – Advertisements – Invitation to treat – In this case,


certain wild live birds were advertised for sale at a stated price and the advertiser
was charged with the offence of “offering for sale” under a wildlife protection
legislation. The advertiser was acquitted as the Court applied the convention
that it was only an invitation to treat and not an offer, 14 reasons for which Mindy
considers unconvincing. 15 However, the solution to this may be to interpret the
legislation broader to cover invitations to treat as well (i.e. offence to cover
soliciting offers).

c. Workspace Consultants Pte Ltd v Teo Seng Siew – Advertisement in paper –


Invitation to treat – In this case, the tenant had sub-leased its premises (which
was meant only for residential uses under the head-lease) to the sub-tenant. The
sub-tenant wanted to move out of the premises and promised to find a
replacement sub-tenant. The sub-tenant had asked a real-estate agent to
advertise the premises, which she did, containing very minimal information,
“D.15 TG. Katong Rd. 900 sq. ft. Fully carpeted. Asking $2000 Move in cond.
92121862 Malpore”. The new sub-tenant used the premises as an office, which
was in breach of the covenants in the head-lease. The new sub-tenant sued the
tenant for breach of contract and tort of negligent misstatement, as well as the
original sub-tenant for negligent misstatement. It was argued that there was no
contractual relationship between the new sub-tenant and the original sub-tenant
as the advertisements were only an invitation to treat, which the Court accepted.
However, the original sub-tenant was found liable for negligent
misstatements.16

12
Pharmaceutical Society of Great Britan v Boots Cash Chemists [1953] 1 QB 401 (EWCA)
("Pharmaceutical Society of Great Britan v Boots Cash Chemists")
13
Mindy Chen-Wishart, Contract Law (OUP, 6th Ed, 2018) at [2.2.1.2].
14
Patridge v Cittenden [1968] 1 WLR 1204 (EWCA) ("Patridge v Cittenden")
15
Mindy Chen-Wishart, Contract Law (OUP, 6th Ed, 2018) at [2.2.1.2].
16
Workspace Consultants Pte Ltd v Teo Seng Siew and Another [1998`] SGHC 372 (SGHC)
("Workspace Consultants Pte Ltd v Teo Seng Siew and Another")

22
d. Carlill v Carbolic Smoke Ball – Advertisement – Offer – In this case, the
manufacturer advertised the “carbolic smoke ball” and offered to pay £100 to
anyone catching the flu after using it in the prescribed manner, adding that
£1,000 had been deposited in the bank to show their “sincerity in the matter”. C
successfully sued for the £100 when she caught the flu after the proper use of
the smoke ball. The Court held that the advertisement was an offer to the whole
world, that a unilateral contract was made to those who met the condition on
faith of the advertisement (by performing their part of the contract). 17

Auctions

14. Generally, a sale by auction takes place in the following steps:

a. The advertisement that an auction will take place on a certain day or the putting
up of goods for sale is merely an invitation to treat;

b. The bid by the purchaser is the offer; and

c. The fall of the hammer indicates the acceptance.

15. As a general position, the auctioneer can refuse to hold the auction at all (e.g. if it is
poorly attended) or refuse to sell to the highest bidder if the bid fails to research the
research (i.e. the minimum price) – this is what a reasonable consumer would
understand. Otherwise, the auctioneer could be exposed to a multiplicity of claims if
the advertisement were to be construed as an offer.

16. However, where the words “without reserve” is added to the advertisement, there is a
reasonable expectation that the auction will take place, no minimum price will be set
and the item will be sold to the highest bidder. In such a case, the transaction can be
construed as two separate contracts, the first being a unilateral collateral contract
between the bidder and the auctioneer by which the auctioneer is obliged to accept the
highest bid from the bidder making it. The main contract for sale is between the highest
bidder and the owner (Barry v Davis).

17
Carlill v Carbolic Smoke Ball [1893] 1 QB 256 (EWCA) ("Carlill v Carbolic Smoke Ball")

23
a. Barry v Davis – advertisement as an offer to hold an auction – In this case,
two machines worth £28,000 pounds were auctioned without reserve, but the
auctioneer refused to accept B’s bids of £200 each. B was awarded £27,600 in
damages (the amount required to obtain replacement machines).18

Tenders

17. The analysis for tenders is similar to that in auctions (see Spencer v Harding):

a. The advertisement calling for tenders is merely an invitation to treat;

b. The tenders submitted are offers; and

c. Acceptance is made when the person inviting the tenders accepts one of the
tenders.

18. However, the Courts have, in certain cases, adopted a two contract analysis in order to
protect the reasonable expectations of bidders and imposed liability for a failure to
consider, or to accept, the lowest or highest tender.

a. Harvela Investments v Royal Trust Co of Canada – R sent a telax to two parties


inviting them to submit competitive sealed tenders. In the telex, it was specified
that “we bind ourselves to accept [the highest offer] provided that such offer
complies with the terms of this telex”. H tendered a fixed bid of $2,175,000 and
X tendered a referential bid of $2,10,000 or $101,000 in excess of any other
offer, whichever is higher. R accepted X’s bid as one for $2,276,000. The
UKHL held that R was bound to accept H’s bid. R’s invitation to tender was an
offer of a unilateral contract to sell shares to the highest bidder. The terms of
the invitation was construed to exclude referential bids – the object was to
ascertain the highest amount each party was prepared to pay and the purpose
would be frustrated by a referential bid. In particular, had both parties bid
referentially, it would have been impossible to determine which was the highest
bid.19

18
Barry v Davies [2000] 1 WLR 1962 (EWCA) ("Barry v Davies")
19
[] Footnote to follow for this case – download UKHL version.

24
b. Blackpool and Fylde Aero Club v Blackpool BC – In this case, the council
invited tenders for a concession to operate flights. It was stipulated in the
invitation that “the council do not bind themselves to accept all or any part of
the tender. No tender which is received after the last date and time specified …
shall be admitted for consideration”. The Aero Club had posted their tender in
the town hall letterbox on which a notice was attached to the box stating that it
was emptied at noon each day but the letter box was not duly processed and the
council did not consider the Aero Club’s bid. The Court considered that there
invitation constituted an offer to consider any conforming tender which was
accepted by the party submitting such tender. The council breached the contract
to consider the Aero Club’s tender.20

Automated machines

19. In the context of automated machines, the offer and acceptance analysis has been
applied in Thornton v Shoe Lane Parking.

a. In this case, the automatic machine outside a car park stated rates, which the
motorist drives in and the machine will issue a ticket. The ticket specifies that
the motorist is bound by the exclusion of liability clause contained in a notice
within the car park. The motorist in this case suffered an injury from the
premises (occupier’s liability) and the issue was whether this liability was
excluded. It was held that the offer was made by the machine with the rates and
the acceptance was by the motorist driving in – the contract comes into being.
The notices were only brought to the motorist’s attention after the contract was
made, and as such was not part of the contract. There was no expectation or
opportunity for negotiation and once the consumer drives in there is also no
scope for withdrawal.21

20
Blackpool and Fylde Aero Club Ltd v Blackpool Borough Council [1990] 1 WLR 1195 (EWCA)
("Blackpool and Fylde Aero Club Ltd v Blackpool Borough Council")
21
Thornton v Shoe Lane Parking Ltd [1971] 2 QB 163 (EWCA) ("Thornton v Shoe Lane Parking Ltd")

25
Online shops

20. There is a conventional interpretation of a few cases (such as Hartog v Colin & Shields,
Smith v Hughes) that if the seller subjectively knew, or objectively had reason to know
and it would be inequitable for him in the circumstances to insist on performance, in
situations of mistake as to terms (i.e. I thought you were promising X). However, Mindy
has suggested that the better interpretation should to say that in unilateral mistake cases,
there is no contract formed at all, on an objective interpretation. 22

a. In Digilandmall, D’s employee mistakenly advertised online commercial laser


printers for $66 each (the actual retail price is $3,854). When the error was
detected, 4,086 orders had already been received and confirmation notes
automatically dispatched within a few minutes. D refused to supply C’s order
for 1,606 printers. The SGCA held that actual knowledge was required to void
the contract under unilateral mistake in law.

b. However, adopting Mindy’s approach, it could be argued that a reasonable


person in the shoes of the consumer would know that there was no offer at all –
the circumstances was such that a reasonable person would know that the seller
did not intend to be bound to the price stated and as such there was no contract
at all.

21. Separately, it has also been clarified that the rules of offer and acceptance are to apply
in the electronic context under the Electronic Transactions Act, save for a few excluded
transactions (which the common law continues to apply and develop). See in particular
the following provisions under the Electronic Transactions Act:

Formation and validity of contracts


11.—(1) For the avoidance of doubt, it is declared that in the context of the
formation of contracts, an offer and the acceptance of an offer may be expressed by
means of electronic communications.
(2) Where an electronic communication is used in the formation of a contract, that
contract shall not be denied validity or enforceability solely on the ground that an
electronic communication was used for that purpose.

22

26
Invitation to make offer
14. A proposal to conclude a contract made through one or more electronic
communications which is not addressed to one or more specific parties, but is
generally accessible to parties making use of information systems, including a
proposal that makes use of interactive applications for the placement of orders
through such information systems, is to be considered as an invitation to make offers,
unless it clearly indicates the intention of the party making the proposal to be bound
in case of acceptance.

Unsolicited goods or services

22. The general rule for unsolicited goods or services is that a consumer may regard them
as unconditional gifts to him from a supplier and is not an offer. The rationale is to
prevent the law from being used by the suppliers who engage in sharp practices, and to
deter such conduct (see the regulation under the Consumer Protection (Fair Trading)
(Opt-out Practices) Regulations 2009).

Unsolicited goods or services

3.—(1) Unless and until the consumer expressly acknowledges to the supplier in
writing his intention to accept and pay for the unsolicited goods or services, the
consumer may use, deal with or dispose of the goods or services as if they were
an unconditional gift to him from the supplier.

(2) Unless the consumer has given the acknowledgment referred to in


paragraph (1), the supplier shall not have a cause of action for any loss, use, misuse,
possession, damage or misappropriation in respect of the goods or services or the
value obtained by the use of the goods or services.

How and when is an offer be terminated?

Revocation

23. A distinction needs to be drawn between unilateral contracts and bilateral contracts:

a. In a unilateral contract, there is an exchange of a promise for a completed act


(or refraining from an act).

b. In a bilateral contract, there is an exchange of a promise for a promise.

27
24. As a general rule, an offeror can revoke her offer any time before the offeree
communicates her acceptance (unless there is a valid contract such as an option or firm
offer23). The exception is in offers for unilateral contracts, which may not be revocable
once the offeree has commenced performance. The policy concern here is that there is
an induced reliance and the offeror’s receipt of unpaid-for benefits. The Courts have
inclined on two lines of reasoning:

a. The unilateral offer may be accepted as soon as performance has begun


unequivocally. The contract is concluded on acceptance, but the offeree cannot
enforce the contract until actual completion of the stipulated performance. This
view has support in Errington v Errington. In this case, the father (deceased)
promised his son and daughter-in-law that a house which they were living in
should belong to them when they have finished paying off the mortgage on the
property. The couple began to pay the mortgage but this was not yet completed.
The issue was whether the mother (as the estate of the father) could revoke the
contract. Denning LJ considered that the father’s promise was a unilateral
contract which could not be revoked by him once the couple entered on
performance of the act of paying the instalments. 24

b. The Courts may find that there is a promise by the offeror not to revoke the offer
once the offeree has started performance under a two-contract analysis. This
view has support by the suggestion of Goff LJ in Daulia v Four Millbank, which
suggested that there is an implied obligation on the part of the offeror not to
prevent the condition becoming satisfied (such as revoking the offer) which
obligation must arise as soon as the offeree starts to perform.25

c. However the Courts have emphasized that there is no universal proposition that
an offeror is not at liberty to revoke the offer once performance of the sought
act commenced (Mobil Oil Australia v Lyndel Nominees).

23
Note that it is insufficient for the offer to merely state that it will be kept open until a particular date
– the Court must still find that there is a valid acceptance and consideration to keep that offer open (see
Dickinson v Dodds). As such, such options are to be made by way of deed or full contracts to be
binding.
24
Errington v Errington [1952] 1 KB 290 (EWCA) ("Errington v Errington")
25
Daulia v Four Millbank [1978] Ch 231 (EWCA) ("Daulia v Four Millbank")

28
i. In this case, franchisees of M argued that a retail marketing manager of
M made 2 promises – for any year that a franchisee achieved above a
certain performance criteria, they would receive (following the expiry
of their agreements) an additional year of tenure – if they had exceeded
the performance in all of the 6 calendar years they would receive an
additional 9 years service station tenure at no cost. Later, M clarified
that they would not grant renewals free of charge. It was held that there
was no ancillary promise – the act of acceptance was ill-defined, no
benefit passed to M from that performance and the franchisees were
contractually bound to M to achieve certain standards in any case. 26

25. A revocation of an offer is only valid if it is communicated to the offeree before the
latter’s acceptance. Exceptionally, an offer may be revoked by a reliable third party
acting without the offeror’s authority, with support being from Dickinson v Dodds.

a. In Dickinson v Dodds, the defendant had made an offer to sell to the plaintiff
property, which was stated to be open until 9am on 12 June. On 11th June, the
defendant had sold the premise to a third party without informing the plaintiff,
but the plaintiff was in fact cognizant of the sale. The plaintiff “accepted” the
defendant’s offer and sued for specific performance of the contract allegedly
formed. The Court refused and held that there was no contract at all.

b. It has been suggested that there could be an implied revocation, and the key
issue is how reliable is the information received by the third party.

Rejection by the offeree

26. An offer is terminated where there is any variation in the terms of the offer itself by the
offeree. It would apply a fortiori that if an offer is rejected, it would be terminated as
well (see the discussion on counter-offer below at [].)

26
Mobil Oil v Lyndel Nominees [1998] 153 ALR 198 (FCA) ("Mobil Oil v Lyndel Nominees")

29
Lapse of the offer

27. An offer lapses after the time fixed in the offer for its validity, expiry of an express or
implied condition for validity, or after a reasonable time where no time was stipulated.

Death of offeror or offeree

28. An offer generally lapses on the death of the offeror or the offeree. However, Errington
v Errington suggests that an offer may remain open if the offeror could not have
terminated the offer during his lifetime, and the performance of the contract does not
depend on the offeror’s personality but can be satisfied out of his estate.

Acceptance

What is an acceptance

29. For there to be a valid acceptance, there must be an unequivocal expression of assent
to the proposal contained in the offer. An acceptance has the effect of immediately
binding the parties to the contract.

30. In this regard, cross-offers and counter-offers are not acceptances in the eyes of the law
as it is not an expression of assent to the proposal contained in the offer.

a. Cross-offer – In Tinn v Hoffman, H wrote to T offering to sell him 800 tons of


iron at 69s per ton. On the same day, T wrote to H offering to buy on the same
terms. It was suggested obiter that these simultaneous offers, made in ignorance
of each other, would not bind the parties. The rationale suggested by Mindy is
that it is impossible, without the nexus of an offer and acceptance, that each
party knows when they are contractually bound, when it is safe to act in reliance
on the contract and when liability will be incurred for departure – a situation in
cross-offers.27

b. Counter-offer and requests for information28

27
See the summary of Tinn v Hoffman and discussion in Mindy Chen-Wishart, Contract Law (OUP,
6th Ed, 2018) at [2.3.2.1].
28
See the summary of cases discussion in ibid.at [2.3.1.2].

30
i. Counter-offer – In Hyde v Wrench, W offered to sell his farm for £1,000.
H said he would only pay £950 which was rejected by W. H then agreed
to pay £1,000 which W also rejected. H’s proposal of £950 was a
counter-offer which terminated the original offer.

ii. Request for information – In Stevenson, Jacques & Co v McLean, M


offered to sell S iron at “40s per ton net cash”. S asked whether M
“would accept forty for delivery over two months, or if not, longest limit
you would give”. M then sold the iron to a third party. Hearing nothing,
S accepted the original offer by telegram before M’s revocation reached
S. The Court held that S merely made an inquiry, not a counter-offer
extinguishing the offer.

iii. The lesson learnt here is an offeree should be polite and request for
more information and not appear as if she is rejecting the original offer,
so as to preserve her power to accept the original offer.

Communication

Generally must be communicated to the offeror

31. Generally, acceptance must be communicated to the offeror by the offeree (Entores Ltd
v Miles Far East Corp).29

a. Note: Entores/Brinkibon were cases dealing with where the contract is


concluded and where/when it takes place – choice of law concerns.

b. In this case, it was further suggested that where correspondences which are
virtually instantaneous are not covered by the postal acceptance rule. However,
the better view is that of what the parties actually intended (1L30G Pte Ltd v
EQ Insurance Co Ltd at [28]). The crux, in the context of an offer and

29
Entores v Miles Far East Corp [1953] 2 QB 327 (EWCA) ("Entores v Miles Far East Corp")

31
acceptance is whether the offeror intends for the postal acceptance rule to
apply.30

c. In Entores, the following were suggested:

i. In a face-to-face oral acceptance was drowned out by a noisy aircraft,


the offeree must repeat the acceptance.

ii. If the telephone goes dead before acceptance is completed, the offeree
must telephone back to complete acceptance.

iii. If the offeror does not catch the clear and audible words of acceptance,
or the printer receiving a telex runs out of ink, but the offeror does not
bother to ask for the message to be repeated, the offeror will be bound.

32. The crux here is that communication is crucial, because it informs the parties as to when
their obligations to perform arise.

Postal acceptance rule

33. Exceptionally, the postal acceptance rule allows for valid acceptance even if it is not
communicated to the offeror. The postal acceptance rule applies where there is an
expressed or implied authorization for acceptance by post, and that the letter must have
been properly stamped and addressed (Household Fire and Carriage Accident v
Grant).31The acceptance occurs at the posting of the letter, once the conditions have
been fulfilled.

a. While the rule may seem to be prejudiced against the offeror, it is open to the
offeror to exclude the postal acceptance rule.

b. Household Fire, the defendant applied for shares in the plaintiff’s company.
The company allotted the shares to the defendant, and duly addressed to him
and posted a letter containing the notice of allotment, but the letter was never

30
1L30G Pte Ltd v EQ Insurance Co Ltd [2017] SGHC 242 (SGHC) ("1L30G Pte Ltd v EQ Insurance
Co Ltd")
31
Household Fire and Carriage Accident Insurance Co Ltd v Grant [1879] 4 Ex D 216 (EWCA)
("Household Fire and Carriage Accident Insurance Co Ltd v Grant")

32
received by him. It was held that there was valid acceptance on communication
and the defendant was liable to pay for his shares (to the liquidator as the
company was in liquidation).

One-way instantaneous

34. In the cases of one-way instantaneous communications, it has been observed that “no
universal rule can over all such cases: they must be resolved by reference to the
intentions of the parties, by sound business practice and in some cases by a judgment
where the risks should lie”. 32 It has been suggested that a test of “when a reasonable
offeror would access the message in all the circumstances” should be adopted so as to
allow the Court to balance the parties’ respective interest (e.g. if a message was sent
out of office hours or to non-business premises, there is no valid acceptance). 33

Method of acceptance

35. An acceptance must be made in an appropriate way so as to properly manifest one’s


intention to be bound. Where the method is not prescribed, any conduct that objectively
evince the offeree’s intention to accept is sufficient.

Prescribed acceptance

36. If an offer requires the offeree to comply with a particular method of acceptance, the
acceptance which deviates from this may not bind the offeror unless the offeror waives
her right to insist on compliance.

37. However, it is open to the Courts to interpret the offer to conclude that the offeror did
not intend to limit the method of acceptance by the stated method of acceptance.

a. For example, in Manchester Diocesan Council v Commercial and General


Investments, M invited tenders on a particular form stating that the person
whose bid was accepted would be informed by a letter sent to the address
contained in the tender. M sent an acceptance of C’s tender to C’s surveyor
rather than to the address in C’s tender. The Court held the offer did not intent

32
Brinkibon v Staghag Stahl [1982] 2 WLR 264 (UKHL) ("Brinkibon v Staghag Stahl")
33
Mindy Chen-Wishart, Contract Law (OUP, 6th Ed, 2018) at [2.3.4.3].

33
to limit the method of acceptance and that as C was not disadvantaged by the
notification to its surveyor, M’s acceptance was effective. In particular, it was
the offeree who made the stipulation (in the template offer) for the offeree’s
benefit, it was open for the offeree to waive strict compliance. 34

Acceptance by silence

38. As a general rule, acceptance cannot be inferred from the offeree’s silence (Felthouse
v Bindley).

a. In this case, F offered to buy his nephew’s horse, adding that “if I hear no more
about him, I consider the horse mine at £30 15s.”. The nephew did not answer
F, but told B, the auctioneer, that the horse had already been sold. B mistakenly
sold the horse and F sued B for conversion. The Court held against F because
his offer to buy the horse could not be accepted by his nephew’s silence.

b. The justifications for the rule is that (i) silence is ambiguous, (ii) acceptance
must generally be communicated to the offeror so that she knows when a
contract binds both parties and (iii) the rule prevents an offeror from exploiting
an offeree’s intertia.

c. Note that Mindy has criticised the rule’s application in Felthouse v Bindley,
arguing that non the facts none of the reasons had applied – the nephew had
manifested his acceptance by informing the auctioneer that the horse had been
sold, the uncle had waived the need for communication of the acceptance and
the nephew had no need for protection from unwanted contractual liability. In
this case, it should be inferred that the nephew’s conduct amounted to an
acceptance.35

34
Manchester Diocesan Council for Education v Commercial and General Investments Ltd [1970] 1
WLR 241 (EWHC) ("Manchester Diocesan Council for Education v Commercial and General
Investments Ltd")
35
Mindy Chen-Wishart, Contract Law (OUP, 6th Ed, 2018) at [2.3.3.2].

34
39. However, the touchstone is still whether the conduct of the parties supports the
existence of an acceptance. Silence is only a factor to be considered (see Midlink
Development Pte Ltd v Stansfield Group).

a. In Midlink Development, the defendant-tenant’s lease had terminated and the


plaintiff-landlord and the defendant had entered into negotiations for a new
lease. The plaintiff had forwarded the new tenancy agreements reflecting the
revised rental for execution but the defendant did not sign the agreements. The
defendant however, continued to pay the reduced rental charges. The defendant
later sought to terminate the lease, but the plaintiff disputed the defendant’s
right to terminate on the basis that there was an agreement to lease the premises
up to 30 June 2004. It was held that there was a concluded agreement reached,
inter alia, that the defendant’s silence by not signing the tenancy agreement was
not decisive. The omission to sign, together with the defendant’s subsequent
conduct was an acceptance (defendant had paid the adjusted rent, all material
terms had been settled) and the signature was a mere formality. In particular the
omission was intended to “take advantage” of the plaintiff, by allowing the
defendant the purported right to terminate anytime it wished.36

Acceptance with knowledge of the offer

40. The general rule is that there can only be valid acceptance if the party purporting to
“accept” the offer had knowledge of the offer before it’s “acceptance” (see the
ignorance case of counter-offers in Tinn v Hoffman). The justification for this rule is
that contracts usually come with liabilities as well as benefits, it would not be fair to
impose such liabilities on a party’s purported acceptance, if that party did not know
about the offer and the liabilities.

41. However, the policy concerns do not raise themselves in unilateral contracts/reward
situations, where generally speaking the offeree is not under any liability, save for the
actual performance. The cases themselves are not easily reconcilable. One possible
interpretation is that provided that one knows of the offer, it is unnecessary for the

36
Midlink Development Pte Ltd v The Standsfield Group Pte Ltd [2004] 4 SLR(R) 258 (SGHC)
("Midlink Development Pte Ltd v The Standsfield Group Pte Ltd")

35
acceptance to be in reliance on that offer, although it does not explain Gibbons v
Proctor.

a. In Gibbons v Proctor, a police officer was entitled to claim a reward because


although the officer was ignorant of the offer when he gave information to a
fellow officer, he knew of it by the time the information reached the
superintendent after passing through other hands. (Knowledge arose after
performance).37 Note that this case has been doubted in R v Clarke by the High
Court of Australia. All that matters is that he is aware of the offer when he
communicates the acceptance (i.e. when he steps forward to claim the reward,
not when performance is done) – Burton.

b. In Williams v Carwardine, the informant had acted because she believed she
would not live long and wished to ease her conscience. The court held that she
was entitled to the reward because she must “have known” of the reward, even
if the informant did not act from the desire to receive the reward. (Knowledge
arose before performance but was not the motive).38

c. In contrast, R v Clarke was a case where C was charged with the murders and
gave the relevant information in order to absolve himself. C’s claim for the
reward was rejected, because he admitted when giving the information, he had
forgotten about the reward. (Knowledge arose before performance but was
forgotten).39

Consideration

Basic idea and justification

42. Note: this is a summary in Mindy’s textbook. 40

37
See the summary and discussion in Mindy Chen-Wishart, Contract Law (OUP, 6th Ed, 2018) at
[2.3.2.2].
38
See the summary and discussion in ibid.at [2.3.2.2].
39
R v Clarke [1927] 40 CLR 227 (HCA) ("R v Clarke")
40
Mindy Chen-Wishart, Contract Law (OUP, 6th Ed, 2018) at [3.1.1].

36
43. The basic idea of the consideration requirement is that in order to acquire the right to
enforce another’s undertaking, that party must undertake to give/actually give
something stipulated by the other as the price for her undertaking.

a. Reciprocal promise from Y against a reciprocal promise from Z (bilateral


contract);

b. Actual performance from Y against a reciprocal promise from Z (unilateral


contract).

44. There are several views as to what consideration serves:

a. Evidence of existence and seriousness of undertaking – the most common


view is that consideration performs an evidentiary, cautionary and channeling
function, analogous to formality requirements (i.e. evidence of an intention to
be bound). However, consideration’s role is not only this – even if promisor
clearly intends to be bound, informal undertaking is still unenforceable without
consideration.

b. Welfare maximizing – welfare theory states that exchanges tend to transfer


resources to those who value them most highly, consideration is the best
indicator of value-maximising transactions. However, it is actually quite a poor
tool to ensure that transactions are value-maximising (donations are welfare
maximizing, buyer who buys under a mistake is not).

c. Reciprocity – it is to protect the norm of reciprocation – a person who has given


something in return is more deserving of protection than one who hasn’t. Once
we establish consideration, it is generally harder for one party to try and avoid
contracts.

d. Gift and exchange distinction – exchanges are generally public enforceable


transactions, whereas gifts are usually private unenforceable agreements. Gifts
are backed by social or moral sanctions instead. Exchanges (in particular with
strangers) lack trust – contract law bridges this gap.

Nexus

45. There must be a sufficient causal connection between the parties and the undertakings.

37
Consideration must move from the claimant

46. Consideration doctrine deals with two questions, namely (1) what undertakings are
enforceable (those which are exchanges), and (2) who can enforce the contract (the
party which has furnished consideration).

Consideration must be requested by the promisor

47. Consideration must be given in return for the promise to be enforced. The consideration
must be requested by the counterparty (Combe v Combe).

a. In Combe v Combe, a husband promised to pay his wife £100 a year on their
divorce. The wife sought to enforce the promise, arguing that she furnished
consideration by not applying for maintenance. The EWCA held that the
consideration was not requested by her husband and there was no binding
contract.

48. One may however invite the Court to imply a request for consideration (Alliance Bank
v Broom).

a. In Alliance Bank v Broom, B promised to provide security for its debt to A.


When A sued to enforce this, B argued that A gave no consideration. The Court
held that while B did not expressly ask A to forbear from suing B to recover the
debt, the Court implied such a request. 41

Past consideration is not good consideration and implied assumpsit

49. Consideration must be given in response to the counter-promise. As such, it cannot be


given before the other’s promise was made. If the “consideration” was given prior, the
law regards the latter promise as a gratuitous and unenforceable response to the earlier
“consideration”.

a. In Re McArdle, 5 children were beneficiaries of an estate (including M). M and


his wife made improvements to a property forming part of the assets under the
estate in 1944 (paid for by the wife). The 5 children signed a document on 1945

41
Case summarized in ibid. at [3.1.2.2].

38
stating that the wife was to be entitled to a sum of money for the improvements
from the estate when distributed. The wife claimed payment, but the other
children objected to the payment. It was held that the works were all completed
prior to the execution of the document and the consideration was past and not
enforceable as a contract.

50. However, under the doctrine of implied assumpsit, if the claimant can show that (1)
the earlier act was given at the promisor’s request, (2) it was clearly understood at the
time of the request that the claimant would be rewarded for the act, and (3) the eventual
promise is one which would have been enforceable if it had been made at the time of
the act, the past act is still good consideration (Pao On v Lau Yiu Long).42

a. In Pao On v Lau Yiu Long, P sold shares to X company and was paid by shares
in X company (share swap). L owned most of the shares in X company and
requested that P not sell 60% of the shares it received for a year to avoid
triggering a fall in their value. To protect P from any fall in value in that period,
L agreed to buy back the shares at $2.50 each at the end of the year. When P
realized that this would deprive them of any increase in the value of the shares
beyond $2.50, P refused to proceed with the main contract (entered in Feb 1973)
unless L agreed to (1) cancel the subsidiary agreement and (2) give a guarantee
by way of indemnity (i.e. L would only buy back if the shares fell below $2.50)
(in April 1973). When the shares fell below $2.50, L refused to indemnify P
arguing lack of consideration from P and duress. It was held that (1) there was
valid consideration from P to go ahead with main contract (pre-existing duty to
3p), (2) valid consideration in promise not to sell shares for a year, because it
was given at L’s request with the common intention that L should protect P
against a drop in value. The intention survived even though there was a
cancellation of the buy-back agreement – the subsequent promise was regarded
as merely fixing the amount payable.

b. In Sim Tony v Lim Ah Ghee, T was a civil servant. The vendor owned some
properties asked T to refer any prospective buyers to him. Two estate agents

42
Pao On v Lau Yiu Long [1980] AC 614 (UKPC) ("Pao On v Lau Yiu Long")

39
found a potential buyer, but wanted to deal with the vendor through L acting as
front man. T agreed to introduce L to the vendor. The vendor agreed to pay L
commission should the sale and purchase be completed during the first meeting.
At the second meeting, the vendor asked L to “take care” of T, which L agreed.
Shortly before completion of the transaction, the estate agents and L agreed to
give a share in the commission to T. T subsequently was dismissed from service,
for engaging in trade. T sued arguing that there was a contract to share the
commission between L and the estate agents. Applying Pao On, the SGCA held
that the second condition (understood that the claimant would be rewarded) was
not made out as T was acting out of friendship. 43

Value

51. A valuable consideration, in the sense of the law, may consist in some right, interest,
profit, or benefit accruing to the one party, or some forbearance, detriment, loss or
responsibility given, suffered, or undertaken by the other (Currie v Misa).44

Consideration must be sufficient but need not be adequate.

52. Consideration must be sufficient (i.e. legally valid) but need not be adequate (i.e. value
of promise A can be much letter in return for promise B) (Chappel & Co v Nestle).45

a. In Chappel v Nestle, the claimant sought an injunction to prevent the promisor’s


infringement of their copyright (by failing to pay them a percentage of the
profits of the sales that the wrappers represented). It was held that the supply of
1s 6d and three wrappers constituted good consideration. Per Lord Somervell,
“A contracting party can stipulate for what consideration he chooses. A
peppercorn does not cease to be good consideration if it is established that the
promisee does not like pepper and will throw away the corn”.

43
Sim Tony v Lim Ah Ghee [1995] 1 SLR(R) 886 (SGCA) ("Sim Tony v Lim Ah Ghee")
44
Case summarized in Mindy Chen-Wishart, Contract Law (OUP, 6th Ed, 2018) at [3.1.3.1].
45
Chappel & Co v Nestle [1960] AC (UKHL) ("Chappel & Co v Nestle")

40
Intangibles and moral obligations

53. Intangibles and moral obligations are generally held to be insufficient consideration
(White v Bluett). However, the line is difficult to draw.46

a. A promise made in consideration of a son’s “natural love and affection” was


held to be insufficient consideration (Bret v JS).47

b. In White v Bluett, it was held that a son’s promise not to annoy the father with
complaints was insufficient consideration. In this case, the son had complained
frequently that the father had distributed his assets unfairly among his children
and agreed to release his son’s obligation to pay under a promissory note (in
favour of the father).48

c. However, in Hamer v Sidway, an American decision, it was held that the


nephew’s refraining from drinking, smoking, gambling and swearing until the
age of 21 was good consideration in return of his uncle’s promise of $5,000. It
was held that the nephew had suffered detriment in giving up his lawful freedom
to drink, smoke, swear and gamble, which was a detriment. 49

i. Note that (1) it could also be said that the son in White v Bluett had
forgone a perfectly lawful conduct, but it was held that the son had “no
right to complain”, and (2) the concept of practical benefits later.

Illusory consideration

54. Where the recipient will inevitably get nothing, the consideration may be illusory and
insufficient (Arrale v Costain Civil Engineering).50

a. In this case, P suffered an injury during employment and had to amputate his
left arm. The P signed an agreement to settle any claim with the defendant
employers, D. P later went to sue D claiming damages at common law for

46
White v Bluett [1853] 23 LJ Ex 36 (ExC) ("White v Bluett")
47
Case summarized in Mindy Chen-Wishart, Contract Law (OUP, 6th Ed, 2018) at [3.1.3.4].
48
White v Bluett [1853] 23 LJ Ex 36 (ExC) ("White v Bluett")
49
Case summarized in Mindy Chen-Wishart, Contract Law (OUP, 6th Ed, 2018) at [3.1.3.4].
50
Arrale v Costain Civil Engineering [1976] 1 Lloyd's LR 98 (EWCA) ("Arrale v Costain Civil
Engineering")

41
negligence. It was argued that D had furnished consideration for P’s claims as
they never intended to invoke a provision in a statute as a defense, but on the
facts it was admitted that D had never intended to pursue that course of action.
It was held that D had not furnished consideration – it is “no consideration to
refrain from a course of action which it was never intended to pursue”.

Motive

55. A person’s mere wish to confer a benefit is not sufficient consideration (Thomas v
Thomas).

a. In this case, the executors of A promised to carry out A’s desire to let his widow
have his house for the rest of her life. It was held that A’s desire was not good
consideration as it was merely the motive for the executors’ promise. However,
as the executors had promised to carry out their promise if the widow had paid
rent and kept the house, the wife’s acts were good consideration. 51

Specific context: Compromise and forbearance to sue

56. Forbearance – Where A has a claim against B, A may agree to refrain from enforcing
the claim for a promise given by B. Such forbearance is good consideration because A
suffers a real detriment in not enforcing his legal rights, while B gains the benefit of
having more time to perform the outstanding obligation.

57. Compromise – A and B may also consider it beneficial to enter into a compromise
whereby one party agrees to surrender his claim in consideration for the other’s
payment or other promises. Such agreements are encouraged as a matter of policy,
which avoids costly and wasteful litigation.

58. Validity of claim and good faith

a. Valid consideration – Valid claim – Where A’s claim is clearly valid, the
compromise or forbearance is good consideration for B’s promise.

51
Mindy Chen-Wishart, Contract Law (OUP, 6th Ed, 2018) at [3.1.3.5].

42
b. Valid consideration – Doubtful claim and good faith– Where A’s claim is
doubtful, the compromise or forbearance is still good consideration for B’s
promise if the grounds are reasonable and A honestly believes that he has a fair
chance of success (Haigh v Brooks).52

c. Valid consideration – Invalid but good faith – Where A’s claim is invalid, the
compromise or forbearance is still good consideration for B’s promise if the
grounds are reasonable and A honestly believes that he has a fair chance of
success (Lim Beng Cheng v Lim Ngee Sing).53

i. In this case, BC had financed NS’s transactions by structuring the


financing as money for option to purchase NS’s property. NS was given
the right to repurchase the option at a higher price (about 50% more than
the option money). Eventually, NS was unable to repay, after several
rounds of negotiations and promised to transfer some interest in a
property and pay a sum of S79,352 to BC, in consideration of BC’s
waiving the debt of $279.352 and withdrawing the caveat over another
property. NS sought to set aside the compromise agreement, arguing that
the promise to withdraw the caveat was bad consideration as the plaintiff
did not have a valid claim. It was held that the consideration was valid
– the forbearance to sue on a doubtful or even “clearly invalid” claim is
good consideration, if there were reasonable grounds for the promisor’s
claim and if the promisor honestly believed that he had a fair chance of
success – this BC believed so.

d. Invalid consideration –Invalid and bad faith – Where A knew that his claim is
invalid, such compromise or forbearance is invalid consideration (Wade v
Simeon).

i. In this case, P sued D to recover a debt, btu agreed to forbear prosecuting


the proceedings in consideration for D’s promise to repay. D failed to
do so and P sued on the compromise agreement, because P knew that

52
Case not read - see ibid.at [3.1.3.6].
53

43
there was no valid cause of action existing in respect of the alleged debt
(Note – had P believed that the original claim was valid, P would had
sued on the original claim instead.) It was held that the surrender of a
groundless claim was neither a benefit to the promisee nor a detriment
to the promisor and P could not be said to have provided consideration.

Specific context: Pre-existing duties imposed by public law

59. The general rule is that a promise to perform/performance of an existing public duty is
invalid consideration in return for a promise. The rationale is to prevent opportunistic
exploitation – it is undesirable to allow civil servants to extract benefits for performing
their existing legal duties.

60. However, where more was promised than was strictly owed under the pre-existing legal
duty, this has been recognized as valid consideration (Glasbrook Bros v Glamorgan
CC).54

a. In Glasbrook, the mine owners were concerned of violence during a strike and
agreed to pay the police a sum of money for protection. The Court enforced the
payment as the police had provided additional protection than they had judged
necessary.

b. In Ward v Byham, the father of an illegitimate child promised to pay the child’s
mother £1 a week provided that the child was “well looked after and happy”.
The father refused to pay, arguing that the mother had provided no consideration
as she was performing her pre-existing legal duty (under the law, fathers of
illegitimate children had no corresponding duty to support at that time). The
EWCA held that the mother had promised to do more than was required under
her duty.55

54
Glasbrook Bros Ltd v Glamorgan CC [1952] AC 270 (UKHL) ("Glasbrook Bros Ltd v Glamorgan
CC")
55
Ward v Byham [1956] 1 WLR 496 (EWCA) ("Ward v Byham")

44
i. Note: Unlike the majority judges, Denning LJ in this case was of the
view that there was factual benefit accruing to the father and did not
consider that the mother had done above and beyond her duty.

61. Note: Mindy has argued that the bar is over-inclusive – the promisee receives not just
the practical benefit, but also the legal benefit (a right to enforce a duty), which the
promisor suffers a corresponding legal detriment (directly liable to the promisee apart
from liability for breach of the public duty). 56 Cf the authors in Phang for the view that
this was artificial.57

Specific context: Pre-existing duties owed to third parties

62. The promise to perform an existing contractual duty owed to a third party is good
consideration for a promise given in exchange. The reason is that the promisee receives
a legal benefit (a right to enforce a duty) and the promisor suffers a corresponding legal
detriment (liable to the promisee apart from liability for breach to the third party) (Pao
On v Lau Yiu Long).

a. In Shadwell v Shadwell, an uncle promised to pay his nephew a sum yearly until
his nephew’s income as a lawyer reached a certain amount. This was held to be
enforceable as the nephew provided consideration by marrying Ellen Nicholl,
which he was already contractually bound to her to do.58

b. In Pao On v Lau Yiu Long, the UKPC held that a promise by A to carry out a
contractual obligation to B to buy shares was valid consideration in return for
C’s promise.59

56
Mindy Chen-Wishart, Contract Law (OUP, 6th Ed, 2018) at [3.1.4.1].
57
The law of contract in Singapore (Academy Publishing, 2012) at [04.040].
58
Case summarized in Mindy Chen-Wishart, Contract Law (OUP, 6th Ed, 2018) at [3.1.4.2].
59
Pao On v Lau Yiu Long [1980] AC 614 (UKPC) ("Pao On v Lau Yiu Long")

45
Specific Context: Pre-existing duties owed to the other party

Outstanding obligations

63. Generally, where both parties have outstanding obligations, agreements to (1) end the
contract or (2) vary the terms of the contract which alters obligations on both sides,
there is sufficient consideration from the discharge/varying on obligations.

More for the same

64. As a starting point, a promise of “more for the same” is not valid consideration (Stilk v
Myrick).

a. In Stilk v Myrick, 2 of the 11 seamen deserted during a voyage and the master
agreed to share the deserters’ wages with the remainder of the crew if they
would continue with the voyage. It was held that Stilk was not entitled to
enforce the contract, as he was contractually obliged to sail the ship home, and
gave no consideration for the master’s promise. 60

65. However, if the promisee gives something more (more for more), than he was obliged
to under the original contract, there is valid consideration (Hartley v Ponsonby).61

a. In Hartley v Ponsonby, 17 out of 36 crew deserted, leaving behind only 4-5 able
seamen. It was held that in agreeing to continue with the voyage, the remaining
crew did more than they were obliged to do. (One interpretation is that the
contract was frustrated from the desertion – it was dangerous and unfair to hold
the existing seamen to their promise. On this view, it would clearly be above
and beyond what the seamen were expected to do).

66. Practical benefits – It has been held in Williams v Roffey Brothers that practical benefit
is valid consideration in return for a promise of more for the same, if at some stage
before A has completely performed his obligations under the contract, B has reason to
doubt whether A will or will be able to complete his side of the bargain.

60
Case summarized in Mindy Chen-Wishart, Contract Law (OUP, 6th Ed, 2018) at [3.1.5.3].
61
Case summarized in ibid.at [3.1.5.3].

46
a. In this case, RB contracted to refurbish 27 flats, subcontracting the carpentry
work to W for £20,000. W finished 9 flats, but may not be able to finish the rest
due to financial problems arising from (a) W underpricing the original job, and
(b) W had deficient supervision of his workers. RB was liable under a separate
contract if it failed to deliver the flats, and promised to pay W an additional sum
of £575 on the timely completion of each of the remaining 18 flats (total of
£10,300). W completed 8 more, but RB did not pay the sums promised. W
discontinued work and sued for £10,847. RN engaged other carpenters to
complete the job and incurred a week’s time penalty under the main contract.
RB argued that W gave no consideration for the additional sums promised.
EWCA awarded W only £3500.

b. The problem with Williams v Roffey Brothers is whether the subsequent


agreement is a bilateral contract – If this is so, the re-promise itself is sufficient
to confer a practical benefit. In this case, 4 practical benefits were identified –
W’s continued performance, avoiding the trouble and expense of obtaining a
substitute, avoiding the penalty for late performance under the main contract
and RB’s promise to pay more only as each flat was completed. The problem
was the award was not the full expectation for W, given that it was RB’s breach
that entitled W to terminate performance of the bilateral contract before
completion.

c. The agreement has been argued to be a unilateral contract. The practical


benefit is the actual performance and this analysis is consistent with a unilateral
contract analysis – W was only awarded for the 8 flats completed, less the mall
deduction for defective and incomplete items. 62

Less for the same

67. The general rule is that a part-payment of a debt in discharge of the whole debt is invalid
consideration (Foakes v Beer).

62
Ibid.at [3.1.5.3].

47
a. In Foakes v Beer, F owed B a sum of money and it was agreed that B would not
enforce the debt if F paid a part of it upfront and the balance in instalments. F
eventually repaid the full sum but B then claimed for interest accruing on the
debt. The UKHL held that the part-payment of a debt was insufficient
consideration.

68. The act of part payment, coupled with other facts however may amount to valid
consideration:

a. A part payment of the debt on an earlier time or different place as requested by


the creditor would suffice as good consideration (the former to recognise the
time value of money, the latter is a change in the debtor’s obligation).

b. Similarly, the addition of a “horse, hawk or robe” would also be sufficient


consideration as there is a change in the debtor’s obligations (Foakes v Beer).

69. There is some uncertainty as to whether the doctrine of practical benefits applies in the
context of part payment of debts.

a. In Re Selectmove, the EWCA expressly declined to expand the doctrine of


practical benefits to part-payments of debt. In this case, S owed the tax
authorities taxes. S sought to enter into an agreement with the authorities to
restructure the debt – in particular, that S would pay it’s future tax obligations
as they fell due (to do what it was already legally obliged to do), and to pay the
arrears in instalments. However, S only paid for the arrears, but failed to do so
with respect to the new tax obligations. It was held by Gibson LJ that “if the
principle in [Williams v Roffey] were extended to an obligation to make
payment, it would in effect leave the principle in [Foakes v Beer] without any
application. When a creditor and debtor who are at arm’s length reach
agreement on the payment of the debt by instalments to accommodate the debtor,
the creditor will no doubt always see a practical benefit to himself in so doing…
It is in my judgment impossible, consistently with the doctrine of precedent, for
this court to extend the principle of Williams’s case to any circumstances
governed by the principle of Foakes v Beer. If that extension is to be made, it

48
must be by the House of Lords or, perhaps even more appropriately, by
Parliament after consideration by the Law Commission”.63

b. In MWB v Rock, the doctrine of practical benefits however was extended to


part-payment of debts. In this case, MWB operated and managed office space
where Rock was a licensee. Rock incurred arrears of licence fees and other
charges, whereupon the parties agreed an oral variation to re-schedule the debt.
Rock paid £3,500 on the same day in accordance with the re-schedule. MWB’s
argument that the variation lacked consideration was rejected. The EWCA held
that Williams v Roffey’s practical benefit applied logically to promises to accept
less. The Court sought to distinguish Foakes v Beers and Re Selectmove on the
basis that:64

i. Per Kitchin LJ: “This is not a case in which the only benefits conferred
on MWB by the oral variation agreement were benefits of a kind
contemplated by [Foakes v Beer] and [Re Selectmove]. MWB derived a
practical benefit which went beyond the advantage of receiving a
prompt payment of a part of the arrears and a promise that it would be
paid the balance of the arrears and any deferred licence fees over the
course of the forthcoming months.” (at [48]). “But second and
importantly, Rock would remain a licensee and continue to occupy the
property with the result that it would not be left standing empty for some
time at further loss to MWB” (at [47]).

ii. Per Arden LJ: “[Re Selectmove] is distinguishable from the present case
and decides only that the benefit which a creditor obtains from a promise
to pay an existing debt by instalments is not good consideration in law”
(at [84]). “My conclusion that [Re Selectmove] can be distinguished in
this case is not inconsistent with [Foakes v Beer] where the only
suggested consideration was the debtor’s promise to pay part of his
existing debt… In accepting that a practical benefit can be good

63
Re Selectmove [1995] 1 WLR 474 (EWCA) ("Re Selectmove")
64
MWB v Rock Advertising [2016] 3 WLR 1519 (EWCA) ("MWB v Rock Advertising")

49
consideration for part payment of a debt, all I am doing is replacing the
words “the gift of a horse, hawk or robe” with a more modern equivalent
in line with the responsibility [in Williams v Roffey]” at ([85]).

iii. With regards to the unilateral contract analysis, while Arden LJ


supported this thesis (at [89]-[90]), McCombe LJ preferred not to base
his decision on this and Kitchin LJ preferred to express no view on it.
However, in Kitchin LJ’s decision, he preferred that the “variation
agreement thereupon became binding upon MWB and it would remain
binding for so long as Rock continued to make payments in accordance
with the revised payment schedule” (at [49]).

Reform of Consideration

70. The question on the reform of consideration is really an exercise of “what function does
the doctrine of consideration serve”.

a. If the doctrine of consideration is merely to serve as an evidentiary and


cautionary function (i.e. make sure that parties have a serious intention to be
bound”, then it follows that the doctrine of consideration should be abolished
and replaced with the tools which better reflect these interests (intention to
create legal relations, vitiating factors) – In this regard, VK Rajah JC (as he then
was) stated in Chwee Kin Keong v Digilandmall SGHC, “Indeed the time may
have come for the common law to shed the pretence of searching for
consideration to uphold commercial transactions. The marrow of contractual
relationships should be the parties’ intention to create legal relations”.

i. Following from this, it is possible to abolish the entire consideration


doctrine and only rely on intention to create legal relations and vitiating
factors.

ii. Alternatively, it can be abolished specifically in the context of contract


modifications, the argument being that consideration is only necessary
at the formation stage – once there is an on-going relationship it would
be fictional to consider any subsequent arrangement to be gratuitous.

50
b. Alternatively, Mindy has argued that the better way forward is to recognise that
the law treats exchanges differently from gifts - consideration should be retained
while recognising that there could be other good reasons for non-contractual
enforcement.

i. In this regard, the main difference is that the full panoply of contract
remedies, in particular damages measured by the value of the promised
performance may not be available to the party enforcing the promise.

ii. This would mean that the Courts can now more overtly find reasons as
to why certain promises which are not exchanges should be enforced,
but tailor the remedies to fit the interests to be protected.

51
Promissory Estoppel

Legal rule

71. If A promises to B that he will not enforce his strict legal rights against B, B relies on
that promise and it would be inequitable to allow A to resile from that promise, then A
is estopped from enforcing his strict legal rights (MWB v Rock).65

Clear Promise

72. For promissory estoppel to be established, there must be a clear and unequivocal
promise that the promissor does not intend to insist on his strict legal rights against the
promisee (Woodhouse v Nigerian Produce).66

a. In Woodhouse v Nigerian Produce, N (sellers) and W (buyers) had a contract


for the sale of cocoa. The buyers were concerned about their inability to pay the
contracts in Nigerian currency. N had replied to W’s request for payment in
Sterling as an alternative, stating “I write to confirm that payment can be made
in sterling and in Lagos… you are at liberty to make payment in sterling not
only with contracts already entered into but also with future contracts”. The
pound sterling was devalued and was worth less than the Nigerian pound. The
issue was whether the representation showed that (1) the payment could be
made as 1 pound sterling for 1 Nigerian pound (which the buyers contend), or
(2) the payment could be made in pound sterling, but in the equivalent of the
contract price in Nigerian pound. It was held that there was no clear
representation of (1) and the understanding was (2) instead.

b. In Al Shams Global Ltd v BNP Paribas, BNP had refused to accept payment
into ASGL’s accounts. Under the contract, BNP was entitled to refuse to accept
deposit if any information or document which it requests for KYC purposes was
not provided or in BNP’s opinion was insufficient or unsatisfactory. BNP had
gave ASGL notice of its decision to close the account – ASGL entered into
negotiations with the Bank indicating that he would prefer if gradual steps were

65
Ibid.at [61].
66
Woodhouse v Nigerian Produce [1972] AC 741 (UKHL) ("Woodhouse v Nigerian Produce")

52
taken to close the account as they were expecting further payments into the
account. BNP asked ASGL to provide them with a list of payments and the
documents it required for KYC purposes. BNP informed ASGL that the bank
would have to return the funds if “appropriate documents” were not received by
25 April 2016. ASGL provided the necessary documents and BNP eventually
took the view that it was unable to process the payment into the account. ASGL
argued inter alia that BNP had represented to ASGL that they would process
the payments once documents were received (without further regard or
consideration). It was held that there was no such representation – the
representation only stated when BNP would return the funds and not when BNP
would process the payments – the receipt of documents was necessary but not
a sufficient or only condition for BNP to process the payment. 67

Reliance

73. Reliance is established if it can be shown that the promisee had a change of position on
the faith of the promise (done something or omitted to do something which he would
otherwise not have done or omitted to do) (The Post Chaser).

74. However, it is unclear whether it is necessary to show that the promisee has to suffer
some detriment as a result. The prevailing view is that there is no requirement for
detrimental reliance (The Post Chaser) in the sense that there must be “detriment” at
the time when the promise was given, but there must be “detriment” at the time when
it was withdrawn (see the oft-cited decision of Lam Chi Kin David v Deutsche Bank
AG).68

a. In The Post Chaser, it was observed by Goff J in the EWHC (as he then was)
that “To establish such inequity, it is not necessary to show detriment; indeed,
the representee may have benefited from the representation, and yet it may be

67
Al Shams Global v BNP [2019] 3 SLR 1189 (SGHC) ("Al Shams Global v BNP")
68
See the oft-cited decision of Lam Chi Kin David v Deutsche Bank AG [2010] 2 SLR(R) 896
(SGHC) ("Lam Chi Kin David v Deutsche Bank AG")

53
inequitable, at least without reasonable notice, for the representor to enforce his
legal rights”.69

b. In Central London Property v High Trees, L represented to T that T could pay


lesser rent, It could be inequitable for L to insist on his strict legal right to the
unpaid rent, because T had conducted his affairs on the basis that he would only
have to pay rent at the lower rate. It was held that the representation bound L
during the period up till the early part of 1945, but from that point on the full
rent is payable.70 There was no obvious detriment to the tenants in High Trees
because they were asked to pay the rent they had contracted to pay.

c. In Abdul Jalil bin Ahmad v A Formation, two properties were subject of a trust
and was leased to D. D did not start paying rental on the date of commencement
of the lease, due to delays in the grant of the temporary occupation permits
which D maintains that it was due to the fault of the trustees. D and the sole
trustee (the other having passed away) entered into a compromise agreement
whereby D would pay the unpaid rental to the trustee by certain dates in
exchange for the waiver of payment of other amounts. The trustee was later
discharged and P was appointed by the Court as replacement. P sought to
recover from D the waived rental arrears and the interest thereon. The issue was
inter alia whether P was estopped from prosecuting the claim as D had relied
on the representations made by the counsel for the trustees in negotiations and
paid the agreed amounts. The Court observe that detriment was not a necessary
requirement, that D having paid all moneys required by the sole trustee, it would
be inequitable to allow P as present trustees to go back on the promise. In any
case, the Court found that the requirement of detriment was met when the
defendant paid part of the rental arrears and also the incurred expenses in
redeveloping the properties. 71

69
The Post Chaser [1982] 1 All ER 19 (EWHC) ("The Post Chaser")
70
Central London Property Trust v High Trees [1947] 1 KB 130 (EWHC) ("Central London Property
Trust v High Trees")
71
Abdul Jalil bin Ahmad bin Talib v A Formation Construction Pte Ltd [2006] 4 SLR(R) 778 (SGHC)
("Abdul Jalil bin Ahmad bin Talib v A Formation Construction Pte Ltd")

54
d. In Lam Chi Kin David SGHC, Chong JC (as he then was) explained that the
term “detriment” may be understood to mean (a) the incurrence of time and
expense, (b) incurring a liability, (c) change of position and (d) the deprivation
of a benefit. The former two categories refer to detriment in a narrow sense
(actual incurrence of time, money or liability), whereas the latter two constitute
“detriment” in the broader sense which would only arise if the promisor were
permitted to resile from its promise.

Inequitable to go back on the promise

75. The Courts will only enforce the promise only if it would be inequitable for the
promisor to resile on his promise. In considering this element, the Court will look at the
following factors:

a. Time lag – The shorter the time lag, the easier it is for the promisee to be
restored to its original position and therefore less prejudice would be suffered
by the promisee. This is illustrated by the case in The Post Chaser. In this case,
the sellers agreed to sell palm oil to the buyers, which contracted to sell the palm
oil to sub-buyers. Under the terms of the contract, the declaration of ship must
be made by the sellers to the buyers in writing as soon as possible after the
vessel’s sailing. The sellers did not give such a declaration until more than 1
month after the ship had sailed. On receipt of the declaration, the buyers made
no protest about the declaration or any statement that they were accepting the
delivery under reserve of their contractual rights. At the request of the buyers,
the sellers handed over documents covering the consignment to the sub-buyers.
The sub-buyers rejected the documents 2 days later and the buyers rejected the
sale the same day. The sellers were forced to sell the oil elsewhere for less than
the contract price and claimed the difference from the buyers by way of
damages. On the facts, it was held that while the sellers had relied on the buyer’s
representation that they would accept the documents, had relied on the
representation by presenting said documents to the sub-buyers, however the
very short time between date of representation and reliance, the position of the

55
sellers was not prejudiced by reason of their reliance. As such, it was not
inequitable for the buyers to enforce their legal right to reject the documents.72

b. Bad behavior on part of promisee – Where the promisee had behaved badly
in procuring the promise, the Court will permit the promisor from resiling from
his promise. In D&C Builders v Rees, R owed D&C a sum of money for
building works, but knew that D&C were in bad financial straits. R offered £300
in full settlement or “nothing”. D&C accepted but later sued for the balance.
The majority of the EWCA refused to recognise the settlement for lack of
consideration. Denning MR however rejected the plea of promissory estoppel
as it was not inequitable for D&C to go back on its promise as the wife had
obtained the promise by intimidation. 73

c. Change in circumstances subsequent to making of the promise – In Williams


v Stern, a creditor promised a debtor extra time to pay before he exercised his
right to seize the debtor’s furniture. The creditor however reneged when he
heard that the debtor’s landlord was about to seize the same assets for unpaid
rent. The Court held that the creditor was entitled to seize, primarily on the
ground that there was no consideration for the promise. Brett LJ however stated
that “it appears that a distress by the plaintiff’s landlord had been threatened;
and under these circumstances, I do not blame the defendant for changing his
mind”.74

d. Failure on part of the promisee to perform its reduced obligations – In Re


Selectmove, the company had itself failed to perform its own lesser promise to
pay taxes. Gibson LJ held that because of this, it was not inequitable for the
promisor to resile on the promise.75

76. There was some suggestion by Adren LJ in Collier v Wright that (1) the part payment
of the debt would automatically render the resiling by the promisor inequitable and (2)

72
The Post Chaser [1982] 1 All ER 19 (EWHC) ("The Post Chaser")
73
D&C Builders v Rees [1966] 2 QB 617 (EWCA) ("D&C Builders v Rees")
74
Williams v Stern [1879] 5 QBD 409 (EWCA) ("Williams v Stern")
75
Re Selectmove [1995] 1 WLR 474 (EWCA) ("Re Selectmove")

56
has the effect of extinguishing the promisor’s rights. 76 However, this was corrected in
the subsequent decision of MWB v Rock, where the EWCA held (Arden LJ including)
there was no automatic inequity from the part payment and that the creditor’s original
rights could be revived on giving reasonable notice. 77

Effect of promissory estoppel: Suspensory or Extinctive

77. The effect of an estoppel is generally suspensory. The promisor may, on giving due
notice, assert her original rights. However, the estoppel can extinguish part or all of the
promisor’s existing rights if the promisee is unable to resume her position (Ajayi v RT
Briscoe).78 The effect depends on what is necessary to ensure that the promisee is not
prejudiced by her reliance on the promise.

a. Total extinction – In Hughes v Metropolitan Railway Co, the landlord’s


implied promise was to suspend his right to timely repairs during negotiations
with the tenant, for the landlord to buyback the lease. The rationale was that it
the property may be reapplied by the landlord for other purposes, which repairs
may be redundant. This right was extinguished because it was impossible for
the tenant to wind the clock back. 79

b. Partial extinction – In High Trees, the lessors were allowed to revert to the full
rent on giving reasonable notice, but the right to the full payments up to the
point of notice was extinguished. 80

c. No extinction – In MWB v Rock, it was held that if there was no binding contract,
the promise will not be given effect as the promisee could resume its original
position (i.e. not inequitable for the promisor to resile).81

76
Collier v Wright [2008] 1 W:R 643 (EWCA) ("Collier v Wright")
77
MWB v Rock Advertising [2016] 3 WLR 1519 (EWCA) ("MWB v Rock Advertising")
78
Ajayi v Briscoe [1964] 1 WLR 1326 (UKPC) ("Ajayi v Briscoe")
79
Hughes v Metropolitan Railway Company [1877] 2 AC 439 (UKHL) ("Hughes v Metropolitan
Railway Company")
80
Central London Property Trust v High Trees [1947] 1 KB 130 (EWHC) ("Central London Property
Trust v High Trees")
81
MWB v Rock Advertising [2016] 3 WLR 1519 (EWCA) ("MWB v Rock Advertising")

57
Shield or sword

78. The starting position is that promissory estoppel can only operate defensively (same for
less type of promises), but cannot create a new cause of action or rights (Combe v
Combe).

a. In Combe v Combe the Denning LJ (as he then was) held that the doctrine does
not create “new causes of action where none existed before”. The rationale was
to prevent an undermining of the doctrine of consideration, as noted by
Denning LJ in the same case, “the doctrine of consideration is too firmly fixed
to be overthrown by a side-wind”.

b. The position in Singapore is the same. In Long Foo Yit v Mobil Oil Singapore,
the SGHC held that for obtaining relief under this doctrine, there must be a
“legal relationship giving rise to certain rights and duties between the parties”
and the promise must be to “not enforce against the other his strict legal rights
arising out of that relationship”. 82

i. In this case, P sued D seeking specific performance of an alleged


contract to renew a dealer licence agreement, under which P claimed to
be entitled to continue operating a Mobil petrol service station for the
next 15 years. P alleged that Mobil had promised to renew the dealer
agreement. It was held on the facts that there was no such promise. Even
if there was such an agreement, P was not entitled to rely on the doctrine
of promissory estoppel to invoke a new legal right which would give
them a cause of action against Mobile (at [58]).

79. However, it may be argued that the doctrine should be reformed:

a. In the context of proprietary estoppel, this doctrine is allowed to be used as a


sword which has created new legal rights (see Crabb v Arun D.C.).

b. The doctrine of promissory estoppel and consideration rests on different bases.


Consideration yields a contractual cause of action for the enforcement of the

82

58
promisee’s full expectation, whereas promissory estoppel seeks to avoid the
detriment arising from the promisee’s reliance on the promise if it would be
inequitable for the promisor to renege.83

c. There is Australian authority which has developed the doctrine to be used as a


sword (Walton’s Stores v Maher).

83
Mindy Chen-Wishart, Contract Law (OUP, 6th Ed, 2018) at [3.2.2.3].

59
Intention to create legal relations

80. There is a requirement that the parties must have intended to create legal relations.
There crux is whether the parties intended that in any disagreement or dispute in relation
to the agreement, parties could invoke the assistance of the court (Gay Choon Ing v
Terence Loh).84

81. In this regard, it is presumed that in social and domestic agreements, parties do not
intend to create legal relations. In the context of commercial agreements, there is a
presumption that parties do intend to create legal relations.

82. The justification is largely based on public policy: (1) the judicial system should not be
swamped with social and domestic disputes, (2) the presumptions are consistent with
contract law’s primary function to facilitate transactions between people who may
otherwise not deal with each other. State coercion should be used to enforce agreements
for which informal social sanctions are absent, and (3) freedom from contract – the
state’s intrusion into the private lives of its citizens should be limited. 85

a. Note that the third reason is not that all convincing – the presumption here (of
intention to create legal relations) is an evidential one, rather than a “legal rule”.
As with most evidential presumptions, these are really commonsense inferences.

Social agreements

83. It is presumed that in social and domestic agreements, parties do not intend to create
legal relations (Balfour v Balfour). The cases below illustrate how the rules are applied.

a. Husband and Wife – In Balfour v Balfour, a husband and wife returned to


England on leave from the husband’s employment in Ceylon (now Sri Lanka).
The wife remained in England on the doctor’s advice and the husband agreed
to pay her an allowance of £30 a month until she rejoined him. When the parties
divorced, the wife sued her now ex-husband for the payments. The EWCA

84
Gay Choon Ing v Loh Sze Ti Terence Peter [2009] 2 SLR(R) 332 (SGCA) ("Gay Choon Ing v Loh
Sze Ti Terence Peter") at [71].
85
Mindy Chen-Wishart, Contract Law (OUP, 6th Ed, 2018) at [2.7.1].

60
denied her claim for lack of consideration and lack of intention to create legal
relations because the agreement was made while they were living “in amity”. 86

i. Consider how the situation would change if the parties were separated
or about to separate. The presumption would be that the parties, in their
agreements, intend to create legal relations.

b. Parents and Children – In Jones v Padavatton, a mother brought a house for


her daughter to live in and to maintain herself from the proceeds of letting the
other rooms. The daughter agreed to give up her secretarial work in Washington
to read for the Bar in London. 6 years later, the daughter had still not passed the
course. The parties fell out and the daughter resisted the mother’s claim for
possession of the house, claiming that the was contractually entitled to remain.
The EWCA held that there was no intention to create legal relations vis-à-vis
the living arrangements since the parties were on good terms when the
arrangement was made and that the terms of the arrangements were too vague,
in particular as to its duration. 87

c. Friends – In De Cruz Andrea Heidi v Guangzhou Yuzhitand Health Products


Co Ltd, ADC and the 5th defendant (“5D”, who is an artist - probably Rayson
Tan) were close friends. 5D “sold” slimming pills to ADC which eventually
caused her liver failure. ADC sought to inter alia sue 5D for breach of contract.

86
In particular, Atkin LJ’s judgment is elegantly written, “The common law does not regulate the form
of agreements between spouses. Their promises are not sealed with seals and ssealing wax. The
consideration that really obtains for them is that natural love and affection which counts for so little in
these cold Courts. The terms may be repudiated, varied or renewed as performance proceeds or as
disagreements develop, and the principles of the common law as to exoneration and discharge and
accord and satisfaction are such as find no place in the domestic code. The parties themselves are
advocates, judges, Courts, sherriff’s officer and reporter. In respect of these promises each house is a
domain into which the King’s writ does not seek to run, and to which his officers do not seek to be
admitted.” Balfour v Balfour [1919] 2 KB 571 (EWCA) ("Balfour v Balfour")
87
Note however the three judges actually came to slightly different conclusions – Danckwerts LJ
considered that there was no intention to create legal relations at all. Salmon LJ however took the view
that there was an initial contract which was valid – the agreement to provide maintenance, with valid
consideration being the daughter’s moving over to England, and there was an intention to create legal
relations. The fact that the contract was uncertain as to the duration was not a bar – he was willing to
infer a reasonable time, but held that the time had expired (5 years) and thereby arrived at his
conclusion that the contract had been brought to an end. Fenton Atkinson L.J. took the view that there
was no consideration and no intention to create legal relations, as well as the terms being too vague and
uncertain.

61
ADC and 5D were friends and were on very good terms. Their SMSes had
undeniable ring tones of friend greeting and teasing friend (ADC had called 5D
Mr Extremly Naughty for e.g.). In the words of the judge, “If the words in the
15 February 2002 message were converted into a picture message, “Mr
Extremly Naughty” would probably be looking at a cute, cheeky smiling face.”
The Court held that there was no intention to create a legal relationship between
the parties.88

84. It has been suggested that the courts refer to policy considerations in deciding whether
the presumption is rebutted. Mindy suggests that where one party has detrimentally
relied upon the agreement, the Courts are more willing to find that there is an intention
to be bound, citing Parker v Clark.

a. In this case, the Clarks and the Parkers agreed that if the Parkers sold their
cottage and came to live with the Clarks, sharing their household expenses, Mr
Clark would leave them a portion of his estate. When the couples fell out, the
Parkers left on the Clarks’ demand and claimed damages for the breach of
contract. It was held that there was an intention to create legal relation – Mr
Clark could not really have supposed that “the law would leave him at liberty if
he so chose, to tell [the Ps] when they arrived that he had changed his mind, that
they could take their furniture away, and that he was indifferent whether they
found anywhere else to live or not”. 89

b. The essence is that the more serious (or the more is at stake) the matter, the
more likely it is that the parties intent the Court’s assistance.

Commercial agreements

85. In the context of commercial agreements, there is a presumption that parties do intend
to create legal relations (Rose and Frank Co v J.R. Crompton & Bros Ltd).

88
De Cruz Andrea Heidi v Guangzhou Yuzhitand Health Products Co Ltd [2003] 4 SLR(R) 682
(SGHC) ("De Cruz Andrea Heidi v Guangzhou Yuzhitand Health Products Co Ltd")
89
Parker v Clarke [1960] 1 WLR 286 (EWHC) ("Parker v Clarke")

62
a. In Edwards v Skyways, an airline company promised to pay a sum to aircrew
who were made redundant, but stated that the payment was “ex gratia”. The
Court held that the meaning of “ex gratia” was merely used to indicate that the
party agreeing to pay does not admit any pre-existing liability on his part, but
not to preclude legal enforceability of the settlement itself. Although there was
some suggest that ex gratia was used to avoid tax liability in the hands of the
recipient, the Court did not find affirmatively that there was no intention to enter
into legal relations. On the balance the Court did not find that the presumption
was rebutted.

b. In contrast, in Rose and Frank Co v Crompton, the presumption was rebutted.


In this case, R was appointed as C’s sole agents to finish in 1920, but C
terminated without notice in 1919. The court found that the presumption of
enforceability was rebutted by the clear “honour clause” in the agreement which
stated that “This arrangement is not entered into… as a formal or legal
agreement, and shall not be subject to legal jurisdiction in the Law Courts… but
it is only a definite expression and record of the purpose and intention of the
three parties concerned, to which they each honourably pledged themselves,
with the fullest confidence – based on past business with each other – that it will
be carried through by each of the three parties with mutual loyalty and friendly
cooperation”.90

90
Rose and Frank Co v Crompton and Bros Ltd [1925] AC 445 (UKHL) ("Rose and Frank Co v
Crompton and Bros Ltd")

63
Chapter 2: Terms

Introduction

86. “The substance of what the parties to a contract have agreed is determined by the terms
of the contract. They may be express: that parties have explicitly agreed to them. They
may be implied: that on the facts, there are terms that the parties must have intended to
include, but for some reason omitted to. Terms may also be implied by operation of law
or by statute, or by custom or usage.”91

Express Terms

Ascertaining the terms

Representations vs Terms

87. As a general proposition, where there is a contractual document, the terms are usually
identified by reference to the document itself. However, there are two situations where
this assumption does not apply:

a. Where the document is an incomplete expression of the bargain between the


parties, such that it may be partly oral and partly written; and

b. Where there is no document and the contract is purely oral.

88. In this case, distinguishing between mere representations and contractual terms is
important. The remedies available for disappointed expectations are available if the
statement is a term (with limited methods of ending the contract), where as a
representation generally only allows for recission and reliance damages/restitution.

89. The test for whether a statement is a term is whether there is evidence of an intention
(objectively ascertained) on the part of either or both of the parties that there should
be contractual liability in respect of the accuracy of the statement (Heilbut, Symons v

91
The law of contract in Singapore (Academy Publishing, 2012) at [06.001].

64
Buckleton).92 The Courts have taken into consideration some relevant factors which
may be indicative of the parties’ objective intention.

a. If the statement is related to a matter of significance vis-à-vis the subject


matter of the contract, it is more likely to be a term of the contract –
(Bannerman v White).

i. In this case, the claimant agreed by contract to purchase some hops to


be used for making beer. He asked the seller if the hops had been treated
with sulphur and told him if they had he wouldn't buy them as he would
not be able to use them for making beer if they had. The seller assured
him that the hops had not been treated with sulphur. In fact they had
been treated with Sulphur. It was held that the statement was a term as
the claimant had communicated the importance of the statement and
further relied on it. The action for breach of contract was successful.93

b. Where the party making the statement possesses special knowledge with
regard to the content of the statement, or is in a better position to ascertain
the truth of the statement, the statement is more likely to be a term of the
contract – (Oscar Chess v Williams, Dick Bentley v Harold Smith).

i. Not a term – Oscar Chess v Williams – In this case, P bought a car from
D (car trade in – in return D could buy a car from P/finance company at
a discount). D told one of P’s salesmen that he wished to buy a new car.
The salesman was a neighbor of D and had been given a lift in the car
on a number of occasions. The salesman thought the car was a 1948
Morris and D described the car as such to him as well, producing the
registration book for it. The salesman checked the registration book,
which showed that 1948 was the date of first registration. The salesman
consulted the Glass’s Guide which gave him the current prices for
second-hand cars according to the year of their manufacture, and offered
the defendant an allowance of £290 for the purchase of a new car. D

92
Heilbut, Symons & Co v Buckleton [1913] AC 30 (UKHL) ("Heilbut, Symons & Co v Buckleton")
93
Bannerman v White [1861] 142 ER 685 ("Bannerman v White")

65
agreed and the transactions were completed. P later discovered that the
car was a 1939 model – if they had known so, they would had offered
less £115. D honestly believed that it was a 1948 model (innocent
misrepresentation). P sought to claim damages of £115. It was held that
the statement was not a term, as per Denning LJ (as he then was), “The
seller had himself no personal knowledge of the year when the car was
made… He must have been relying on the registration book… If the
seller was asked to pledge himself to it, he would at once have said “I
cannot do that. I have only the log-book to go by, the same as you”. 94

ii. A term – Dick Bentley v Harold Smith – In this case, P told D that he
was on the lookout for a ‘well vetted Bentley car’. D found one and
bought it for 1500 pounds. He informed P of his acquisition. P then went
to see the car. D told him the car had been fitted with a replacement
engine and gearbox and that it had done only 20000 miles since the work
had been carried out. The speedometer on the car showed only 20,000
miles. The car however proved to be a “considerable disappointment”
to P, who sued D for breach of contract. In this case it was held that the
statement was a term of the contract, per Denning MR, “Here we have
a dealer… who was in a position to know, or at least to find out, the
history of the car. He could get it by writing to the makers. He did not
do so… When the history of this car was examined, his statement turned
out to be quite wrong… Smith stated a fact that should be within his
own knowledge. He had jumped to a conclusion and stated it as a fact.
A fact that a buyer would act on.”

c. Where the maker of the statement asks the other party to verify the truth of
his statement, it is unlikely that the statement will amount to a term – (Ecay v
Godfrey, Schawel v Reade).

i. Not a term – Ecay v Godfrey – In this case, P had orally agreed to buy a
cruiser from D. Prior to entry into the contract, D stated that the cruiser

94
Oscar Chess v Williams [1957] 1 WLR 370 (EWCA) ("Oscar Chess v Williams")

66
was in good condition, but asked P if P wanted to take a survey of the
boat, which P said no. It was held that the statement was not a term, as
D’s query showed that D was unwilling to take responsibility for the
soundness of the boat.95

ii. A term – Schawel v Reade – In this case, P bought a horse from D for
stud purposes. While P was examining the horse, D told P that the stud
was sound and that if there was anything wrong with the horse D would
inform P and told P there was no need to get a vet to check him out. It
turned out later that the horse had a hereditary eye disease. It was held
that the statement was a term as D had taken responsibility by stating
expressly that there was no need to verify its truth. 96

d. Where the statement was merely passed on and not initiated by the maker, it
is unlikely to be a term and lapse of time – (Routledge v McKay).

i. In this case, P bought a second-hand motor car from D. In this case, each
seller in the chain was passing on information about the year the car was
made by relying on a false entry in the registration book. D in this case
was not the person who altered the registration book (not the originator)
but a mere innocent passer-on. It was held that the statement was not a
term.97 The Court also considered that there was time for the buyer to
check the statement (lapse of time from when statement was made to
entering into the contract), which weighed in favour of finding that it
was a mere representation.

Written contracts and the parol evidence rule

90. Under S93 and S94 of the Evidence Act, where the terms of the contract have been
reduced into a document, no evidence shall be given to prove the terms of the contract

95
Ecay v Godfrey [1947] 80 Lloyd's LR 286 (EWHC) ("Ecay v Godfrey")
96
Case summarized in Mindy Chen-Wishart, Contract Law (OUP, 6th Ed, 2018) at [5.1.1.2].
97
Routledge v Mckay [1954] 1 WLR 615 (EWCA) ("Routledge v Mckay")

67
except the document itself, and no extrinsic (oral) evidence is admissible to contradict,
vary, add to or subtract from its terms. However,

a. the parol evidence rule does not apply if the document in the first place was not
intended to represent the entire agreement between them, objectively
ascertained. The Court will take cognizance of the extrinsic evidence or the
surrounding circumstances of the contract, as well as the attributes of the
document in question (e.g. standard form contracts are examples par excellence
of contracts that look complete to the parties) (Zurich Insurance v B-Gold
Interior).98

b. The parole evidence rule is subject to the exceptions in the provisos to S94 of
the Evidence Act.

Collateral contracts (oral)

91. Under S94(b) of the Evidence Act, the existence of a separate oral agreement on a
matter which a document is silent and which is not inconsistent with its terms, may be
proved.

92. Unlike the common law exception, proviso (b) only allows the admission of evidence
of a oral collateral contract on matters which are not inconsistent with the written
agreement.

a. City and Westminister Properties v Mudd – T was induced to sign a new lease
(limiting the use of the premises to business purposes) by the landlord’s oral
assurance that the tenant could continue to live on the premises. The landlord
was prevented from forfeiting the lease against the tenant for doing so.99

b. Mendelssohn v Normand Ltd – M parked in N’s garage on terms that N would


“accept no liability for any loss or damages in particular, sustained by the
vehicle its accessories or contents howsoever caused”. M left the car unlocked

98
Zurich Insurance v B-Gold [2008] 3 SLR(R) 1029 (SGCA) ("Zurich Insurance v B-Gold") at
[110],[112].
99
Case summarized in Mindy Chen-Wishart, Contract Law (OUP, 6th Ed, 2018) at [10.3.2]

68
on the insistence of N’s employee who undertook to lock it for him. It was held
that M successfully sued for valuables stolen from the car. 100

c. However, the Courts are cautious in finding a collateral contract – in Ang Sin
Hock v Khoo Eng Lim, the Court emphasized that the finding of a collateral
contract is on the particular facts of the case, and must constitute all the requisite
legal criteria for contracts (it’s a legal settlement case).101

d. In particular, it was emphasized in Wen Wen Food Trading v Food Republic, it


was held that an appropriately worded entire-agreement clause may prevent any
pre-contractual or collateral agreement of legal effect. In this case, W argued
that it relied on a misrepresentation of a 6 year licence in entering into the
contract. The contract however specified a 2 year licence instead and contained
an entire agreement clause. It was held that the representations/extrinsic
evidence of the 6 year licence was excluded. 102

e. Note also that S17(1) of the CPFTA disapplies the parol evidence rule.

Incorporation of Terms

Incorporation by Signature

93. As a general rule, a person is bound by the contents of a contractual document that the
person signs, whether or not that person has read or understood it (L’Estrange v
Graucob).103 The red hand rule does not apply where there is a signed contract (Press
Automation Technology v Trans-link Exhibition Forwarding).

a. L’Estrange v Graucob – In this case, E bought an automatic cigarette vending


machine from G which stopped working after a few days. G denied E’s claim
for breach of an implied term as to fitness for purpose because E signed an order
form which excluded liability for all implied and express terms and statements.

100
Mendelssohn v Normand Ltd [1976] 1 WLR 1078 (EWCA) ("Mendelssohn v Normand Ltd")
101
Ang Sin Hock v Khoo Eng Lim [2010] 3 SLR 179 (SGCA) ("Ang Sin Hock v Khoo Eng Lim")
102
Wen Wen Food Trading v Food Republic [2019] SGHC 60 (SGHC) ("Wen Wen Food Trading v
Food Republic") at [22].
103
L'Estrange v Graucob [1934] 2 KB 394 (EWCA) ("L'Estrange v Graucob")

69
The exemption clause was in “regrettably small print”, printed on brown paper
and in an unexpected place. It was held that E was bound by her signature.

b. Consmat Singapore v Bank of America National Trust – In this case, C had a


contract with B for bank services. B honoured and paid 15 cheques drawn on
C’s account. The signatures on the 15 cheques were forged. C sued B to recover
the amount and B argued that an onerous clause in the contract precluded C
from raising the argument that the signatures were forged. It was held that in
this case, the parties had signed it and was deemed to have accepted all the
terms.104

c. Press Automation Technology Pte Ltd v Trans-link Exhibition Forwarding Pte


Ltd – In this case, T contracted with P to transport an exhibition machine. T
gave P quotations for their services, stating that the business was transacted with
the SFFA conditions (incorporation). Clause 27 of the SFFA conditions
restricted the carrier’s liability to $100,000 while clause 30 stated that there was
a time bar of 9 months for claims. T did not forward the SFFA conditions to P,
or drew attention to the clauses. It was held that the clause was incorporated by
signature. The Court refused to follow Tilden the reason being (1) parties must
care for their own legal positions, (2) the reasonableness is now protected
under UCTA.105

94. However, the signature rule does not apply if the document is not contractual in nature
(Grogan v Robin Meredith Plant Hire).

a. In this case, an oral contract was made for the hire of machinery. Timesheets
were signed after contract commenced. These time sheets contained additional
alleged terms of contract. It was held that the Timesheets were signed after
contract formed (oral) and that the timesheet was merely administrative, only
allowing parties to implement their prior agreement. 106

104
Consmat Singapore v Bank of America National Trust [1992] 2 SLR(R) 195 (SGHC) ("Consmat
Singapore v Bank of America National Trust")
105
Press Automation Technology v Trans-link Exibition Forwarding [2003] 1 SLR(R) 712 (SGHC)
("Press Automation Technology v Trans-link Exibition Forwarding")
106
Grogan v Robin Meredith Plant [1996] CLC 1127 (EWCA) ("Grogan v Robin Meredith Plant")

70
95. The signature rule has been criticised for being unfair as contracting practice reveals
that people often do not scrutinize all the clauses of a contract before signing them. To
do so is impossible practically. One possible solution to result in a fair outcome might
be the approach taken in Tilden Rent-a-Car V Clendenning, where in this case the
Ontario Court of Appeal held that a signatory is bound only when it is reasonable for
the enforcing party to believe that the signature manifests assent to the onerous term
sought to be enforced. In a hurried, informal or consumer transaction with such finely
printed, unexpected and harsh clauses, the signature does not truly represent an
agreement to the unusual and onerous terms. The enforcing party is required to take
reasonable steps to draw such terms to the attention of the other party. 107

96. The signature rule is justifiable on the basis of certainty, efficiency, administrative
convenience and the parties’ autonomy. The obstacles of giving informed consent to
every term in a standard form contract of any complexity is simply insurmountable. A
person who refused to contract without being adequately apprised of the fine print
would deny themselves most means of living in a modern society – given that the parties
clearly consent to the main subject matter of the contract, the signature is a proxy for
consent in order to make the contract enforceable. Any unfairness is mitigated by other
legal rules in determining the contents and the meaning of the contract, or invalidating
unfair terms.108

Incorporation by notice in unsigned documents

97. A term may be incorporated into a contract by notice if it was given at or before contract
formation, in a document intended to have contractual effect and must have reasonable
notice (Parker v South Eastern Railway Co).109

a. Parker was a case concerning a jury trial, where the EWCA held that there was
a misdirection by the judge of the jury. In this case, P deposited his bag at a
clock room at a railway station at a charge of 2d. P received a ticket on the face
which states “see back” and there was a notice to limit the cloakroom’s liability.

107
Mindy Chen-Wishart, Contract Law (OUP, 6th Ed, 2018) at [10.3.3.1].
108
Ibid.at [10.3.3.1]; Press Automation Technology v Trans-link Exibition Forwarding [2003] 1
SLR(R) 712 (SGHC) ("Press Automation Technology v Trans-link Exibition Forwarding") at [40].
109

71
A placard containing the same terms was hung up in the cloak room. P’s bag
exceeded the limit. The Court ordered a retrial on the basis of misdirection of
jury, holding that the direction to be: “if he knew that there was writing on the
ticket, but did not know or believe that the writing contained conditions,
nevertheless he would be bound, if the delivering of the ticket to him in such a
manner that he could see there was writing upon it, was, in the opinion of the
jury, reasonable notice that the writing contained conditions.” 110

b. Insufficient notice – Thornton v Shoe Lane Parking – In this case, the EWCA
held that there was insufficient notice on the facts of this case (action for
personal injury sustained in the defendants’ parking area). The ticket referred at
the bottom left-hand corner (and in small print) to the exception clause, which
was placed in a pillar within the carpark. The case stands for two propositions:
First, the notice must be given prior or at the time of contract (which it did not)
and second, the degree of notice must commensurate with the harshness or
unexpectedness of the terms i.e. the red-hand rule).111

c. Insufficient notice – Interfoto Picture Library v Stiletto Visual Programmes –


P ran a photography transparency lending library. D wanted to borrow 47
transparencies, which P delivered to D along with a delivery note containing 9
printed conditions. Condition 2 stipulated that all transparencies had to be
returned within 14 days of delivery, otherwise a holding fee of £5 a day and
VAT will be charged for each transparency. D had not used P’s services before
and did not read the conditions, returning the transparencies 4 weeks later. P
invoiced D for a sum of £3,783.50. D refused to pay. It was held that the clause
was onerous and unusual, and that P had done nothing to draw D’s attention to
condition 2, as such the condition was not incorporated into the contract.

110
Parker v South Eastern Railway Co [1877] 2 CPD 416 (EWCA) ("Parker v South Eastern Railway
Co")
111
Thornton v Shoe Lane Parking Ltd [1971] 2 QB 163 (EWCA) ("Thornton v Shoe Lane Parking
Ltd")

72
However, P was entitled to recover a reasonable sum under the law of unjust
enrichment.112

d. Insufficient notice – Hakko Products v Danzas (Singapore) – in this case, P


had brought a very high-precision and sensitive machine from Germany. D was
a freight forwarding company which submitted a quotation to P for airfreighting
the machine and transporting it to P’s factory. The quotation included at the
bottom a clause incorporating the SAAA conditions (which contained an
exemption and limitation of liability clause). The clause was in fine print and
was illegible and were not brought to P’s attention. The transportation was
negligent and one of the unit was left in the rain and was corroded. P sued for
the additional cost of replacement. It was held that D who was seeking to
enforce the clauses should have brought them to the notice of but failed to do
so, applying Interfoto Picture.113

Incorporation by previous course of dealing or trade usage

98. A term may be incorporated by a consistent course of previous dealing between the
parties (Henry Kendal v William Lillico, cf Hollier v Rambler Motors). (Note: the line
between the previous dealing and trade custom are not clearly demarcated. In R1, the
Court used the trade practice to bolster the incorporation in the course of dealing).

a. Incorporated – Henry Kendal v William Lillico – In this case, the parties had
made a contract orally over the phone. After the oral contract, a document was
dispatched containing the challenged exemption clause which had been
consistently used over 100 transactions between the parties over three years. It
was held that the clause was incorporated for a consistent course of previous
dealing between the parties. 114

b. Incorporated – R1 International v Lonstroff – In this case, L bought rubber


from R in 5 separate transactions over the course of a year. Each transaction

112
Interfoto Picture Library v Stiletto Visual [1989] 1 QB 433 (EWCA) ("Interfoto Picture Library v
Stiletto Visual")
113
Hakko Products v Danzas [1999] 1 SLR(R) 651 (SGHC) ("Hakko Products v Danzas")
114
Case summarized in Mindy Chen-Wishart, Contract Law (OUP, 6th Ed, 2018) at [10.3.3.3].

73
comprised of the parties concluding the headline terms, followed by R sending
an email confirmation of those headline terms and L sending across a purchase
order. R would later send across the contract note, with a request that L
countersigns and returns a copy. Each contract note would state that the IRAC
terms would apply, which provided that disputes would be resolved by
arbitration. The notes were never countersigned or returned and L never
protested. The issue was whether the arbitration clause was validly incorporated.
It was held that the terms were incorporated, with the courts taking into account
of the industry (this was a common practice in the industry) and the course of
dealings between the parties. The buyer did not protest in the subsequent
transactions.115

c. Not incorporated – Hollier v Rambler Motors – In this case, H had signed


forms exempting R (a garage) from liability three or four times over five years
but had, on this occasion, contracted over the phone without mentioning the
exemption. It was held that the term was not incorporated, and R was liable for
negligent damage to H’s car by fire. 116

d. Not incorporated – MGA International v Wajilam – In this case, W and M had


entered into several financing transactions. W argued that in this transaction
(Marina I), it had a discretionary commission term incorporated by a prior
course of dealing. The Court held at [98] that only 1 transaction had such a
discretionary transaction and notwithstanding the several transactions, there
was insufficient evidence of “continuous notice of such a term”. There was
insufficient consistency in the prior dealing and as such there was no
discretionary commission term in favour of W. 117

99. A term may be incorporated by the custom of the relevant trade (British Crane Hire
Corp v Ipswich Plant) (Note: incorporation under this would not require several
transactions).

115
R1 International v Lonstroff [2015] 1 SLR 521 (SGCA) ("R1 International v Lonstroff")
116
Hollier v Rambler Motors [1972] 2 QB 71 (EWCA) ("Hollier v Rambler Motors")
117
MGA International v Wajilam Exports [2010] SGHC 319 (SGHC) ("MGA International v Wajilam
Exports")

74
a. Incorporated – British Crane Hire Corp v Ipswich Plant – In this case, B hired
a crane urgently from Ipswitch. Ipswitch subsequently sent out a printed form.
Although the parties had only contracted twice previously on these forms, the
document was incorporated because: (1) the parties were of equal bargaining
power; (2) both were in the crane hire business; (3) the terms relied upon were
habitually used in the trade, including by B itself when hiring out. The crane
sunk into soft ground and the issue was who was responsible for the recovery
of the claim and expenses (the conditions stated that the renting party was to be
liable).118

Interpretation of Terms

Is the evidence admissible to aid interpretation?

General rules and parol evidence of subjective intent

100. The admissibility of evidence is subject to the Evidence Act.

a. Extrinsic evidence (except for the drafter’s subjective intention) is admissible


under S94(f) of the Evidence Act to ascertain if the contractual text is (1) plainly
clear, (2) patently ambiguous or (3) latently ambiguous. Evidence of the
surrounding circumstances include facts and circumstances which were (or
ought to have been) in the mind of the drafter when he used those words. This
does not include parol evidence of the drafter’s subjective intention (Sembcorp
Marine SGCA at [64]).

b. If the extrinsic evidence reveals that the text is plainly clear, no parol evidence
of the drafter’s subjective intention is admissible under S96 of the Evidence Act
(Sembcorp Marine SGCA at [53]-[65]).

i. S96 Evidence Act – Illustration: A conveys to B by deed “my estate at


Kranji containing 100 hectares”. A has an estate at Kranji containing

118
British Crane Hire v Ipswich Plant [1975] 1 QB 303 (EWCA) ("British Crane Hire v Ipswich
Plant")

75
100 hectares. Evidence may not be given of the fact that the estate meant
was one situated at a different place and of a different size.

c. If the extrinsic evidence reveals that the text is patently ambiguous, no parol
evidence of the drafter’s subjective intention is admissible under S95 of the
Evidence Act (Sembcorp Marine SGCA at [53]-[65]).

i. S95 Evidence Act – Illustration (a): A agrees in writing to sell a horse


to B for $500 or $600. Evidence cannot be given to show which price
was to be given.

ii. S95 Evidence Act – Illustration (b): A deed contains blanks. Evidence
cannot be given of facts which would show how they were meant to be
filled.

d. If the extrinsic evidence reveals that the text is latently ambiguous, parol
evidence of the drafter’s subjective intention is admissible under S97-100 of
the Evidence Act (Sembcorp Marine SGCA at [53]-[65]).

i. S97 Evidence Act – Illustration: A conveys to B by deed “my plantation


in Penang”. A had no plantation in Penang, but it appears that he had a
plantation in Province Wellesley, of which B had been in possession
since the execution of the deed. These facts may be proved to show that
the deed related to the plantation in Province Wellesley.

ii. S98 Evidence Act – Illustration (a): A agrees to sell to B for $500 “my
white horse”. A has 2 white horses. Evidence may be given of facts
which show which of them was meant.

iii. S98 Evidence Act – Illustration (b): A agrees to accompany B to Halifax.


Evidence may be given of facts showing whether Halifax in Yorkshire
or Halifax in Nova Scotia was meant.

iv. S99 Evidence Act – Illustration: A agrees to sell to B “my land at X in


the occupation of Y”. A has land at X, but not in the occupation of Y,
and he has land in the occupation of Y, but it is not at X. Evidence may
be given of facts showing which he meant to sell.

76
v. S100 Evidence Act – Illustration: A, a sculptor, agrees to sell to B “all
my mods”. A has both models and modelling tools. Evidence may be
given to show which he meant to sell.

Evidence of prior negotiations and subsequent conduct

101. There is some uncertainty as to whether evidence of pre-contractual


negotiations or conduct subsequent to formation is admissible in Singapore. The
English law position is that such evidence is inadmissible (Chartbrook Ltd v Persimmon
Homes).119

a. In Chartbrook v Persimmon, Lord Hoffmann considered that practical and


policy considerations militated admission of evidence of pre-contractual
negotiations and subsequent conduct under English law:

i. Admission of pre-contractual negotiations would lead to uncertainty in


outcome in disputes over interpretation and add to the legal costs.
Everyone will have to read the correspondence and statements, which is
a time consuming and expensive exercise and the scope of disagreement
would increase.

ii. The document should so far as possible speak for itself.

iii. Parties may (similar to ministers and others in relation to statutory


interpretation) make statements in their correspondence in hope of
influencing the construction courts will give to the agreement.

119
As noted in Sembcorp Marine v PPL Holdings [2013] 4 SLR 193 (SGCA) ("Sembcorp Marine v
PPL Holdings")

77
b. In Zurich, the VK Rajah JA suggested that prior negotiations and even
subsequent conduct may be admissible for the purpose of interpretation in
principle.120

c. In Sembcorp, the SGCA left the issue open, but suggested the policy tensions
between costs and efficiency (as raised in Chartbrook) against the concerns of
fairness of outcome.121

d. In Xia Zhengyan, the SGCA left the issue open as well: 122

i. At [65], the Court considered that generally reliance on draft agreements


without more cannot amount to a clear and obvious context in so far as
the Court is left in the dark with regard to the actual bargaining process
undertaken by the contracting parties. The addition, removal, or
variation of any contractual term is a result of the bargains and
exchanges, beyond the actual words found in the draft agreements.

ii. However, there may be instances where such prior negotiations may
satisfy the context requirement. The SGCA cited Inglis v John Buttery
as an example – This case concerned a contract for works on a ship for
a fixed sum of £17,250. In the draft contract, the following provision
stated that “if any new plating is required the same to be paid for extra”.
In the course of negotiations, the agent for the shipowners wrote to the
shipbuilder stating that “we must ask you to erase all stipulations after
the word “repaired”” (which was basically the quoted provision), and
that the sum covered the new plating. The SGCA recognized that while
Inglis was excluded, it would most likely be admissible locally given
the flexible approach adopted by the SGCA in Zurich Insurance (at [67]).

iii. In Xia Zhengyan itself, the parties did not argue whether the drafts were
inadmissible. The Courts considered that in this case, the evidence

120
Zurich Insurance v B-Gold [2008] 3 SLR(R) 1029 (SGCA) ("Zurich Insurance v B-Gold")
121
Sembcorp Marine v PPL Holdings [2013] 4 SLR 193 (SGCA) ("Sembcorp Marine v PPL
Holdings")
122
Xia Zhengyan v Geng Changqing [2015] 3 SLR 732 (SGCA) ("Xia Zhengyan v Geng Changqing")

78
performed a confirmatory role rather than a pivotal role. The Court
suggested that such evidence could be admitted if the situation (such as
that in Inglis was extremely clear) and the evidence was merely a
confirmatory one (at [69]).

e. In Ding Pei Zhen v Yap Son On, the SGHC considered that the Zurich test
restrictions would exclude evidence of subsequent conduct as the requirement
of reasonable availability to all contracting parties and clear and obvious context
appears to refer to the extrinsic evidence being so available at the time of
contracting, but noted that given the SGCA did not close the door on evidence
of such nature, it is arguable that the requirements were not intended to be
limited temporally (at [95]).123

f. Goh Yihan has argued in “Towards a consistent use of subsequent conduct in


Singapore contract law” that:124

i. The Evidence Act allows admission of subsequent conduct. Common


law restrictions are incompatible with the Evidence Act.

g. Lord Nicholls, writing extra judicially argues against the exclusions in My


Kingdom for a Horse: The Meaning of Words:125

i. The exclusionary rules leads to injustice – it allows one party to contend


for a meaning he knows was not intended.

ii. Best evidence – it would be perverse to be barred from having regard to


what may be the best evidence of all. A reasonable person with
knowledge of the background circumstances minus the negotiations will
have an incomplete picture of the background to the contract.

123
Ding Pei Zhen v Yap Son On [2015] 5 SLR 911 (SGHC) ("Ding Pei Zhen v Yap Son On")
124
Goh Yihan, "Towards a consistent use of subsequent conduct in Singapore contract law" (2017) 5
JBL 387
125
As summarized in Mindy Chen-Wishart, Contract Law (OUP, 6th Ed, 2018) at [10.4.4].

79
iii. Many exceptions – evidence of prior negotiations admissible to resolve
questions about formation of contract through course of dealings,
rectification and claims for misrepresentation, plea of estoppel etc.

iv. Transparency – evidence of parties’ actual intentions often does become


known by and influences judges on disputes about interpretation.

v. Costs – the solution lies in case management and control over evidence
sought to be adduced (relevancy).

vi. Comparative law – exclusions are inconsistent with most other legal
systems and international restatements of contract law such as Vienna
Convention on Sale of Goods; UNIDROIT Principles of International
and Commercial Contracts; Principles of European Contract Law.

Added requirements

h. Before the evidence is admissible, it must be relevant, reasonably available to


the contracting parties at the time of contract and must relate to a clear or
obvious context (Zurich Insurance). 126

i. Relevancy: The evidence is relevant if it would affect the way in which the
language of the document would have been understood by a reasonable man.

j. Reasonable availability: The extrinsic evidence must have been reasonably


available to the contracting parties at the time of the contract.

k. Clear and obvious context: The extrinsic evidence must relate to a clear or
obvious context before the court can say with any certainty that such evidence
is of assistance to the Courts.

102. Beyond this, the 4 requirements under Civil Procedure must be met (in the
Singapore Supreme Court Directions) (see Sembcorp Marine): 127

126
Zurich Insurance v B-Gold [2008] 3 SLR(R) 1029 (SGCA) ("Zurich Insurance v B-Gold")
127
Sembcorp Marine v PPL Holdings [2013] 4 SLR 193 (SGCA) ("Sembcorp Marine v PPL
Holdings")

80
a. first, parties who contend that the factual matrix is relevant to the construction
of the contract must plead with specificity each fact of the factual matrix that
they wish to rely on in support of their construction of the contract;

b. second, the factual circumstances in which the facts in (a) were known to both
or all the relevant parties must also be pleaded with sufficient particularity;

c. third, parties should in their pleadings specify the effect which such facts will
have on their contended construction; and

d. fourth, the obligation of parties to disclose evidence would be limited by the


extent to which the evidence are relevant to the facts pleaded in (a) and (b).

Principles of interpretation

103. The Courts will consider the following principles (Zurich Insurance
summarized in Master Marine):128

a. Objective approach

i. First, the aim of the exercise of construction is to ascertain the meaning


the document would convey to a reasonable business person;

ii. Secondly, the courts are concerned with the objective expressed
intention of the parties and not their actual intentions;

b. Contextual approach

i. Thirdly, the courts will not excessively focus on particular phrases or


words. The emphasis is on the document as a whole;

ii. Fourthly, the courts are prepared to look into the legal, regulatory and
factual matrix constituting the background in which the document was
drafted to inform them on how to interpret the document.

128
Master Marine AS v Labroy Offshore [2012] 3 SLR 125 (SGCA) ("Master Marine AS v Labroy
Offshore")

81
iii. Fifthly, the courts will give regard to the overall commercial purpose of
the parties in entering into the transaction.

c. Bias to lawfulness

i. Sixthly, preference will be given to an interpretation that makes the


contract and its performance lawful and effective.

d. Bias towards fairness

i. Seventhly, where the contract appears to be one-sided or onerous, it will


be construed strictly against the party seeking to rely on it.

ii. Eighthly, an interpretation that leads to very unreasonable results will


be avoided unless it is required by clear words and there is no other
tenable construction.

e. Specific overrides the general

i. Ninthly, a specially agreed provision should override an inconsistent


standard provision which has not been individually negotiated.

ii. Tenthly, a more precise provision should override an inconsistent


general provision.

Illustrative cases

Y.E.S F&B Group v Soup Restaurant

104. This case concerned an interpretation of a sublease agreement between Y and


S. Y and S were sister companies when the sublease agreement was entered into in
2009, but this relation came to an end in 2012. Y and S operated adjacent restaurants at
Vivocity. Y entered into a 3 year lease with the landlord. S entered into a lease with the
landlord for an adjacent unit and did not require the entire premise. S sublet a part of it
for Y’s use. The sublease provided that the sublease shall survive as long as “the
Company’s lease with the landlord is not terminated” (S being “the Company”). S
argued that “the Company’s lease” referred to the 3 year lease, whereas Y argued that
it referred to a generic reference to any lease S had with the landlord.

82
a. The SGCA held that the lease was a generic reference. It considered that the
agreement was drafted by laypersons in the business context without any legal
assistance. The Court in such circumstances eschew a strict construction of the
relevant language and adopt a more common-sense approach which considered
the reasonable and probable expectations that the parties would have had.

b. Y and S were commercially-related sister companies at the time of the


agreement – it was reasonable to infer that the parties would have expected their
commercial partnership to continue for the foreseeable future, which militated
against S’s construction.

c. Y and S were aware of the purpose for which the sub-leased premises were used
by Y. The investment by Y in the premises was disproportionate to a 3 year
lease.

Ngee Ann Development v Takashimaya Singapore

105. In Ngee Ann Development SGCA, the case concerned a lease in which the tenant
was to lease the premises for an initial term of 20 years. After the first 5 years, a rent
review was to be conducted to determine the rent payable for each of the successive
five-year periods up to the end of the initial 20-year-term. The parties were to endeavor
to agree on the “prevailing market rental value of the [premises]”, which would
represent the new rent for each rent review period, but if they failed to agree, the
“prevailing market rental value of the [premises]” would be determined by a licensed
valuer.

a. The disputes were resolved by the appointed valuer who adopted the existing
configuration as the basis for determining the prevailing market rental value.
Takashimaya sought to exercise its option to renew the lease for an additional
10 years. Under the lease, in a procedure similar to which governed the rent
reviews, the parties were to agree on the “prevailing market rental value of the
[premises]” failing which the value was to be determined by a licensed valuer.
The landlord argued that the valuer was permitted to posit a different and
hypothetical configuration – one which would reflect the “highest and best use”
of the premises.

83
b. The Court considered from the text Takashimaya had a wide discretion to decide
on the configuration of the premises, that the relationship was akin to a joint
venture, and application of a hypothetical configuration would be inconsistent
with the nature of the parties’ agreement.

c. The Court relied on the landlord’s failure to object to the prior and consistent
usage of the existing configuration as it was cogent evidence (at [103]). (Note
that in Centre for Laser v GPK Clinic SGCA, the Court acknowledged that this
was permissible but eventually left the issue open again as to whether such
evidence was admissible (??) at [51], [53], [54]).

Yap Son On v Ding Pei Zhen

106. In this case, the parties collaborated to assist a Chinese company in its efforts
to list on the Frankfurt Stock Exchange, agreeing to expend efforts and to bear the
expenses associated with the listing in exchange for share capital that would be issued.
5.5m shares were registered in the names of various companies owned by Yap. Parties
agreed to a share allotment agreement, with Ding being entitled to 10.35% block, out
of the “19%” of the shareholding. Yap argued that 19% referred to 19% of the shares
allotted to Yap, which was 2,144,541 shares. Ding argued that 19% was an
indosyncratic way of expressing the totality of the shares held by the Yap companies,
which was 2,996,053.

a. It was held that the text was clear – a percentage was a fraction out of 100 and
to agree with Ding’s interpretation, the lower court Judge had crossed the line
from interpretation to a variation of the contract.

b. Ding had given oral evidence of her subjective intention. This was inadmissible
on the facts given that there was no ambiguity – subjective intention evidence
was only admissible in cases of latent ambiguity.

c. It was held that the pleadings were not sufficiently particularized. There was no
explanation of how this figure of 5.5 million shares came to be expressed in the
Allotment Agreement by the use of the term “Total 19%”. (failed to meet the
civil procedure requirements).

84
d. Most of the evidence did not satisfy the 3 Zurich criteria. For example, the 2007
agreement was not shown to be within the knowledge of Yap.

CISG v Ong Puay Koon

107. CIFG was an investment vehicle established in 2007, to enter into a set of
convertible bond subscription agreements (CBSA) with the defendants. The 5
defendants were Polimet and Polimet’s 4 initial shareholders. The CBSA contained a
general indemnity clause in favour of the plaintiff. The plaintiff also obtained charges
over the assets of all the companies in the Polimet group, personal guarantees from Lee
and Ho (limited to their initial 50% shareholding in Polimet). Polimet defaulted. The
issue was whether the shareholders were personally liable under the general indemnity,
jointly and severally.

a. The Court considered the document and how the contract was drafted, as well
as the entirety of the commercial documents.

b. The Court also considered the pre-contract negotiations, but only because it was
not controversial or disputed (at [23]).

c. The CBSAs had made specific provision to allocate risks to the defendants.

d. The negotiations showed that Ho and Lee limited their liability to the extent of
their shareholding, which militates against the interpretation that the general
indemnity clause would impose liability on all parties jointly and severally.

e. The clause was introduced as a boiler-plate, but its scope and effect were not
discussed.

f. The Court considered that the general indemnity provision was a gap-filling
provision, only to cover matters if they were not already covered elsewhere – it
could not override the commercial structure of the deal and the calibrated
allocation of risk that is reflected elsewhere in the suite of agreements entered
into.

85
Implied terms

Terms implied in fact

108. For a term to be implied in fact, there must be (Sembcorp Marine v PPL
Holdings) at [101]:129

a. There must be a true gap (i.e. the parties had not contemplated the issue);

b. The term must be necessary to give business efficacy to the contract (business
efficacy test);

i. The Moorcock130 – the plaintiff’s vessel was damaged when docking at


the defendant’s jetty. The contract required the plaintiff to bring his
vessel alongside the wharf but said nothing about any warranty that the
riverbed at that location would permit this. It was argued that a term
should be implied imposing on the defendant wharfingers a duty to
exercise reasonable care to ascertain that the riverbed adjacent to the
jetty would not damage the vessel. The court implied the term on the
basis that it was necessary for business efficacy to do so.

c. The term must be so obvious that it goes without saying (officious bystander
test – officious bystander proposes, the parties suppresses).

i. Shirlaw v Southern Foundries131 – Mr Shirlaw had been the managing


director of Southern Foundries Ltd, which was in the business of iron
castings. But then another company called ‘Federated Foundries Ltd’
took over the business. The new owners had altered article 8 of Southern
Foundries Ltd's constitution, empowering two directors and the
secretary (who were friends of Federated Foundries) to remove any
director. Then they acted on it, by sacking Mr Shirlaw. Mr Shirlaw's
contract, signed in 1933 stated that he was to remain in post for ten years.

129
Sembcorp Marine v PPL Holdings [2013] 4 SLR 193 (SGCA) ("Sembcorp Marine v PPL
Holdings")
130
The Moorcock [1889] 14 PD 64 (EWCA) ("The Moorcock")
131
Shirlaw v Southern Foundries [1939] 2 KB 206 (EWCA) ("Shirlaw v Southern Foundries")

86
Mr Shirlaw sued the company for breach of contract, claiming for an
injunction to stay in office or substantial damages. It was held that there
was an implied term in the 1933 agreement that the company would not
remove Mr Shirlaw from his directorship for the time in which he was
appointed as managing director – “Would it not be well to put in a
provision that the company shall not exercise or create any right to
remove Mr Shirlaw from his directorship, and he have no right to resign
his directorship”

Illustrative case

109. Sembcorp Marine – In this case, Sembcorp and PPL were joint venture partners
who each owned 50% of the joint venture company, PPL Shipyard. Under the terms of
the joint venture agreement and PPL’s articles of association, Sembcorp and PPL were
entitled to appoint three directors, as long as they held 50% of the shares in PPL
Shipyard. Sembcorp increased its stake to 85% subsequently by buying 35% from PPl.
Sometime later, PPL sold it’s remaining shares to a third party. Sembcorp’s nominated
directors took a number of steps to reduce PPL Holdings’ board influence and executive
control in PPL Shipyard. PPL had argued that the resolutions passed by the Sembcorp-
nominated directors were invalid as they did not satisfy certain quorum requirements,
whereas Sembcorp argued that there was an implied term which disapplied certain
clauses relating to quorum and control once the 50-50 joint venture proportion changed.

a. It was held that there was a true gap as there was no evidence to suggest that
the parties had addressed their mind to the issue (at [118]).

b. The nature of the transaction was commercial, and there is no need to depart
from the general presumption that the parties contracted on the basis of business
efficacy. It was necessary as a matter of business efficacy to imply the term, as
all the clauses related to powers and mode of operations nominated by the
shareholders when they retained equal shareholding. There was a need to imply
a term to deal with the provisions on quorum and control when a party has no
right to appoint a director (at [119]).

c. If the officious bystander asked Sembcorp and PPL Holdings whether the
clauses on quorum and control would cease to apply when one party obtained

87
more than 50% of the share capital, the parties would have responded “Oh, of
course!” (at [126]).

Terms implied in law

110. A term may be implied by law – once a term has been implied, such a term will
be implied in all future contracts of that particular type. 132 There is authority which
suggests that a term is implied in law only if it is necessary to do so, and not merely if
the term is reasonable (Liverpool CC v Irwin). Singapore however seems to allow the
implication of a term, if there are “general reasons of justice and fairness as well as of
public policy” to justify doing so (Jet Holding v Cooper).

a. Liverpool CC v Irwin – Three 15-storey tower blocks were built in Everton,


Liverpool in 1966. Each had 70 units, a stairwell, two lifts, and a rubbish chute.
Mr and Mrs Irwin were tenants from July 1966. The common parts were
vandalised, the lifts did not work, the stair lights failed, the chute was blocked,
lavatory cisterns blocked and overflowed. The blocks became nicknamed "The
Piggeries" (in EWCA). The tenants, conducting a rent strike, refused to pay rent.
In an action by the council to eject them, they counterclaimed that the council
was in breach of a duty to keep the common parts of the estates in decent repair.
It was held that the landlords had an implied in law obligation to their tenants
to take reasonable care to maintain the common property of the tower blocks,
which was fulfilled as it spent more on repairs than it received in rents.

b. Jet Holding v Cooper – JHL owned an oil rig. A slip joint on the rig had
fractured. JSL was the former owner. JDL was the manager. MEP was the
assignee of the rights from JHL, JSL and JDL. JDL had contracted Cameron on
JSL’s behalf to refurbish and repair the joints. Cameron subcontracted the works
to VDH who re-assembled a single operational joint from the two slip joints.
Cameron subcontracted the refurbishment of the remaining unused parts to
Stork, who created a new slip joint (standby slip joint). One of Stork’s tasks as
part of the refurbishment contract was to dismantle and inspect the remaining
unused parts (“the pre-inspection”). During the trial, Stork’s quality control

132
Jet Holding v Cooper [2006] 3 SLR(R) 769 (SGCA) ("Jet Holding v Cooper")

88
inspector (“Prabhuram”) gave evidence that although he knew that Stork was
required to conduct a dimensional inspection, he did not do so since he did not
think it a standard or common practice to conduct wall thickness tests on the
Riser Box. Counsel for Stork placed great reliance on the fact that Cameron
failed to furnish it with the relevant dimensional drawings of the Riser Box,
which allegedly would have made it impossible for Stork to detect any
dimensional defects in the Riser Box wall even if the dimensional inspection
had been conducted. The SGCA held that there was a breach of an implied
term that Cameron was required to take reasonable care (in law) and to deliver
the dimensional drawings of the Riser Box to Stork. Note: Phang has never
said what the policy was or what was the category of contracts which such a
term would be implied in.

111. Note: such terms can be contracted out

Terms implied in statute

112. The most well-known instance of implication by statute is to be found in


sections 11 to 15 of the English Sale of Goods Act 1979. It is now the Sale of Goods
Act (Cap 393, 1994 Rev Ed) in SG.

a. S11 – when condition to be treated as a warranty;

b. S12 – implied terms about title etc;

c. S13 – sale by description;

d. S14 – implied terms about quality or fitness;

e. S15 – Sale by sample.

113. Note: unlike terms implied by law, certain terms cannot be excluded even if
parties agree – See S6 Unfair Contract Terms Act.

89
Invalidation of exception clauses

Exception (limiting or excluding contractual liability) vs defining contracting parties’

legal obligations?

114. A clause which seeks to limit or exclude or act as a defence to contractual


liability will be governed by the law relating to exception clauses. A clause which
merely defines the contracting parties’ legal obligations would fall outside (The law of
contract in Singapore at [7.004]). It has been held that clauses which limits or totally
excludes liability are exception clauses, where as clauses such as liquidated damages
merely defines the obligations of the parties (genuine pre-estimate of damages) (Emjay
Enterprises Pte Ltd v Skylift Consolidator).

a. In Emjay, the plaintiff had obtained interlocutory judgement against the


defendant for breach of contract with damages to be assessed. However, at the
stage when damages were being assessed the defendant sought to introduce the
exception clause concerned. The Court held that the exception clause (total or
partial) both relate to the liability of the parties, being the primary obligations
of the parties (in their performance) and their secondary obligations (obligations
to pay damages). As such, the exception clause could not be introduced at the
stage of assessment of damages. (at [24]-[30]).

115. However, the Court’s attitude is that limitation of liability clauses are
interpreted less stringently than total exclusion of liability clauses (Emjay Enterprises
v Skylift Consolidator).

Fundamental breach as a rule of construction

116. A fundamental breach of the contract does not necessarily and automatically
destroy the efficacy of an exception clause. Whilst the primary obligations come to an
end, the secondary obligation (to pay damages) remains and an exception clause might
cover this last-mentioned liability. The Courts will adopt a rule of construction in
ascertaining what the parties intended for the clause to effect (Emjay Enterprises v
Skylift Consolidator).

a. the wider the clause in protecting the breaching party, the less likely that parties
could have intended the clause to be interpreted as applying to the breach.

90
b. However, very clear wording can exclude liability for fundamental breaches or
deliberate repudiations of the contract.

Contra proferentem rule

117. The contra proferentem rule requires any ambiguity in a contractual term to be
construed against the party who introduced it. The contra proferentem rule applies only
where there is an ambiguity in the contract which cannot be resolved by interpreting
the term in the context of the overall contract (LTT Global Consultant SGHC at [56]).

118. The rule operates in standard term contracts and is not applicable to the case of
contracts that have been individually negotiated (LTT Global Consultant SGHC at [58]).

a. In LTT Global Consultant, the Court held that while the phrase “bi-monthly”
was ambiguous, this was not a case of a standard form contract but a negotiated
contract. The Court declined to apply the contra proferentem rule. The
agreement in question was between LTT and BMC to offer a professional
course leading to an LLB – in the case, Dr Siva argued that BMC had breached
the contract inter alia to give full disclosure of the fees collected on a “bi-
monthly” basis. The ambiguity was whether the phrase meant twice a month or
once every two months – it was held that it meant twice a month.

119. The contra proferentem rule in the context of exemption of negligence liability
is laid down in Canada Steamships v The King, which have been accepted in Marina
Center Holdings v Pars Carpet Gallery (at [7]-[8]):

a. If the clause contains language which expressly exempts the person in whose
favour it is made (hereafter called ‘the proferens’) from the consequence of the
negligence of his own servants, effect must be given to that provision.

b. If there is no express reference to negligence, the court must consider whether


the words used are wide enough, in their ordinary meaning, to cover negligence
on the part of the servants of the proferens. If a doubt arises at this point, it must
be resolved against the proferens.

c. If the words used are wide enough for the above purpose, the court must then
consider whether ‘the head of damage may be based on some ground other than

91
that of negligence’. The existence of a possible head of damage other than that
of negligence is fatal to the proferens even if the words used are prima facie
wide enough to cover negligence on the part of his servants.

120. In particular, in Ailsa Craig v Malvern Fishing, it was held that if the clause
which is broad enough to cover negligence and only limits rather than excludes liability,
the term is effective.133

a. In Marina Center, P leased premises from M. Water seeped through the ceiling
and damaged P’s goods. The issue was whether the insurers of P (subrogation)
could sue M for breach of covenants for quiet enjoyment and repair in the lease,
and negligence at common law. M argued inter alia that cl 36.1(b) excluded
liability, which stipulated that the appellant and its officers, servants, employees
or agents should not be liable or in any way responsible for any injury or damage
to persons or property or any consequential loss resulting from an entire list of
events “unless caused by the wilful misconduct of [the appellant] or [its] officers,
servants, employees or agents”. It was held that (1) although the clause did not
expressly exempt negligence, it was wide enough to cover negligence under the
phrase “willful conduct”; (2) the clause did not cover other heads of damage
apart from negligence – it did not cover the breach of covenants as the clause
specified certain events which could only be caused by negligence (short circuit,
leaks etc). As for nuisance and Rylands v Fletcher, this was not covered by the
clause as the presence of the pipes/electricity were for the benefit of P, P had
already consented to the source of danger. The clause was thus read only to be
limited to negligence liability and was effective.

b. Note: at the DC level, the Court held that liability for all 3 heads (quiet
enjoyment, repair, negligence) was made out, but held that the clause exempted
all 3 heads of liability. P appealed to the High Court challenging only the part
of the decision that the clause exempted liability for negligence, which the High

133
As summarised in Mindy Chen-Wishart, Contract Law (OUP, 6th Ed, 2018)

92
Court agreed (because it covered the other 2 heads). The SGCA disagreed and
held that the clause ONLY exempted liability for negligence.

Unfair Contract Terms Act

What kind of liability and contracts does UCTA apply to?

121. Negligence for death and personal injury – Cannot be excluded by any
contract term or notice at all (S2(1) UCTA).

122. Negligence for loss or damage – cannot exclude or restrict unless term or
notice is reasonable (S2(2) UCTA). Awareness or agreement of the notice itself is not
an indication of voluntary acceptance of any risk (S2(3) UCTA).

123. Contract liability

a. Only applies between contracting parties where one of them deals as consumer
or on the other’s written standard terms of business (S3(1) UCTA).

b. Against that party, the other cannot

i. Exclude/restrict liability his liability in respect of the breach; or


(S3(2)(a) UCTA);

ii. Claim to be entitled

1. to render a contractual performance substantially different from


that which was reasonably expected of him; or (S3(2)(b)(i)
UCTA)

2. in respect the whole or any part of his contractual obligation, to


render no performance at all (S3(2)(b)(ii) UCTA);

Unless the term is reasonable.

What terms does UCTA apply to?

124. UCTA addresses clauses which “exclude or restrict” a liability, obligation or


duty. The Court is concerned about the substantive effect of the term or notice, and not
the form or identification (i.e. basis clauses which are duty-defining (non-reliance/non-

93
representation) are viewed substantively the same as exclusion of liability clauses (shall
not be liable for…)) (Deutsche Bank v Chang Tse Wen, S13 UCTA).

a. In this case, the Court suggested that clauses which defined the scope or nature
of the relationship between the parties would be subjected to the UCTA where
applicable, as UCTA is concerned about the substantive effect of the term,
rather than the form or identification (at [63],[68]).

b. Smith v Eric S Bush – In this case, P applied to D for a loan on the security of
a house they had selected to buy on the council’s standard mortgage application
form and paid the valuation and administration fee. The form provided that the
valuation was confidential and intended solely for the information of the council.
No responsibility was implied or accepted by the council for the value or
condition of the property by reason of such inspection and report. The applicants
were advised to instruct their own surveyor/architect to inexpect the property.
The valuation conducted by an employee of the council was inaccurate, and P
sought damages from the valuer’s negligence as servant/agent of the council.
The UKHL held that the clause which prevented the duty from arising was
caught by the English equivalent of S13(1) of the UCTA.

c. S13(1)(a) UCTA prevents also

i. making liability or enforcement restrictive or onerous;

ii. excluding or restricting any right or remedy in respect of the liability/


subjecting a person to prejudice in consequence of his pursuing of such
right or remedy;

iii. excluding or restricting rules of evidence or procedure;

iv. or terms and notices which exclude or restrict the relevant obligation or
duty.

Reasonableness test

125. Time of assessment – S11(1) UCTA states that the time of assessing
reasonableness is during the time when the contract was made, taking into account the
circumstances known to be or in the contemplation of the parties at that time.

94
126. Burden of proof – S11(5) UCTA – the burden of proving reasonableness lies
on the party seeking to rely on the term.

127. Practically a single test of reasonableness – The test is whether the term is a
fair and reasonable one, having regard to the circumstances which were, or ought
reasonably to have been, known to or in the contemplation of the parties when the
contract was made (S11(1) UCTA). Although there are 3 tests which appears in UCTA,
the Courts will consider all the factors in the guidelines where they appear relevant, and
they are non-exhaustive. These factors include:

a. Resources of parties;

b. Availability of insurance;

c. Relative bargaining positions of parties;

d. Inducement to agree to term;

e. Choice of which party to contract with

f. Awareness of existence of term etc.

128. Illustrative cases:

a. Smith v Eric S Bush – In this case, the Court held that the term excluding
negligence liability of the valuer was not reasonable. The reasons are that: the
valuer is paid for his services, many purchasers cannot afford a second
valuation, most purchasers in fact rely on his valuation and do not commission
their own survey, mortgagees are trustworthy and they appoint careful and
competent valuers – which purchasers trust, the valuer knows that his failure
may be disastrous to the purchaser.

b. Consmat Singapore v Bank of American National Trust – In this case, P was


a customer of BANT. Consmat signed a general agreement for Commercial
Business and opened an account with BANT, containing a term which states
that the Customer is obliged to verify the correctness of the statement of account
accompanying cheques or vouchers received from the banks. If within 7 days
they do not object to any incorrect entries, it shall be conclusive evidence that

95
the account is correct, and that the bank shall be free from all claims in respect
of that account. Consmat later discovered a total of 15 cheques were forged,
which BANT had honoured. Consmat sued BANT and BANT relied on the
exclusion clause. It was held that the term was valid – the agreement was for
commercial business. Consmat could negotiate for a modification if necessary.
Forgeries are extremely difficult for a bank to detect (as they could only verify
signatures against specimen signatures). The customers have a better ability to
identify the signatures and detect forgeries. Consmat had the resources to verify
the statements and cheques as well.

c. Tjoa Elis v UOB – In this case, P (an individual) was a customer of UOB and
was subject to a similar clause to that in Consmat. The Court held that the
provision was fair and reasonable and that the customer should be required to
check his statement and notify the bank promptly (adopting Consmat).

d. Jiang Ou v EFG Bank AG – In this case, P opened an account with EFG. EFG’s
employee had executed a series of 160 high-volume and/or high risk
transactions fraudulently, in the absence of any instructions from P. The Court
observed that clauses which exclude liability for the fraud of banks’ employees
would, in the Court’s judgment, stand contrary to public policy considerations
and run afoul of the reasonableness test vis-à-vis UCTA. Individual and
corporations entrust banks and employees of banks with their savings and
investments. Public confidence in the system is fundamental, founded on mutal
trust and reasonable expectation of honest dealings by employees of banks.
Shifting the attendant risk and liability for fraud and willful misconduct of
employees of banks by way of conclusive evidence clauses, strikes at the heart
of presumed integrity of the system. The Bank is better placed to ascertain such
fraud on their employees, and the negative impact on public confidence and
trust in the modern banking system would render such clauses to be
unreasonable under UCTA.

CPFTA

129. Applies to consumers – S3 CPFTA. Consumer means an individual who,


otherwise than exclusively in the course of business receives or has the right to receive

96
goods or services from a supplier; or has a legal obligation to pay a supplier for goods
or services that have been or are to be supplied to another individual.

130. The CPFTA cannot be contracted out (see S13 CPFTA and Speedo Motoring
v Ong Gek Sing).

131. The CPFTA gives consumers additional remedies (S6 CPFTA – right to sue)
against suppliers if the latter engages in unfair practices (as defined in S4 CPFTA).

132. The Small Claims Tribunal has wide powers (S7 CPFTA):

a. Order restitution;

b. Award damages;

c. Order specific performance;

d. Order supplier to repair goods or provide parts;

e. Order to vary the contract between the supplier and the consumer.

133. Unfair practices – defined very broadly to include:

a. “Taking advantage of a consumer if the supplier knows or ought reasonably to


know that the consumer is not in a position to protect his own interests; or is not
reasonably able to understand the character, nature, language or effect of the
transaction or any matter related to the transaction” (S4(c) CPFTA);

b. “Taking advantage of a consumer by including in an agreement terms or


conditions that are harsh, oppressive or excessively one-sided so as to be
unconscionable” (Para 11, Second Schedule CPFTA);

c. “Using small print to conceal a material fact from the consumer or to mislead a
consumer as to a material fact, in connection with the supply of goods or
services” (Para 20, Second Schedule CPFTA).

97
Chapter 3: Frustration

Historical development of the law

134. Historically, the common law rule was strict and insists on the literal
performance of contracts. The change in circumstances after a promise was made did
not excuse the promisor from the performance, even if this was impossible (Paradine
v Jane).

a. In Paradine v Jane, a tenant of a farm was dispossessed for 2 years, due to an


act of the King’s enemies. The tenant claimed that he was not liable to pay the
rent for the period which he was dispossessed. It was held that the tenant was
obliged to pay, notwithstanding the change of circumstances.

135. However, the common law eventually developed further and the doctrine of
frustration was created, to address such situations – the genesis is often attributed to the
case of Taylor v Caldwell.

a. In Taylor v Caldwell, P and D entered into a contract, where D agreed to loan


the P the use of Surrey Gardens and the Music Hall on 4 days for a concert, in
return for rent. There was a fire the day before the concert, which destroyed the
Music Hall. P sued D for the expenses, which they had incurred in advertising
and making preparations for the concerts, arguing that D was in breach of their
obligations to provide the venue. The court held that there was an implied
condition that “impossibility of performance arising from the perishing of the
person or thing shall excuse the performance”. As the Music Hall was essential
to their performance and ceased to exist, without fault on either party, both
parties were excused from the contract.

b. Note that in this case, the Court utilised the fiction of an implied condition.

Juridical basis

136. National Carriers v Panalpina [1981] AC 675 at 687 (Lord Hailsham) (MK
744 / 740):

98
“At least five theories of the basis of the doctrine of frustration have been put
forward at various times...”.

Implied term

“The first is the ‘implied term’ or ‘implied condition’ theory on which


Blackburn J plainly relied in Taylor v Caldwell ... The weakness ... is that it
raises once more the spectral figure of the officious bystander intruding on the
parties at the moment of agreement. In the present case, had the officious
bystander pointed out to the parties ... the danger of carrying on the business ...
and asked them what they would do in the event of a temporary closure of
Kingston Street ... after the lease has been running for over five years, I have
not the least idea what they would have said, or whether either would have
entered into the lease at all.”

Total failure of consideration

“...the respondent sought to argue that Taylor v Caldwell ... could as easily been
decided on the basis of a total failure of consideration. This is the second of the
five theories. But Taylor v Caldwell was clearly not so decided, and in any event,
many if not most, cases of frustration which have followed Taylor v Caldwell
have occurred during the currency of a contract partly executed on both sides,
when no question of total failure of consideration can possibly arise”.

Just and reasonable solution

“In Hirji Mulji v Cheong Yue Steamship ... Lord Sumner seems to have
formulated the doctrine as a ‘...device [sic], by which the rules as to absolute
contracts are reconciled with a special exception which justice demands’... The
weakness of the formulation ... is that, though it admirably expresses the
purpose of the doctrine, it does not provide it with any theoretical basis at all”.

Foundation of the contract

“Hirji Mulji ... is, it seems to me, really an example of the more sophisticated
theory of ‘frustration of the adventure’ or ‘foundation of the contract’
formulation, said to have originated with Jackson v Union Marine Insurance ...

99
This, of course, leaves open the question of what is, in any given case, the
foundation of the contract or what is ‘fundamental’ to it, or what is the
‘adventure’.”

Construction of the contract/radical change in obligation

“Another theory ... is that the doctrine is based on the answer to the question:
‘What in fact is the true meaning of the contract?’... This is the ‘construction
theory’. In Davis Contractors Ltd v Fareham ... Lord Radcliffe put the matter
thus, and it is the formulation I personally prefer: ‘... frustration occurs
whenever the law recognises that without default of either party a contractual
obligation has become incapable of being performed because the circumstances
in which performance is called for would render it a thing radically different
from that which was undertaken by the contract. Non haec in foedera veni. It
was not this that I promised to do’.”

137. Alliance Concrete Singapore Pte Ltd v Sato Kogyo (S) Pte Ltd [2014] SGCA
35 at [38] (Phang JA):

“Although the doctrine of frustration is an established one in the common law


of contract, it is an exception to the norm of sanctity of contract and therefore
is to be applied to discharge the parties from their contract in only exceptional
circumstances. At this juncture, it is important to note that the doctrine of
frustration itself embodies an equally valid idea of justice and fairness. This
idea of justice and fairness is that it would be unjust and unfair to hold the parties
to their contract where an external event has rendered further performance of
their contract something radically or fundamentally different from that which
they had originally contemplated at the time they entered into that contract. This
last-mentioned point is important inasmuch as it is not some general (still
less, subjective) idea of justice and fairness that is within the scope of the
doctrine. To this end, we endorse the following passage in Chitty on Contracts
vol 1 (Sweet & Maxwell, 31st Ed, 2012) at para 23-017:

... As we have noted, some judges have maintained that the doctrine seeks to
give effect to the demands of justice, but these statements cannot be invoked to

100
justify the conferral upon the courts of a wide-ranging discretion to re-write the
parties' bargain in the name of 'fairness and reasonableness'.”

138. The better view is that no consent is the rationale – the radical change test
assumes that parties only consent to perform in a limited (although wide) range of
circumstances. When events radically change the circumstances, taking them outside
that range, the consent runs out (Chen-Wishart, Contract Law).

Test for frustration

139. A contract is discharged by operation of law under the doctrine of frustration


where (Davis Contractors v Fareham Urban District Council):

a. there was a supervening event, which occurred after the formation of the
contract;

b. without the fault of either party; and

c. the circumstances in which performance is called for would render the


obligation a thing radically different from that which was undertaken by the
contract.

140. This test involved a multi-factorial approach and only applied to discharge the
parties from their contract in only exceptional circumstances (Alliance Concrete
Singapore v Sato Kogyo at [37]-[38]).

141. The factors which the Courts will consider include (Alliance Concrete
Singapore v Sato Kogyo at [37]):

a. The terms of the contract, matrix or context;

b. The parties’ expectations, assumptions and contemplations, in particular as to


risk, as at the time of contract, which can be ascribed mutually and objectively;

c. The nature of the supervening event; and

d. The parties’ reasonable and objectively ascertainable calculations as to the


possibilities of future performance in the new circumstances.

101
Illustrations in specific factual situations

Supervening impossibility

142. A contractual obligation may be rendered radically different in the situation of


supervening impossibility – the circumstances render the performance of the contract
impossible (even though literal impossibility is not required) (Alliance Concrete v Sato
Kogyo at [48]).

Destruction of subject matter

143. A contractual obligation may be impossible to perform if the subject matter is


destroyed (Taylor v Caldwell).

a. Performance hall destroyed by fire – Frustrated – Taylor v Caldwell – see


135 above.

b. Structure on land destroyed by fire, but main purpose of contract – Not


frustrated – Chiang Hong Pte Ltd v Lim Poh Neo – In this case, CH and LPN
were parties to an agreement where CH would pay a sum of money to LPN for
LPN to vacate and deliver up vacant possession of certain premises to CH.
Before all the money was paid to LPN, a fire completely destroyed the wodden
factory built on the premises. CH sought a declaration that the agreement was
frustrated. The SGCA agreed with the SGHC reasoning that CH was interested
to only recover vacant possession of the land, the structure was inconsequential
and had to be demolished anyway – CH in their negotiations agreed that they
would bear the task and costs of demolishing the structure. As such, the
subject matter was the vacant possession, and not the structure.

Unavailability of subject matter

144. The unavailability of the subject matter may result in the contractual obligation
becoming impossible to perform:

a. Compulsory acquisition of land in sale and purchase of land – Frustrated


– Lim Kim Som v Sheriffa Taibah – In this case, ST (vendor) and LKS
(purchaser) entered into an agreement to purchase a piece of property (at
$2,138,200). Completion was slated on 7 July 1983, but the purchaser delayed.

102
On 13 July 1983, ST’s solicitors sent a notice to complete to the LKS’s solicitors.
The property was subsequently gazette for public purpose on the same day, and
was compulsorily acquired on 23 August 1984 (for $450,000). It was held that
the contract was frustrated – the purchaser had bargained for the legal estate and
the use of the property but what he would had gotten was an estate which was
unusable and unsaleable. This was a different thing from that contracted for
by the parties.

b. Compulsory acquisition of land in lease – Frustrated – SG Woodcraft v Mok


Ah Sai – In this case, D had a tenancy for certain premises at Orchard, which D
subleased to P. The property was subsequently compulsorily acquired. P sued
for a refund of the deposit and moneys paid under the contract. D sought to
enforce the contract (which was for 10 years and did not provide for earlier
termination). The Court held that the contract was frustrated by the compulsory
acquisition – P’s claims were allowed and D’s counterclaim was dismissed.

c. Malaysian offshore ringgit unavailable – Frustrated – Shenyin Wangou-APS


v Commerzbank – In this case, Ps deposited MYR in offshore accounts with D,
which was due to mature on 3 Sept 1998. On 1 Sept 1998, the Malaysian
government imposed exchange control measures to stabilize the MYR. The
entire offshore market for MYR was wiped out. D made repayment in US
dollars at a rate less favourable than the official rate, and P had to purchase
MYR at the official rate to fulfill their own commitments and sustained losses.
P sought to claim this loss against D. It was held that the contract was frustrated.

Failure of a particular source

145. The failure of a source may operate to frustrate the contract, depending on
whether one or both of the contracting parties intended or contemplated that particular
source (Alliance Concrete v Sato Kogyo at [48]).

a. Source is stipulated in contract – Frustrated – Where the source is referred


to in the contract and the source fails, the contract is generally discharged by
the doctrine of frustration – Howell v Coupland.

103
i. In Howell v Coupland, a farmer contracted with a merchant to sell the
merchant potatoes grown on the defendant’s land in Whaplode. The
crops failed due to disease and he was unable to supply the full quantity
contracted for. The Court held that the contract was discharged because
the performance to sell specific things had become impossible.

b. Only one party intended an unspecified source – Not frustrated – Where


only one of the contracting parties intended for a particular source such that the
source is not provided for in the contract, then the contract will not be
discharged when that source fails – Blackburn Bobbin Co Ltd v TW Allen &
Sons Ltd.

i. In Blackburn Bobbin, the seller contracted to sell “Finland Birch timber”.


The seller did not keep stocks of this timber in England, but got it by
shipping through the Baltic. However, due to the outbreak of WW1, the
seller’s usual supply was disrupted. The contract was not frustrated,
since the buyer did not know that the seller had to ship the timber
through the Baltic.

c. Both parties contemplated an unspecified source – Frustrated – where the


source is not referred to in the contract, but both parties contemplated that the
unspecified source the contract will be frustrated – Alliance Concrete v Sato
Kogyo.

i. AC was a supplier of ready-mixed concrete (“RMC”), while SK was a


contractor. AC had contracted with SK to supply SK with RMC.
However, in 2007, there was a sand ban which banned export of
Indonesian sand – a key ingredient for RMC. In this case, the Court held
that parties contemplated that Indonesian sand would be used in the
preparation of RMC by AC, even though this was not specified (there
was an implicit understanding, given that particular types of sand have
different properties). The Court held that the contract was discharged.

146. There is some authority suggesting that where the failure of the source is partial,
the contract is only partially frustrated (Sainsbury v Street).

104
a. In this case, S agreed to sell Sainsbury 2755 tons of barley grown on S’s farm,
but the yield was only 140 tons, which S sold to a third party. The Court held
that Sainsbury should have the option of requiring S to deliver the tonnage
actually produced, otherwise in times of failing crops, the supplier could
disregard its contracts and profit from rising prices by selling elsewhere.

Impossibility of method of performance

147. Where the method of performance is impossible, the contract may be frustrated
(Nicholl & Knight v Ashton Edridge). However, the mere fact that there was a change
in the method of performance itself does not amount to frustration (Tsakiroglou v
Noblee Thorl).

a. Different method of performance – Not frustrated – Tsakiroglou v Noblee


Thorl – In this case, there was a contract for the sale of Sundanese groundnuts
cif Hamburg during November/December 1956. On 2 November 1956, the Suez
Canal was closed to navigation, and the goods have to be shipped round the
Cape of Good Hope. The route was more than twice as long and the freights
were more costly (3 times). The sellers failed to ship the goods. The Court held
that the contract was not frustrated – the route of delivery is normally not
relevant to the buyer; the goods were not perishable; there was no date fixed for
delivery; there was no shortage of available shipping carrying goods via the
Cape. As such, the contract is not frustrated, in particular if the alternative
means of performance does not fundamentally differ from the intended means
(even where stipulated in the contract).

b. Method of performance impossible – Frustrated – Nicholl & Knight v Ashton


Edridge – In this case, there was a contract of sale of goods “to be shipped per
steamship Orlando from Alexandria during… January”. The ship however ran
aground and could not reach Alexandria in January. The Court held that the
contract for the sale of goods was frustrated.

Death/incapacity of contracting party in personal contracts

148. A contract of personal service may be frustrated by death or incapacitation of


one of the contracting parties (Notcutt v Universal Equipment).

105
a. Frustrated – In this case, an employee (skilled workman whose job was to
operate a universal milling machine) suffered a heart attack and count never
work again. In this case, the issue was when the contract was terminated – if it
was terminated earlier by contract, the employer is not required to pay sick pay
to the employee, whereas if it was terminated later, the employer was required
to pay sick leave if the employee was under notice. The Court held that the
employment contract was frustrated.

Supervening illegality

149. Contractual performance rendered legally impossible by a change in law or


circumstances may frustrate the contract (Fibrosa v Fairbairn).

a. Frustrated – In Fibrosa v Fairbairn, P agreed to buy some machinery for 4800


pounds from D. in july, P made a payment of 1000 pounds as part of the
agreement. By September, Germany invaded Poland and Britain had declared
war. P attempted to get the payment back but D refused, arguing that the
invasion frustrated the contract. P sued D. It was held that the contract was
frustrated when Poland was occupied by Germany in the second world war and
that it was illegal to trade with the enemy in times of war.

Delay and hardship

150. Few contracts are “impossible” to perform. Delay and hardship are generally
insufficient to render a contract frustrated.

Increased costs/lower profits

151. In general, a mere increase cost or hardship in performing the contract is


insufficient to discharge the parties from the contract by frustration. This is in line with
the general notion that parties should have contemplated the risks and thus bears the
risks when such risks turn into reality (Davis Contractors v Fareham UDC).

a. Not frustrated – Davis Contractors v Fareham – In this case, D agreed to build


78 houses for F. The work took almost 3 times longer and cost £17,000 more
than was planned because of serious labour shortages and difficulties in
obtaining building supplies. D completed performance but argued that the
contract was frustrated so that they were not confined to the contract price but

106
could claim an extra £17,000 on a quantum meruit basis. The UKHL held that
the contract was not frustrated: (1) D took the risk of increased costs and delay
by agreeing to a fixed price; (2) the difficulties were foreseeable and D could
have provided for them in the contract; and (3) although D’s performance was
significantly more onerous, it was not radically different from that originally
undertaken.

b. Not frustrated – Glahe International v ACS Computer – Glahe sought to


recover US$233,680 advanced to the respondent, ACS Computer for the
purchase of computers under a sale agreement. The computers were intended
for the Russian market, to be purchased by MS-Group. However, MS-Group
was unable to enter into contracts directly with ACS as they had difficulty
obtaining US dollars. Glahe was meant to be the go-between and assumed the
risk of MS-Group not performing their part of the contract (to buy the
computers). Glahe claimed that it was relieved from performing its obligations
by a force majeure clause, or that the contract was frustrated by the supervening
political and economic events taking place in Russia. The Court held that Glahe
was still obliged to take the computers – the increase in customs duty by 15%
on the goods imported into Russia and the inflation did not prevent Glahe from
importing the computers into Russia and selling them there. The contract was
merely unprofitable.

c. Not frustrated – MP-Bilt Pte Ltd v Edy Yumianto – In this case, D signed a sale
and purchase agreement to purchase a flat from P. The payment was to be made
in installments and D failed to pay the 2nd installment. D argued that due to the
resulting inflation, rioting and civil unrest in Indonesia, D’s business in
Indonesia had suffered severe losses and he could no longer pay the installments
under the agreement and that it was frustrated. The Court held that there was no
frustration – (1) the financial capacity of the purchaser was his own concern;
(2) the subject matter (flat) was not affected by the crisis in Indonesia; (3) D
might borrow money or have money already set aside. He cannot walk away
merely because financial difficulties had come upon him.

152. However, there is dicta suggesting that if the increase in the cost of performing
the contract was astronomical (such as 100 times), it would be a fundamentally different

107
situation which had unexpectedly emerged for the parties, and may be a ground for
frustration (Holcim v Precise Development).

Failure of underlying purpose of the contract

153. A contract may be frustrated although performance of the contract is not illegal,
impossible or more onerous. The recipient of the performance claims that the event has
undermined the purpose of the contract for her. The purpose must be common to both
parties and must be thwarted to a very high degree (Krell v Henry).

a. Frustrated – Krell v Henry – In this case, Henry agreed to hire from Krell a flat
in Pall Mall for June 26 and 27 to view the coronation procession of KE7. The
contract contained no express reference to the coronation processions, or to any
other purpose for which the flat was taken. A deposit was paid when the contract
was entered into. As the processions did not take place on the days originally
fixed, the defendants declined to pay the balance of the agreed rent. The Court
inferred the common purpose from (1) the position of the flat; (2) flat owner’s
advertisement for windows to view the coronation; (3) an enhanced price was
charged for two days excluding the nights. The Court held that the contract was
frustrated.

b. Not frustrated – Herne Bay Steamboat v Hutton – In this case, HBS hired out
a pleasure boat “for the purpose of viewing the naval review and for a day’s
cruise around the fleet” to H, who then charged passengers to see the said naval
review and trips around the fleet. The naval review was cancelled along with
the coronation. However, the Court regarded H’s venture to charge passengers
for the cruise as his own and at his own risk, and that the purpose was not
frustrated as well as H could still bring passengers to cruise around the fleet.

Temporary unavailability/delay

154. Unavailability of subject matter or delay in performance may be grounds for


finding frustration (Jackson v The Union Marine Insurance). The question is however
one of degree.

a. Frustrated – Charterparty and ship running aground – Jackson v The Union


Marine Insurance – In this case, on the first day of the charterparty (to deliver

108
rails), the ship ran aground and was not refloated and repaired for 7 months. The
Court held that the damage occasioned such delay that it amounted to practical
commercial destruction – performance after such delay would be of no use to
the charterer.

b. Not frustrated – Charterparty for 5 years, unavailable for 2 – Tamplin SS v


Anglo-Mexican Petroleum – In this case, a ship chartered for 5 years was
requisitioned by the government after two years and altered for use as a
troopship. The shipowner claimed substantial compensation from the Crown,
arguing frustration of the charterparty. The Court held that there was no
frustration, as the charterparty expressly permitted the charterer to sublet the
ship on Admiralty or other service, and there remained many months when the
ship might be still free for commercial use.

c. Not frustrated – Lease for 10 years, unavailable for 20 months – National


Carriers v Panalpina – In this case, a 10 year lease of a warehouse was rendered
inaccessible for some 20 months, after 5 years into the lease. The Court held
that the local authority’s act of rendering the warehouse inaccessible did not
frustrate the warehouse, given that there was more than 3 years of remaining
unexpired lease. The Court seemed to suggested that had the prohibition been
permanent, the lease may had been frustrated.

Exceptions and limits of frustration

Self-induced frustration and power to elect

155. Where a party enters into a number of contracts and an external event partially
destroys her supplies so that she cannot satisfy all her contracts, she must elect which
contracts to allocate remaining supplies to, and which to leave unperformed. The fact
that a party can elect bars the frustration of that contract (Maritime National Fish v
Ocean Trawlers, Super Servant Two). However, frustration is permitted if (i) the event
completely destroys the party’s supplies, or (ii) only one mode of performance was
specified or a specific supply was allocated for the contract. Fault here is not limited to
acting deliberately or in breach of an actionable duty but includes carelessness (Super
Servant Two – "whether it is an event which the party seeking to rely on it had the
means and opportunity to prevent but nevertheless caused or permitted to come about")

109
a. Not frustrated – Maritime National Fish v Ocean Trawlers – M chartered from
O a trawler for otter trawling. M applied for 5 licences but received only 3,
which they allocated to other trawlers (including two of their own). M sought
to argue that the other contracts were frustrated. The UKPC held M to be liable
to O for the hire, because O’s trawler was only not usable for otter trawling due
to M’s own election. The contract with O would have been possible to perform
if M had allocated a licence to O’s trawler.

b. Not frustrated – The Super Servant Two – In this case, W agreed to transport
L’s drilling rig between 20 June and 20 August using, at its option, either The
Super Servant One or the The Super Servant Two. W internally allocated The
Super Servant Two to this contract and committed The Super Servant One to
other contracts. Prior to performance, The Super Servant Two sank. The EWCA
rejected W’s claim of frustration because it was W’s own election not to use the
remaining ship that had led to its non-performance. However, the carrier was
excused under the force majeure clause in the contract, which covered “perils
or dangers and accidents of the sea”.

156. The cases above can be criticised for leaving a seller or supplier of goods in an
impossible situation where her source partially fails due to an unforeseen event. The
doctrine of frustration should allow some of the contracts to be frustrated. Prof Mindy
raises some arguments in support of this:

a. The conclusion of the EWCA in The Super Servant Two was unnecessary.
Unlike in the case of Maritime National Fish where the complainant could
perform all its contracts (by not allocating a licence to its own boat), this was
not the case in The Super Servant Two, where the party’s only choice is as to
which of her contracts would be left unperformed.

b. The problem is not choice but favoritism. The real objection is the potential for
the complainant to give preferential treatment to her most profitable contract
partners.

c. The solution is to eliminate the complainant’s choice by requiring her to allocate


her remaining supplies fairly – such as in the cases of partial failure of supply.

110
Where this is impossible, the supplier should be made to perform her contracts
in the order they were made or where performance is required.

Construction of the contract and foresight

157. Faced with a defence of frustration, the Court must decide whether the contract,
on its proper construction, has positively allocated the risk of the supervening events.
Only if the contract has not expressly or impliedly provided for what should happen in
the new circumstances does the frustration doctrine step in to determine whether the
parties are absolved from liability for non-performance (RDC Concrete v Sato Kogyo
at [63])).

Express provision: Force majeure and hardship clauses.

158. Parties may stipulate in the contract (i) the circumstances excusing further
performance of the contract (widening or narrowing the scope of frustrating
circumstances) and/or (ii) the consequences of the triggering circumstances (RDC
Concrete v Sato Kogyo at [55]). The issue is one of interpretation of contracts.

a. FM clause applied – Holcim v Kwan Yong Construction – In this case,


Indonesia had imposed a sand ban and Holcim was unable to supply ready made
concrete. Clause 3 of the contract states that “the Suppler shall be under no
obligation to supply the concrete if the said supply has been disrupted by virtue
of… shortage of material… or any other factors arising through circumstances
beyond the control of the Supplier”. The SGCA held that the FM clause applied
and the supplier had a defence for their failure to perform. The supply was
disrupted, taking into account of considerations of commercial practicality (i.e.
even though parties on the facts could still carry out further performance. In this
case, disruption is a standard is lower than that in frustration at [56]).

b. FM clause read down, frustration applied instead – Metropolitan Water Board


v Dick Kerr – In this case, the contractors had agreed to construct a reservoir in
six years. The contract was to continue, and time extension provided in the event
of delays “whatsoever and howsoever occasioned”. There was a supervening
regulatory intervention, whereby the government ordered work to be stopped
and the plant sold. The Court read down the FM clause to only apply to
temporary difficulties, and not where the interruption is of such a character

111
and duration that it vitally and fundamentally changes the conditions of the
contract, and could not possibly have been in the contemplation of the parties
(outcome was commercially nonsensical – extension infinitum).

Implied provision and foresight

159. There are conflicting judicial statements as to whether foreseeability of the risk
would oust the operation of the doctrine of frustration. The better view is that
foreseeability is a factor as to whether the parties have implicitly allocated the risk of
the supervening event to the performing party (Chen-Wishart, Contract law at [7.4.2]).
The higher the degree of foreseeability and detail, the more likely that the non-
performing party would have assumed the risk.

160. Cases with judicial pronouncements barring frustration

a. RDC v Sato Kogyo – “whereas a force majeure clause is an agreement as to how


outstanding obligations should be resolved upon the onset of a foreseeable event,
the doctrine of frustration concerns the treatment of contractual obligations
from the onset of an unforeseeable event”.

b. Glahe International v ACS – “This doctrine of frustration operates on a


completely different level from a force majeure clause. Whereas a force majeure
clause is an agreement as to how to resolve outstanding obligations upon the
onset of a foreseeable event, the doctrine of frustration concerns the treatment
of contractual obligations upon the onset of an unforeseeable event.” In this case,
the Courts held that foreseeability was an important factor – “ACS contract,
significant changes in terms of rates of exchange, currency depreciation,
inflation, imposition of taxes and political situations were already taking place
in Russia. Such matters were, and must have been, in the contemplation of the
parties”.

i. Note: On the facts, MS-Group (Russian company) was interested in


computers exhibited by ACS and wanted to import ACS for local
distribution. MS-Group approached Glahe to act as an intermediary – as
Glahe was not a Soviet company, Glahe would not encounter any
problem in dealing with hard currency. MS-Group’s Societ customers

112
would pay US Dollars directly to Glahe in Switzerland and Glahe would
pay ACS the moneys they had received. The entire purpose of imposing
Glahe was that it assumed the risk of MS-Group not performing their
obligation to pay the contract sum for the computers (at [33]).

c. Walton Harvey v Walker – In this case, D granted P a contractual right to display


an advertising sign on top of their hotel for 7 years. The hotel was subsequently
acquired compulsorily. D was held liable for the breach of contract – there was
no frustration because the defendants foresaw the risk of compulsory
acquisition.

d. HDB v Microform Precision – In this case, P leased land to D, who intended to


build a factory. The land was land-locked. D signed the lease, but the
government refused to grant access. D was aware that there was a risk that the
authorities would not grant access to the plot of land before the contract was
concluded (“it was a known problem and one that was known for a long time”),
but took on the risk of such a foreseeable event happening. The Court held that
the contract was not frustrated – they had assumed the known risk that approval
for access may not be granted, but hoped that the authorities would eventually
relent.

161. Cases with judicial pronouncements not barring frustration

a. The Eugenia – Per Lord Denning, “It has frequently been said that the doctrine
of frustration only applies when the new situation is ‘unforeseen’ or
‘unexpected’ or ‘uncontemplated’, as if that were an essential feature. But it is
not so. The only thing that is essential is that the parties should have made no
provision for it in their contract … The only relevance of it being ‘unforeseen’
is this: If the parties did not foresee anything of the kind happening, you can
readily infer they have made no provision for it: whereas, if they did foresee it,
you would expect them to make provision for it. But cases have occurred
where the parties have foreseen the danger ahead, and yet made no provision
for it…”

b. Lim Kim Som v Sheriffa Taibah – “‘The relevance of foreseeability to the


doctrine of frustration is not free from doubt…In our opinion, the rigid

113
insistence on the fact that the event ought to have been foreseen cannot be an
adequate solution and it would negate the very test propounded by Lord
Radcliffe in the Davis Contractors case’ Ultimately takes reference from what
parties agreed to, whether or not they foresaw certain contingencies.”

i. In any case, on the facts, compulsory acquisition was not foreseen or


foreseeable.

c. Alliance Concrete v Sato Kogyo – “The “radical change in obligation” test


involves a multi-factorial approach… Among the factors which have to be
considered are the terms of the contract itself, its matrix or context, the parties’
knowledge, expectations, assumptions and contemplations, in particular as to
risk, as at the time of contract, at any rate so far as these can be ascribed
mutually and objectively, and then the nature of the supervening event, and the
parties’ reasonable and objectively ascertainable calculations as to the
possibilities of future performance in the new circumstances. Since the subject
matter of the doctrine of frustration is contract, and contracts are about the
allocation of risk, and since the allocation and assumption of risk is not simply
a matter of express or implied provision but may also depend on less easily
defined matters such as ‘the contemplation of the parties’, the application of
the doctrine can often be a difficult one.”

Effects of Frustration

At common law

162. At common law, the initial position was that when a contract is discharged by
frustration, the loss would lie where it falls.

163. Money paid or due:

a. As such, any sums paid before the frustrating event were unrecoverable, and
sums due before the frustrating event still had to be paid (Chandler v Webster).

i. In this case, W agreed to let C a room on Pall Mall to watch the King’s
coronation. Money for the room was pre-paid. C had hired the room for
a customer of his, but the customer eventually did not want the room,

114
due to a death of a relative. C wrote to W to confirm the hiring of the
room and paid £100, and was liable to pay a further £41. The contract
was eventually frustrated, and the issue was whether C could recover his
£100 and not pay the £41. It was held that C could not recover the £100
and was liable to pay £41 as it was an obligation accruing prior to
frustration, and the loss lies where it falls.

b. This position has been modified in Fibrosa v Fairbairn, such that the money
paid/payable would be recoverable or need not be paid, if there is a total failure
of basis.

i. In this case, the buyer contracted to buy some machinery from the seller,
with £1000 pre-paid and £600 payable prior to the frustrating event (out
of £4800). The Court held that the sum paid was recoverable and the
money payable need not be paid. It was held that because there was a
total failure of basis (no part of the machinery delivered), the buyer
could recover the money.

164. Non-money benefits: The value of goods or services conferred prior to the
frustrating event is not recoverable, unless payment was due prior to the frustrating
event (Appleby v Myers).

a. In this case, P contracted to make and erect machinery in D’s factory and to
maintain the machinery for two years. Payment was to be upon completion of
the work. After part of the machinery had been erected, an accidental fire
destroyed the factory and machinery and frustrated the contract. It was held that
the claimants could not recover in respect of their work because they were only
entitled to payment when performance was completed. As the fire had prevented
completion, they were not entitled to payment.

Frustrated contracts act

165. The Frustrated Contracts Act, and its English equivalent (The Law Reform
(Frustrated Contracts) Act 1943 governs the prevention of unjust enrichment and some
adjustment of losses following the discharge of a contract for frustration. The statute

115
has immediate significance with the law of unjust enrichment, in so far as the statute
has flavors of unjust enrichment.

166. After the case of Fibrosa v Fairbairn, the common law of unjust enrichment
suffered from three defects: First, it was assumed that the failure of the basis must be
total, such that the receipt of 1 machine in Fibrosa v Fairbairn would had denied
recovery in unjust enrichment. Second, the cases around Fibrosa v Fairbairn were
decided in a time when the cases were brought under forms of action, and that failure
of basis only worked for money claims. Third, the losses incurred under the contract
(reliance expenses) could not be apportioned, but this is an area which lies outside the
law of unjust enrichment in general (Andrew Burrows, the Law of Restitution 3rd Ed at
361-362).

Restitution of money (S2(2),(3) FCA)

167. Under S2(2) FCA, payments made under the contract are to be returned, and
payments due but are unpaid are no longer payable. As such there is no requirement
that money can only be recovered where the failure of consideration is total.

168. Under S2(3) FCA, if the party entitled to the sums had incurred expenses before
the time of discharge/performance of the contract, the Court may, if it considers just to
do so, allow that party to retain the sums, not being an amount in excess of the expenses
so incurred (i.e. the cap is the lower of the sums retained or the expenses).

169. In BP v Hunt (No.2), Goff J took the view that the aim of the section (and
S2(4)(a)) was a statutory recognition of change of position, and took the view that the
scheme is wholly concerned with the law of unjust enrichment, and “it is not designed
to apportion the loss between the parties”. However, it is doubtful whether this
interpretation is correct. In Gameco SA v ICM/Fair Warning, Garland J took the view
that the section confers on the court a discretion to engage in loss apportionment,
because the Court can reduce the amount to be repaid if it is “just” to do so.

Restitution of benefits (S2(4) of the FCA)

170. Under this section, there is a discrepancy in the way how it deals with
unperformed obligations (as compared to S2(2) of the FCA, S1(2) of LR(FC)A 1943).
If a contractual obligation to transfer a non-money benefit has fallen due before the

116
frustrating event, and remains unperformed, the party subject to that obligation is liable
in damages for failing to perform it. Such a damages award would fall outside the
scheme of the act.

171. In BP v Hunt (No.2), Goff J (as he then was) interpreted the section as calling
for two stages of analysis: first, the valuable benefit must be identified, and then the
just sum must be assessed. In that case, Goff J (as he then was) took the view that the
benefit of services rendered to the defendant would depend on whether an end result
would result from the services. If there was, that benefit would be identified as the
“end product of the services”, whereas if there was no end product, the benefit would
be the services themselves. As such, where the end product had been reduced by the
frustrating event (such as that in Appleby v Myers), it would seem that there would be
no benefit obtained by the person contracting for the services and end product.

a. In this case, H owned an oil concession in Libya, and contracted with BP to


exploit the oil. Under the contract: (i) H was to transfer to BP half the
concession; (ii) BP would transfer to H “farm-in” contributions in cash and oil;
(iii) BP would explore for and develop the oil; (iv) BP would provide for all the
funds until the oil was found; and (v) the profits would be shared, but 3/8 of
Hunt’s share would go to BP until 125% of the farm-in contributions and half
the costs of BP were covered. A massive oil reserve was found in 1967, but
there was e regime change in Libya, which expropriated the concession. BP
claimed that the contract was frustrated.

b. It was held that the contract was frustrated. The value of the benefit received by
Hunt was half the value of oil reaped and compensation for confiscation ($85m)
at stage 1. The court considered that there was an end product (the
enhancement of value of H’s share of the concession), and not the cost to BP of
its work.

c. In assessing the just sum due to BP, the Court took the just sum (value of
contractual performance of $98m), subtracted by the value of performance
received from Hunt ($63m), which the Court considered that this was the
reasonable value of BP’s work and expenses conferred on H. As such, the award
of $35m was awarded in full as it came well under the ceiling of $85m.

117
172. Difficulties of BP v Hunt – This case poses a few challenges:

a. Stage 1: benefits.

i. Where B transfers goods to A, the benefit is easily identifiable (goods).

ii. Where B performs services which is intended to yield an end product,


the benefit is the end product.

iii. Where B performs pure services, which are never intended to produce
end products, the benefit is the value of the services themselves.

iv. Generally, there are no difficulties where the performance is completed,


and there is an objective value from B’s services. However, two
situations brings difficulties:

1. Partial performance – where B only partially performed before


the frustrating event, the precise value is problematic – what is
half a haircut? Mindy’s solution is to allow for subjective
devaluation in Stage 2, rather than in stage 1.

2. Reduction of benefit by frustrating event: If the frustrating event


reduces or completely destroys the value of the end product, this
is considered in the Stage 1 assessment. The preferable approach
is that the effect of the frustrating event on the benefit should be
dealt with in Stage 2, in the assessment of the “just sum”.
Furthermore, there is nothing in the plain reading of the section
to require that the benefit of the services to be conceived in its
end product, and seems to only be directed to contracts for sales
of goods simpliciter.

118
Chapter 4: Breach of contract and termination for breach

What is a breach of contract

173. A party breaches a contract, when, without lawful excuse, he or she fails to
perform any of his or her contractual obligations (Chen-Wishart, Contract Law).

Breach and fault

174. As a starting position, fault is not needed for the innocent party to establish that
the counterparty had breached the contract (RDC v Sato Kogyo at [134]). However, it
is possible for the parties to provide that breach is conditioned on fault (see e.g. Lim
Sze Eng v Lin Choo Mee):

a. Expressly: In Lim Sze Eng v Lin Choo Mee, the Court reaffirmed the earlier
proposition (citing KS Energy Service v BR Energy) that a “reasonable
endeavours” clause should have the same test applied as a “best endeavours”
clause.

i. The obligor has a duty to do everything reasonable in good faith with a


view to procuring the contractually stipulated outcome within the time
allowed. This involves taking all those reasonable steps which a prudent
and determined man, acting in the interests of the oblige… and anxious
to procure the contractually-stipulated outcome within the available
time, would have taken.

ii. The test is an objective one. The obligor can take into account its own
interests.

iii. The obligation is not a warranty to procure the contractually-stipulated


outcome.

iv. Note: there are other general principles for interpretation.

b. Impliedly: Generally, in services contract, liability is often based on fault –


services by professional persons (solicitors, architects, accountants or doctors)
impose duties of care only, and obviously does not guarantee a result.

119
Order of performance and breach

175. The order of performance may mean that the failure to perform one’s obligation
is not a breach by that party:

a. Condition precedent – If the performance is conditioned on an event, the non-


occurrence of that event may mean that A’s performance need not occur.

i. Obligation to start work may be depending on parties acquiring


regulatory licence/planning permission.

ii. A’s performance may be dependent on B’s performance – A can only


lend money, if B had successfully provided the necessary
documentation for drawdown (corporate authorisations).

b. Frustration – If the performance is after the supervening event amounting to


frustration, the parties’ future obligations are discharged.

Right to terminate for breach

Why would innocent party want to terminate the contract?

176. Non-termination “keeps the contract alive for the benefit of the other party as
well as his own; [the innocent party] remains subject to all his own obligations and
liabilities under it, and enables the other party not only to complete the contract…
notwithstanding his previous repudiation of it, but also to take advantage of any
supervening circumstance which would justify him in declining to complete it” (Frost
v Knight at 112). As such the advantages are:

a. Innocent party no longer bound to perform his end of the contract;

b. Innocent party is no longer bound to accept/pay for further performance.

c. Prevents the operation of frustration in the future as a defence.

d. In some circumstances, allows the innocent party to mitigate his losses and sue
for damages immediately, without having to wait for the contractually stipulated
time of performance.

120
i. In Frost v Knight, D promised to marry P as soon as D’s father dies.
During D’s father’s lifetime, D refused absolutely to marry P. It was
held that P could sue on the contract, even though D’s father is still
alive.

When does the innocent party have a right to terminate for a breach?

177. There are four situations which entitle the innocent party to elect to treat the
contract as discharged (RDC at [113]):

a. Situation 1: where the contractual term in question clearly and unambiguously


states that, should an event or certain events occur, the innocent party would be
entitled to terminate the contract (RDC at [91]).

b. Situation 2: where the party in breach of the contract, by its words or conduct,
simply renounces the contract inasmuch as it clearly conveys to the innocent
party that it will not perform its contractual obligations at all (RDC at [93]).

c. Situation 3(a) – Condition/Warranty: where the term breached is a condition of


the contract, the innocent party is entitled to terminate the contract (RDC at
[97]). Where the term is not a condition, it is presumed to be an intermediate
term (under Situation 3(b)), unless parties expressly agree that the term is a
warranty (Sports Connection SGCA at [50]).

i. This applies only where it is unclear what the contracting parties’


intentions are with respect to the nature of that particular term (RDC at
[118]).

d. Situation 3(b) – Intermediate term: where the breach of the intermediate term
deprives the innocent party of substantially the whole benefit which it was
intended to obtain from the contract (RDC at [99]).

Situation 1: Express termination clause

178. Under Situation 1, where the contractual term in question clearly and
unambiguously states that, should an event or certain events occur, the innocent party
would be entitled to terminate the contract (RDC at [91]).

121
a. RDC Concrete v Sato Kogyo – In this case, Sato Kogyo was engaged to
construct an MRT station with LTA. RDC was to provide ready-mixed concrete
to Sato Kogyo. LTA subsequently instructed Sato Kogyo to suspend RDC’s
supply of concrete due to an unacceptable amount of cube failure. Subsequently,
RDC was allowed to resume concrete supply, but RDC had failed to supply the
concrete, citing shortages of raw material and plant breakdowns. Sato Kogyo
deducted the cost differential incurred (from buying concrete from alternative
suppliers at higher rates) from outstanding amounts due to RDC. RDC relied on
the force majeure clauses as a defence, suspended its supply of concrete. Sato
Kogyo terminated the contract. Clause 8 provides that “In the event that your
supply is unable to meet LTA’s requirements, or you are unable to continue
your supply, Sato Kogyo (S) Pte Ltd reserves the right to terminate your
contract and retain and use both the retention sum and any outstanding payment
due to you to seek for alternative source of supply. In addition, Sato Kogyo (S)
Pte Ltd also reserves the right to seek from you any direct cost incurred due to
your non-compliance.” The court held that pursuant to cl 8 of the contract, Sato
Kogyo was entitled to terminate the contract as RDC was unable to meet
LTA’s requirements and to provide uninterrupted supply.

b. Fu Yuan Foodstuff – In this case MWS ran a nursing home for destitute persons.
MWS contracted with FYF to provide in-house catering services at the home.
Clause 3.2 of the clause permitted MWS to terminate the contract without notice,
should the appellant breach any item under, inter alia, cl 2.7. Clause 2.7.2 stated
that the appellant “shall comply with all Singapore laws and regulations,
especially with regard to food establishments and employment of staff”. MWS
terminated the contracts as 6 foreign workers had been illegally deployed at the
home (they were employed by another entity instead, and FYF cannot use these
workers). It was held that the case fell squarely within “Situation 1” in RDC, as
cl 3.2 clearly and unambiguously stated that should FYF breach any item under
cl 2.7, MWS would be entitled to terminate the contract. There was no
requirement to read down the clause, as the literal language had accurately
reflected the intentions of the parties – (1) MWS was a charitable organization
and did not want to be implicated in any criminal offence; (2) it is well known
in Singapore that there are strict rules governing the employment of foreigners;

122
and (3) in this context, the literal language accurately reflected the intentions of
the parties, distinguishing Rice v Great Yarmouth BC.

c. Chua Chian Ya – In this case, CCY entered into a music publishing agreement
with M&M for 3 years. CCY extended the agreement for another 3 years. CCY
assigned her rights to her works to M&M, M&M is responsible for promoting
CCY’s works and collecting royalties for CCY’s behalf. M&M was obliged to
provide CCY with a statement of accounts under clause 10, a breach of which
CCY is entitled to terminate under clause 12 (“In the event that the Publisher
fails to account and make payment hereunder or fails to perform any obligations
required hereunder and in the event that such failure is not cured within thirty
(30) days after written notice has been served on the Publisher… then and in
any such events the Writer… may elect to cancel or terminate this
agreement…”). The Court held that cl 12 was a clear example of an express
termination clause under “Situation 1” which entitled CCY to terminate the
agreement.

179. The Court may read down an express termination clause and avoid the literal
interpretation of a termination clause (Rice v Great Yarmouth). However, the Court
may not do so, where the literal language accurately reflected the intentions of the
parties, taking into account of the contractual context (Fu Yuan Foodstuff).

a. Rice – in this case, GYBC and Rice had a contract to maintain and manage the
sports facilities, parks, gardens and playgrounds of the borough for a period of
four years. Rice was awarded the contract pursuant to a tender process. The
termination clause in the contract provided that “if the contractor… commits a
breach of any of it obligations under the Contract… the Council may, without
prejudice to any accrued rights or remedies under the Contract, terminate the
Contractor’s employment under the Contract by notice in writing having
immediate effect”. GYBC invoked this clause and had DSO (an affiliated
organization) to take over from Rice, and Rice sued for wrongful termination.
The Court held that there was a precondition to the termination under this clause,
that a repudiatory breach or an accumulation of breaches that as a whole
could properly be described as repudiatory was required. On the facts, the
clause did not consider which terms were conditions, or which terms were

123
considered to be so important that any breach (whether material or trivial) would
justify termination. Such a literal interpretation on the other hand would be
uncommercial, in the context of a contract lasting for at least 4 years, involving
substantial investment by at least 1 party, and a myriad of obligations of
differing importance and varying frequency.

180. Difficulties with this area of law:

a. It is odd taxonomy. Under RDC, it would seem that this is to be expressly


provided, and has the substantive legal effect of a condition. It would also seem
that it is possible for such a clause to be an innominate term. However, this
would lead to substantial overlap with situation 3(a) and (b). The taxonomy is
odd because we consider the construction of a term to be dealing with what the
term means expressly (see interpretation of terms), as such, there is no reason
to create a separate class of situation 1 altogether.

b. In addition, there are often situations where you may have an express
termination clause without the need for any breach of the contract – consider
force majeure clauses and termination of employment contracts without breach
(i.e. termination by notice). These clauses are arguably a separate class of its
own and should be classed in situation 1 (but without fault).

c. See Sports Connection at [55] – may not have full measure of damages as with
Situation 3(a).

Situation 2: Renunciation

181. Where the party in breach of contract, by its words or conduct, simply renounces
the contract inasmuch as it clearly conveys to the innocent party that it will not perform
its contractual obligations at all, the innocent party is entitled to terminate the contract
(RDC at [93], San International at [20]).

a. Renunciation – Alvin Nicholas Nathan v Raffles Assets SGCA – in this case,


the landlord prematurely ended a 2 year lease agreement at the 1 year mark. It
was held that this amounted to a repudiatory breach of the lease agreement by
renunciation, which gave the tenant an immediate right to terminate the contract
(Note: in this case, all the terms were repudiated).

124
b. No renunciation – Alliance Concrete v Sato Kogyo – In this case, SK claimed
that ACS did not supply the RMC as contracted. The Courts considered the
letters between the parties as evidence of them re-negotiating for a higher price
(by AC) to share in the increased cost of sand. This was not AC evincing an
intention to renounce its obligations altogether, but merely attempting to vary
the contract (at [105]). (Note: this is a renegotiation case).

c. No renunciation – Biofuel v V8 SGCA – In this case, BFI was in the business


of disposing waste wood and woodchips. V8 had an agreement (Biomass
Supply Agreement) whereby BFI was V8’s exclusive disposal service provider
and in exchange V8 would enjoy lower disposal fees. V8 terminated the BSA
alleging that BFI had renounced the contract – BFI wrongfully rejected V8’s
deliveries of waste wood and wood chips, unilaterally raised prices for its
services on several occasions. It was held that BFI’s conduct did not
demonstrate an intention to no longer be bound by the BSA and did not amount
to a repudiation of the BSA. While BFI behaved poorly, these did not amount
to renunciation. BFI’s attempt was negated by the issuance of credit notes. The
reason for BFI’s rejection was that the wood chis were oversized or that it ran
out of storage space, which was not illustrative of an intention to renounce the
BSA.

Situation 3(a): Condition-warranty

Condition and factors

182. Definition of Condition: where the term breached is a condition, the innocent
party may terminate the contract for such a breach. The test is whether the intention of
the parties to the contract was to designate that term as one that is so important that any
breach, regardless of the actual consequences of such a breach, would entitle the
innocent party to terminate the contract (RDC v Sato Kogyo at [97]).

183. In construing the intentions of the parties, the Courts will consider the following
factors (Man Financial v Wong Bark Chuan at [159]-[174]):

a. Classification by statute as an implied condition – where a statute classifies


a specific contractual term as a “condition”, then that term will be a condition,
such as the Sales of Goods Act).

125
i. Arcos v Ronaasen – In this case, buyers wished to purchase wooden
staves for the making of wooden barrels. The agreement with the sellers
stipulated some variation for length and breadth of the staves, but not
for the thickness, which were all specified as half an inch. The buyers
purported to reject the staves. It was held that under the SGA, there was
an implied condition that the goods will correspond with the description,
as such, the buyers were entitled to reject the goods for non-conformity
with the contractual description – harsh because buyers was trying to
escape a bad bargain as there was a fall in timber prices.

ii. Chai Cher Watt v SDL Technologies – In this case, CCW contracted to
buy a drilling and boring machine from SDL. The drilling machine was
specified to be 11m in length, but the machine delivered was 13.5m in
length. It was held that S13 of the SGA (implied condition that the goods
will correspond with the description) was breached.

iii. Note- S15A SGA modifies the contract of sale where if the buyer does
not deal with a consumer, and the breach is slight. The breach is treated
as a breach of a warranty instead.

b. Express stipulation – where the contractual term itself expressly states that it
is a “condition”, then it would be held to be a condition. However, the courts
may read down such a term, if it was not utilized as a term of legal art, but in a
lay sense (L Schuler AG v Wickman Machine Tools).

i. In this case, W was S’s distributor. Cl 7 of the contract stated “It shall
be [a] condition of this agreement that:- (i) Sales shall send its
representatives to visit the six firms whose names are listed in the
Schedule hereto at least once in every week for the purpose of *248
soliciting orders for panel presses; (ii) that the same representative shall
vis- it each firm on each occasion unless there are un- avoidable reasons
preventing the visit being made by that representative in which case the
visit shall be made by an alternate representative and Sales will ensure
that such a visit is always made by the same alternate representative.”
This clause required W to make a total of 1400 visits. W failed in their

126
obligation. The majority of the Court held that the word “condition” was
not used as a term of art/legal sense. The result was unreasonable – the
failure to make even one visit would allow S to terminate the contract.
However, there was a Cl 11, which provided that either party might
determine the agreement if the other party committed a "material
breach" and failed to remedy it within 60 days of being required to do
so in writing. There was a potential clash between Cl 7(b) and Cl 11.

ii. This has been doubted in Man Financial – [166] – [170] and endorsed
the dissenting judgment by Lord Wilberforce, which warned against
rewriting the clear intention of the parties that cl 7 was to be designated
as a condition in a legal sense.

c. Judicial precedent classifying a term as a condition – Where a prior


precedent is available, the Courts will be persuaded to take the same
interpretation with the precedent (The Mihalis Angelos).

i. This case concerned a charterparty, under which the shipowner let the
vessel to the charterer to load cargo in Vietnam, expected ready to load
on 1 July. The vessel was still stuck in Hong Kong on 17 July. The
charterer ‘threw up’ (i.e. terminated) the charter lawfully as the
‘expected readiness to load’ term was considered a condition, rather than
a ‘mere warranty without a right to cancellation’. The court held that an
“expected readiness” clause was a condition on the ground, inter alia,
that the same conclusion had been reached by its own previous decision.

ii. However, in Man Financial, the Court stated that there needs to be an
inquiry as to whether the analysis and reasoning in the prior precedent
passed muster in principle, and the Courts may depart where appropriate.

d. Mercantile transactions – In the context of mercantile transactions, certainty


and predictability is important. As such, courts are likely to classify contractual
terms as conditions in this context, especially where they relate to timing (The
Bunge and The Mihalis Angelos).

127
i. Duration of notice period to load – Condition – Bunge v Tradax – In
this case, B agreed to purchase from T 500 tons of soya bean meal.
Under the terms of the agreement, B was to nominate the vessel and give
notice to T to load (15 days’ notice). B gave late notice to T (10 days’
notice). T terminated the contract. The Court held that the term of giving
notice was a condition. The reason for such a clause was to enable each
party to organise his affairs to meet obligations arising in the future
under the contract and not merely to determine, with the benefit of
hindsight, the appropriate remedy when a breach occurred.

ii. Time of readiness to load – Condition – Mihalis Angelos – In this case,


MA was fixed to sail to Haiphong and load a cargo for delivery in
Europe. The owners stated that the ship was expected to load about 1
July 1965. The charterparty provided that “should the vessel not be
ready to load (whether in berth or not) on or before 20 July 1965,
charterers have the option of cancelling this contract, such option to be
declared, if demanded, at least 48 hours before vessel’s expected arrival
at port of loading”. On 17 July 1965, the ship was still at Hong Kong,
discharging cargo from her previous voyage. It was physically
impossible for the MA to finish discharging and reach Haiphong by 20
July 1965. It was held that the term of expected readiness was a
condition and the charterer was entitled to rely on the breach to terminate
the contract.

iii. Time of delivery – Condition – LED Linear (Asia) v Kristile – In this


case, LED contracted to deliver lighting to Kristile. The contract was
silent as to whether the time of delivery was a condition of the contract.
It was held that time was of the essence for delivery, and the contract
was breached by LED when it failed to deliver the lighting on time.
Krislite was entitled to terminate the contract and refuse delivery.

iv. Time of payment – Not a condition – Ioannis Valilas v Valdet Januzaj


– In this case, there was an agreement between the appellant and
defendant dentists for a facilities contract (appellant was to pay
defendant 50% of his receipts for the right to use the

128
premises/equipment/services of dental nurses of the defendant. One of
the issues was whether the obligation to make payments when they fell
due was a condition – the Court held that this was not a condition as
generally, it is not important to a commercial contract. The requirement
to pay on time was an innominate term.

e. Illustrative cases – Construction of contract

i. Man Financial v Wong Bark Chuan David– In this case, David was the
MD/CEO of Man Financial. D was asked to resign, and executed a
termination agreement with MF, which contained a non-solicitation
clause. The Clause was breached and MF refused to provide
compensation. The issue was whether the clause was a condition – if it
was, MF would not have an obligation to continue its performance and
D would not be able to claim the compensation. The Court held that the
term was a condition, because (1) the clause was integral and was the
consideration D furnished for payment, (2) there was express terms
“save in the event that you breach any of the terms… the sums… above
will be paid to you in full and without deduction…”, (3) parties had
negotiated the ambit and scope so that it was acceptable to them.

ii. Phosagro Asia – In this case, PI was the MD of P. PI was previously the
sole director and shareholder of P, but he sold the shares off to another
company, remaining however as the MD. PI was given wide ranging
powers to run P, did not have to report to anybody when making day-to-
day financial decisions and no corporate governance rules were imposed
on him by the new owners. Cl 3 of the employment contract obliged PI
to faithfully serve the Company in all respects and use his best
endeavours to promote the interests of the company. It was held that this
clause was a condition as from the aforementioned context, given that it
would have been intended by the parties to be of the utmost importance.
There was thus a breach (mismanagement of companies funds) which
entitled P to terminate the contract without notice (or payment in lieu of
notice).

129
iii. PT Banyan Resources v BCBS – In this case, BR and BCBCS had
entered into a joint venture agreement for the construction and
commissioning of a coal briquette processing plant. BR had stopped
supplying coal to the joint venture company (KSC). The Court held that
this was a breach of a condition which entitled BCBCS to terminate the
agreement – the purpose of the joint venture could never have been
achieved if BR ceased the supply of coal to KSC, since it was BR’s coal
that was meant to be treated and upgraded. It was fundamental to the
joint venture.

Definition of a warranty

184. Definition of Warranty: where the term breached is a warranty, the innocent
party may not terminate the contract for such a breach. The test is whether the intention
of the parties to the contract was to designate that term as one that is not so important
that any breach, regardless of the actual consequences of such a breach, would not
entitle the innocent party to terminate the contract (RDC v Sato Kogyo at [98]).

185. Where the term is not a condition, it is presumed to be an intermediate term


(under Situation 3(b)), unless parties expressly agree that the term is a warranty (Sports
Connection SGCA at [50]).

Situation 3(b): Hongkong Fir

186. The breach of an intermediate only confers on the aggrieved party a right to
terminate if such breach deprives the aggrieved party of “substantially the whole benefit
which it was intended he should obtain from the contract” (Hong Kong Fir v Kawasaki
Kisen Kaisha).

a. Seaworthiness clause – not successful – Hongkong Fir – In this case, HKFS


hired out their ship under a 2 year charterparty to KKK, to sail from Liverpool
to Osaka via Panama. A term in the charterparty required the ship to be
seaworthy and to be “in every way fitted for ordinary cargo service”. However,
the crew were both insufficient and incompetent to deal with her old fashioned
machinery. The voyage resulted in the ship’s engines suffering several
breakdowns and the ship was off-hire for a total of 5 weeks. On arrival at Osaka,
a further 15 weeks of repairs were needed. By this time, only 17 months of the

130
charter remained. Once in Osaka, the freight rates happened to fall and KKK
terminated the contract for HKFS’s breach. HKFS responded that KKK were
now in breach themselves. It was held that the clause was neither a condition
nor a warranty, and the breakdowns/delays did not substantially deprive the
charterer of the whole benefit of the contract.

b. Shipment of citrus pulp to be made in good condition – not successful – The


Hansa Nord – In this case, the sellers agreed to sell to the buyers 12000 tons of
citrus pulp pellets for use as animal feed. The contracts specified that the
shipment was to be made in good condition. The buyers paid £100,000. The
pellets were shipped with 1260 tons in 1 hold, and 2053 tonnes in the other. The
market price of those pellets had fallen to some £86,000. Much of the cargo in
hold 1 was damaged, while the goods in the other were in substantially good
condition. The buyers rejected the cargo from both holds and claimed
repayment of the price on the ground that the shipment was not made in good
condition. The sellers rejected the claim. The buyers subsequently bought the
same pellets for about £30,000. The Courts held that the SGA did not compel
the Courts to consider all terms in a sale of goods as a condition or a warranty.
On the facts, this was an innominate term and the breach did not go into the root
of the contract. They were not entitled to reject the goods but were entitled to
damages for the breach.

c. Confidentiality obligation – not successful – Cousins Scott William v RBS – In


this case, P was employed by D as their Chief trader (and subsequently Senior
trader). P requested D to either make him redundant or demote him, which arose
from an altercation between P and his colleague. P continued his request to be
made redundant, which D accepted. Under the redundancy agreement, he was
offered payments, in return to inter alia, observe the obligation of secrecy and
confidentiality. After signing the agreement, P had forwarded some emails from
his workstation to his personal account (confidential information), which was
discovered by D. P was subsequently summarily dismissed. The issue was
whether D was entitled to dismiss P (for the breach of the confidentiality
obligation). It was held that there was a breach, however it did not deprive D
the whole benefit of the agreement. The breach caused no loss to D, as P had

131
deleted the emails and reinforced his confidentiality obligation by executing a
statutory declaration – it did not entitle D to terminate the redundancy
agreement.

d. Generators delayed and less than satisfactory quality – successful – Aero-Gate


Pte Ltd v Engen Marine Engineering – In this case, D was obliged to deliver 10
generators under 2 purchase orders – in PO1, 4 generators by end-janguary 2012,
under PO2, 4 by 1 Nov 2011 and 2 by 1 Jan 2012. The defendant had only
delivered a total of 2 generators on 16 January 2012, but made no further
deliveries (there was a failure to meet the deadlines under PO 2 – several months
worth of delay). P terminated both PO1 and PO2. It was held that there was a
breach of both PO1 and 2, which deprived the plaintiff of substantially the
whole benefit which it was intended P would obtain from the contract.

e. Delay in delivery of hollow core slabs – successful – CAA Technologies v


Newcon Builders – In this case, Newcon contracted with JTC to build a medical
tech hub. CAA was subcontracted to design, produce and install pre-cast
concrete hollow core slabs. CAA failed to deliver the HCS on time contributed
to delay in the main contract works. The HCS were on the “critical path” of
Newcon’s main contract works (i.e. Newcon could not install columns if slabs
were not first installed). This exposed Newcon to payment of liquidated
damages to JTC. It was held that Newcon was entitled to terminate, and there
was substantial deprivation.

Anticipatory breaches

187. Where a defendant’s conduct evinced a clear intention not to perform its
obligations under the contract, this is a breach of the contract, notwithstanding that the
time for its performance had yet to arrive under the contract (STX Mumbai at [51]). The
basis is that this is an actual breach, but notified in advance, and thus would apply to
executed as well as executory contracts (at [63]).

a. Hochster v De la Tour – In this case, DlT employed H to commence work for


3 months from 1 June, but repudiated the contract on 11 May. H could claim
damages immediately (before 1 June).

132
b. Bowdell v Parsons – In this case, P agreed to sell 12 loads of hay to B. After the
delivery of the 1st load, P sold the rest to a third party without B’s consent. It
was held that this was a breach of the contract.

c. Universal Cargo Carriers v Citati – In this case, U chartered a ship to C, who


agreed to provide the cargo and nominate a berth and a shipper before a certain
date. C wanted to perform but had done nothing 3 days before the due date. This
left it so late that performance would inevitably be delayed, and this would
frustrate the commercial purpose of the contract. The court held that C had not
renounced the contract, but its inability to perform entitled U to terminate and
find another charterer.

188. Under STX Mumbai (at [64]-[67]), an anticipatory breach would

a. Not fall under Situation 1;

b. Typically fall under Situation 2;

i. Note: historically, San International considered situation 2 as merely


anticipatory breaches of 3(a) and 3(b).

c. Fall under Situation 3(a), provided that it is clear that the breach will occur.
There must be a serious absence of readiness or willingness to perform on the
part of the defendant. (i.e. must be inevitable – The Bulk Uruguay) – STX
Mumbai at [66].

d. Fall under Situation 3(b), if the court is able to extrapolate or project the effects
of the breach (STX Mumbai at [68]).

i. Chua Chay Lee v Premier Properties – In this case PP had acquired 24


apartments by en bloc for redevelopment. The appellants were prior
owners who had exchanged their apartments for the redeveloped sites.
Each appellant would receive a banker’s guarantee as security for the
developer’s performance. The developer was obliged to hand over the
apartments to the appellants not later than 3 months after the old
apartments were handed over to them. There was a delay in starting the
redevelopment of a year, although the architects considered that the

133
work could be completed on schedule. The appellants sought to
terminate the contract and sue on the performance guarantee. It was held
that time of completion was not a condition in the agreement. The
construction was moving along by the time the owners purported to
terminate, and the delay did not go to the root of the contract.

189. Under STX Mumbai, the doctrine of anticipatory breach would apply to both
executory and executed contracts. In particular, the AB doctrine protects the innocent
party with outstanding obligations, to avoid his wasteful performance. Similarly, it
protects the innocent party who has already provided his performance as well.

Election and limits

Exercise of right and timing

190. A breach which allows the innocent party a right to terminate does not
automatically bring a contract to an end. It provides the claimant a right to elect whether
to terminate or affirm the contract (Geys v Societe Generale, London Branch).

a. The termination must be clear and unequivocal. Mere inactivity will not
normally suffice. It must be communicated to the contract-breaker, but requires
no particular form (Vitol SA v Norelf).

191. The innocent party is not bound to elect at once (Stocznia Gdanska SA v
Latvian Shipping No.2). However, the innocent party must remain willing and able
to perform and runs the risk that (1) the law will treat him as having affirmed the
contract, which may be overtaken by another event which prejudices his right, such as
frustration or his own breach; and (2) the party in breach may resume performance,
thereby ending the right to elect.

a. The length of time the claimant may wait while making up her mind would
depend on the facts of each case, and whether there is urgency (Force India
Formula One v Etihad Airways) In this case, FIF1 owed obligations to its
sponsor. When the team changed owners, the team name was rebranded, and
was held to be a repudiatory breach. The issue was whether the sponsor’s initial
lack of complaint was an affirmation of the contract, which the Court held not
to be so, given that the case concerned a complex medium term relationship

134
which required time for the consequences to become clearer and for the innocent
party to consider his position.

b. Similarly, in Aero-Gate v Engen Marine, the Court held that Aero-gate’s


conduct in treating PO2 as alive was merely a time taken for him to wait and
see what the consequences of the breach are, and not an affirmation of the
contract.

Loss of right to terminate

192. The claimant’s right to terminate a contract may be lost by:

a. Affirmation (delay or positive affirmation).

i. The Kanchenjunga – In this case, the charterers had chartered the


Kanchenjunga , but nominating Kharg Island to load a cargo of crude
oil (which was not a safe port). The owners accepted the charters’
nomination, and thus lost their rights to reject nomination, but only
retained their right to claim damages.

b. Acceptance – See also acceptance of goods under Sales of Goods Act S11(3)
and S35.

c. Waiver (not in syllabus).

Consequences of election

Termination

193. Termination operates prospectively, thereby releasing parties from their future
obligations, but leaving intact rights accrued prior to termination (Photo Production v
Securior Transport). It is possible for some clauses in a contract to survive termination,
such as:

a. Jurisdiction/choice of law clauses;

b. Dispute resolution clauses;

c. Restraint of trade/confidentiality clauses.

135
Affirmation

194. Both parties are obliged to continue performance, but the innocent party has a
right to damages for prior breaches.

195. Once a contract has been affirmed, the affirmation cannot be revoked. However,
the aggrieved party may be able to treat a continuing non-performance as a fresh act of
repudiation, which gives rise to a fresh right to terminate the contract (Johnson v
Agnew).

a. Johnson v Agnew – In this case, the owner of land contracted to sell it to a


purchaser. The purchaser failed to complete the contract. The vendor obtained
an order for specific performance, but the purchaser still did not complete. The
vendor goes back to Court and asked for the order for specific performance to
be dissolved and the contract terminated. The Court held that the vendor was
entitled to terminate the contract and sue for damages.

b. National Skin Center v Eutech Cybernetics – In this case, Eutech was to


complete a software system for NSC. Eutech was in breach at the initial deadline,
but was given the opportunity to continue the contract with the extensions of
time. NSC subsequently gave a notice for the system to be delivered within 3
months, which Eutech failed to deliver. NSC was held to be entitled to terminate
the contract when Eutech was unable to meet the new deadline.

c. See also Safehaven v Springbok, The Fortune Plum for the same principles.

196. The innocent party, if she elects to affirm, must actually perform her obligations
in order to affirm successfully and obtain the contract price (Doherty v Fannigan
Holdings).

a. In this case, D had failed to provide money for the purchase of shares from FHL.
While FHL could sue for specific performance or damages, it could not sue for
the price or serve a statutory demand as it had not transferred the shares to D.

136
Restriction on affirmation – White & Carter

197. The claimant may be barred from affirming the contract if her performance
requires the contract breaker’s cooperation, or that the claimant has no legitimate
interest in performing the contract (White & Carter v McGregor).

a. In this case, M contracted for his garage business to be advertised on rubbish


bins that W&C supplied to the local council for 3 years. M tried to (on the same
day) cancel the contract, but W&C refused. The UKHL held that W&C was
entitled to sue M for the agreed price (by 3:2).

Cooperation

198. Contract breaker’s cooperation – It has been suggested in MP Bilt v Oey


Widarto that a contractor undertaking to repair a house for a lump sum needs
cooperation of the owner who has control over access of the property. The contractor
will need an order of court to force the house owner to give him access, which ordinarily
would be refused as damages would suffice – specific performance would usually not
be awarded.

Legitimate interest

199. Legitimate interest – In The Aquafaith at [23], citing The Dynamic, it was held
that:

a. the burden of proof is on the contract breaker too show that the claimant has no
legitimate interest in performing the contract.

b. It is insufficient to show that the benefit of affirmation to the claimant is small


in comparison with the loss to the contract-breaker.

c. this will only be made out where damages would be adequate and an affirmation
would be wholly unreasonable (The Dynamic).

200. The following are some examples of what amounts/does not amount to a
legitimate interest:

a. Legitimate interest – Difficulty of obtaining damages/potentially speculative


– The Odenfield – In this case, the Court held that the owners were permitted to

137
affirm the contract, because it would be difficult to find other employment for
the ship (the charterer could sub-let the vessel), termination would put them in
breach of other contracts, and damages would be difficult to assess.

b. Legitimate interest – Non-pecuniary interest in performance – Ministry of


Sound v World Online – in this case, the claimant had continued to publicise the
defendant’s services as contracted, despite the defendant’s refusal to pay. The
claimant had a legitimate non-financial interest in proceeding with the contract,
since its name would be associated with World Online’s publicity – at [47].

c. Legal liability to third parties – The Odenfield – In addition to the above facts,
the ship was financed by a loan, which was secured by with the long charter. If
the owners had been barred from affirming the charter, this would put them in
breach of their loan obligations. Given that the loan agreement and assignment
were known to the charterers it is clear that the owners had a legitimate interest
in enforcing the contract.

d. No legitimate interest – Manifestly wasteful – Puerto Buitrago – In this case,


Charterer had chartered a ship for several months and returned the ship before
the end of the hire period due to engine trouble. The charterer however was
bound to carry out the necessary repairs during the charter £2,000,000 – whereas
the ship’s value was only £1,000,000 after full repairs. It was held that the
Owner could not refuse the repudiation – to claim the continuing agreed hire
and to insist on the repair would be economically wasteful.

Defaulting party rights and order of performance

201. The issue here is whether the defaulting party have any rights which it may
claim, notwithstanding the fact that the contract may have been terminated. The
defaulting may have a claim, depending on the relationship between its obligation and
the claimant’s obligation.

Independent obligations

202. Independent obligations – Where the claimant’s reciprocal obligation is


independent of the defaulting party’s performance, the claimant must continue
performance, but the defaulting party will be liable for its own breach.

138
a. Taylor v Webb – In this case, the Court held that a tenant’s covenant to pay rent
is independent of the landlord’s covenant to repair. The tenant cannot withhold
payment, even if the landlord fails to repair.

b. Alliance Concrete v Comfort Resources – In this case, the Court held that AC’s
non-late payment of the sums due and under-ordering of sand from Comfort
was independent of Comfort’s breach subsequently in refusing to supply sand.
Comfort’s obligation to supply sand (after the stipulated date) was not a
condition precedent to AC’s obligation to pay sums due and owing for sand that
had already been delivered at a time prior to the stipulated date.

Dependent obligations

Divisible obligations

203. Where the obligation to do work is divisible, the obligation can be said to consist
of entire stages, such that the completion of each specific stage will entitle the party in
default to the claimant’s rateable obligation (Taylor v Laird).

a. Taylor v Laird – In this case, the defendant had contracted with the plaintiff for
the plaintiff to command a steamer. The plaintiff was inter alia to be paid at a
monthly rate. The plaintiff however only commanded the steamer for a few
months, and subsequently repudiated the contract. The Court held that the
obligation was divisible – the defendant was entitled to claim as per the contract
for the months of service rendered, even though he had failed to complete the
entire voyage.

b. Tong Aik v Eastern Minerals & Trading – In this case, P had sought to claim
from D compensation for their work, labour and materials supplied to D at D’s
manganese mine. The contract stipulated that P should supply D with a
minimum of 5,000 tons of manganese ores, to be paid for $6.50 per ton for the
operations at the mine and $7.50 per ton for transportation. It was held that the
contract was a divisible one – this is because there was also a clause which states
that the contract could not be determined until the first 6 months – i.e. this was
a trial period.

139
Entire obligations

204. A obligation which is entire must be completely performed before the other
party is obliged to perform (Cutter v Powell).

a. Cutter v Powell – In this case, C agreed to work on a ship sailing from Jamaica
to England for 30 guineas, the payment is on completion of the voyage. C died
before completing the journey and the widow was denied recovery of wages for
his part-performance. This case is exceptional – the Court considered that his
rates were high compared to that generally prevailing (8 guineas), which may
had influenced the court into putting the risk of non-completion on to him.

205. Nevertheless, the defaulting party may rely on the following rules to circumvent
the entire obligations rule:

a. Under the doctrine of substantial performance, the innocent party cannot


withhold performance if the defaulting party had substantially performed her
obligations (Hoenig v Isaacs).

i. Success – Hoenig v Isaacs – In this case, P was employed by D to


renovate D’s flat, payment of the balance to be on completion. D refused
to pay the balance on the ground that the work done and furniture
supplied were defective. It was held that P had substantially performed
his obligations, D was liable for the balance, less a deduction for the
damages.

ii. Failed – Bolton v Mahadeva – In this case, P agreed with D to install


central heating for a lump sum. D complained that the work was
defective and refused to pay. The system was defective, and gave off
insufficient/uneven heat in each room. It was held that there was no
substantive performance.

iii. Treitel’s interpretation – even if a builder was under an entire


obligation as to the quantity of work, there is no such obligation as to
its quality. Bolton v Mahadeva could be considered as quantity of work
done, but qualitative defects were major, or work was done so badly
there was no quantitative completion.

140
b. Under unjust enrichment, the party in breach may claim for restitution, if there
was enrichment at his expense and the enrichment was unjust (Sumpter v
Hedges).

i. Part performance may not benefit the innocent party (subjective


devaluation – what is the value of half a hair-cut?). However, the benefit
may be proved if the other party has an option to reject the enrichment,
but voluntarily accepts it. In Sumpter v Hedges, the Court considered
that there was an enrichment for the materials for construction, but no
enrichment for the services of building the property (able to accept the
first, but not the second).

ii. The enrichment may be considered unjust, if there is a total or partial


failure of basis/counter-performance.

141
Chapter 5: Remedies for breach of contract

Introduction

206. Where a contract has been breached, the law offers several remedies to the
aggrieved party. We will examine in some detail these judicial remedies, ie, remedies
ordered by the court at the successful conclusion of a suit for breach of contract. These
are to be distinguished from “self-help remedies”, some of which have already been
considered in the previous chapter.

207. Among judicial remedies, we concentrate on the common law remedy of


“damages”. Besides damages, we shall also consider specific remedies viz. an order to
do what was promised. These consist of the “action on an agreed sum” at common law
and the discretionary remedies of “specific performance” and ‘injunction”.
Additionally, we shall examine the judicial attitude toward the different agreed
remedies i.e. remedies that parties have stipulated in the contract. These include:
liquidated (agreed) damages and deposits.

Damages

Compensatory principle

208. The starting position is that contractual damages serve to compensate the
non-breaching party for the loss caused by the breach.

a. Robinson v Harman – “The rule of the common law is, that where a party
sustains a loss by reason of a breach of contract, he is, so far as money can do
it, to be placed in the same situation, with respect to damages, as if the contract
had been performed”.

i. Note: strictly, this case is about the expectation interest.

b. PH Hydraulics v Airtrust – “We take as our starting point the well-accepted


proposition that ‘the purpose of the law of contract is not to punish wrongdoing
but to satisfy the expectations of the party entitled to performance’… That is
why the general aim of damages for breach of contract is to compensate: the
plaintiff is to be placed, as far as a payment of money allows, in the same

142
position as if the contract had been performed... Besides orthodox compensatory
damages, which are assessed by reference to the plaintiff’s pecuniary loss, there
are other measures by which contractual damages can be assessed, which we
will return to later. But it suffices to note that these other remedial options serve
also to protect the plaintiff’s interest in contractual performance and remain
primarily compensatory in purpose.”

209. Given the primary compensatory rationale, the law of contract:

a. “Generally eschews the concept of punishment” – Turf Club Auto Emporium


v Yeo Boong Hua at [198], “It is clear, in our view, that the arguments against
the award of [punitive damages] far outweigh the arguments in favour of such
an award… In the circumstances, we are of the view that there ought to be a
general rule that punitive damages cannot be awarded for breach of contract” –
PH Hydraulics v Airtrust.

Non-pecuniary losses

210. As a starting position, non-pecuniary losses are not recoverable under contract
law (Addis v Gramaphone)

a. Addis v Gramaphone – In this case, a company had wrongfully dismissed its


manager in a way that was “harsh and humiliating”. It was held that the plaintiff
could recover the damages for loss of salary and commission, but not for the
injury to his feelings caused by the manner of his dismissal.

b. Arul Chandran v Gartshore – In this case, P was the vice president of a club. D
were the members of the club’s general committee. The plaintiff was removed
by the defendants and P sought to have his removal declared unlawful. P
pleaded his cases on the basis of breach of the membership contract between
the parties and sought damages for inter alia, mental distress, humiliation and
defamation (defendants consented judgment). The Court awarded nominal
damages only – in respect of mental distress, this could not be claimed as a
matter of policy; for loss of reputation, this was protected under the tort of
defamation instead, and thus not awarded unless financial loss was proved for

143
contract law; and had the plaintiff been suspended or deprived of his club
membership, the Courts could have given him damages for loss of amenities.

211. However, damages for non-pecuniary losses could be awarded, if it comprises


of loss of satisfaction (such as enjoyment or peace of mind) or distress, and it was an
important object of the contract that the claimant should have satisfaction or should not
suffer distress (Farley v Skinner).

a. Farley v Skinner, P was considering to buy a house in Sussex some 15 mines


from an airport, and engaged the defendant as his surveyor. P specifically asked
D to investigate, in addition to the usual matters, whether the property would be
affected by aircraft noise. D reported that he considered it unlikely that the
property would suffer greatly from the noise. P spent a lot of money to refurbish
and modernize the house. After moving in, P discovered that the property was
substantially affected by the aircraft noise, but decided not to sell. It was held
that damages could be awarded for P’s disappointment of his loss of amenity
(which had no economic value) arising from the ensuring of pleasure, relaxation
or peace of mind. It was sufficient if the provision of the amenity had been a
major or important part of the contract, rather than the sole object. It was
immaterial that D did not guarantee the result but merely undertook to exercise
reasonable care in achieving that result.

b. Kay Swee Pin v SICC – K had her membership suspended by the SICC.
However, it was held in the substantive trial that SICC had breached their rules
of natural justice in failing to give K an opportunity to respond to the allegations
against her and that the club rules did not apply to K at the relevant time. K’s
suspension was held to be invalid. K sought damages for (1) the deprivation of
her rights and privileges as a member, (2) damages for the humiliation,
embarrassment, anguish and mental distress caused by the wrongful suspension
(In this case, notices of her suspension were put out without her being notified
prior), and (3) aggravated, exemplary and punitive damages. It was held that
(1) could be recovered - $32,000; (2) – non-pecuniary damages recoverable
under the principle of Farley v Skinner as the SICC membership is one for the
provision of “mental benefits”; (3) not successful.

144
i. In Kay Swee Pin, it was further suggested that there should not be a
blanket restriction on damages for mental distress, and that the rules on
remoteness would be sufficient as the safeguard against such claims
(“Crumbling edifice”).

Proof of loss

212. The non-breaching party may have inadequate evidence of the loss. If no
substantial loss can be proved, only nominal damages will be recoverable. However,
the courts may make allowances for imperfect evidence where they are satisfied that
substantial losses were suffered. The Court will not demand that the plaintiff prove
with complete certainty the exact amount of damage that he has suffered, but will
take a flexible approach – where precise evidence is obtainable, the Court naturally
expects to have it. Where it is not, the Court must do the best it can (Robertson Quay
v Steen Consultants).

a. In Robertson Quay, RQI engaged the respondents to design and construct


Gallery Hotel. The design were found to be underdesigned. The drawings were
corrected, but the respondents gave the building contractor the uncorrected
version. As a result, there were structural deficiencies in the hotel, delaying the
completion of the project. The respondents admitted liability and the issue was
whether RQI was entitled for (inter alia) its claim for interest, incurred on the
shareholder and bank loans during the period of delay. The Courts held that RQI
only established the quantum of the alleged damage by adducing statements
evidencing the interest, but failed to establish that the damage was as a result
of the delay caused by the respondents. RQI failed to establish a factual link
between the delay in completion and the additional interest. It must show that
the full/partial repayment of the loans would have been made using the income
generated from the Hotel’s operations, thus establishing the necessary link
between the breach of contract and the loss alleged by RQI. RQI must establish
evidence on how repayment of the Loans were to be made upon completion of
the project. In particular, none of the loans were contractually repayable upon
the timely completion of the project. (Note this was because the loans had either
no repayment date, or were much later than the completion date. As such, even

145
if there was no delay, the interest may still have to be paid by the RQI. Need to
show partial repayment etc).

Date of assessment

213. The starting point is that damages are assessed at the date of the breach but if
justice requires it, the court may take into account subsequent events to reduce (or
possibly increase) the damages (The Golden Victory).

a. Note: this is relevant where the market for the subject matter of the contract
fluctuates. The amount of compensation depends on when damages are
assessed. In a rising market, the earlier the time is fixed for assessment, the less
will be awarded. Usually, the loss is measured at the earliest date that the
claimant can be expected to mitigate (at the date of the breach), and the claimant
cannot sit back in the hope of claiming a greater loss at the date of the trial
(Chen-Wishart).

b. Date of breach – Tay Joo Sing v Ku Yu Sang – In this case, TJS and his brother
owned a property as TIC. TS signed a document with KYS expressing that TJS
and his brother had agreed to sell the property to KYS for $470,000. KYS gave
TJS a cheque for $5,000 as part payment of the option. TJS later informed KYS
(4th May 1987) that he did not wish to sell the property, and the part payment
was returned. KYS refused the cheque, tendered the balance of the 10% of the
purchase price + earlier cheque. KYS commenced proceedings against TJS,
suing for specific performance for his undivided half-share, and/or damages for
breach of the agreement, but only after 25 months later. It was held that damages
were to be assessed at the date of breach of contract, in particular given that the
purchaser ought to have but failed to mitigate his loss. In this case, the
agreement was silent on completion, and the date was held to be the date when
the sale and purchase of the property could reasonably be completed (August
1987).

c. Subsequent events considered – The Golden Victory – In this case, the owners
accepted the charterer’s repudiation on 17 December 2011, when the
charterparty was not due to end until 6 December 2005. On March 2003, prior
to the original award of the arbitrator, the Second Gulf War began, and the

146
charter argued that damages should not be awarded for the period beyond that
date since it would have exercised its right to cancel under a War Clause. The
Majority held that to award damages for the period after 20 March 2003 would
be inconsistent with the overriding compensatory principle.

i. Minority: The charterer’s argument that the owners would be over-


compensated if the breach-date rule was adopted is not persuasive.
Contracts are made to be performed, not broken. If the charterers had
promptly honoured their obligation to pay damages, the transaction
would have been well settled before the Second Gulf War became a
reality. The owners were entitled to be compensated for value of what
they had lost on the date it was lost – a charterparty with slightly less
than 4 years to be run. For the interests of certainty, the Court should
uphold this approach.

ii. Majority: Fundamental principle is that damages should compensate


the victim for the loss of his contractual bargain. If the contract would
have been terminated early on the occurrence of a particular event, the
chance of that event happening must be taken into account in the
assessment of damages – he is not entitled to the benefit of more
valuable contractual rights than those he has lost. “Certainty is a
desideratum and a very important one, particularly in commercial
contracts. But it is not a principle and must give way to principle”.

d. Subsequent events considered – Bunge SA v Nidera – In this case, the buyer


terminated a contract for the sale of Russian wheat, due to the seller’s
repudiation. However, the seller would had been entitled to cancel the contract
4 days later when an export ban came into effect. The contract contained a
damages clause providing a scheme for establishing damages on default by
either party, but the UKSC awarded only nominal damages, because (1) account
had to be taken of what would have happened had the defendant not repudiated
and since the export ban remained in force throughout the period the claimant
suffered no loss; and (b) the damages clause did not deal with the effect of
subsequent events which would have resulted in the original contract not being
performed.

147
Measure of damages and interests protected: Expectation, reliance and restitution

Expectation interest

214. The expectation interest refers to the expectation of the non-breaching party that
the contract will be performed, measured in money.

a. Robinson v Harman – “The rule of the common law is, that where a party
sustains a loss by reason of a breach of contract, he is, so far as money can do
it, to be placed in the same situation, with respect to damages, as if the contract
had been performed”.

b. Typically, this refers to the profit/gain the contract would have produced.

3 typical measures: Cost of cure, diminution in value, loss of amenity

215. Expectation damages are typically calculated in 3 ways, the cost of cure, the
diminution in value and the loss of amenity (Ruxley Electronics v Forsyth, Yap Boon
Keng Sonny v Pacific Prince).

a. Diminution of value: refers to the market value of the performance that the
defendant promised, minus that actually given.

b. Cost of cure: refers to the cost of buying substitute performance from another
party, including undoing any defective performance.

c. Loss of amenity: the non-pecuniary loss to the claimant.

216. Generally, the cost of cure measure tends to be higher than the other two value
measures. In such a situation, the cost of cure will not be awarded, unless the claimant
has incurred the cost of cure or intends to do so and it was, or will be reasonable for
the claimant to incur the cost of cure (Ruxley Electronics, Yap Boon Keng Sonny).

a. Ruxley Electronics – In this case, R agreed to build a swimming pool for F for
£17,797. The pool actually built was shallower than the specified depth, but it
was still safe for diving and F’s property was no less valuable for the breach. F
claimed £21,560 as the cost of cure, to demolish and rebuild the pool to the
specified depth. The UKHL held that the cost of cure was unreasonable and
irrecoverable by taking into account (a) the claimant’s purpose(s) in

148
contracting; (b) whether the claimant has cured or intends to cure the breach;
and (c) the proportionality between he cost of cure, the contract price, the
benefit already received by the claimant and the benefit he would receive from
cure. The UKHL held that the purpose was that F wanted the comfort of a deep
pool (it was 20% shallower – 45.72cm) – there was a personal, subjective and
non-monetary gain; however, F had no genuine intention to cure the defect – R
had already rebuilt the pool once to rectify other defects, but F still refused to
pay, alleging various minor deficiencies. F’s statements made no mention of the
pol’s debt until 5 years later; Given the benefit F had already received, the cost
of cure was wholly disproportionate to the disadvantage F suffered. F still had
a useable and safe pool.

b. Yap Boon Keng Sonny – YBK contracted with PPI to build a 3 story semi-
detached house. PPI however did not fully performed the contract, and YBK
sued for defective works and undersized bedrooms. With respect to the
undersized rooms, the homeowner sought to recover the cost of cure. The Court
held that the purpose of the contract was to provide a house suitable for the
occupation of the homeowner and his family. The objective was achieved even
though 3 of the bedrooms were somewhat smaller. It would have been excessive
to spend substantial sums to reconstruct the bedrooms. The Court awarded a
loss of amenity sustained by the shortfall in the sizes of the bedroom.

Loss of chance

217. Where the breach deprives the claimant of an opportunity to make a gain, but it
is uncertain that the claimant would actually have made a gain, the Courts may award
damages for the loss of chance. The claimant must show that (1) the defendant’s acts
have caused the loss of chance and (2) the chance lost was a real or substantial one
(MK Distripark v Pedder, Asia Hotel Investments v Starwood). Such losses are confined
to cases where the causation of loss rests on the hypothetical actions of a third party –
if it rests on the hypothetical actions of the claimant, the claimant must show on a
balance of probabilities that it would had used the chance.

a. Awarded – Chaplin v Hicks – In this case, Chaplin entered into a beauty contest
organized by Hicks. The claimant got to the final 50 but did not receive her

149
invitation for an interview (where 12 would be selected for employment), until
it was too late to attend. Chaplin brought an action based on her loss of chance
of gaining employment and was awarded £100 for her loss of chance (by jury
assessment). Hicks appealed. The EWCA held that Chaplin was entitled to
recover for her loss of chance of gaining employment, and did not need to
demonstrate that she would have been successful at interview.

b. Too speculative – McRae v Commonwealth Disposals Commission – In this


case, CDC contracted with McRae to salvage an oil tanker lying on Jourmand
Reef. It later turned out that there was neither a tanker nor the existence of
Jourmand Reef. In this case, the parties claimed for reliance losses, as the loss
of chance was too speculative.

c. Awarded – MK Distripark – In this case, Mapletree held a JTC lease. Mapletree


leased it to MKD, which then further sublet the premises to Pedder. The 49
month sublease was to be renewed annually upon obtaining JTC approval.
Pedder was obliged to continue with the sublease if the new terms were as
favourable than the original terms of the sublease. MKD negotiated with
Mapletree to novate the MKD lease to Pedder, thereby dispensing the need to
get JTC approval year after year. However, Pedder refused to the novation given
that the terms were more onerous. MKD did not apply to JTC for approval to
renew the sublease as well. MKD sued Pedder for repudiation of the sublease
(arguing that there was an obligation to accept the novation), while Pedder sued
for the loss of chance to continue the sublease. The court held that Pedder was
not required to accept the novation (as the terms were less favourable). The
Court held that there was a real chance that JTC approval would be given, due
to JTC’s subletting policy. Although there was a lack of evidence of the
percentage probability of obtaining JTC approval for the continued sublease,
this was not a bar – court specified 60% chance (the benefit was clear – true
rental value – contractual value).

d. Awarded – Asia Hotel v Starwood – In this case, AHI wanted to invest in Grand
Pacific Hotel. In doing so, AHI would need to (1) acquire LS’s majority shares
in PSD (PSD owned the hotel), (2) enter into arrangements with financial
institutions for a loan, (3) secure a management contract with a hotel operator,

150
(4) obtain Pongphan’s waiver of his right of first refusal (to acquire LS’s shares).
AHI entered into a contract with Starwood for a non-circumvention agreement
(i.e. Starwood could not solicit any source introduced by the other party, or enter
into any agreement with such source for 12 months). AHI however failed to
conclude the deal before its MOU with LS expired. Starwood later collaborated
with Narulas to proceed with the acquisition, in breach of the non-
circumvention agreement. AHI sued Starwood (a reliable hotel operator) for the
loss of chance to acquire the LS shares.

i. Majority – held that (1) AHI’s lack of progress in its negotiations with
LS and the FIs were not important – the most critical factor was in
acquiring an international operator with a 5-star brand (starwood), and
thus was not anxious when LS refused to extend the MOU/Narulas was
a competitor – note if AHI had actively pursued, it would only drive the
price up in competition with Narulas; (2) Starwood’s action caused the
loss of chance by assisting Narula.

ii. Minority – surrounding facts and objective evidence did not show that
the AHI have the intention to recommence negotiations with LS or the
FIs which were necessary to proceed with the deal. The Majority was
wrong in holding that the appellant was entitled to stay by the sidelines
and wait till the deal with the Narulas fell through.

Reliance interest

218. There is considerable confusion about the role of the so-called “reliance
damages”. There is one overall aim of compensatory damages (to protect the
expectation or performance interest) – reliance damages are merely a method of
achieving that aim (Restatement, Andrew Burrows at pg 123). A claimant, instead of
directly seeking its lost profits, can claim its reliance loss. However, the claimant cannot
claim reliance damages to escape from what the defendant proves to be a bad bargain
(C&P Haulage v Middleton, Turf Club Auto at [124]). The assumption here is that the
expectation would at least equal the expenditure (and is a proxy method of assessment
of the expectation interest).

151
a. McRae v Commonwealth Disposals Commission – In this case, CDC contracted
with McRae to salvage an oil tanker lying on Jourmand Reef. It later turned out
that there was neither a tanker nor the existence of Jourmand Reef. In this case,
the parties claimed for reliance losses, as the loss of chance was too speculative.
The Court awarded McRae the price paid it had earlier to CDC (for the
opportunity to salvage), and the wasted expenditure McRae had incurred in
attempting to find and salvage the ship.

b. C&P Haulage v Middleton – In this case, M (an engineer) was granted a


contractual licence to occupy premises from C&P (plant hire business). M
expended money in making the premises suitable for the purposes of his work,
even though it was expressly provided that fixtures put by him were not to be
removed at the expiry of the licence. M was subsequently unlawfully ejected
from the premises by C&P after a disagreement. As a temporary measure, M
had obtained permission from the local authority to use his own home for his
work, which he did until well after the 6 months’ term would have expired (from
the original license). The appellant claimed against the licensor for the cost of
improvements effected by him in the premises. The Court held that the
appellant had not suffered any loss as a result of the licensor’s breach as he
was no worse off than if the contract had been fully performed (he did not
incur any expenses such as alternatively renting another place). He was not
entitled to claim for the reliance interest, as that would compensate him at
C&P’s expense for the bad bargain, only nominal damages were awarded.

c. Turf Club at [127] – “The crucial point to note is that both measures of damages
are compensatory and based on the plaintiff’s loss. Indeed, as observed by the
English High Court in Omak Maritime Ltd v Mamola Challenger Shipping Co
[2010] EWHC 2026 (Comm) (“The Mamola Challenger”), the two measures
are not awarded on a different juridical basis, and reliance loss may be
characterised as an application of the compensatory principle laid down in
Robinson, premised on the assumption that the plaintiff would have, at the very
least, recouped his expenditure had the contract been performed (at [42]–[57]).
This analysis explains why, although the plaintiff generally has an unfettered
choice to claim either expectation loss or reliance loss, he will not be permitted

152
to claim reliance loss where he has made a bad bargain (ie, where the reliance
loss exceeds the expectation loss).”

d. See Smile Inc Dental v OP3 SGHC at [52]-[55] – It is possible to claim for both
the expectation and reliance loss, depending on how it is particularized. For
example, if a claim for expectation loss such as loss of profits is made on a net
basis (e.g. does not take into account of expenses and costs) then the loss of
profits is compatible with the reliance losses claim (expenses and costs). The
sum total puts the non-breaching party in a position as if the contract had been
performed. However, if the claim is made on a gross basis (e.g. really the
income stream), then a claim for reliance will lead to double compensation.

219. Pre-contractual expenses may be recovered, so long as it was incurred in


anticipation of the contract (Anglia Television v Reed).

a. In Anglia Television v Reed, R contracted to star in A’s film but repudiated the
contract at the last moment. A was unable to find a replacement and abandoned
the project. It was impossible to assess how profitable the film would have been
if R had performed. But A was awarded its expenses, even though these were
incurred before the contract was made, since it was foreseeable that they could
be wasted on breach.

Loss of chance to bargain, restitution interest

220. Wrotham Park damages is a recognized head of contractual damages, which


aims to compensate the plaintiff for the loss of the performance interest. They can be
awarded where: (1) orthodox compensatory damages and specific relief are
unavailable; (2) there was a breach of a negative covenant; (3) the case is not one where
it would be irrational or totally unrealistic to expect the parties to bargain for the release
of the relevant covenant, even on a hypothetical basis (Turf Club Auto v Yeo Boong
Hua).

a. Orthodox compensatory damages and specific relief: expectation/reliance loss,


specific relief. Seems to also require no non-pecuniary loss claim at [220]. Mere
difficulty of assessing damages is not sufficient – citing Robertson Quay.

153
b. Breach of negative covenant: usually most acute in the context of negative
covenants, as usually no financial loss is suffered by the plaintiff. Traditional
measures are usually irrelevant. If it is a positive obligation, plaintiff can usually
obtain substitute performance – however, does not rule out the possibility for
positive obligations, in particular variation of contract situations.

c. Hypothetical bargain: will not apply if irrational/totally unrealistic to negotiate.


AG v Blake is one situation where to do so would be legally impermissible.
However, where one or both parties would in practice have refused to make a
deal, this is to be ignored (mere unwillingness is not sufficient) (at [236]).

221. The Wrotham Park damages should be measured by such a sum of money as
might reasonably have been demanded in exchange for relaxing the covenant, and is
determined by a hypothetical bargain. The quantum of the breaching party’s gain itself
is only relevant as a matter of evidence in assessing the compensation, but other metrics
beyond the gain may be considered (Turf Club Auto at [247]).

a. Wrotham Park – Parkside built 55 houses on its own land in breach of a


restrictive covenant with Wrotham Park Estate in Wrotham Park, Hertfordshire,
despite the latter's objections. Wrotham Park sued for breach of the covenant.
The Court awarded 5% of the developer’s anticipated profit as compensation.

b. Turf Club – In this case, the SAA group and the Respondents held shares in two
companies incorporated pursuant to a joint venture. SLA leased a site, which
was to be developed and the JV Companies would (as sub-tenants) grant sub-
sub tenancies to the ultimate tenants. The two group fell into disputes, but
reached a settlement, recorded as a consent order by the SGHC (in another trial).
The respondents succeeded in showing that there was a repudiatory breach of
the consent order (SAA had appropriated the SLA headlease for itself, interfered
with the valuation, did not preserve the status quo), and the issue was what was
the remedy that should be imposed (against the appellants). The SGCA declined
to award Wrotham Park damages – the respondents could be awarded orthodox
compensatory damages, assessed by reference to their value of their
shareholding in the JV Companies at the time of the repudiatory breaches, with
a premium of 15% to accurately reflect their true expectation losses. It was also

154
irrational and totally unrealistic to expect the parties to enter into a bargain for
the release of SAA’s obligation to refrain from upsetting the status quo, so that
the bidding exercise (i.e. Respondents or SAA group would buyout each other
depending who was the higher bidder) could proceed, even on a hypothetical
basis – it would render the purpose of the Consent Order defeated, the
hypothetical negotiation was already provided under the Bidding Exercise –
parties would not have to enter into such a negotiation on the eve of the Bidding
Exercise.

222. Exceptionally, it may be possible for the Courts to disgorge all of the promisor’s
gains (AG v Blake). However, the primary difficulty in recognising this head of damage
was the uncertainty of the legal criteria to be applied in awarding such damages. The
concept of “legitimate interest in preventing the defendant’s profit making activity”
was general and even vague (Turf Club Auto).

a. In AG v Blake, Blake was an agent of the UK government, but passed secret


information to agents of the Soviet Union. He was convicted of offences under
the Official Secrets Act, but escaped from prison and fled to Moscow. He
published a book (which contained information which was no longer
confidential, but was still an offence under the Official Secrets Act). The breach
however caused no loss to the Crown and after the publication the remedy of
injunction was no longer available. The Court held by a majority that the Court
could award an account of profits, thereby stripping Blake of his gains.

Limiting damages

Causation, contributory negligence

223. The damages claimed must have a factual and legal link with the breach. A
factual link may be broken by external events such as natural disasters or the conduct
of a third party/the non-breaching party.

224. Where the conduct of the non-breaching party contributes to the loss but does
not break the causation chain, we speak of contributory negligence. Contributory
negligence in a contractual context was historically ignored while in tort it operated as
a complete defence to a claim. The Contributory Negligence and Personal Injuries Act
(Cap 54, 2002 Rev Ed) provides that contributory negligence would be available as a

155
defence in contract where the defendant’s liability in contract was the same as his or
her liability in the tort of negligence independently of the existence of any contract (Jet
Holding v Cooper Cameron, following Vesta v Butcher).

a. Jet Holding v Cooper – In this case, Stork (2nd defendant) was to dismantle and
inspect the unused parts from the refurbishment of the vessel’s slip joint (Stork
was subcontractor of Cooper). Stork did not conduct a dimensional inspection
(was in breach of duty). Cameron sought to obtain an indemnity from Stork for
its liability to the plaintiffs (it was also sued jointly). Stork claimed that
Cameron was contributorily negligent – it had failed to furnish Stork with the
dimensional drawings. The Court implied a term in law that Cameron owed
Stock a duty to take reasonable care in contract, that the defence of contributory
negligence was available as a defence in contract, given that Stork’s liability in
contract was the same as his liability in tort independently of any contract (to
Cameron). Cameron’s contributory negligence could be pleaded against it.

Remoteness of damage

225. The doctrine of remoteness is imposed by law to protect the contract breaker
from infinite damages. The doctrine aims to strike a balance between full reparation
for the loss suffered by an innocent victim of another’s culpable conduct, and the
excessive burden that would be imposed on human activity if a wrongdoer were to be
held to answer for all consequences of his default (Robertson Quay at [70]). The
doctrine also aims to distinguish between the rules of remoteness in contract and those
in tort (Robertson Quay at [71]).

a. There are two limitations on the extent of a contract breaker’s liability for
damages, the first being the parties’ express agreement to allocate risks of
certain losses in their contract by exclusion or limitation provisions (note:
internal) and the second was where the law imposed limits on the extent of the
contract breaker’s liability (note: remoteness) Out of the Box at ([12],[13]). This
is because parties do not specifically address their minds to the question of
damages or more generally of remedies at the time they enter into the contract.

226. The party in breach will be liable for damages arising naturally from such
breach of contract itself, or may reasonably be supposed to have been in the

156
contemplation of both parties, at the time they made the contract, as the probable result
of the breach of it (Hadley v Baxendale).

a. Under the first limb, there is no requirement to prove that the breaching party
knew about the events. Such knowledge is imputed to him. For special damages
under the second limb, it must be shown that the breaching party had actual
knowledge of the special circumstances and facts (Victoria Laundry, Robertson
Quay).

b. The loss foreseeable must be of a serious possibility, and not merely a slight
possibility (as in tort) (Heron II).

c. Under the Second Limb, the Courts will consider the circumstances which the
special facts were brought home to the defendant and in light of that knowledge
and circumstances, whether the damages in question would be foreseeable as a
not unlikely consequence that he should be liable for (Out of the Box v Wanin).

i. The court will consider the extent to which such knowledge should be
taken into account when assessing the defendant’s liability (Out of the
Box v Wanin at [21]).

227. Illustrative cases

a. Hadley v Baxendale – In this case, a shaft in the claimants’ mill broke and had
to be sent to the makers at Greenwich to serve as a pattern for the production of
a new one. The defendants agreed to carry the shaft to Greenwich but, as a result
of their breach of contract, its delivery was delayed so that there was a stoppage
of several days at the mill. The claimants claimed damages of £300 in respect
of their loss of profits during this period. The Court held that the stoppage was
not the “natural” consequence of the delay. It was not contemplated by the
carrier that delay in delivering the shaft would keep the mill idle. The claimants
might have had a spare shaft, or been able to get one. The stoppage was not
contemplated by both parties (only the claimant) at the time of contracting –
the defendants were not told that any delay by them would keep the mill idle.
If they had been told of this, they might have attempted to limit their liability
(i.e. not limb 1, nor limb 2).

157
b. Victoria Laundry v Newman Industries – In this case, D sold a boiler to buyers
who, as D knew, wanted it for immediate use in their laundry business. The
boiler was delivered some 5 months after the agreed date, so the buyers suffered
loss of profits. The Court held that the defendants knew that the buyers wanted
the boiler for immediate use in their business, and would be liable for loss of
profits that would ordinarily result from such use (limb 1). They were not liable
for loss of exceptionally lucrative government contracts, which the buyers
would have been able to make if they had received the boiler in time. D did not
know about these contracts and could not reasonably have foreseen such a loss.

c. Heron II – In this case, ship was chartered to carry sugar from Constanza to
Basrah. At the time of contracting, the charterer intended to sell the sugar as
soon as it reached Basrah. The shipowner did not actually know this, but he
knew that there was a market for sugar at Basrah – if he had thought about the
matter he must have realized at least it was not unlikely that the sugar would be
sold in the market at market price on arrival. The shipowner was in breach of
contract, and reached Basrah nine days late. The price fell significantly. The
UKHL rejected the view that the test of remoteness in contract was
“reasonable foreseeability” (i.e. tort), the loss must be a serious possibility, and
not merely easily foreseeable/not unlikely. The Court held that the difference in
price was recoverable as there was a serious possibility that the value would
decline.

d. Robertson Quay –In Robertson Quay, RQI engaged the respondents to design
and construct Gallery Hotel. The design were found to be underdesigned. The
drawings were corrected, but the respondents gave the building contractor the
uncorrected version. As a result, there were structural deficiencies in the hotel,
delaying the completion of the project. The respondents admitted liability and
the issue was whether RQI was entitled for (inter alia) its claim for interest,
incurred on the shareholder and bank loans during the period of delay. It was
held that the additional interest was not too remote to be recoverable under the
first limb of Hadley. Third party financing was inevitable for large commercial
construction contracts, which parties to the contract, as reasonable people, must

158
be imputed with the knowledge that a delay would give rise to additional
financing costs.

e. MFM v Fish & Co – In this case, Fish & Co owned a chain of restraurants. F&C
sued the 2nd appellant (a former employee) for breaching non-competition
obligations. The parties settled and recorded a settlement deed, with MFM
included as a party. F&C alleged that MFM (1 st appellant) and the 2nd appellant
had breached the settlement deed (MFM dishes/advertising similar to F&C).
The parties entered consent judgment with damages to be assessed. F&C
claimed damages for the period where the breaches occurred, and where the
period after the breaches have ceased but the effects continued to result in losses
(sales figures). The Court held that the nature of the loss fell within the first limb
in Hadley.

f. Out of the Box v Wanin – In this case, OOTB engaged WI to manufacture a new
sports drink called “18 for life”. The contract was a routine contract for the
supply of modest quantities of a generic sports drink. Unknown to WI, OOTB’s
plan was to thrust 18 into popular demand through its own advertising genius.
Although virtually nothing was spent on developing the drink itself, OOTB had
incurred an outlay in the region of $779,812.30) on advertising and promoting
18. WI later breached the contract as a shipment of 18 changed color and was
contaminated with insects. The 18 brand was damaged beyond repair. OOTB
sued WI for reliance damages – it was held that the heads of damages were too
remote. The strategy meant that OOTB was exposed to risks which were
different from the average beverage distributor – the particular facts include the
scale of OOTB’s ambitions, the approach towards realizing these ambitions
through advertising and promotion. None of these facts were brought home to
WI – WI would have approached the contract on the footing simply that it was
a contract to manufacture a generic sports drink, which would have brought WI
a modest sum of at least $12,360 (i.e. the profit of WI) – WI could not have
contemplated that on a contract on this sort, it would be liable for such open-
ended losses as were incurred by OOTB.

159
Mitigation

228. The aggrieved party must take all reasonable steps to mitigate the loss
consequent on the defaulting party’s breach, and cannot recover damages for any loss
which it could have avoided but failed to avoid due to its own unreasonable action or
inaction (British Westinghouse Electric, cited in the Asia Star). The burden of proving
that the aggrieved party has failed to fulfil its duty to mitigate falls on the defaulting
party. The aggrieved party may recover expenses reasonably incurred in the course of
taking mitigation measures (the Asia Star).

a. There is no single persuasive explanation for the duty to mitigate. Economic


efficiency is one factor in this doctrine.

b. The underlying policy is that the non-breaching party should be proactive in


reducing its loss and not expect the party in breach to fully indemnify it.

c. Unfairness to the non-breaching party is reduced, by taking into account the


subjective circumstances. The non-breaching party need not expose itself to
moral or financial risk.

d. Inaction may be worse than an unreasonable action (a factor only).

229. A benefit received by a claimant could only be brought into account in assessing
the damages payable for a defendant’s breach of contract, if the benefit was caused
either by the breach, or by a successful act of mitigation (Globalia Business).

a. In this case, there was an early return of a ship in 2007 in breach of a


charterparty. The owners claimed for their lost profit on the charter hire. They
had also sold the ship for USD 23.7m. The Charterers argued that if they had
returned the ship only in 2009, the value of the ship would have been worth
USD 7m. This increase in capital account was argued to be brought into account.
The UKSC held that the fall in value was irrelevant – the breach did not make
it necessary to sell the vessel at all. It might at most be the occasion for the sale
and not its legal cause – indeed, the ship could have been sold during the term
of the charterparty. As such, the owners succeeded on the full claim of their
lost profit.

160
230. Illustrative case

a. The Asia Star – In this case, the shipowners were obliged to present the vessel
at the nominated ports for loading, but failed to do so at the stipulated time. The
charterer rejected the vessel and no cargo was loaded (it was found that the
vessel was unsuitable to receive the cargo also). The Charterer failed to engage
an available alternative vessel (pure inaction). The Court held that the Charterer
had acted unreasonably: (1) the cost of chartering was significantly less than the
profits it would have made from the sale of the palm oil, (2) there was no
evidence that the charterer was impecunious, or that the cost was financially
prohibitive, (3) not established that dead freight was a concern, (4) the charterer
should have communicated with the defaulting party on the various mitigation
measures available and ask if the defaulting party is willing to bear/share the
additional costs.

231. However, the doctrine of mitigation does not apply to the election by the
aggrieved party to affirm a contract (MP Bilt v Oey Widarto). As such, where the
contract has been affirmed, the aggrieved party may sue for the debt (action on agreed
sum) without having to mitigate his losses.

Agreed remedies

Liquidated damages

232. A liquidated damages clause may be unenforceable as a penalty, if the sum is


not a genuine pre-estimate of the claimant’s loss from the breach (Dunlop Pneumatic
Tyre v New Garage and Motor, Xia Zhengyan SGCA).

a. It will be a penalty if the sum stipulated for is extravagant and unconscionable


in amount in comparison with the greatest loss that could conceivably be proved
to have followed from the breach.

b. It will be a penalty if the breach consists only in not paying a sum of money, ad
the sum stipulated is a sum greater than the sum which ought to have been paid
(apart from interest).

161
c. There is a presumption that it is a penalty when a single lump sum is made
payable by way of compensation, on the occurrence of one or more or all of
several events, some of which may occasion serious and others but trifling
damage.

d. It is no obstacle to the sum stipulated being a genuine pre-estimate of damage,


that the consequences of the breach are such as to make the precise pre-
estimation almost an impossibility.

233. In Cavendish Square/ParkingEye joint appeals, the UKSC recently


reconsidered the law on penalties and held that even though a stipulated term is not a
genuine pre-estimate of loss, it is not a penalty if it protects a legitimate interest of the
claimant (in the performance of the contract) and is not out of all proportion in doing
so. The traditional focus on (non-excessive) compensation is only one of the legitimate
interests that the claimant may protect (at [152]). The test is whether the provision is a
secondary obligation which imposes a detriment on the contract-breaker out of all
proportion to any legitimate interest of the innocent party in the enforcement of the
primary obligation. The doctrine of penalties does not apply to primary obligations and
the Courts will take a substance over form approach.

a. Cavendish Square v Makessi – In this case, M was to sell shares to CS by


instalments. M was not allowed to compete with CS after the sale but this was
breached. CS was entitled to decline further payment and M’s remaining shares
could be bought at a lower price (excluding goodwill).

i. Neuberger & Sumption considered it to be a price adjustment clause and


not a secondary provision (at [74]), but also considered that there was a
legitimate interest for the clause (at [75]).

ii. Lord Mance considered that Cavendish had a legitimate interest – this
assumed that the provisions were secondary obligations, but equivocally,
acknowledged that the clauses redefined the parties “primary
relationship” at [183].

162
iii. Lord Hodge expressly construed cl 5.6 as a secondary obligation. With
respect for 5.1, the analysis was a little more obscure, and assumed it
was a secondary obligation (at [271]-[278]).

iv. Lord Toulson stated he agreed with Hodge and Mance (without
appreciating the distinctions).

v. Lord Clarke agreed with the reasoning of Neuberger, Sumption, Mance


and Hodge (confusingly).

vi. Unclear if there was a clear majority on the nature of the obligations
under the clauses.

vii. However, all agreed that there was a legitimate interest.

b. ParkingEye v Beavis – In this case, the carpark operator charged £85 for
overstaying in the carpark (maximum stay was 2 hours). The Court held
unanimously that the clause was a secondary obligation, but not a penalty.

i. ParkingEye had a legitimate interest in charging for overstays. Their


interest was in the efficient use of the parking spaces for thebenefit of
other users of the shopping centre, which could only be served by
deterring drivers from occupying parking spaces for long periods of
time. The charge was no greater than was necessary to achieve that result.

234. The rule against penalties does not apply to primary obligations. The court will
assess as a matter of substance, (a) the overall context in which the bargain in the clause
was struck; (b) reasons why the parties agreed to include the clause; and (c) whether
the clause was entered into and contemplated as part of the parties' primary obligations
under the contract to secure some independent commercial purpose, or whether it was
to hold the affected party in terrorem in order to secure his compliance with his primary
obligations (Leiman Ricardo v Noble Resources SGCA at [101]).

a. L was employed by NRL. When L resigned, both parties agreed on L’s


departure terms ("Settlement Agreement"), as well as that L would provide
advisory services to NRL ("Advisory Agreement"). Cl 3(a) of the Settlement
Agreement stated that he was entitled to receive payments and benefits in cl 3

163
and the payment schedule to the Settlement Agreement if he complied with the
non-competition and confidentiality obligations under Employment Agreement
and Settlement Agreement. cl 3(b) provided for remuneration under the
Advisory Agreement. Cl 3(c) provided certain share options to L. Cl 3(d)
awarded certain shares to L, that would vest on specific dates. Cll 3(c) and (d)
were subject to the condition that L is not to act to the detriment of NRO. NRL
decided that he had acted to their detriment and denied him of his share options
and certain payments.

b. The SGCA held that Clause 3(a) was a penalty clause. Looking at the rights
under clauses 3(b) (Advisory Agreement) and (e) (entitlement to 2011 bonus),
they were based on considerations independent of L's continued compliance
with his non-competition and confidentiality obligations (i.e. no link). It is
unclear what independent commercial purpose would be served by requiring L
to continue to comply with the non-competition and confidentiality obligations
to be entitled to those rights. However, there was an independent commercial
purpose in extracting from L an agreement to subject his rights under clauses
3(c)/(d) to his being a "good leaver", and the parties had specifically provided
for a "good leaver" condition under these clauses. Clause 3(a) imposed an
additional hurdle on L, subjecting all his rights under cl3 to the additional
condition that he not breach his contractual obligations of non-competition and
confidentiality, and Cl 3(a) was a secondary obligation in substance (though not
in form).

c. As a secondary obligation, clause 3(a) was unenforceable regardless of whether


Dunlop or Cavendish was applied. It was not a genuine pre-estimate of damages,
as it disentitled L from receiving fixed benefits regardless of the nature/extent
of his breach of the non-compete and confidentiality obligations. Cl 3(a) also
failed Cavendish as it was unclear what legitimate interest NRL had in
upholding this clause, beyond punishing L if he breached his obligations of non-
competition and confidentiality.

d. Cl 3(c) and (d) were not penalty clauses. Cl3(c), L gave up unqualified ability
to exercise some vested rights, in return for an extension of time to exercise
those rights and the grant of more rights that he would not otherwise have been

164
entitled to. The consummation of those enhanced rights was subject to the
condition that he not act to Noble’s detriment. This was a condition mutually
arrived at by the parties. In short, the parties agreed to clause 3(c) as part of their
primary obligations, so that Leiman could exchange one set of entitlements for
another in return for being a “good leaver”. Moreover, this did not impose on
Leiman any secondary obligation to pay damages to or compensate Noble for
any breach of his contractual obligations. Clause 3(d) similarly set out a fresh
primary obligation on Noble to vest the Shares in Leiman on the condition that
he not act to Noble’s detriment, as determined by the R&O Committee in the
event of a dispute. As such, clauses 3(c) and 3(d) superseded the original terms
of the grant of the Share Options and the Shares, as part of a fresh bargain that
Leiman struck with Noble, and imposed fresh primary obligations on Noble to
honour Leiman’s enhanced rights in respect of the Share Options and the Shares.

235. Illustrative cases in Singapore

a. Primary obligations, not penalty – iTronics v Tan Swee Leon – In this case, the
parties had entered into convertible loan agreements, under which the plaintiffs
agreed to extend loans to the defendant to assist the latter’s bid to list his
company. In return, the plaintiffs were granted an option of converting the value
of the loan to shares in the Company after it was listed. In the event that the
listing did not take place by the stipulated date, the defendant was to repay the
principal loans with certain compensation sums. The listing did not take place
and the plaintiffs sought to recover the sums. Applying Cavendish SGHC held
that the penalty rule did not apply – it did not impose an obligation on the
defendant to procure the company’s listing. The obligation to pay the sums was
a conditional primary obligation which crystallised on the occurrence of an
event – the failure of the Company to list by the stipulated date. Even if the
penalty rule applied (Dunlop), the sums were not so extravagant or exorbitant
– 10% interest rate. The agreements were thoroughly negotiated with the advice
of solicitors.

b. Default interest not penalty – CIFG v Polimet – In this case, the defendants
sought to argue that a default interest of 2% per month clause was a penalty.
The Court held that the clause was not a penalty – the defendants had not

165
adduced any evidence to show that this amount was out of line with that
imposed in comparable loans. In addition, the amount did not exceed the
statutory guidelines under the Moneylenders Rules 2009 (4%), which indicates
that the amount was not extravagant or unconscionable. The contracts were
between properly advised parties of comparable bargaining power and the terms
were specifically negotiated. As such, the clause was not a penalty (applying
Dunlop).

236. In Australia, the HCA in Andrews v ANZ rejected the “breach limitation” and
held that “a stipulation prima facie imposes a penalty on a party if, as a matter of
substance, it is collateral to a primary stipulation in favour of a second party and this
collateral stipulation, upon the failure of the primary stipulation, imposes upon the first
party an additional detriment, the penalty, to the benefit of the second party”.

a. However, this could be criticised for increasing commercial risks, by


broadening the application of the rule to a number of common ontractual
mechanisms that do not involve breach (J W Carter et al, “Contractual Penalties:
Resurrecting the Equitable Jurisdiction”.)

Deposits

Deposits and advanced payment

237. A deposit is a security for damages for breach of contract, and is forfeitable
upon default. In contrast, an advance payment may be recoverable under the law of
unjust enrichment (Lee Chee Wei v Tan Hor Peow Victor).

a. If there is no specification whether a sum paid upfront is forfeitable, there is a


potential fight as to whether the sum paid is an advance payment, or a deposit.

b. If the sum is forfeitable, the question is whether this is a reasonable sum


(deposit), or an advance payment which is forfeitable. The former is immune
from the penalty rule, but not the latter (see below).

c. In Lee Chee Wei v Tan Hor Peow, P entered into an agreement to sell his shares
in DMS to the 4th D. The issue in that case was that D had paid a sum of
$750,000 under the contract (it was not stated if this was forfeitable), and sought

166
to recover the sums (D was in default). The payment was stipulated in the
contract as being by installments, which the Court held that the parties intended
this to be an advance payment and could be recovered by D (party in breach).

238. The penalty doctrine does not apply to a true deposit – a reasonable, moderate,
customary sum given as earnest. However, if the sum is not a true deposit but rather a
forfeitable advance payment, this is subject to the penalty doctrine – including the part
as would have been reasonable as an earnest. This is because a true deposit is not
compensatory and may be retained even if no loss is suffered (Hon Chin Kong v Yip
Fook Mun).

a. In Hon Chin Kong v Yip Fook Mun, P wanted to acquire D’s shares in CDX.
The payment was made in 3 tranches of $300,000, with the first being a “down
payment deposit”, and D being obliged to transfer the shares only after all
tranches were paid. P repudiated the agreement and demanded the first payment.
D argued that the first tranch was a deposit paid and was forfeitable upon P’s
failure to make payment of the balance purchase price. The Court held that
while there was no express agreement to forfeit, the use of the language “deposit”
in the context showed that this was intended to be forfeitable. The Court held
that the sum was also a true deposit – although it was about 36% of the purchase
price (higher percentage than precedent cases where deposits were found to be
unreasonable), P in this case had already delayed payment multiple times prior
to the variation of the payment into 3 tranches (earlier it was lumpsum). D had
kept CDX off the market longer than originally agreed – it was reasonable for
D to stipulate a deposit of that amount to assure themselves of P’s earnestness.

239. In contracts concerning with interest in land, the Court may grant relief from
forfeiture of a deposit if it was unconscionable (objectionable terms in a morally
reprehensible manner). However, it is unclear whether this applies to other contracts
(Tan Wee Fong v Denieru Tatsu). (balance of certainty against fairness).

Pre-contract deposits

240. A pre-contractual deposit is paid before any binding contract has been formed,
and is an expression of seriousness of intention on the part of the prospective purchaser.
The basis for such payment is usually that the contract will subsequently come into

167
existence – if no contract materializes, the basis would have failed and the deposit must
be returned under the law of unjust enrichment (note that there could be other bases,
such as external events) (Simpson Marine v Jiacipto Jiaravanon).

a. In Simpson Marine, JJ (dead) was in negotiations with SM (dealer) to purchase


yachts. JJ signed an invoice agreeing to pay a deposit of €1m to secure two
specific yachts (and later, another 2 yachts after SM sold off one of the initial
ones). JJ eventually refused to purchase the yacht – the parties had a
compromise whereby half the deposit was applied to the purchase price of
another yacht which was already purchased. The remaining was the subject of
appeal which JJ’s estate wished to recover. The Court held that the fresh basis
(to secure two specific yachts) did not fail, because both yachts were kept off
the market by SM. As such, JJ was not entitled to obtain the remainder.

Specific Remedies

Specific performance

241. A Court may make an order of specific performance (equitable remedy) which
requires the party in breach to perform a positive contractual obligation. However,
specific performance will only be awarded where damages are inadequate (EC
Investment v Ridout). The Courts will consider a number of factors, such as the need
for constant supervision/wastefulness/uncertainty/hardship on the defendant in
assessing whether to grant specific performance (Cooperative Insurance v Argyll
Stores).

a. No SP Granted – EC Investment v Ridout – in this case, Anwar wished to raise


some financing, which was done by way of an option to purchase granted by
Ridout (which held the property on trust for anwar), to EC. The option could be
cancelled if Ridout paid a sum back in return to EC. Ridout failed to have the
option cancelled, and sought to have the option exercised. The issue was
whether the Court should grant specific performance. The Court held that (1)
Ridout did not approach the courts with unclean hand, (2) damages was
adequate as ECI was content to forgo its right to acquire the property if it
obtained compensation, ECI permitted Ridout to look for other buyers as well
(to a third party), (3) allowing the sale would cause hardship on Orion which

168
had a charge over the balance of the sale proceeds (the price under the option
was much lower – and Orion would not be able to recover anything). Note: This
case departed from the usual assumption that in the context of contract for the
sale of land, specific performance is typically granted as of practice.

b. No SP Granted – Cooperative Insurance v Argyll Stores – In this case, C leased


out the anchor unit in its shopping centre to AS to operate a supermarket. AS
covenanted that it would, for 35 years, keep the demised premises open for retail
trade during the usual hours of business in the locality and the display windows
properly dressed in a suitable manner in keeping with a good class parade of
shops. The supermarket started to run at a loss with 19 years of the lease still to
run. AS stopped trading and stripped out the fittings. C sought specific
performance. The UKHL denied SP, holding that (1) it was settled practice that
SP was not suitable when this would require a defendant to carry on an activity
(not a well defined single act here – unclear scope); (2) the ultimate sanction for
SP is contempt of Court, which is not suitable for commercial disputes; (3) there
is a need for constant supervision, which was wasteful of the resources (repeated
court rulings; (4) Both parties were commercial, which meant that they knew
the likely remedy for breach was damages; (5) the order would cause undue
hardship on AS – to run the business at a loss for 19 years of the lease.

c. No SP Granted – Patel v Ali – In this case, the claimant sought SP 4 years after
the defendant contracted to sell her home. The defendant at that time had (1)
suffered cancer necessitating amputation of her leg, (2) borne 2 more children
(3 in all), (3) husband was adjudged a bankrupt and imprisoned; (4) her poor
English made her highly dependent on the support of friends and relatives living
close by – The Court held that to compel her to move would cause a hardship
amounting to injustice.

Injunctions

242. A mandatory injunction compels the defendant to undo the effects of breaching
a negative undertaking (e.g. tear down houses which were built in breach of covenant).
A prohibitory injunction specifically enforces a contractual obligation not to do
something.

169
243. Generally, a prohibitory injunction is the main remedy to restrain threated or
future breaches of negative undertakings. The grant is usually a matter of course – this
is because it is less restrictive of the defendant’s individual liberty, less likely to run up
against the claimant’s duty to mitigate and the problem of constant supervision, and
avoids the difficulty of assessing loss from a breach of a negative undertaking. However,
where the grant of a prohibitory injunction amounts to an indirect specific performance,
the Courts will not grant it.

244. In Warner Brothers v Nelson, Betty Davis entered into a contract with the
plaintiffs, film producers, for fifty-two weeks, renewable for further periods of fifty-
two weeks at the option of the plaintiffs whereby she agreed to render her exclusive
services as such artist to the plaintiffs, and by way of negative stipulation not, during
the period of the contract, to render such services to any other person. In admitted
breach of this agreement the defendant entered into a contract to perform as a film artist
in this country for a third person. The Court granted the injunction – it did not amount
to specific performance of a personal services contract. The defendant could earn in
other ways as well, and given that damages was not an adequate remedy, the Court
awarded the injunction.

Action on agreed sum

245. When the breach consists in a failure to pay a sum of money stipulated in the
contract, the claimant can sue for that sum eg the price for goods or services rendered.
The claimant need not prove any loss, the claim cannot be reduced for her failure to
mitigate loss (MP Bilt v Oey Widarto). The doctrine of penalties does not apply to an
action for a debt (Stansfield v Vithya).

a. MP Bilt v Oey Widarto – In this case, MP was the developer of a condominium


project. D agreed to purchase a unit while it was under construction. The SPA
provided that 20% of the purchase price was payable on signing of the
agreement, and the balance would be paid in instalments. P had given notice of
the completed stages but the defendant failed to pay for the instalments. P sued
the defendant for the payment of the outstanding instalments. The Court held
that P was entitled to summary judgment – the debt had already accrued, the
doctrine of White v Carter did not apply as the developer was under a legal

170
obligation/practical compulsion to complete performance of the contract in
question. As such, P was entitled to the full debt.

b. Stansfield Business v Vithya Sri Sumathis – In this case, a student was enrolled
in a course, rendering her liable for the fees irrespective of her completing the
course. The student withdrew during the term. The Court held that the school’s
services were made available, and as such they could sue for the agreed sum.
The doctrine of penalty did not apply.

171
Chapter 6: Privity of Contract

Privity rule and consideration rule

246. The privity of contract doctrine only permits those who are party to the contract,
to enforce the terms of the contract and seek contract law remedies for breaches of the
contract (Dunlop Pneumatic Tyre v Selfridge, 134 Tweddle v Atkinson).

a. Tweddle v Atkinson – William Tweddle married the daughter of William Guy.


Prior to the wedding, William Guy entered into a verbal agreement with John
Tweddle (William Tweddle's father), under which both promised to give their
children marriage portions. After the wedding, they entered into a written
agreement, which was intended to give effect to their verbal promises, under
which William Guy agreed to pay £200 to William Tweddle and John Tweddle
agreed to pay him £100. The agreement further contained that "it is hereby
further agreed by the aforesaid William Guy and the said John Tweddle that the
said William Tweddle has full power to sue the said parties in any Court of law
or equity for the aforesaid sums hereby promised and specified". William Guy
failed to pay and William Tweddle sued the executor of William Guy's estate
for the sum. The claim failed – the judges held that WT did not provide any
consideration ("consideration must move from the party entitled to sue
upon the contract").

i. Note the confusion between the consideration rule and privity rule.
The rule has been separated in its analysis in Dunlop.

b. Dunlop Pneumatic – In this case, Messrs Dew agreed to buy tyres from Dunlop,
and Dunlop agreed to give Dew discounts off their list price – Dew in return
agreed to not sell Dunlop's goods to anyone at less than list prices. However, it

134
Dunlop Pneumatic Tyres v Selfridge [1915] 1 AC 847 (UKHL) ("Dunlop Pneumatic Tyres v
Selfridge")

172
was also agreed that Dew could give genuine trade customers a limited discount
off Dunlop's list prices, if as agents of Dunlop, Dew obtained from the trader a
similar written undertaking that it would observe the list prices. Selfridge
ordered Dunlop tyres from Dew. Dew agreed to give Selfridge certain discounts
off Dunlop's list prices, and Selfridge agreed not to sell any Dunlop tyres to
private customers at less than list prices. Dunlop sued Selfridge for breach of
this undertaking. The UKHL held that Dunlop was not entitled to sue as (1)
Dew contracted not as agents, but as principals and (2) assuming even if Dew
contracted as agents, Dunlop did not furnish any consideration for the
contract.

247. The rationale behind the privity rule is that no promises have been made to third
parties, who may or may not even be aware of the existence of the contract. The parties
elect to bring themselves into a consensual, reciprocal and exclusive legal relationship.

248. Consideration may be furnished jointly, such that even consideration moved
only from 1 promisee to the promisor, it is considered to be given on behalf of them
all (Coulls v Bagots).

a. In this case, C entered into a contract to allow ONC to quarry part of his land.
In exchange, ONC was to pay royalties to C and his wife as joint tenants.
Following C's death, his executor sought to determine whether ONC was
required to pay the royalties to the estate or C's wife. The majority held that the
royalties were payable only to the estate on the ground that C's wife was not
party to the contract. However, it was suggested that had C's wife become a
party to the contract, it could be considered that C and his wife were joint
promisees – consideration even if it was furnished from 1 of them, it would be
regarded as given on behalf of them all, and therefore moving from all of
them. In such a situation, the parties should sue together.

Contracts conferring benefits to third parties

249. In a contract which confers a benefit to a third party, and such contract is
breached by the promisor, this will affect the promisee and the third party. What rights
and remedies does each of them have? Which is the party which is planning to sue on
on the contract?

173
Promisee rights

Damages

250. The promisee may claim damages where he has suffered loss as a result of the
promisor's failure to perform in favour of the third party. However, he cannot recover
damages for breach of a contract made for the benefit of a third party, in respect of loss
suffered not by the promisee, but by the third party (Woodar Investment Development
v Wimpey Construction, Beswick v Beswick, McAlpine v Panatown, Chia Kok Leong v
Prosperland).

a. Jackson v Horizon Holidays – The principle that a promisee could not recover
damages measured by the third party loss was doubted by Lord Denning in
Jackson v Horizon Holidays.

i. In Jackson v Horizon Holidays, D contracted with C to provide holiday


accommodation for C, his wife and their two three-year-old children.
The accommodation fell far short of the promised standard, and C
recovered damages, including £500 for mental distress.

1. Lord Denning considered that the award would be excessive


compensation for the claimant's own distress, but upheld the
award on the basis that the claimant had made a contract for the
benefit of his family and could thus include loss suffered by the
family.

2. James LJ however regarded the amount as compensation for C's


own distress – increased by witnessing the distress suffered by
his wife and children.

b. Woodar Investment Development v Wimpey Construction – The UKHL


disapproved Lord Denning's approach to the question of damages in Jackson's
case.

i. In Woodar, a contract for the sale of land provided that, on completion,


the purchaser should pay £850,000 to the vendor and also £150,000 to a
third party. The vendor claimed damages on the footing that the

174
purchaser had wrongly repudiated the contract. The UKHL held that
there was no such repudiation, and thus the issue of damages did not
arise.

ii. However, Lord Wilberforce, in delivering the judgment (with support of


the other judges) seemed to assume that Woodar was not agent/trustee
for Transworld and that Woodar itself did not sustain any loss. He
preferred to/endorsed James LJ's view on damages.

Albazero Exception (Narrow ground)

251. A promisee may recover a third party's loss on a contract for the benefit of a
third party if (1) the contracts relates to property; (2) the parties contemplated that
property may be transferred to a third party, so that the consequences of any breach of
contract will be suffered by that third party;135 and (3) the third party does not itself
have a right against the promisor to recover the third party's loss (Albacruz v Albazero,
Alfred McAlpine Construction v Panatown). The promisee is accountable to the third
party for the proceeds of his judgment (Albacruz v Albazero, Alfred McAlpine
Construction v Panatown). The justification for the exception is to avoid the
disappearance of a substantial claim into a legal black hole.

a. Albacruz v Albazero – In this case, a preliminary question of law was raised for
the consideration of the court – whether the charterer of a bill of lading, having
endorsed it over to a third party, was still entitled to claim for substantial
damages upon the vessel and its cargo becoming a total loss in the course of the
voyage. At the time of the loss, the plaintiff-charterer had indorsed the bill of
lading, but was only received by the indorsee the day after the loss occurred.
The plaintiff claimed that the measure of damages which it was entitled to
recover was the arrived value of the goods, notwithstanding that at the time the
goods were lost, it no longer had any property in the goods and suffered no loss
by reason of their non-delivery at the destination. In that case, the Court held

135
Note that Burrow’s restatement only requires contemplation that the loss to the property will be
suffered by a third party. This is probably the better statement – transfer of property in a property law
sense is not required. Darlington v Wiltshier did not require any transfer of the affected property to the
third party.

175
that the plaintiff in Albazero could not claim for substantial damages, because
under the Bill of Lading Act, the indorsee of a blll of lading was entitled to
enforce the contract directly.

b. McAlpine v Panatown – In this case, Panatown employed McAlpine to


construct a building on land owned by a third party, UIPL, a company related
to Panatown. The work was defective and considerable expenses would have to
be incurred by UIPL to put things right. Panatown argued that it had the right to
sue for substantial damages pursuant to The Albazero exception. The dispute
wet to arbitration – the issue was whether Panatown was able to sue for
substantial damage.

i. The majority held that due to the existence of a duty of care deed
between McAlpine and UIPL, UPIL obtained a direct remedy against
McAlpine. There was no legal blackhole whatsoever (3 judges). This
was also accepted by the minority in the decision vis-à-vis the narrow
ground.

252. To deprive a party of The Albazero exception, there has to be an express


contractual right in favour of a third party or something akin to it. The possibility of a
claim in tort is insufficient (Chia Kok Leong v Prosperland).

a. Recall in McAlpine, that the existence of a duty of care deed (contractual) meant
that the exception was not available.

b. In Chia Kok Leong, Prosperland was the developer of a condominium, while


CKL were the architects. The MCST was later constituted and became the
proprietor of the common property. Prosperland commenced an action against
the main contractor for defective works, and against CKL for breach of contract
which resulted in defective works. At this time, P was no longer the owner of
the condominium and had not spent any money to effect the repairs, and had
not been sued by the MCST in respect of the defects. It was suggested that the
MCST had a direct legal remedy in tort against the appellants.

i. The Court held that the case fell within the narrow ground – the fact that
subsequent purchasers of the unit had a limited right in tort did not

176
remove the legal black hole completely – It is subject to other
requirements such as proximity, foreseeability and defences. (In
addition, the obligations under contract may be higher than that in tort
as well).

253. The narrow ground exception is not available to permit a promisee to recover
losses accruing to undisclosed principals, because the promisor would be totally
unaware of the existence of the third party, and thus impossible for the contracting
parties to have contemplated any transfer of proprietary interest between the promisee
(agent) and the third party (undisclosed principal) (Family Food Court).

a. In this case, FFC operated several food courts. SBL and his wife operated a
chain of duck rice stalls. SBL sued FFC for repudiation of a licence agreement.
SBL argued that it had validly terminated the agreement pursuant to an oral
agreement where by FFC would grant the licence to SBL only when SBL took
up a concurrent lease of another stall. It later transpired that SBL might not be
the owner of the business (family business with multiple names) and his wife
was applied to be joined as his undisclosed principal. The trial judge held that
both SBL and his wife were not the owners of the business, but the first
respondent could recover damages as agent for the unidentified, undisclosed
principal. On appeal:

i. The SGCA agreed with the TJ that the existence of the oral tie-up
arrangement was not proven – the appellant had wrongfully repudiated
the licence agreement.

ii. SBL was really the owner and there is no undisclosed principal – he
acted for himself.

iii. The narrow ground is unavailable for the undisclosed principal situation
– parties could not have contemplated any transfer of the proprietary
interest in the matter as one party was simply unaware of the existence
of the third party.

177
iv. Unclear if Darlington’s extension (requirement that property belongs to
promisee and subsequently transferred to 3p) should be accepted in
Singapore (at [56]).

Broad Ground

254. Under the broad ground of recovery, the loss suffered by the promisee is
reconceptualized as the loss of his performance interest in the contract – that the
promisor has failed to deliver the performance contracted for by the promisee who
intends to benefit a third party – and should be entitled to recover for substantial
damages for these losses (Panatown, Prosperland, Family Food Court).

a. The Court in Panaown endorsed the broad ground, but with varying degrees of
enthusiasm.

i. The majority held that the existence of direct contractual rights under
the DCD by the 3rd party with the contractor preclude the employer from
recovering damages under the broad ground.

ii. Lord Millett adopted a more cautious approach, and that the broad
ground should be restricted to building contracts/other contracts for the
supply of work and materials where the claim [was] in respect of
defective or incomplete work or delay.

iii. Lord Clyde took the view that there must be an obligation on the part of
the plaintiff/promisee to account to the third party for any damages
awarded.

iv. Lord Jauncey was of the view that there could be no recovery if the
plaintiff/promisee did not spend money in entering into the contract, or
did not intend to remedy the breach.

b. The SGCA in Singapore has unequivocally adopted the broad ground principle
in both Prosperland and Family Food Court.

i. In Prosperland, the Court regarded that the promisee’s entitlement to


substantial damages flows from the fact that the promisee did not receive

178
what he bargained and paid for. This prevents the black hole problem
from materializing by modifying the damages rule.

ii. The performance interest claimed must be a genuine one – it must be


reasonable, so as to curb any windfall accrued to the promisee (Family
Food Court).

iii. It is not necessary for the promisee to always demonstrate an intention


to utilize the damages sought to benefit the third party before he is
entitled to claim for substantial damages (Prosperland, Family Food
Court).

iv. Although there is a need to guard against the promisor being doubly
liable (to the promisee, and an action by the third party), this could be
resolved procedurally by joining all the parties together, to determine
all the rights and liabilities of all parties at the same time (Family Food
Court).

v. SGCA in Family Food Court considered that the narrow and broad
ground are conceptually inconsistent with each other and cannot apply
simultaneously, but declined to make a determination here.

Specific Performance

255. A promisee may request for specific performance of the contract as a remedy –
where this is available, the third party will be the one receiving specific performance
(Beswick v Beswick).

a. In this case, P was a coal merchant and his nephew assisted him in his business.
P had his leg amputated and was in poor health. The nephew, anxious to get
hold of the business before P died, entered into an agreement that P was to
assign his business to his nephew. In return, the nephew would employ P for
the rest of his life, and then paying a weekly annuity to his wife. The nephew
refused to pay the annuity to his aunt, arguing that she was not a party to the
contract and thus barred by the privity rule from enforcing the contract.

179
b. The aunt had sued in two capacities – first as a third party beneficiary and
second as administratrix of her husband’s estate. In the former capacity, that
would be barred by the privity rule. In the latter capacity, the UKHL assumed
that the estate would had only been entitled to nominal damages. As such,
specific performance was ordered to enforce the promise.

c. Question: In light of recent developments, where administratrix could sue for


broad/narrow ground and thus no longer nominal damages, would SP be
awarded?

Injunctive relief

256. Where the promise is negative in nature, the most obvious remedy is an
injunction to restrain the promisor’s breach. The Court will grant such a remedy if the
promisee has sufficient interest in enforcing the promise (Gore v Van der Lann). This
remedy is available for example, where A promised B not to sue C, e.g. for a debt by C
to A, and B is under an obligation to indemnify C. If, in breach, A sues C, B may wish
to start a suit for an injunction to restrain A from proceeding with the first action. This
is to avoid an undesirable multiplicity of legal proceedings.

a. A sues C. – A gets 100 – C loses 100

b. C sues B. – B loses 100, C is restored to her original position.

c. B sues A. – A loses his 100 (restored to original position), B gains 100 (restored
to original position).

d. Gore v Van Der Lann – In this case, LC gave free bus passes for pensioners. P
applied for such a pass and signed an application form which excluded liability
for LC and its servants/agents for injury caused. P fell while boarding an LC
bus, and claimed damages against the bus conductor for negligence. The bus
conductor denied negligence and sought to rely on the exclusion clause in the
free pass, while the LC applied to stay all further proceedings under its' rights
in the free pass. It was held that, inter alia, the LC did not have sufficient
interest entitling them to an injunction – there was no contractual obligation
on the part of the corporation to indemnify the bus conductor.

180
e. Snelling v Snelling – In this case, 3 brothers lent money to a family company,
of which they were directors. They agreed that if any one of them resigned, they
would forfeit the money due to him from the company. One of them resigned
and sued the company for the amounts due to him. The company relied on the
agreement between the brothers. The two brothers applied to be joined as
defendants to the action, adopted the company's defence and counterclaimed for
a declaration that the third brother's loan was forfeited. An injunction was
granted.

i. No discussion was made as to whether the brothers had sufficient


interest. There was no evidence of any legal obligation by the brothers
to indemnify the company if the company was sued. However,
reconcilable – sufficient interest need not be limited to legal obligation,
may extend to the brothers' financial interest in the company for example.

f. Elbe Maru – In this case, CIL shipped goods on the Elbe Maru. The bills of
lading, issued for NYK as carriers, provided a limitation of liability clause in
favour of the carrier and its sub-contractors, against claims from the Merchant.
NYK sub-contracted the carriage of goods to SC and while the goods were in
SC's custody, part of them were stolen. IIE were indorsees of the bills of lading,
and claimed against SC for the lost goods. NYK sought an injunction against
IIE suing SC as being in breach of the limitation of liability clause.

i. It was shown that under the contractual arrangements, NYK would


ultimately have to indemnify SC. As such, NYK would suffer financial
loss if IIE were entitled to sue, which constituted sufficient interest
under Gore v Van Der Lann.

Third party enforcement

Changing the status of the third party

257. It may be possible to construe the arrangements between the parties such that
the third party is now in a direct contractual relationship with the promisor. This may
be done in two ways:

181
a. First, where there are direct representations as between the third party and
the promisor, this may form the basis of a contract. In Shanklin Pier v Detel
entered into a main contract to sell paint to a contractor, who used the paint to
paint Shanklin's property. However, the paint was not durable. Shanklin was
able to sue Detel directly in this case, despite not being the purchaser of the
paint as (1) the court found that Detel had promised Shanklin that the paint was
durable, and (2) in consideration of the promise, Shanklin had caused the
specification in their contract with the contractors to be amended to allow for
Detel's paint to be used. As such, Shanklin was now in direct contract with Detel
and could sue (Shanklin Pier v Detel).

b. Second, where there are no direct representations between the third party and
the promisor, a collateral contract + agency analysis may be used (Scruttons v
Midland Silicones). This may succeed if (1) the contract makes it clear that the
third party is intended to be benefitted by the clause, (2) the contract makes it
clear that the party to the contract is contracting on behalf of the third party, (3)
the party has authority from the third party, or the contract is subsequently
ratified by the third party, and (4) the third party has furnished consideration
(Scruttons v Midland Silicones, The Eurymedon).

i. In Scruttons v Midland, A drum of chemicals had been shipped under a


contract between shipper and carrier evidenced by a bill of lading which
limited the liability of “the carrier” to $500. After the rights and duties
under this contract had passed to the claimants by transfer of the bill of
lading, the drum was damaged by the negligence of a firm of stevedores
who had been employed by the carrier to unload the ship. It was held
that the stevedores could not rely on the exclusion of liability clause, as
they were not parties to the contract of carriage, and the carrier did not
act as their agents for the making of a contract between the stevedores
and the shipper.

ii. The Eurymedon – In this case, a drilling machine was to be shipped


from Liverpool to New Zealand. The bill of lading limited the liability
of the carrier, and extends the limitation to servants, agents and
independent contractors of the carrier. The stevedore negligently

182
damaged the drill while unloading it and sought to rely on the immunity
clause in the contract between the carrier and Satterthwaite (hirer). The
clause was upheld – this was a offer for a unilateral contract, between
the shippers and the stevedores. This became a full contract when the
appellant performed services by unloading the goods (actual completion
of performance).

1. Quare – what happens if the actual completion was not done (i.e.
drill dropped into the sea during unloading) – see Mersy Docks.

2. The courts will generally not encourage a search for fine


distinctions which would diminish the general applicability of
the Himalaya clause (The New York Star). This is because such
clauses are established practice in shipping (don't read this
damned case).

iii. Burke v Mersy Docks – In this case, P bought motor cycles. The sellers
acted as P's agent in arranging transportation of the 60 motorcycles from
the seller's premises to Canada. D operated a dock, and agreed to to
provide a berth for ships owned by Manchester Liners, and provides
services for loading and discharge of containers. The sellers arranged
for the shipment of the motorcycles by Manchester Liners' ships, which
bills of lading contained a Himalaya clause. D's employees unloaded the
Hollandia, and in the course of the discharging operation, a can varrier
collided with a container containing corrosive liquid, which escaped and
damaged P's motorcycles. The motorcycles were never shipped and no
bill of lading was ever issued. The Court held that in order to accept P's
offer in the bill of lading the defendants must have performed services
referable to the carriers' contract – However, no act was performed by
D which constituted an acceptance of any offer by the plaintiffs, and
the exclusion clause did not operate.

183
CRTPA

Right of third party to enforce contractual term

258. A third party may enforce a term of the contract if the contract expressly
provides that he may (S2(1)(a) CRTPA).

259. Where the contract purports to confer a benefit on him (S2(1)(b) CRTPA), it is
presumed to be enforceable by the third party, unless the contract shows that the parties
did not intend the term to be enforceable by the third party (S2(2) CRTPA).

a. S2(1)(b) CRTPA only applies to intended and not incidental beneficiaries


(CLAAS Medical Centre v Ng, Columbia Asia Healthcare v Edward Hong).

i. Columbia Asia Healthcare v Edward Hong, there was an agreement to


a sale and purchase of shares in the target company from the vendors to
the purchasers. The SPA provided that the vendors would indemnify the
purchaser (Columbia) under the SSA for breaches of the SSA – the SSA
provided that the purchase was to be free from all liabilities, including
liabilities of PTNM (a subsidiary of the target company). It later turned
out that a debt from PTNM to Thermal Industries (owned by one of the
vendors) were not paid. PTNM was sued on that debt, and sought to rely
on the SSA indemnity clause, joining Columbia to the suit. It was held
that the vendors were obliged to indemnify the purchasers, for the
sums claimed within the indemnity, but PTNM was not entitled to rely
on the indemnity as it was an incidental beneficiary. The warranties
were meant for the benefit of Columbia, with PTNM being an incidental
beneficiary.

1. I agree with a bookshop to buy 100 Enid Blyton books.


Bookshops probably an incidental beneficiary.

b. It is not necessary that benefitting the third party had to be a predominant


purpose or intent, as long as it is the purpose of the parties, objectively
ascertained (CLAAS Medical v Ng).

i. CLAAS Medical v Ng – In this case, Ng was a doctor who entered into


a joint venture with other doctors to setup CLAAS Medical Centre. Ng

184
and the other 6 doctors entered into a shareholders agreement, which Ng
sold his practice to CLAAS. CLAAS was a third party. The SHA
contained a restraint of trade provision, restraining the parties from
being engaging in a competing business as long as they were
shareholders and for 3 years thereafter. The party in breach had to pay
$1m inn LD to CLAAS. Ng breached this term. Ng claimed money from
CLAAS, who sought to set-off debts owed to Ng of $1m for Ng's breach
of the SHA. The SGCA held that CLAAS was entitled to sue on the
SHA as it was an intended beneficiary. The fact that (1) there was no
express term conferring the right of the 3p to enforce the contract, (2)
that parties had the right to vary the contract, (3) consent to assign rights
under the contract generally did not rebut the S2(2) CRTPA presumption
(as such rights were not inconsistent with the enforcement by CLAAS).

260. The third party must be expressly identified by name, class or description, but
need not be in existence at the time of the contract (S2(3) CRTPA).

261. The CRTPA may be excluded S2(2) CRTPA (Koh Chong Chiah v Treasure
Resort).

a. In this case, an option to purchase, pursuant to which the Club was sold to TR,
excluded the application of the CRTPA. It was held that the option had
effectively precluded the plaintiffs from claiming any rights under the option.

Limits to variation and cancellation of contract (common law position and CRTPA)

262. At common law, parties to the contract may agree to vary the contract by
agreement without C's consent, such that the benefit conferred to C is deprived.
However, this ability is limited if the promisee holds the contractual right on trust on
C's behalf (as suggested in Re Schebsman).

a. Note – in this case, no trust was found and the parties were entitled to vary the
contract.

263. The contract may expressly confer the contracting parties:

185
a. The right to vary or cancel the contract without the consent of the third party
(S3(3)(a) CRTPA); or

b. The right to vary or cancel, and the circumstances in which consent of the third
party is required (S3(3)(b) CRTPA).

264. Where the contract is silent, there can be no variation or cancellation if one of
the following conditions is satisfied (S3(1), 3(2) CRTPA):

a. The third party had communicated to the promisor his assent (by words or
conduct) to the relevant term. The postal acceptance rule is disapplied where
assent is by post.

b. The third party had relied on the term and the promisor is aware of it.

c. The third party has relied on the term, and the promisor could reasonably have
foreseen that the third party would do so.

265. There is a limited discretion for the court or arbitral tribunal to dispense with
the third party's consent in certain situations (S3(4), (5) CRTPA).

Defences available to the promisor

266. Unless otherwise provided in contract, promisor can, generally, raise against
third party any defence that could have been raised against promisee (s 4(2),(3),(5)
CRTPA)

267. Promisor may also rely on defences, set-off, etc against third party which arise
from dealings between them (s 4(4) CRTPA)

268. Third party may not rely on term of contract (eg. exclusion or limitation clause)
if he could not have done so had he been party to contract (s 4(6)). Eg UCTA, s2(1)
where personal injury or death is involved.

269. Additional protection for promisor: s 2(2) UCTA inapplicable where third party
is suing to enforce a term of the contract against promisor who can rely on exclusion or
limitation clause as a defence (s 8(2) CRTPA)

186
Double liability

270. Right of promisee to enforce contract specifically preserved (s 5 CRTPA)

271. Compensation recovered by promisee from promisor in relation to term falling


within s 2 to be taken into account in any award subsequently made to third party (s 6
CRTPA)

Exceptions

272. Certain contracts are excluded from the provisions of the CRTPA (S7 CRTPA).

a. However, for contracts for carriage of goods by sea, road, rail or air, a third
party may rely on an exclusion or limitation clause contained in the contract
(S7(4) CRTPA).

Contracts imposing burdens on third parties

273. Generally, a promise between parties will not bind anyone else (i.e. cannot
impose burdens on third parties). This is so because the third party has not voluntarily
accepted those burdens. Exceptionally, this may be allowed in certain circumstances
such as property law.

274. In bailment situations, A (bailee promisor) promises B (the sub-bailee


promisee) to exclude or limit B's liability as a sub-bailee. If B is sued by C (the bailor,
a third party), B may rely on its term in the contract (between A and B) to exclude
liability and limit C's ability to sue.

a. The owner (bailor) is bound by the conditions if he has expressly or impliedly


consented to the bailee making a sub-bailment containing those conditions
(Morris v Martin).

i. Morris v Martin – In this case, Morris sent her coat to a furrier (Beder).
Beder stated that he did not do cleaning himself, and it was
subcontracted to Martins. Martins collected the fur coat under certain
terms which limited their liability. Morris sued Martins claiming that
they had not exercised reasonable care in maintaining the coat, with
Martins relying on their terms with Beder. It was held that while Martins

187
was entitled to rely on their terms against Morris, the conditions did not
exempt Martins from their negligence liability.

ii. The Pioneer Container – P contracted with carriers for carriage of P's
goods by container from Taiwan to HK. Carries issued P with bills of
lading, which allowed the carrier to sub-contract on any terms the
handling, storage, or carriage of goods. The carries subcontracted the
carriage to D, who issued feeder bills of lading acknowledging receipt
of P's containers for shipment. The feeder bills incorporated an exclusive
jurisdiction clause governed by Chinese law and for disputes to be
determined in Taiwan unless carrier agreed. D's ship sank and suffered
total loss of cargo following a collision with another vessel. It was held
Morris v Martin applied, and P was bound by the terms on which the
goods were sub-bailed as he had expressly consented to the bailee
making such a sub-bailment containing those conditions.

188
Chapter 7: Misrepresentation

General conditions of liability

275. A contract is voidable where a party to the contract entered into it in reliance on
a misrepresentation, whether fraudulent, negligent or innocent, by the other contracting
party.

Misrepresentation

Understanding the representation

Objective meaning

276. Objective meaning – There must be a false representation made by one party
to another. Whether any and if so what representation has been made is to be "judged
objectively according to the impact of whatever is sad may be expected to have on a
reasonable representee in the position and with the known characteristics of the actual
representee" (MCI WorldCom International v Primus Telecommunications Inc at [30]).

a. The appellant (P) appealed against summary judgment given in favour of the
respondent (M) and against a refusal to allow P to amend its defence and
counterclaim. P and M were US corporations which supplied
telecommunication services. M sold circuit capacity to P on a capitalised lease
basis. M had claimed against P for outstanding lease rental and an additional
sum pursuant to an upgrade agreement. P's defence had been that it was entitled
to rescind the lease agreements. Following the issue of M's application for
summary judgment, P had sought to amend its defence and counterclaim. The
proposed amendments pleaded entitlement to rectification of one of the
agreements, breach of contract and fresh allegations of misrepresentation which
P alleged had induced it to enter into the agreements. It was argued inter alia
that if the representee had subjectively understood the representations in a
particular way, then it is irrelevant whether a reasonable listener in his position
would have understood them in that sense (i.e. M argued that P could not have
understood the representations in the sense that they objectively bore – wanted
to stop P from giving evidence in the full trial).

189
Express and implied representations

277. Express and implied statements of fact – In the case of an express statement,
the "Court has to understand what a reasonable person would have understood from the
words used in the context in which they were used", and in the case of an implied
representation, whether implied from an express statement of fact or from an opinion,
the court has to "consider what a reasonable person would have inferred was being
implicitly represented by the representor's words and conduct in their context" (IFE
Fund SA v Goldman Sachs International at [50]).

a. In IFE the defendant arranger of a syndicated loan supplied to the claimant, as


a potential lender, pre-acquisition reports on the target company which had been
purchased but failed to pass on post-acquisition reports which it was claimed
cast doubt on the reliability of the earlier reports. It did so pursuant to an
ifnromation memorandum in which it was stated that no representation, express
or implied, was made as to the "accuracy or completeness" of the information
supplied or that the information supplied "will be updated". The Court rejected
implying a continuing representation that the arranger knew nothing which
showed that the information which had been supplied was or might be materially
incorrect. The Court rejected the proposed implied representation, stating that
the only implied representation was that in supplying the information the
defendant was acting in good faith (i.e. it did not know of anything which made
the information given misleading).

Representations by conduct (including concealment, silence)

278. Words or conduct – Representations can be made by words or conduct. They


can thus be implied from conduct as well, applying the objective approach (see IFE v
Goldman Sachs above, Spice Girls v Aprilia). As such, half-truths are actionable
misrepresentations as there is a concealment of what is left unsaid (Dimmock v Hallett).

a. In Spice Girls v Aprilia, Spice Girls Ltd made a contract to promote AWS's
mortorcycles on 6 May 1998. One member left the band three weeks later,
causing considerable loss to the defendant. Since all 5 members participated in
a commercial photo shoot, supplied logos, images and designs, the EWCA held
that the Spice Girls had made an implied misrepresentation by conduct that they

190
did not know and had no grounds to believe that any of the Spice Girls were
intending to leave when they knew this was untrue.

b. In Dimmock v Hallett, a statement that the farms "were let", whilst literally true
when made, did not go on to say that two of the tenants had given notice to quit
– this was a truth which hid the true status of the land when it was to be
purchased and was held to be an actionable misrepresentation.

279. There is generally no duty to disclose at common law in commercial


transactions (Trans-world (Aluminium) v Cornelder China at [65]). Whether there is
such a duty of disclosure is to be assessed by reference to how a reasonable person
would view the silence in the circumstances. Such a duty might arise out of the
relationship of the parties and/or other circumstances in which the silence was
maintained (Broadley Construction v Alacran Design at [28]).

a. In Broadley Construction, BC and AD entered into a contract for the supply of


equipment, which BC used to fulfill its contractual obligations with its main
contractor. The MC stopped paying the BC, and consequently BC began to
default on its payments. BC, AD and MC discussed and signed an undertaking
which authorized the MC to pay AD on BC's behalf, and stated that BC was
released of all its liability to AD. MC did not pay and AD sued BC for the
outstanding sum. AD had verbally told BC that BC would remain liable if MC
failed to pay and BC had remained silent. It was held that BC's silence was not
a misrepresentation – his silence would not be viewed by a reasonable party as
an unequivocal assent to AD's position. The parties were negotiating the
payment of the outstanding sums from opposing positions and interests and
expected a written agreement to be forthcoming. The BC's managing director
did not shrug/nod/did anything by conduct to suggest agreement.

b. In Trans-World, P entered into a contract with M for the purchase of cargo. The
cargo was in custody of D's warehousemen and collateral managers. P alleged
that D's employee represented to them that the cargo carried no risk as to title
and delivery. The cargo however was already subject of an injunction – P sued
D for misrepresentation. On the facts, it was held that there was no contract

191
between P and D and the claim failed under the Misrepresentation Act – further
held that there was no duty of care owed by the D to disclose information to P.

Self-induced misrepresentation

280. Where the misrepresentation was self-induced (i.e. no representation from the
other party), the claim will fail (Eng Hui Cheh David v Opera Gallery).

a. In this case, E purchased a limited edition of a bronze sculpture from OG,


unilaterally thinking it was an exclusive piece. It turned out not to be so and E
sought to sue for misrepresentation. The Court held that this was a case of self-
induced misrepresentation – OG made no representation to this effect.

Actionable representations – statements as to fact and law

Statements of fact

281. For a representation to be actionable, it must be one of present or past fact. This
is often contrasted with opinion, belief, intent, promise and matters as to the future.
However, these statements of non-fact can be re-characterised as statements of fact.

Statement of opinion (recharacterized as statement that opinion maker holds the opinion,

statement that opinion maker had reasonable grounds for making statement)

282. Stating one's opinion about a matter is not the same as stating the truth of that
matter – an opinion is the maker's subjective judgment as to the particular matter, based
on his present state of knowledge. A statement of opinion thus is not an actionable
statement (Smith v Land and House Property Corp). However, such statements of
opinion may imply that the maker had reasonable grounds for holding that particular
opinion. In this regard, the fact that the party making the assertion is in a better
position to know the true facts as compared to the recipient is a factor in favour of the
courts implying such a representation (Smith v Land and House, Esso Petroleum v
Mardon, cf Bisset v Wilkinson). Similarly, the statement of opinion is also actionable,
if the maker did not believe in it (Smith v Land and House).

a. Smith v Land – In this case, vendor of a property in the sale of hotel stated that
the hotel was let to a most desirable tenant. The vendor knew that the tenant's
rent had remained unpaid and was late in instalments. The Court held that given

192
that the vendor is in a better position than the representee to know the truth, the
Court implied a statement of fact that the representor had reasonable ground for
his or her opinion.

i. "The statement of such opinion is in a sense a statement of a fact, about


the condition of the man's own mind… If the facts are not equally known
to both sides, then a statement of opinion by the one who knows the facts
best involves very often a statement of a material fact, for he impliedly
states that he knows facts which justify his opinion" (per Bowen L.J. at
pg 15).

b. Esso Petroleum v Mardon – M was induced to lease a petrol station then under
construction by E's statement that the estimated future annual turnover was
200,000 gallons per year. E reaffirmed this estimate even after the local
authority refused planning permission for the original layout. This change
reduced the turnover to 78,000 gallons. M incurred substantial losses from
operating the station. It was held that the statement was a profits forecast made
with special knowledge and skill of E. The Court held the maker of statement
liable for the misrepresentation (that it had reasonable grounds) when there was
no reasonable basis for it.

c. Bisset v Wilkinson – In this case, the vendor of a farm in New Zealand told the
prospective buyer that he thought the land could carry 2,000 sheeps. Both
parties knew that the land was untried as a sheep farm and were in the same
position to form an opinion on this. The vendor also had no idea about the
answer. The Court held that the vendor's statement was an honest opinion,
which does not imply that he knew facts justifying it, hence the contract was
not rescinded for misrepresentation (given that there was none).

Statement of intention/future conduct (recharacterized as representor holding that intention

represented)

283. A statement as to what the maker will or intends to do in the future is not a
statement of a present or past fact (Tan Chin Seng v Raffles Town Club). However, there
is an implied representation that the maker does in fact hold that intent (Edgington v
Fitzmaurice, Wales v Wadham).

193
a. Edgington v Fitzmaurice – in this case, a prospectus was issued by the directors
of a company inviting subscriptions for debentures, stating that the object of the
issue of the debentures was to raise funds for alterations to buildings owned by
the company, the purchase of horses and vans, and for the development of the
company's business. However, the evidence showed that the real object was to
enable the directors to pay off the company's pressing liabilities. The directors
were stating as fact something that was untrue and were found liable for
misrepresentation. "The state of a man's mind is as much a fact as the state of
his digestion", per Bowen LJ at pg 483.

b. Wales v Wadham – in this case, the husband and wife were in divorce
proceedings (husband wanted to leave wife for another woman). They entered
into a compromise agreement whereby the husband would pay the wife money
for full settlement of the wife's claim on the divorce petition for maintenance.
The wife had earlier represented to the husband that she had no intentions of
remarrying, but later did so. The Court held that the wife made an honest
statement of intention as to her future conduct (she was just 50 when she made
them, did not meet the man she remarried to later, made the statement in order
to preserve the relationship – even proposed husband to have 6 month trial
marriage with the 3rd party) and thus no misrepresentation. (Note – fight was
because maintenance obligations is until the other party re-marries and usually
for monthly payments).

c. Tan Chin Eng v Raffles Town Club – In this case, TRC was incorporated to own
and manage raffles town club. RTC initiated a membership drive. In the
promotional materials, RTC promised that it would deliver a premier club with
first class facilities. After the club was finally built, the plainitffs experienced
crowdedness at the premises, found that close to 19,000 persons had been
admitted as founder members. The club was no longer a premier club. The
SGCA held that the statements were promises as to the future and not actionable
as representations. However, a term was implied that RTC would provide a
premier club – the plaintiffs were only entitled to damages for breach of this
implied term.

194
d. Deutsche Bank AG v Chang Tse Wen – In this case, Chang and Lim met a
relationship manager of DB, who presented them with a brochure detailing DB's
services. Lim signed an account application form while Chang said he would
sign a similar form after receiving proceeds from his sale of Tanox Inc shares.
Chang eventually opened up an account for investment purposes and he suffered
huge losses. Chang owed the bank US$1,788,855.41 and he brought a
counterclaim in, inter alia, misrepresentation. The court held that the
information in the brochure were statements of future intention (what DB would
do for Chang). It was honest belief – they were meant to interest Chang into
entering into separate relationships whereby DB would provide advisory or
wealth management services. In addition, the Court held that the statement that
DB was amongst the best compared to other international banks was a statement
of fact (at [89]), given that it was not obviously untrue and that it was made in
the presentation to a potential client – in such circumstances this was not a mere
hyperbole. However, the statement was not shown to be false (industry awards
suggested the contrary).

Statements of the law

284. Traditionally statements of the law were not actionable in misrepresentation. A


similar constraint prevented the recovery of money paid out under a mistake of law
until the abolishment in Kleinwort Benson v Lincoln City Council. In Pankhania v The
London Borough of Hackney, the Court considered that the "misrepresentation of law"
bar should be abolished, following the decision of Kleinwort Benson as well.

Puffs

285. A mere puff is a statement so vague that they have no effect as a representation.
For example, to describe land as "fertile and improvable" is mere sales talk which
affords no ground for relief (Dimmock v Hallett). Whether a statement is a sales puff or
a representation of fact depends on the degree of its untruth, the circumstances of its
making and the expertise and knowledge attributable to the person to whom it was made
(Deutsche Bank v Chang at [87]-[89]).

195
a. In Dimmock v Hallett, the vendor of land informs a prospective purchaser of
land that the land was "fertile and improvable". The Court held that this was not
a misrepresentation, but a mere puff.

b. In Deutsche Bank v Chang, the representation that the services provided by


DBPWM were amongst the best when compared to other international banks is
one of fact and not a puff. It was not obviously untrue as DB had won many
industry awards and accolades over the years. The representation was not
hyperbole in the context of a presentation to a potential client and was
actionable – however, there was no evidence that this was false (at [89]).

The representation must be false

286. The plaintiff must allege and prove that the representation was false – when the
facts asserted do not correspond with the facts as they exist. It needs to be substantially
false – it need not be false in every respect, nor is it invariably sufficient if it is false in
a single respect. The test is whether "the discrepancy between the facts as represented
and the facts as they existed would have reasonably influenced the mind of a normal
representee, in considering whether to alter his position as he did" (Ernest Ferdinand
Perez De La Sala v Compania De Nagegacion Palomar SGCA at [173]).

Continuing representations and change of circumstances

287. A person may have to disclose material facts which come to his notice before
the conclusion of a contract if they falsify a representation previously made by him
(With v O'Flanagan).

a. In this case, negotiations for the sale of a medical practice begun in January,
when the practice was said to be worth £2,000. A contract of sale was made on
1 May, by which the practice had become worthless due to the intervening
illness of the vendor. The contract was set aside on the ground that the vendor
ought to have communicated this change of circumstances to the purchasers.

The representation must be made to the party misled

288. The statement must be addressed to the party mislead (Peek v Gurney).

196
a. In this case, the appellant purchased shares on the faith of false statements
contained in a prospectus issued by the promoters of the company. The
appellant was not a person to whom shares have been allotted to on the
formation of the company, but merely purchased shares from the allottees. The
Court held that the prospectus was only addressed to the first applicants for
shares. It could not be supposed to extend to others than those – the appellant's
action failed since the false statements were not addressed for him. However,
indirect representation may be effective if the representor intended it to be
passed to the representee.

b. See also Changi Makan Pte Ltd v Development 2003 Holding at [49] – The law
does not require the representation to be made directly to the plaintiff. It is
sufficient if the representation is made to a third party to be communicated to
the plaintiff or a class of persons of whom the plaintiff is one, or even if it is
made to the public generally with a view of it being acted on.

The misrepresentation must have induced the misled party to enter into contract

Inducement and reliance

289. The misrepresentation must have induced the misled party to enter into the
contract. The misled party must show that it had relied on the misrepresentation (Wee
Chiaw Sek Anna v Ng Li-Ann Genevieve). The inducement need not be the sole cause
so long as it played a real and substantial part and operated in the representee's mind,
no matter how strong or how many were the other matters which also played on the
representee's mind (Anna Wee v Ng Li-Ann at [94]).

a. Proof of reliance is still required even if fraud is established – however, the fact
that there was fraud bolsters the inference that the representee was so induced
(at [45]).

b. In this case, the appellant had a separation agreement with her ex-husband who
is now dead. During the proceedings, the ex-husband filed an affidavit of assets
and means, stating he was willing to pay RM 1,200 per child in maintenance.
He did not reveal the existence of two other contracts he made, worth USD$25m
and RM900k. The appellant claimed that the deceased ahd made fraudulent
misrepresentations, intending to induce her to forgo division of assets. She had

197
relied on this when she agreed not to ask for a division of assets. The appellant
only discovered the 2 agreements 9 years later and sought to set aside the
separation agreement on the ground of fraudulent misrepresentation. The Court
held that there was no evidence that the deceased's financial position was false.
The Court gave the dead husband the benefit of doubt that "despite my financial
situation" was an ambiguous statement (statement was made to settle the sole
outstanding issue of care and control, maintenance and custody of children and
not division of assets). There was also no reliance by the appellant as she had
already made up her mind that the deceased had no assets worth dividing –
she thought the deceased was a man of straw.

290. Illustrative case – Ong Keh Choo v Paul Huntington Bernado [2020] SGCA
65.

a. In this case, P granted an option to D1 and D2 to purchase a property, for a fee


of $316,000. P was a real estate agent, whose formal name was Ong Keh Choo.
She advertised the sale of her property as Jeanette Ong. The cheque for the
option fee was subsequently countermanded, which P sued for breach of
contract. Ds alleged that (1) P had represented herself being the agent and not
the owner of the property, (2/3) that P had told the cheque was for show/the
cheque would not be cashed, (4) that P had represented by her action/conduct
that it was normal practice to pay 10% of price as option, (5) that the signature
on the OTP (by R2) was to acknowledge cancellation of certain words.

b. It was held that the identity of the seller is ordinarily immaterial/not important
(at [85]). There were no facts suggesting that the identity of the seller was
important. The allegation that if the Ds knew that P was the owner, the Ds would
not believe P when she said she would get a good deal for them – Ds had claimed
P to be rule and unprofessional/rushed them into viewing. Also – evidence
showed afterwards that after knowing the identity they still continued
negotiations – affirmation/waiver possible as a defense. Main reason for asking
the cheque back was that they had not taken into account of Ds own expenses.

198
c. 2/3 representations were not true – evidenced from circumstances (money was
reduced. Ds tried to destroy evidence by marking out counterfoil in cheque book
as well).

d. Rep 4 – no evidence to support – rep not proven.

e. Rep 5 – R2 signature was not necessary to establish OTP legally binding on R2.

291. Note that there is some uncertainty as to the requisite test for reliance in
misrepresentation.

a. Orthodox contract law takes the view that but for causation must be established:
but for the representation it would not have entered into the contract (if no
misrepresentation was made, he would have done something differently) – see
The Law of Contract in Singapore at [11.074], A Restatement of the English
Law of Contract at pg 191, Chitty 33rd Ed at [7-039].

b. However, "but for" causation is not required for rescission for fraud – it is
sufficient if there is evidence to show that he was materially influenced by the
misrepresentation merely in the sense that it had been actively present to his
mind (BV Nederlandse Industrie van Eiproduckten v Rembrandt Enterprises at
[32]).

Representee could have discovered the truth – not a bar

292. Where the representee could have discovered the truth by exercise of reasonable
care, it is not a bar to establishing reliance (Redgrave v Hurd) – he must have discovered
the truth.

a. In this case, P made a statement about the turnover of his practice to the buyer.
The buyer declined invitation to examine further documents, but later refused
to complete the transaction on the ground of misrepresentation. The Court held
that the buyer's failure to examine documents closely was not a bar to claiming
misrepresentation.

199
Representee knows the truth – quare if this is a bar?

293. There is authority which suggests that where the claimant suspected or possibly
even knew that the statement was untrue, this may not be a bar to establishing reliance
in the context of fraudulent misrepresentation (Hayward v Zurich Insurance).

a. In this case, the insurers entered into a settlement agreement with the defendant
(who had sued his employers for a back injury suffered at work). The employer's
insurers suspected that the defendant had exaggerated his claim but would not
have been able to prove it in court and so agreed to settle the claim for £135,000.
The defendant's neighbours provided evidence to substantiate the insurer's
suspicion and they sought to rescind the settlement agreement. The UKSC held
that the insurers did not prove that they had settled because they believed the
defendant's misrepresentations were true – just had to show that they were
influenced by those misrepresentations.

b. The better view however is that the issue in Hayward v Zurich Insurance is
better dealt with under the law of duress (economic/lawful act duress) instead –
there was no falsity in the circumstances (Pre-Contractual Misrepresentations:
Mistaken Belief Induced by Mis-statements, Prof Kelry Loi).

Representee relied on third party/due diligence not a bar

294. A person who has relied on a third party or conducted his own due diligence is
not barred from claiming misrepresentation unless he has come to learn of the
misrepresentation before entering into the contract or does not rely on the
misrepresentation at all (Jurong Town Corporation v Wishing Star).

a. In this case, JTC, which was developing a research complex (biopolis), had,
through its subsidiary, JCPL, invited tenders for certain works. The plaintiff
WSL was awarded the contract. In its tender, WSL had made certain
representations which claimed compliance with the conditions of the tender.
These representations turned out to be false. The contract was terminated on the
ground of misrepresentation. WSL sued for wrongful termination, where JTC
applied for the issue of misrepresentation to be tried first. The court held that
the fact that JCPL had evaluated WSL's representations, JTC did not rely on the
representations but on JCPL's evaluation. The court disagreed – this would

200
penalize a party who has chosen to act carefully but failed (whether through
negligence or otherwise, to discover the fraud). Such a proposition would
encourage fraud and the indolent.

Misrepresentation was corrected - bar

295. Where the misrepresentation was corrected, there will be no such


misrepresentation going forward for the person to rely on (Broadley Construction v
Alacran).

a. In this case, the misrepresentation (assuming there was one where the party had
impliedly misrepresented by silence (see above)) was corrected in a later draft
agreement which expressly provided that BC was released from liability for the
outstanding sum, such that AD could not have been induced by the
misrepresentation into signing the agreement.

Inducement, materiality and fraud

296. If the representation is material (i.e. it is an important statement vis-à-vis the


contract), it is a factor towards a finding that the representee had relied on a statement.
However, it is not a presumption of law but only a presumption of fact (i.e. one possible
inference to be drawn) (Lim Koon Park v Yap Jin Meng Bryan at [53]).

a. In this case, the respondents sought to rescind the agreement to setup a joint
venture company on the account of the appellant's misrepresentation as to the
land's plot ratio. The court found that there was no misrepresentation on the
facts. Even if there was, the trial judge was wrong to conclude that the
respondents had relied on the appellant's statement as to plot ratio only because
it was of a material nature. The inference was inappropriate in the circumstances
as there was clear evidence that the respondents had at no relevant time placed
any reliance on the appellant's representation.

297. Where the representation was made fraudulently, this is a factor towards finding
that the representee had relied on the statement. However, this is merely a presumption
of fact (i.e. logical inference) and it is very difficult to rebut, but the onus is still on the
representee to show that he was induced (BV Nederlandse Industrie Van Eiproduckten
v Rembrandt at [43]).

201
a. In this case, the claimant contracted tu supply the defendant with egg product
for set prices per kilo, on the condition that the claimant's procedures received
regulatory approval. Prior to approval being granted, the claimant informed the
defendant that due to unanticipated extra regulatory costs, the prices have to be
increased – a new contract was entered into between the parties with increased
set prices. The defendant suspended its performance of the contract on several
grounds, including fraudulent misrepresentation – the increase in sale price
contained no element of extra profit but was calculated by reference to the extra
regulatory costs alone. The Court held that fraud was made out – the
presumption of fact (of inducement) was not rebutted.

298. Note: probably not an independent requirement – the immateriality of the


misrepresentation should not defeat a claim in misrepresentation (Chitty at [7-041]).

Types of misrepresentation

Fraudulent misrepresentation

299. Fraud is proved when it is shown that a false representation has been made (i)
knowingly false, or (ii) without belief in its truth, or (iii) recklessly, whether it be true
or false. (Derry v Peek). This requires an ascertainment of the maker's mind,
subjectively and the maker must intend the appellant to act in reliance on that statement
(Wee Chiaw Sek Anna v Ng Li-Ann at [51]).

a. In Derry v Peek, the company ran trams with animal power. Although its
application to run trams with steam had not yet been approved, it claimed in its
prospectus that it had obtained approval. Unexpectedly, the application was
rejected. Peek bought shares on the basis of the prospectus and sued in the tort
for deceit. The Court held that the director had honestly believed that the
approval was forthcoming. The directors were not liable merely because they
acted unreasonably in failing to check the truth of the statement – it must be
shown that they did not care at all about the truth (and thus had no honest belief
in the statement when it was made).

b. See Anna Wee at [34] – "recklessness must not be equated with negligence or
carelessness… Not caring… did not mean not taking care, it meant indifference

202
to the truth, the moral obliquity which consists in a willful disregard of the
importance of truth".

c. Wang Xiaopu v Goh Seng Heng [2019] SGHC 284 – Illustrative of how a
misrepresentation action is argued. Probably not too important – no new
principles/statement of law. – In this case there was fraud – SPA agreements
tainted by (a) misrepresentation about price being discounted, (b)
misrepresentation that it would not be sold prior to IPO without plaintiff consent.
However, misrepresentation about business growth is not substantially false –
trepresented 10m, but profit is about S$9,374,703.

d. Changi Makan v Development – statement not false, ergo no fraud. About rental
capacity and changed circumstances – did disclose the change to the plaintiff
and substantially correct rental.

Negligent misrepresentation

300. To establish negligence misrepresentation, it must be shown that there is a


breach of a duty of care owed (i.e. a reasonable person would have done something
more/refrained from doing something) (Hedley Bryne v Heller).

a. In this case, the plaintiffs were advertising agents who booked, for a company
Easipower Ltd, orders for advertising time on television programmes and for
advertising space in certain newspapers on terms that they themselves became
personally liable to the television and newspaper companies for the cost of the
orders. Prior to doing so, the plaintiffs obtained, through their own bankers,
references regarding the creditworthiness of Easipower Ltd from the defendant
bank, who were the company’s bankers and who knew the purpose for which
the references were requested. These references were inaccurate but were
stipulated to be “without responsibility on the part of the bank or its officials”.
Relying on the references, the plaintiffs placed the orders which resulted in a
loss of some 17000 pounds when the company went into liquidation. The HL
held that, because of the disclaimer, the defendant was not liable for the
negligent misstatement. However, had there been no disclaimer, a duty of care
would have been owed by the defendant to the plaintiffs to take reasonable care
in making the statements contained in the reference.

203
Innocent misrepresentation

301. An innocent misrepresentation is one which is made without fraud or negligence


(Redgrave v Hurd).

Remedies

Primary remedy: Rescission

302. Misrepresentation renders a contract voidable, not void. The representee


therefore has to elect whether to rescind the contract or not (Car and Universal Finance
v Caldwell). It must be communicated with the representor generally, but where the
representor was a fraudster, the representee is only required to have taken all possible
steps to try to rescind (Car and Universal Finance v Caldwell).

a. In this case, C was fraudulently induced to sell their car in return for a bad
cheque. The fraudster disappeared. C sought help from police and automobile
association immediately. In the meanwhile, the fraudster sold the car to a third
party who in turn sold to CUF, who purchased in good faith. The Court held
that C had effectively rescinded the contract by his actions before CUF had
acquired rights to the car, thereby revesting title to the car to C. Notice to the
fraudster was not needed and it suffices if C had taken all possible steps to
recover the goods.

Bars to rescission – affirmation

303. If the representee elects to affirm the contract and this decision is communicated
to the representor, the representee loses the power to rescind. This can be by way of
conduct or words, but it must be unequivocal (Long v Lloyd, Straits Colonies v SMRT
Alpha).

a. In Long v Lloyd, Long was induced by Llyod's representation that the lorry he
wanted to buy was in excellent and first class condition. Long noticed many
defects after viewing lorry. He was advised by an expert that the lorry was not
in roadworthy condition, but Long still bought the lorry and used it. It was held
that Long had affirmed the contract and could not rescind subsequently.

204
b. In Straits Colonies v SMRT Alpha, a landlord and a retailer of a shopping mall
sued a tenant for payment of rent due. The tenant's defenace was that the
landlord had misrepresented, inter alia, that the premises could be used for the
operation of a pub, bar and club with live entertainment when the landlord had
not obtained the requisite permission for these uses from the authorities.
However, the tenant's conduct envinced a clear and unequivocal intention to
affirm the lease despite the misrepresentations – the tenant signed the lease
agreement without making any qualifications more than a month after it
received word of the URA's decision, made no protest when taking possession
of the Premises, commencing business and paying rent. There was also no
evidence of any gentlemen's agreement to renegotiate the rent.

304. The representor need not show that the representee knew of its legal right to
rescind the lease – it was sufficient that the representee knew of the facts giving rise to
its right of rescission. The position was that the representee is not allowed to hide
behind its ignorance of the law, and avoided practical difficulties such as requiring the
representor to inquire and seek confirmation that the representee was aware of his legal
rights (Straits Colonies v SMRT Alpha at [61]-[64]).

Bars to rescission – lapse of time

305. A representee who fails to rescind the contract (even when she remains ignorant
of the non-fraudulent) may be barred from rescission by a substantial passage of time
(Leaf v International Galleries). However, this position has been questioned recently
by the EWCA in Salt v Stratstone Specialist, and it was suggested that it is only the
lapse of a reasonable time such that it would be inequitable in all the circumstances to
grant a rescission which constitutes a bar to the remedy (at [43]), which is in essence
the principle of laches.

a. Leaf v International Galleries - "Salisbury Cathedral" by John Constable was


what Ernest Louis Leaf thought he was buying on 8 March 1944 from
International Galleries. International Galleries said it was a Constable. Leaf paid
£85. Five years later when he tried to auction it, Leaf was told that it was not a
Constable. He claimed rescission of the contract against International Galleries,
to get back his money. The Court held that recission was barred – It was held

205
that the purchaser ought to verify or disprove the representation within a
reasonable time, and there should be an interest in the finality of the bargain.

b. Salt v Stratstone – the purchaser was told by Stratstone that the car was brand
new when it was not. Stratstone sought to reject the vehicle a year later as the
car exhibited a number of defects and asked for his mony abck. Stratstone
refused, and the purchaser sued for damages that the car was not of
merchantable quality. Later in documents disclosed by Stratstone, it was
revealed that the car was not new and the purchaser amended his claim to seek
rescission of the contract on the basis of misrepresentation. The EWCA held
that the delay of over 3 years between the sale and seeking recission was not a
bar to relief. The lapse of time on its own was not a bar – the ground was only
known to the claimant on the disclosure of documents – most of the subsequent
delay was due to the litigation process and the defendant's own wrongful refusal
to take the car back.

i. Leaf was doubted by the EWCA because (1) it was decided prior to
when the misrepresentation act was passed – it is now doubtful whether
a representor should be in a no worse position than if the representation
had become a term of the contract; (2) the new sale of goods act provides
that a buyer is not deemed to have accepted the goods unless he has had
a reasonable time to examine the goods – which includes whether the
buyer had such opportunity to examine the goods (departing from the
earlier sales of goods act which provided that a buyer accepts the good
after a lapse of reasonable time) – as such, it is doubtful as to whether
leaf is still good law.

Bar to rescission – third parties

306. The representee would generally be unable to exercise his right to rescind the
induced contract if this would prejudice the rights of innocent third parties (i.e. title to
the goods has passed). The onus lies on the representee who seeks to rescind to prove
that the third party took the property with notice of the fraud or otherwise than in good
faith (Car & Universal Finance Co v Caldwell).

206
a. In this case, CUF was the third party who had acquired the car (C sold to
fraudster, who sold it to M (who had notice to the defect of title), who sold it to
CUF later). The Court held that if prior to C's exercising of his right to rescind,
the 3p had in food faith given value for the car without notice, he would obtain
good title.

Bars to rescission – restitutio in integrum impossible

307. Rescission is only possible if the parties can be paced in their original positions
(mutual restitution). Precise restitution is not needed, and the court may grant rescission
on terms (Alati v Kruger).

a. The respondent was induced by the appellant's fraudulent misrepresentation to


purchase a business conducted on leasehold premises. The respondent took
immediate steps, on discovering the falsity of the representations, to have the
contract set aside. Until judgment was given on the facts the respondent
continued the business, but while the court was considering its decision on the
question whether restitution in integrum was possible, the respondent ceased to
trade and the landlord reentered upon the premises. The High Court of Australia
held that it was not necessary for the parties to be restored to exactly the same
positions as those they originally held and that the respondent was entitled to a
decree of rescission – (lease was surrendered as appellant took back possession
– some of the chattels cannot be re-delivered) – judgment varied to order
respondent to return to appellant the chattels remaining, while appellant had to
refund purchase money to respondent, deducing the chattels which could not be
returned + use value of property used.

Remedies: Damages

Fraudulent misrepresentation

308. Damages for fraudulent misrepresentation included all losses that flowed
directly as a result of the entry by the plaintiff (in reliance upon the fraudulent
misrepresentation) into the transaction in question, regardless of whether or not such
loss was foreseeable and included all consequential loss as well – the purpose of
damages here is to put the victim in the position in which he would have been, if the
tort had not been committed (Wishing Star v JTC at [26]-[28]).

207
a. In this case, WSL's bid was around $54m, with the next lowest bid being $63m
(Liang Huat). Upon termination, JTC took up a new contract with BLL with a
bid of $61.81m. JTC sought to recover the difference between the BLL and
WSL bid - $7.81m. The SGCA held that JTC was not entitled to so recover – if
there was no misrepresentation made, JTC would had entered into a contract of
a higher pricing. However, other losses, such as expenses incurred by JTC for
JCPL to China to inspect WSL's facilities, costs for JCPL to attend to WSL and
costs of engaging a surveyor for JTC's inspection of WSL's facilities.

309. Note that even if the loss recoverable is broader, loss suffered must be mitigated
by the innocent party (Smith New Court v Scrimgeour).

Negligent misrepresentation

310. Damages for negligent misrepresentation included all losses which were not too
remote (i.e. reasonably foreseeable) (Hedley Bryne v Heller).

Innocent misrepresentation prior to the Misrepresentation Act

311. Prior to the enactment of the misrepresentation act, a non-fraudulent


misrepresentation gave no right to damages, unless the misrepresentation was a term of
the contract (or after Hedley, constituted a negligent misrepresentation) (Whittington v
Seale-Hayne).

a. In this case, Mr Whittington bred prize poultry. He bought a long farm lease,
induced by Seale-Hayne’s representation that the premises were sanitary and in
good repair. But the water supply was poisoned, Mr Whittington’s manager got
very ill and the poultry died. Under the lease, Mr Whittington had covenanted
to carry out repairs required by the council, and these were needed after the
council declared the premises unfit for habitation and the drains needed
renewing. It was undisputed that Whittington was entitled to indemnity for rates
paid or repairs costs. Whittington sought rescission and indemnity for loss of
poultry, profits and medical expenses. Farwell J held no further losses could be
claimed because it was beyond the ambit of the indemnity to which Mr
Whittington was entitled. These losses did not result in a benefit to Seale. Since
the representation was non-fraudulent, there could be no damages and therefore
no compensation either. It was not the case that the rescinder should be in a

208
position status quo ante because ‘to make good by way of compensation for the
consequences of the misrepresentations is the same thing as asking for damages.’

Damages under the Misrepresentation Act

Damages under the fiction of fraud

312. Where a person has entered into a contract after a misrepresentation has been
made to him by another party thereto and as a result thereof he has suffered loss, then,
if the person making the misrepresentation would be liable to damages in respect
thereof had the misrepresentation been made fraudulently, that person shall be so liable
notwithstanding that the misrepresentation was not made fraudulently, unless he proves
that he had reasonable ground to believe and did believe up to the time the contract was
made that the facts represented were true. (S2(1) Misrepresentation Act).

313. The key elements which needs to established are:

a. P has entered into a contract after a misrepresentation;

b. The representation was made by the other party;

c. P has suffered loss;

Once these elements are established, the other contracting party will be so liable
(for the losses) notwithstanding that the misrepresentation was not made
fraudulently, unless he shows that he had reasonable ground to believe and did
believe up to the time the contract was made that the facts represented were true.
This basically requires the representor to establish that the misrepresentation was
made innocently.

314. What are the key advantages pursuing a claim under the Misrepresentation Act
then?

a. Once a misrepresentation is established, damages is available to the representee


as of right. The representee need not further establish that the misrepresentation
was made fraudulently or negligently to be entitled to damages. The onus is now
on the representor to prove honesty and reasonableness.

209
b. Unlike Hedley Bryne, there is no need to establish a separate duty of care owed
by the representor to the representee for negligent misrepresentation. The fact
that there is a contract between them is sufficient.

This is illustrated in the case of Howard Marine v Ogden.

In this case, O wished to hire barges. H told O that their barges could carry 1600
tonnes, based on the Lloyd's Register for barges. The Lloyd's Register in this
case was however incorrect and the true capacity was 1055 tonnes. H knew that
the German shipping documents contained the correct figure, but preferred the
Lloyd's Register. The contract stated that O's acceptance of the barges showed
that they were in every way satisfied. When the barges proved to be insufficient
for the job, O refused to pay the full price. H terminated the contract and sued
for the outstanding payments, while O counterclaimed for inter alia a breach of
S2(1) of the Misrepresentation Act and negligent misrepresentation under
Hedley. The majority of the court held that there was no tortious duty of care
owed by H to O under Hedley, but there was a breach of S2(1) of the
Misrepresentation Act. The majority of the court considered that the burden of
proving reasonableness was not discharged, since the owner could have
discovered the true state of affairs by looking at the documents in possession.
The exclusion clause failed to pass the reasonableness test under S3 of the
Misrepresentation Act.

315. To escape liability, the representor must show that it had subjectively believed
in the truth of the representation and that there were objectively reasonable grounds
for that belief (based on the considerations which were subjectively present in the mind
of the person) – (i.e. what a reasonable person in the defendant's shoes have believed)
(RBC v Defu Furniture at [77]).

a. Defu entered into a contract to lease from RBC the first storey of an industrial
building, and the sole purpose of the lease was as a furniture showroom.
However, RBC made misrepresentations assuring that all necessary approvals
had been obtained for the premises to be used as a furniture showroom, which
was argued to be not the case because the Singapore Land Authority, acting on
behalf of the State had not given its approval for the premises to be so used. The

210
Court held that the misrepresentation was innocent: Even though RBC had
failed to review the terms of the State Lease at all, on an assessment of the facts,
(1) the building was developed from the start for the Premises to be used as a
showroom and (2) SLA was silent as to whether a differential premium was
assessable in the first place (as a change of use warranted a premium). As such,
a reasonable person would not have checked the State Lease or inquired with
the SLA – there was also evidence that SLA would only communicate with
parties privy to the State Lease and not RBC (merely a subtenant), and that the
state lease did not mandate that approval might need to be sought. Legal checks
may not have uncovered this specific issue.

316. What is the measure of damages under S2(1) of the Misrepresentation Act?
The UKHL held that the measure was that of fraud in Royscot Trust v Rogerson.

a. In this case, a car dealer wished to sell a car to a customer for a cash price of
£7,600, of which the customer would pay the dealer a deposit of £1,200. To
finance this, the dealer would propose to a third party financier the purchase
price and deposit of £8,000 and £1,600 respectively (the financier had a policy
of not entertaining hire-purchase financing unless a deposit of at least 20% of
the cash price was made paid). The third party financier bought the car from the
dealer at £6,400, and entered into a hire-purchase agreement with the customer.
The customer paid instalments for part of the sum due to the financier, but
ceased to pay later and dishonestly sold the car. The Court had to consider if the
representor was liable for all the consequences of the misrepresentation, or if
the customer's independent act of selling the car broke the chain of causation.
The Court held that the measure of damage is that of fraud – the loss arising
from the car being sold could be considered and thus the loss from what the
customer owed to the financier could be recovered.

317. This position however has been left open in the UKHL decision of Smith New
Court v Scrimgeour per Lord Steyn at 283). The position has been doubted in the SGCA
decision of RBC v Defu Furniture:

a. The rationale for the more generous measure of damages for fraud is to
discourage fraud (at [82]);

211
b. The language of the statute suggest that it was meant to extend the remedy of
damages to situations that fell short of fraud (at [83]);

c. If only liability is extended, given that the common law had only recently
developed Hedley Bryne, this is merely a statutory analogue for the common
law in terms of liability (at [83]); and

d. As such, it should follow that the measure of damages to be awarded under


S2(1) is that for negligent misrepresentation and to depart from Royscot Trust.

318. However, even under the fraud measure, there is a duty to take all reasonable
steps to mitigate his loss once he has discovered the fraud (Smith New Court v
Scrimgeour).

Damages in lieu of recission

319. Where the misrepresentation is not fraudulent, the Court has a discretion under
S2(2) of the Misrepresentation Act to award damages in lieu of rescission. The court
will consider inter alia, the nature of the misrepresentation, the loss that would be
caused to the representee if the contract were upheld and the loss the recission would
cause to the representor (William Sindall v Cambridgeshire CC).

a. William Sindal v Cambridgeshire CC – In this case, WS bought land from CCC.


WS was asked if there were any rights of easement or public rights affecting the
property other than those disclosed in the contract. WS later discovered a foul
sewer under the land. At the same time, the site worth less than half of the
purchase price. The court held that there was no misrepresentation, but even if
there were, the court will award damages in lieu of rescission because the
misrepresentation was relatively minor in relation to the £5m sale; the cost of
diverting sewer at £18,000 was much less than the loss to the representor (refund
if purchase price + interest is about £8m).

b. Tiong Swee Eng v Yeo Khee Siang – In this case, P and D had a settlement
agreement in relation to matrimonial assets for divorce. P later discovered that
assets were omitted from the agreement. P relied on the misrepresentation but
the Court held that this was not a deliberate omission from D. In fact, P herself
(as a joint owner of the significant asset) had also forgotten about the property.

212
The Court exercised its discretion under S2(2) of the Misrepresentation Act and
declined to order rescission as it was disproportionate on the facts. The
misrepresentation did not go into the heart of the agreement (less than 1% of
assets); P did not suffer any harm as she was still the joint owner; rescission
would throw parties back into litigation with little likelihood a better outcome
could be achieved.

c. RBC v Defu Furniture – In this case, the Court held that it would not award
damages in lieu of rescission. This was because the misrepresentation went to
the heart of the contract and there was little merit in upholding the lease. In
particular, damages in lieu of rescission would usually be awarded where the
misrepresentation is slight or relatively unimportant, such that rescission would
be disproportionately harsh on the representor.

Indemnity on rescission

320. A representee is entitled to recover costs incurred in discharging legal


obligations necessitated by the contract (Whittington v Seale Hayne, RBC v Defu
Furniture). Only obligations which have been created by the contract into which he has
been induced to enter by the misrepresentation can be indemnified (RBC v Defu
Furniture).

a. In Whittington v Seale Hayne, In this case, Mr Whittington bred prize poultry.


He bought a long farm lease, induced by Seale-Hayne’s representation that the
premises were sanitary and in good repair. But the water supply was poisoned,
Mr Whittington’s manager got very ill and the poultry died. Under the lease, Mr
Whittington had covenanted to carry out repairs required by the council, and
these were needed after the council declared the premises unfit for habitation
and the drains needed renewing. It was undisputed that Whittington was entitled
to indemnity for rates paid or repairs costs. Whittington sought rescission and
indemnity for loss of poultry, profits and medical expenses. Whittington was
entitled to indemnity for the amounts spent on the rents, rates, cost of repairs
which were obligations under the lease were recoverable (i.e. in discharge of
the obligations) but not for the other losses (which was not linked to an
unwinding or reversal of the parties' obligations – e.g. loss of breeding season,

213
value of poultry stock lost, costs incurred for removal of storage and other
incidental losses).

b. RBC v Defu – In this case, Defu was entitled to recover certain payments made
to RBC, such as security deposit and rent, as these were obligations created
under the lease. However, fitting out cost is not recoverable as it was not an
obligation on Defu under the contract to fit out the premises.

Exclusion clauses/basis clauses/non-reliance and no representation clauses

321. At common law, a person could not contract out of liability for fraud inducing
the making of a contract with him where the fraud was his own (S Pearsonn v Dublin
Corp).

322. However, at common law, it is possible for a person to:

a. Exclude liability for misrepresentation; or

b. State that a party has not relied on a non-contractual representation (the so called
"contractual estoppel", see Springwell Navigation Corp v JP Morgan).

i. Parties could agree that a state of affairs would be the basis of their
contractual dealings with one another, even if they knew that it was not
the case (Springwell at [143]).

ii. A contractual estoppel unlike other estoppels do not require a


demonstration that it would be unconscionable for the other party to
resile from the common state of affairs that the parties have assumed, or
reliance (Springwell at [177]).

iii. Kelry Loi argues that in reality, this is merely an enforcement of a


contractual term – a party is precluded from resiling from such clauses
because to do so would be a breach of a contract. Preclusion is the
remedial consequence of the threatened breach (Contractual Estoppel
and Non-reliance clauses). This has been endorsed in Chen v Ng at [30].

323. The question is how effective are such clauses? S3 of the misrepresentation
act limits the enforceability of clauses which attempt to exclude or restrict liability

214
for pre-contractual misrepresentation or remedies available for such
misrepresentation, unless it is shown to be reasonable as defined in S11(1) of UCTA.
The onus is on the party relying on the term to show that it is reasonable.

Agency situation

324. Terms which restrict the authority of an agent falls entirely out of S3 of the
Misrepresentation Act. The principal is entitled to limit the ostensible authority of his
agent (Overbrooke v Glencombe Properties).

a. In this case, the auctioneer misrepresented that the authorities had no plans for
the land. The auction catalogue stated that the auctioneer was not authorized to
make representations or warranty on the property. It was held that the auction
contract could not be rescinded and the buyer had to make payment. The
condition was valid – it was not to exclude liability, but to define their duty so
that responsibility for the statement was never assumed (i.e. in the first place no
duty owed, maintained no such duty owed).

325. However, where the term merely disclaims responsibility for the statements by
the agent, but the agent had authority to make representations, such a clause was
subject to the reasonableness analysis under S3 of the Misrepresentation Act
(Cremdean Properties v Nash).

a. In this case, C contracted to buy property from N. C relied on representations


by N's agents that there was planning permission for 17,900 square feet of
officers (they wanted to redevelop the land). The true figure was much lower.
Cremdean sought rescission or damages for misrepresentation, N sought to rely
on a footnote clause in the invitation to tender document that said such
statements are believed to be correct but their accuracy not guaranteed, errors
would not annul the sale and pre-contractual statements did not form part of the
offer. The purchaser must satisfy himself by inspection. C argued that this
exclusion fell within S3 of the Misrepresentation Act and was unreasonable.
The Court held that the footnote was an exclusion clause within S3.

215
b. "Humpty Dumpty would have fallen for this argument. If we were to fall for it,
the MA would be dashed to pieces which not all the King's lawyers could put
together again." – Scarman LJ.

Representations made by the representor

326. Although there is some dicta that non-reliance/no representation clauses are
outside the scope of S3 of the Misrepresentation Act (see Watford Electronics v
Sanderson at [40]), this view has been largely discredited in the modern cases.

a. Thomas Witter v TPB – In this case, the clause provided that the purchaser
acknowledges that it has not been induced to enter into this Agreement by any
representation or warranty other than statements contained or referred to [in the
schedule]. TWL bought the business from TIPL. TIPL and its parents made
certain misrepresentations as to the accounts of the business in question. TWL
sought to rescind the contract. The Court held that the clause was subject to S3
of the Misrepresentation Act and that it was unreasonable as it excluded
fraudulent misrepresentation.

i. Quare if the decision is technically correct – The court took a binary


approach towards interpreting the clause. The misrepresentation on the
facts was non-fraudulent, but the Court struck the clause down
wholesale. Cf cases such as Tjoa Elis v UOB and fraud from third party.
Probably need to limit it to negligence at best?

b. Watford Electronics v Sanderson – In this case, a clause which stated that "these
terms and conditions… represent the entire agreement… and that no statement
or representations made by either party have been relied upon by the other in
agreeing to enter into the contract" was held to be outside of S3 of the
Misrepresentation Act (at [40]-[41]).

i. Note – this has been read down in First Tower Trustees v CDS at [56] –
Chadwick LJ was not interpreting the clause in question (clause 14), but
a separate clause (clause 7.3).

c. Springwell – The court seemed to accept that non-reliance clauses ("without


reliance on CMB or CMIL or CMp… you have represented… that… you are

216
fully familiar with and able to evaluate the merits and risks associated…" were
not exclusion of liability clauses but contractual estoppel clauses which fell out
of S3 of the Misrepresentation Act. There was separately a clause which states
that "the Holder has not relied on and acknowledges that neither CMSCI nor
CMIL has made any representations…" which the Court consider it to be an
attempt to exclude/restrict liability.

i. The latter was picked up in First Tower Trustee v CDS at [62].

d. Deutsche Bank AG v Chang Tse Wen –In this case, the Court suggested that
clauses which defined the scope or nature of the relationship between the parties
would be subjected to the UCTA where applicable, as UCTA is concerned about
the substantive effect of the term, rather than the form or identification (at
[63],[68]).

i. Note however, there is a specific provision in UCTA which provides for


this, see S13(1) UCTA – stuff which excludes or restricts the relevant
obligation or duty.

e. First Tower Trustees v CDS – In this case, the EWCA took the view that a clause
which defined the party's primary obligations under the contract would not be
subject to S3 of the Misrepresentation Act, because it merely defines the parties'
obligations. However, where the duty is imposed by law and not because it was
a term of a contract agreed between the parties, such a contractual estoppel
clause is seeking to exclude a liability which would otherwise be there. In this
regard, the Misrepresentation Act creates a statutory tort which filled a
perceived gap in the common law by imposing a liability in damages for loss
cause by a misrepresentation without the need to show fraud. This is by
operation of law. A term which tries to claim "no reliance" would be a term
excluding liability (at [97]).

217
Chapter 8: Mistake

Introduction

327. It is normal for books on contract to have a special chapter on mistake. This is
traditional. But it does not follow that there are any special rules for mistake: what these
chapters may be saying is rather that the fact that one or both parties are mistaken may
trigger off other rules. One difficulty with the ‘mistake’ argument is this: at common
law, the argument is premised on the notion that the mistake renders void the agreement
between the parties. Mistakes are common enough in everyday life. Allowing mistake
to be used liberally would mean that there is a ready means of escape for those who
were mistaken or who would use mistake to get out of bad bargains. This is not
consistent with the common law orientation to uphold agreements rather than to
discharge them. The consequence – rendering void the contract – also gives room for
pause. A wide doctrine of mistake potentially undermines the security of transactions
and puts at risk third party interests.136

328. Mindy has pointed out that the law in this area is riddled with very difficult
distinctions, some of which have only superficial logic and can cut across each other to
yield contradictory results.

Communication/formation mistakes

Mutual mistake/latent ambiguity

329. In most cases, the application of the objective test of formation will preclude a
party entering into a contract under a mistake from setting up his mistake as a defence
to an action against him for breach of contract. However, where the parties are
genuinely at cross-purposes as to the subject matter of the contract, the result may be
that there is no offer and acceptance of the same terms because neither party can
show that the other party should reasonably have understood his version.
Alternatively, the terms of the offer/acceptance may be so ambiguous that it is

136
Contract law NUS reading list 2018/19.

218
impossible to point to one or other interpretations – there is no contract formed
(Wellmix Organics v Lau Yu Man at [58]).

a. Wellmix Organics v Lau Yu Man SGHC – In this case, there was a dispute as
to whether an unless order was a consent unless order. The SGHC held that
defendant counsel was under the impression that the compromise arrived
between the parties was one where the parties would exchange their respective
AEICs. Counsel for plaintiff however was not (he wanted an additional potential
sanction of an unless order – a consent unless order when breached would have
severe consequences as opposed to a mere unless order) – at [58], ]: “Put simply,
this particular aspect of the law relating to mistake is simply the result of a lack
of coincidence between offer and acceptance. In other words, both parties are
at cross-purposes and hence, no agreement or contract has been formed as a
result.”

b. Raffles v Wichelhaus – In this case, the defendants contracted to buy a cargo


of cotton to arrive "ex Peerless from Bombay". There were two ships of that
same name and both sailed from Bombay, one left in October while the other in
December. The description of the goods pointed equally to either cargo. In an
action for refusal to accept goods from the December shipment, the defendant
pleaded that the agreement referred to the October one. The plaintiff demurred,
but the court gave judgment for the defendants, apparently taking the view that
it was open to the latter to adduce parol evidence as to which ship was meant
(no evidence was given). The judgment does not indicate what the position
would be if the parol evidence failed to point to one cargo rather than the other,
but the court did not express any disagreement with counsel’s proposition that,
if the defendant meant one Peerless and the plaintiff the other, there would be
no contract.

i. Note – In a modern case of a similar character it would have to be shown


that each party’s interpretation was as reasonable as the other’s, and it is
unlikely that the facts proved would be so sparse as not to give some
ground for adopting one interpretation of the contract rather than the
other.

219
c. Tamplin v James – In this case, James purchased a public house at auction,
believing it included a field the previous publican had used. The particulars
stated the property correctly, but James did not read them. James, on
discovering his mistake, refused to complete the transaction – the Court made
an order for James to specifically perform the contract (i.e. he must buy).
Objectively, the reasonable man would assume that the sale was made on the
basis of the particulars, hence no objective offer and acceptance at cross
purposes.

Unilateral mistake

As to identity

330. The starting position is that an offer can only be accepted by the offeree to whom
it was addressed (Shogun Finance v Hudson, Tribune Investment Trust v Soosan
Trading). The key issue is who objectively the court will construe that the innocent
party intended to deal with (i.e. the fraudster or someone else). If the objective
interpretation of the parties' communications shows that the claimant's offer was meant
for the fraudster, the acceptance by the fraudster is effective – a mistake as to the
attributes is insufficient (e.g. solvency, character, or social position).

a. The criticism is "ultimately this must refer to a physical person, but a


physical person can only be identified by describing his or her attributes"
– Peter Millett LJ, Shogun Finance v Hudson at [73].

b. "A man's very name is one of his attributes. It is also a key to his identity. If
then, he gives a false name, is it a mistake as to his identity? Or a mistake as to
his attributes? These fine distinctions do no good to the law" – Denning MR,
Lewis v Averay at 207.

331. The cases however have not been wholly consistent with their outcomes. The
following are some key factors which the court will consider.

332. "Generations of law students have struggled with this problem. They may be
forgiven for thinking that it is contrived by their tutors to test their mettle. After all, the
situation seems artificial and is one which is seldom likely to arise in practice, at least
in the absence of fraud. Unfortunately, fraudulent impersonation is not at all uncommon

220
today. The growth in the number of credit transactions, often entered into electronically
between persons unknown to each other, has led to a surge in what has been called
"theft of identity" – Peter Millett LJ, Shogun Finance v Hudson at [57].

Importance of the identity/Existence or non-existence

333. The Court may find that the identity was not important to the offeror (e.g. TITI
v Soosan at [46] – "Admittedly, the identity of the person with whom one is contracting
or proposing to contract is often immaterial… [T]his was not a case of an over-the-
counter cash sale or a sale concluded at a public auction. In these instances, there is no
doubt that the identity of the buyer would be completely irrelevant to the seller for the
seller is only concerned with selling his wares at the agreed price. It does not matter to
him who is buying the goods."

a. If the claimant failed to check the fraudster's identity, this strongly suggests that
the identity was unimportant to him.

b. If the entity is non-existent, it is easier to establish that the identity was


unimportant (King's Norton Metal v Edridge, Merrett and Co).

i. In this case, K sent goods in response to an order from the fictitious


'Hallam & Co', written on headed stationery with a picture of a large
factory and a list of overseas depots. The fraudster then sold the goods
to E. Although the contract was in writing, the contract was only
voidable – K intended to contract with the writer of the letter, being
mistaken only as to his attributes – solvency and respectability. If it
could be shown that there was a separate entity called Hallam & Co,
then the case might come within the decision in Cundy v Lindsay.

ii. Note: This case is problematic – why is it not enough that K believed H
existed when it did not? If the rationale is simply that B cannot accept
an offer known not to be meant for her, why did this not apply?

c. If there is a particular reason why the identity is important, then this is a factor
in favour for interpreting that the identity is important (Bolton v Jones).

221
i. In this case, the defendant had been used to deal with Brocklehurst,
against whom he had a set-off. He sent Brocklehurst a written order for
some goods (naming Brocklehurst as the party). On the very day that the
order was sent, Brocklehurst had transferred his business to his foreman,
the plaintiff. The plaintiff dispatched the goods without informing the
defendant of the change of ownership. The Court held that the defendant
was not liable as there was a void contract – the identity was important
as the defendant had a right to set-off against Brocklehurst, had
previously dealt with Brocklehurst and intended to rely on that set-off
right.

Written contracts

334. Where a contract is reduced to writing, it can only be between the persons
named in a written contract as the parties to the contract (Cundy v Lindsay, Shogun
Finance v Hudson). The identification of the parties to the agreement is a question of
the construction of the putative contract. If an individual is unequivocally identified by
the description in the writing, that precludes any finding that the party to the agreement
is anyone other than the person so described (Shogun Finance v Hudson). This will
apply to e-mail and written correspondence. The face to face rule may be extended to
calls however.

a. Cundy v Lindsay – In this case, a fraudulent person named Blenkarn wrote to


the plaintiffs offering to buy certain goods, and so contrived his signature ot
resemble that of Blenkiron & Co, a prosperous firm carrying on business in the
same street and with whom the plaintiffs had previously dealt. The plaintiffs
despatched the goods in the belief that they were dealing with Blenkiron & Co
and the goods eventually came into the hands of an innocent purchaser, the
defendant. The Court held that the mistake was one as to the identity of the
contracting party and the contract was void.

i. This decision was upheld on the basis that the negotiations were by
correspondence and the offer was made only to the person identified in
the writing (i.e. respectable firm of Blenkirons) by the majority in
Shogun Finance v Hudson.

222
ii. However, the minority in Shogun doubted this approach – Lord Millett
accepted that a person cannot accept an offer that is made to another, but
there should be a presumption that the mistaken party intends to deal
with the person with whom he is physically dealing (present person or
writer of the letter). Lord Nicholls agreed that Cundy v Lindsay should
not be followed.

b. Shogun Finance v Hudson – In this case, a rogue wanted to acquire a vehicle


displayed by a car dealer and showed the dealer a driving licence in the name
of a Mr Patel. The dealer contacted the claimants and after the claimants had
checked Mr Patel's credit details, a financing agreement with the claimants was
arranged in the name of Mr Patel. After the rogue had paid a deposit partly in
cash and partly by cheque (which was later dishonoured), the dealer allowed the
rogue to take the vehicle. The defendant bought the vehicle in good faith.The
defendant claimed that he was protected by Hire Purchase Act 1964 s.27. This
provides that when a motor vehicle has been bailed under a hire-purchase
agreement or agreed to be sold under a conditional sale agreement, and before
the property has vested in the debtor he disposes of it to a private purchaser who
buys it in good faith, the purchaser will obtain good title. The UKHL by a
majority (3:2), held that the defendant had not acquired the vehicle from a
“debtor” under a hire-purchase agreement as there was no valid agreement. The
finance company was only willing to do business with the person who appeared
to have identified himself in the written document – Mr Patel.

i. "Surely it is fairer that the party who was actually swindled and who had
an opportunity to uncover the fraud should bear the loss rather than a
party who entered the picture only after the swindle had been carried out
and who had none", Peter Millet LJ dissenting at [82].

Face to face contracts

335. When the parties are dealing with face-to-face, there is a strong presumption
that the mistaken party "intends" to deal with the person physically present (Shogun
Finance Ltd v Hudson).

223
a. Phillips v Brooks – in this case, one North entered into the plaintiff's shop and
selected several pieces of jewellery. He wrote out a cheque for the price, saying
"I am Sir George Bullough" – a person known by reputation to the plaintiff. He
took away some of the jewellery and pledged it with the defendant who received
it in good faith. P had checked up the information, finds that there is such an
individual at the directory. The plaintiff sought to recover the jewellery pledged,
arguing that there was a mistake as to identity. It was held that the plaintiff
intended to contract with the person in the shop.

b. Ingram v Little – In this case, a rogue, in the course of negotiating to buy a car
from three ladies, who were reluctant to take his cheque, stated that his name
was one Hutchinson and gave an address in Caterham. One of the ladies went
to the local post office and ascertained from the telephone directory that there
was indeed one Hutchinson who lived at that address. The ladies parted
possession of the car in exchange for a worthless cheque. The majority held that
the identity of Hutchinson was significant and it was with him that the vendors
intended to deal. Devlin LJ dissented – the identity of the purchaser was
immaterial, although his creditworthiness was not.

i. Doubted in Lewis v Averay which declined to follow it, nearly


overruled by the UKHL in Shogun Finance.

c. Lewis v Averay – In this case, the plaintiff advertised his car for sale in a
newspaper. A rogue telephoned and asked to see it. He arrived and told the
plaintiff and his fiancée that he was Richard Green (actor of Robin Hood – well
known and famous). A sale was agreed and the rogue wrote out a cheque for
the purchase price. The plaintiff allowed the rogue to take the car before the
cheque was cleared, whereupon the rogue produced an admission pass to
Pinewood Studios with an official stamp on it bearing Richard Green's name
but with the rogue's photograph. On sight of the pass, the plaintiff allowed the
rogue to take the car and the documents that related to it. The cheque bounced
and the rogue sold the car to the defendant, pretending at this point he had the
plaintiff's name. The EWCA held that there was a valid contract and good title
had passed.

224
As to terms

At common law

336. A contract is void for unilateral mistake if (a) one party had made a mistake, (b)
the mistake must be fundamental as to a term, and (c) the non-mistaken party had actual
knowledge of the mistake (Chwee Kin Keng v Digilandmall).

337. The mistake must be as to the terms (Smith v Hughes, Hartog v Colin & Shields,
Statoil ASA v Louis Dreyfus Energy Services)

a. Smith v Hughes – In this case, a racehorse trainer agreed to buy oats from a
farmer after inspecting a sample, which the trainer believed to be old oats. When
the oats turned out to be new and useless to him, the trainer refused to pay for
them. The jury did not distinguish clearly between the seller's knowledge that
(a) the buyer believed the oats too be old, or (b) that the seller was promising
them to be old. The lack of evidence for (b) suggested that the jury had
erroneously voided the contract on (a) and a retrial was ordered.

b. Hartog v Colin & Shields – In this case, a seller offered to sell 3,000 Argentine
hare skins at a fixed price "per pound" when he really meant "per piece". Since
there were three pieces to the pound, he mistakenly offered the hare skins at
one-third of his intended asking price. The buyer purported to accept and sued
for damages when the seller refused to delivery at that low price. The Court
found no contract, as they had always discussed the price "per piece" and never
"per pound". The buyer thus must have realised that a mistake had occurred in
the seller's offer.

c. Statoil ASA v Louis Dreyfus Energy Services – In this case, the parties had
reached a compromise over the amount of demurrage due. One party had made
an offer, basing its calculations on a mistaken assumption as to the date the ship
had completed its unloading. The mistaken party was not entitled to relief even
though the other party was aware of the mistake when it accepted the offer and
decided to say nothing – it was not a term of the contract that the discharge
was completed on the date the claimant supposed.

338. The mistake must be fundamental (Chwee Kin Keong v Digilandmall at [34])

225
a. CKK v Digilandmall – at [34], "[I]t does not follow that every mistake would
vitiate a contract. It has to be a sufficiently important or fundamental mistake
as t a term for that t happen. There is no dubt that the error in the present case
as to the price is a fundamental one. Accordingly, it is wholly unnecessary for
us to deal with the question as to what nature of mistake would constitute a
serious mistake sufficient to vitiate a contract. It is also unnecessary for us to
address a related controversial question whether a mistake as to quality, or the
substance of the thing contracted for, is of sufficient gravity to negate an
agreement".

b. Affirmed in Broadley Construction v Alacran, Olivine Capital Pte Ltd v Chia


Chin Yan.

339. The non-mistaken party must have actual knowledge of the mistake (Chwee
Kin Keong v Digilandmall at [41]).

a. "Absence of an express admission or incontrovertible evidence, the fact of


knowledge would invariably have to be inferred from all the surrounding
circumstances, including the experiences and idiosyncrasies of the person and
what a reasonable person would have known in a similar situation" CKK v
Digilandmall at [41].

b. CKK v Digilandmall – In this case, D's online portal mistakenly posted laser
printers for sale at a much lower price ($66 dollars vs $3,854). A total of 4,086
printers were sold online due to the error, of which the appellants had bought a
significant number (in the thousands). It was held that the buyers had actual
knowledge of the mistake (email evidence showing that appellants know they
posted wrong price).

c. Broadley Construction v Alacran Design – In this case, BC and AD entered


into a contract for the supply of equipment, which BC used to fulfill its
contractual obligations with its main contractor. The MC stopped paying the
BC, and consequently BC began to default on its payments. BC, AD and MC
discussed and signed an undertaking which authorized the MC to pay AD on
BC's behalf, and stated that BC was released of all its liability to AD. MC did
not pay and AD sued BC for the outstanding sum. It was argued that AD was

226
mistaken as to the effect of the undertaking and that BC was aware. It was held
that there was no operative mistake. In addition, there was no actual knowledge
by BC – BC was entitled to assume that AD had read the undertaking (AD took
the document back to office) which contained the express wording of the release
of liability.

d. Quare if actual knowledge requirement is correct – See Mindy's argument in


Contractual mistake, intention in formation and vitiation: the oxymoron of
Smith v Hughes.

In equity

340. A contract is voidable for unilateral mistake if (a) one party had made a mistake,
(b) the mistake must be fundamental as to a term, (c) the non-mistaken party had
constructive knowledge of the mistake with an element of impropriety such as sharp
practice or unconscionable conduct (Chwee Kin Keng v Digilandmall).

341. The non-mistaken party had constructive knowledge of the mistake with an
element of impropriety such as sharp practice or unconscionable conduct

a. Chong Sze Pak v Har Meng Wo – P purchased the property from D at $315 per
square food. However, the contractual document said that the final price will be
based on the actual built in area as shown in the title documents or according to
a survey by a registered surveyor. P sought to reduce the price as the property
was much smaller. The Court found that built in area meant the area of the flat
and not the land. However, the court also found that there was a mistake by the
defendant – the purchaser knew or would have known that at the low price of
$315 per square foot, the area would have to be 1,633 sq ft. All the documents
relating to the land was shown to be 1,633 sq ft. P knew that the seller was
mistaken, and allowed them to remain under their delusion instead of pointing
out the mistake. It would be unconscientious for the purchaser to avail himself
of the legal advantage at the lower price.

b. Broadley Construction v Alacran Design – On the facts, equitable unilateral


mistake is not made out. BC had remained silent at the meeting, drafted the
agreement with the exclusion of liability, sent the agreement (barely 10 lines

227
long) and gave AD ample time to review and sign agreement. BC had thus no
suspicion that AD was mistaken as to the terms of the agreement.

c. Note: UK position uncertain whether there is a doctrine of unilateral mistake in


equity – Statoil ASA v Louis Dreyfus at [105], without a valid misrepresentation.

d. Note: Phang J (as he then was) prefers to have only 1 doctrine of unilateral
mistake – the difference lies not in the formulations of the doctrine, but the
consequences and actual application – Wellmix Organics v Lau Yu Man – should
have the same formulations of unilateral mistake, but just subsume it in equity
(i.e. always voidable).

Expectation/Common Mistake

Mistaken assumptions at law

342. A contract may be set aside for mistaken assumptions if (a) there is no allocation
of risk to either party of the consequences occasioned by the mistake, (b) the mistake
was without the fault of either party, (c) the mistake is shared and relates to facts or law
before the contract was concluded and (d) the mistake must render the subject matter
fundamentally different from the subject matter which the parties contract on as
constituting the basis of their contract (Olivine Capital v Chia Chin Yan at [67]).

a. Note – the approach is broadly similar to that in Great Peace v Tsavliris and
Associated Japanese Bank v Credit du Nord SA.

b. Note – the approach is also similar to that of frustration. The difference is one
of timing.

Contractual allocation of risk

343. The first question is whether the contract has dealt with the risk of the mistake.
If the contract has done so, it governs the situation – only where the contract is silent
can the mistake doctrine apply (McRae v Commonwealth Disposals Commission,
Associated Japanese Bank v Credit du Nord).

a. Risk allocated to one party – McRae v CDC – In this case, CDC invited tenders
for an oil tanker lying on the Jourmand Reef which is said to contain oil. M

228
tendered successfully and embarked on an expensive salvage expedition but
neither the tanker nor the Jourmand Reef existed. M successfully claimed
damages for the breach – the Court construed the obligations and found that
there was a promise by the CDC that there was a tanker in the position specified
– i.e. risk of mistake was allocated contractually to CDC.

b. The contract may provide for how the risk is managed – Associated
Japanese Bank v Credit du Nord – In this case, a fraudster purported to sell to,
and then lease back from AJB 4 machines, which did not actually exist. CN was
sued as the fraudster's guarantor on the fraudster's bankruptcy. The EWCA held
that, since the guarantee stipulated that the machines could only be substituted
with CN's consent, this amounted to an express condition precedent that the
guarantee was for the lease of existing machines. Alternatively, the Court was
willing to imply such a condition on the facts.

Fault

344. A party cannot be allowed to rely on a common mistake where he had no


reasonable grounds for such belief (Associated Japanese Bank v Credit du Nord).
Where a party contracts with minimal knowledge of the facts to which the mistake
relates but is content that is a good speculative risk (reckless/negligent), that party will
be at fault (McRae v Commonwealth Disposals Commission).

Shared fundamental mistake

345. There must be a sufficient degree of disparity between the performance f the
contract on the mistaken state of affairs and its performance on the actual state of affairs.
The following are some illustrations of mistakes which may be sufficiently fundamental.

346. Mistake as to the existence of the subject matter (Res Extincta in Bell v Lever
Brothers).

a. Voided – Courtier v Hastie – Sale of thing that has ceased to exist – There was
a sale of a cargo of corn which was believed to be in transit from Salonica to
the United Kingdom. Unknown to either party the cargo had deteriorated and
had already been sold by the master of the ship. The liability of the purchaser to
pay the price depended upon the construction of the contract. If the contract was

229
a contract for the sale of that specific cargo of corn, then the consideration for
the contract had totally failed and the seller was not entitled to the price. If,
however, as the seller contended, it was a contract for the sale of the adventure,
the seller had performed his side of the contract by offering to deliver the
shipping documents and the purchaser was liable to pay the purchase price. The
House of Lords, confirming the decision of the Exchequer Chamber that had
reversed a contrary decision by the Court of Exchequer, held that the contract
was for the sale of a cargo and therefore the purchaser was not bound to pay.

i. Note case was not decided on mistake, but total failure of consideration
– language of mistake is not used, but subsequent cases regarded it as an
example of mistake.

ii. Note – codified in S6 of SGA – contract is void if goods for sale which
initially exist ceased to exist.

b. Voided – Scott v Coulson – In this case, a contract for the sale of a life policy
was entered into by both parties in the belief that the assured was still alive.
Between the date of the contract and the assignment, the purchaser had received
information which led him to believe that at the date of the contract, the assured
was dead, which after the date of the assignment was ascertained to have been
the fact, but the purchaser never disclosed this information to the vendor. It was
held that the vendor was entitled to have the contract set aside.

c. Note: Cf McRae v CDC and Assoc Japanese Bank v Credit du Nord for
contractual allocation of risk.

347. Mistake as to the ownership of the item (Res Sua in Bell v Lever Brothers).

a. Void if brought at common law – In Cooper v Phibbs, C agreed to lease a


fishery from P. Unknown to them, P had no title while C was already tenant for
life for the fishery. The lease was set aside.

i. The UKHL in Bell v Lever Brothers considered that the common law
would have ruled the contract void for mistake.

348. Mistake as to quality (Bell v Lever Brothers)

230
a. Not void – Bell v Lever Brothers – In this case, L paid £50,000 to terminate the
employment of two employees as part of its corporate reorganization. Unknown
to L, the employees had breached their contracts by speculating in cocoa on
their own account, thereby entitling L to dismiss them without compensation. L
sought the return of the £50,000 for fraud. When this failed, L relied on the
alternative ground of mistake – the UKHL held by a 3:2 majority that the
mistake was not sufficiently fundamental to void the contract.

b. Not void – In Leaf v International Galleries, the complainant bought a picture


which both parties mistakenly believed was painted by the famous artist
Constable. It was held that the contract was not void for fundamental mistake –
mistake is as to the quality – they were buying a painting, not a "constable"
painting.

i. Note the context – attribution of works of art to particular artists is a


matter of great controversy and increasing difficulty – divergent views
which may change.

c. Not void – Great Peace v Tsavliris – T agreed to provide salvage services to the
ship the Cape Providence, which had suffered serious structural damage in the
South Indian Ocean and was in imminent danger of going down with her crew
and cargo. Since the tug that T proposed to use was some fve days away, T
looked for merchant vessels in the vicinity that could evacuate the crew of the
CP in the meantime if necessary. Relying on information from a third party that
one vessel, the Great Peace (GP), was about 35 miles away, T hired the GP for
a minimum of fve days to divert to the CP as back-up pending the tug’s arrival.
In fact, the GP was 400 miles away while another vessel was substantially closer
to the CP. T refused to pay the hire, claiming that the contract was (i) void for
common mistake (ie that the GP was ‘in close proximity’ to the CP) under Bell
v Lever Brothers, or (ii) voidable for mistake in equity under Solle v Butcher
(1950) (6.3.6). The Court of Appeal rejected both grounds.

i. Note the context – the defendants did not want to cancel the agreement
with the Great Peace until they knew whether they could get a nearer
vessel to assist. The Great Peace would arrive in time, provide several

231
days of escort service – the contractual adventure was still possible to
perform.

d. Not void – Oscar Chess v Williams – In this case, the complainant purchased a
car which both believed to be a 1948 model, when it was in fact a less valuable
1939 model. The contract was not set aside.

e. Not void – Ho Seng Construction v Nian Chuan Construction – In this case,


HSL agreed to lease metal-forms to NC for NC's use at certain construction
projects. The quantities of metal-forms actually leased did not tally with the
quantities stated in the agreements (lesser), as NC had took over the project
from KS and they all simply based the rental on KS quantities (KS had removed
some metal-forms). When NC failed to pay the rental moneys, HSL commenced
proceedings. NC argued that the agreements were inter alia vitiated by common
mistake – although the quantities actually leased were less, they were more than
sufficient for NC to perform its work. Notwithstanding the shortfall, the subject
matter of the agreements was not essentially and radically different from that
which both believed to exist at the time the contract was executed.

Mistaken assumptions in equity

349. Singapore has accepted that there is a doctrine of common mistake in equity in
Singapore (Olivine Capital at [63], see also Chwee Kin Keong v Digilandmall.com).
However, there is some uncertainty as to what is the relationship between the doctrine
at common law and in equity.

a. Solle v Butcher – In this case, the parties agreed a £250 yearly rental when they
were in fact subject to a £140 limit under the Rent Act unless a notice of increase
was served. This was not served due to a common mistake about the status of
the property. The Court granted the landlord recission on terms of the lease, but
on the terms that he offer the tenant a new lease for £250.

b. This was doubted in Great Peace v Tsavliris – there are four reasons:

i. Solle v Butcher is s not merely supplementing or mitigating the common


law, but to outflank and contradict it.

232
ii. There is no precedent for a more expansive equitable jurisdiction in Bell
v Lever Brothers itself or any case earlier. Cooper v Phibbs is not good
authority for a lower threshold as it involved mistakenly acquiring one's
own property, which would have voided the contract at common law.

iii. Uncertainty – although in Solle v Butcher, Denning LJ stated that the


mistake must be fundamental, it should be "less fundamental" than that
at common law (otherwise, what is the point?). However, subsequent
cases have not identified the test of mistake which differs from that in
common law.

iv. Illegitimacy – largely motivated to avoid unjust outcome – unfair for


tenant to pay £140 rent when market rent is £250 for the remaining 5
years.

Rectification

Rectification of contractual documents for mistakes as to terms

350. The essential idea here is that something has went wrong with the recording of
the agreement. Such mistakes in the recording often occurs, even where documents are
prepared by professionals and are typically unavoidable (see e.g. in LBG Capital No.1
PLC v BNY Mellon Corporate Trustee Services where the EWCA corrected an obvious
mistake in a tradable financial instrument (definition of CET1)).

a. A written contract is not the contract itself, but only evidence/record of the
contract (Wake v Harrop). As such, Courts only rectify documents and not the
contractual bargain – the documents are changed so as to reflect what is actually
agreed to (Agip SpA v Navigazione Alta, "The Nai Genova"). In order to order
rectification, the court must be satisfied first that there was a prior agreement
between the parties, and that the contractual document did not reflect that
agreement (Etablissements Georges v Adderley Navigation, "The Olympic
Pride").

b. Rectification for mistake is not precluded by an entire agreement clause – the


agreement which constitutes “the entire agreement” is to be found in the
contract as rectified and not in the contract which, ex hypothesi , does not reflect

233
the true intention or agreement of the parties (Borough of Milton Keynes v
Viridor at [78]).

i. In this case, P engaged D to carryout waste recycling in Milton Keynes


for 15 years. D is to make fixed and variable payments to the claimant.
In D's final bid, there was a fixed payment component "indexed for
inflation", but when the final contract documents were put together by
the claimant's consultants, an earlier version of the payment mechanism
was included which made no reference to indexation. The court held that
there was a common mistake which was shared by the parties (D had
notice of the gaps after the contract had been signed and pointed out the
gaps – thought it wasn't that prior to contract). In the alternative, if
Hamblin did notice the gaps prior, he did not alert P – D knew of mistake
and omitted to draw it to claimant's notice and could be liable for
unilateral mistake.

Common mistake as to terms

351. Rectification may be available if parties made a binding agreement to execute


a document containing particular terms, but instead executed a document containing
different terms. In such cases, the terms of the contract to which the subsequent
document is made to conform must be objectively determined (Chartbrook Ltd v
Persimmon Homes, FSCH Group Holdings v GLAS Trust Corporation).

352. However, rectification may also be available if the parties reached a common
consensus on an issue, but this fell short of a binding agreement and they executed a
document containing different terms (Joscelyne v Nissen, FSCH Group Holdings v
GLAS Trust Corporation).

a. Joscelyne v Nissen - A father entered into a written contract with his daughter
by which he transferred to her his car hire business in return for her agreement
to pay him a pension and discharge certain expenses. In their discussions it had
been agreed between them that these expenses should include the father's gas,
electricity and coal bills and the cost of home help. However, the daughter
argued, and the trial judge held, that the signed contract did not on its proper
interpretation provide for payment of these expenses. The judge nevertheless

234
rectified the written contract to provide for these expenses to be paid by the
daughter, although he found that no binding contract had been concluded
until the document was signed.

353. For rectification to be available, it must be shown that (Chartbrook Ltd v


Persimmon Homes, FSHC Group v Glas Trust):

a. The parties had a common continuing intention, whether or not amounting to


an agreement, in respect of a particular matter in the instrument to be rectified;

b. There was an outward expression of accord;

c. The intention continued at the time of the execution of the instrument sought to
be rectified;

d. By mistake the instrument did not reflect that common intention.

354. Where rectification is (FSHC Group v Glas Trust):

a. Based on a concluded binding antecedent contract as to what would be in the


executed document, then the antecedent agreement to be enforced must be
construed objectively (at [141]);

b. Where it is based on a non-binding antecedent common intention, rectification


is made pursuant to the equitable principle of good faith and conscience and a
subjective approach is to be taken to construing the common intention.

355. Illustrative cases

a. Rose v Pim – No literal disparity – The plaintiffs, who were London merchants,
had been asked by Egyptian buyers to supply "feveroles". Not knowing what
this term meant, they asked the defendants' representative, who responded that
"feveroles" meant horsebeans. Relying on this information, the plaintiffs
contracted to buy a quantity of horsebeans from the defendants, which they then
sold on as "feveroles" to the Egyptian buyers. To fulfil the contract, the
defendants purchased "horsebeans" from an Algerian supplier. There are in fact
different varieties of horsebeans and those supplied were "feves", which were
less valuable than "feveroles". The Egyptian buyers claimed the difference in

235
value as damages from the plaintiffs, who then sought to rectify their contract
with the defendants by adding the word "feveroles" after the references to
"horsebeans".

i. Note: doubted in FSHC at [71]. The problem is that Rose v Pim was
decided at a time when the law had not yet recognized that rectification
was available where parties had deliberately chosen words but were
mistaken about their meaning – best justification was that rectification
involved amending the contract which was a part of a chain of contracts
involving third parties – it would not be fair to rectify one of the
contracts without doing so for all three, which is obviously impossible
(at [71]).

b. Kok Lee Kuen v Choon Fook Realty – D granted 1st P an option to purchase a
property. During negotiations conducted between the 2nd D and the independent
agent, which was on the basis that the Ps wanted both the land and the houses
in the development for rental collection in the first instance but intended to
redevelop the land eventually, Ps were given a site plan showing that the
property had an area of 23,090 sqft. However, they did not know that the land
area included, as a separate parcel, a thin strip of land fronting the road that was
owned by the remaining Ds, without which the development would be
landlocked. The strip of land only had a public drain, and on its own had no
commercial value to the owner. Ps only intended to buy the entire site, assuming
that the development sat on the entire area. Ps sought rectification of the option
to include the additional lot as Ds refused to convey the lot without further
consideration. The Court allowed rectification as it was clear the Ds intended to
sell the whole development site (pointless to keep additional lot without value,
landlocked land without that lot), holding that knowledge of the designation of
the additional lot was not material – it was what the parties intended to sell and
buy that mattered. The standard of proof is the civil standing of balance of
probability, but convincing proof is necessary to rebut the written document.

236
Unilateral mistake as to terms

356. Rectification may be allowed for a unilateral mistake in recording the contract
if (1) the non-mistaken party had actual knowledge of the mistaken party's intentions
and of the mistake; (2) the non-mistaken party had failed to draw the mistaken party's
attention to the mistake; (3) the mistake must be such that the non-mistaken party would
derive a benefit or the mistaken party would suffer a detriment, if the inaccuracy in the
document were to remain uncorrected. It is not necessary that the conduct amounts to
fraud, all is necessary is that the knowledge or conduct of the non-mistaken party must
be such as to make it inequitable for that party to object to rectification (Sheng Siong v
Carilla at [67]).

a. Historical rationale is that the higher threshold is needed because the contract is
enforced on the mistaken party's understanding. This has been criticised – the
doctrine should be regarded as situations where the courts give effect to the true
agreement between the parties as objectively determined (Littman v Aspen Oil
at [33] for some support, see also The "Drastic" Remedy of Rectification for
Unilateral Mistake).

357. Illustrative cases

a. Thomas Bates v Wyndham – T's fixed term lease was renewed in 1970. It
provided for a rent review after five years: the new rent was to be such "as shall
have been agreed" between lessor and lessee. The parties had intended to
include a provision for arbitration in default of agreement. T realised, but did
not point out to L, that there was no such provision. In 1975 L sought to review
the rent. The judge granted rectification of the rent review clause to include a
provision for fixing the new rent by arbitration in default of agreement, and
granted L a declaration that the rent on review was to be a market rent. On
appeal by T, held, allowing the appeal in part, that (1) although L's mistake had
been unilateral, the parties had had a common intention to include the clause for
arbitration, and L had thought that the provision was included; T had known of
the omission and failed to bring it to L's notice, and had benefited thereby;
accordingly, rectification should be ordered; (2) however, the rent review clause
referred to such rent "as shall have been agreed between the parties" as opposed

237
to the rent "agreed for the demised premises," and the rent on review should be,
not the market rent, but the rent at which it would be reasonable for the
particular parties to agree having regard to all the circumstances.

b. Centrovincial Estates v Merchant Investors – In this case, the defendants were


tenants of offices with an annual rent of £68,320, subject to review from 25 Dec
1982. The rent review clause stated that on that date, the rent should be increase
to current market rental value as agreed between the parties, and in no
circumstances should rent be lower than figure payable immediately before the
review. Agreement was reached on a figure of £65,000 before the plaintiffs
realised that they had made a mistake, since they claimed that they had intended
to propose a figure of £126,000. The EWCA refused to find that there was an
operative mistake, on the ground that it had not been established that the
defendants knew or ought reasonably to have known of the plaintiffs' error when
they reached agreement on the figure of £65,000. – EWCA thought it was at
least arguable that defendants considered that rent might be reduced, despite of
the proviso in the rent review clause.

c. Littman v Aspen Oil – The lease was of commercial premises in London for a
term of five years. Clause 10 of the lease provided that either party could
terminate the lease at the end of the third year of the term by giving not less than
six months' notice in writing provided that up to the termination date "in the
case of a notice given by the landlord" the tenant had paid the rent and duly
observed and performed the tenant's covenants. T purported to bring the lease
to an end pursuant to that clause. L disputed the effectiveness of that notice on
the grounds that T was in breach of its obligations under the lease as at the
termination date and contended that the words "in the case of a notice given by
the landlord" were nonsensical and should be read as if they said "in the case of
a notice given by the tenant" or "in the case of a notice given to the landlord".
On appeal, T accepted that the clause as it stood was commercially nonsensical.
The EWCA held that there had been a mistake in drafting the clause. It was the
kind of mistake that could easily be made by any draftsman – it was argued that
to allow rectification would be to impose a contract which the non-mistaken
party would never want – "Besides in reality Mr Levan's conduct amounted to

238
an agreement—by accepting the clause knowing what the other side thought it
meant, he was accepting just that and equity should not allow him to resile."
Per Jacob LJ at [24].

d. Sheng Siong Supermarket v Carilla – In this case, P sought to rent from D a 3


storey leasehold property, which HDB was the reversionary owner. A MTS was
drafted by D and signed by parties. Cl 10 of the MTS provided that 40% of the
General Floor Area (GFA) is for retail and 60% is for entertainment, offices,
etc. MTS further provided that the terms would be incorporated into the final
tenancy agreement. However, the subsequent drafts and final version of the
tenancy agreement omitted various terms in the MTS, in particular Cl 10. Annex
1 of the executed tenancy agreement however included a plan of the Premises,
which depicted a supermarket. HDB rejected proposals that the premises be
used as a supermarket. D conveyed this to P and asked that it adjust the use of
the premises accordingly. P sued for the return of the deposit paid under the TA,
taking the position that the lease was conditional on HDB's permission to use
the premises as a supermarket. The Court considered that had an action for
rectification been brought by P, the Court would have been inclined to allow for
rectification.

Non Est Factum

358. Non est factum is a specific category of mistake that operates as an exception to
the general rule that a person is bound by his signature on a contractual document, even
if he did not fully understand the terms of the document. If successfully invoked, the
document so signed is void. There must (1) be a radical difference between what was
signed and what was thought to have been signed, and (2) the party seeking to rely
upon the doctrine must prove he took care in signing the document (Mahidon Nichar
v Dawood Sultan Kamaldin, Saunders v Anglia).

a. Mahidon Nichar – In this case 3 siblings signed a deed in the mistaken belief
that it was to appoint Dawood as the sole administrator of their father's estate.
However, the actual deed was in fact a renunciation of their beneficial interests
in their father's estate. The Court held that this was radically different legal
outcomes. The 3 siblings were also not negligent in their signing – they were

239
lay and unsophisticated clients (modest education) going to the solicitors in the
expectation that the solicitors would look out and advise them properly. The
deed was thus void.

b. Saunders v Anglia – In this case, S wanted to help her nephew raise money on
the security of her house, provided she could remain there for life. She signed a
document without reading it, having broken her glasses and having been told
that it was a deed of gift to her nephew. She actually signed a sale of her house
to L, her nephew’s friend (who had agreed with the nephew to raise money in
his own name for the nephew because the latter wanted to avoid his wife’s
maintenance claim). L mortgaged the house to A but kept the money it raised.
Although a gift to her nephew seems fundamentally different from a sale to his
friend, the court held that it was not: her purpose of assisting her nephew by
raising money on her house would have been achieved by the actual sale, had
the nephew’s friend paid up. Furthermore she could at least have checked the
identity of the transferee and thus was negligent.

c. Lee Siew Chun v Sourgrapes – P signed a document granting a mortgage to a


bank over her property. Her son had tricked her into signing the document,
claiming it was a testimonial as to his character (did he have any?) P was semi-
literate in English, but could recognise numbers, her English name, her son's
English name, the address of her property. The documents contained details of
her property, floor area but no reference to her son's name. She signed the
document but did not check. The Court held that she was negligent – a
reasonable person with semi-literacy would have checked the documents at
least.

d. Note: fraud is not necessary to make out non est factum (OCBC v Frankel Motor
at [25]).

240
Chapter 9: Illegality

General illegality

Denying enforcement

359. The doctrine of illegality and public policy in the context of unlawful contracts
encompassed two stages of inquiry. The first stage of the inquiry was to ascertain
whether the contract was prohibited either a statute (expressly or impliedly) and/or an
established head of common law public policy. If the contract was thus prohibited, there
could be no recovery (enforcement) pursuant to the illegal contract. Where the
contract is not unlawful per se, but was entered into with the object of committing an
illegal act (e.g. contracts with the object of using the subject matter of the contract for
an illegal purpose, contracts entered into with the intention of using the contractual
documentation for an illegal purpose, as well as contracts which were intended to be
performed in an illegal manner), the principle of proportionality will be applied to
determine if the contract was enforceable (Ochroid v Chua Siok Lui at [64]).

Statutory illegality: Express prohibition

360. Where it is alleged that the contract is prohibited by statute, the court will have
to examine the legislative purpose of the relevant provision in order to determine
whether the provision was intended to prohibit the contract (and not merely the illegal
conduct). This is a question of statutory interpretation. The fundamental question is
whether the statutory provision concerned is intended to prohibit only the conduct of
the parties, or whether it is intended to prohibit not only the conduct but also the
contract (Ochroid v Chua Siok Lui at [27]-[28], Ting Siew May at [106]).

361. Some examples of express statutory illegality include:

a. Competition Act S35 – Cartelistic behavior – agreements between undertakings,


decisions by associations of undertakings or concerted practices which have as
their object or effect the prevention, restriction or distortion of competition
within Singapore are prohibited unless they are exempt…

b. Human Organ Transplant Act S14(1) – Sale of organs – [A] contract …. under
which a person agrees, for valuable consideration… to the sale or supply of any

241
organ or blood from his body or from the body of another person, whether
before or after his death or the death of the other person, as the case may be,
shall be void.

c. Moneylenders Act S14(2) – Loansharking – Where any contract for a loan has
been granted by an unlicensed moneylender, or any guarantee or security has
been given for such a loan — (a) the contract for the loan, and the guarantee or
security, as the case may be, shall be unenforceable….

362. Illustrative cases

a. Ting Siew May v Boon Lay Choo – Backdating option to circumvent LTV ratio
– This case involved an option to purchase a property, granted by the appellant
to the respondents on 13 Oct 2012. The Option was backdated to 4 October
2012 at the respondents' request, so that they could obtain a housing loan from
a bank on more favourable terms allowed prior to the amendment to MAS
Notice No. 632 issued by the MAS on 5 October 2023, which lowered the
permissible loan-to-value ratio of residential property loans for borrowers in the
respondents’ position. Subsequently, the appellant withdrew her offer as
provided in the Option, stating that she did not want to be a party to any illegality
or irregularity. Correspondence between the parties’ solicitors ensued. Amongst
other things, the respondents’ solicitors proposed exercising the Option on the
basis that it was dated 13 October 2012, the actual date of the appellant’s
signature, and that the respondents would obtain financing for the purchase on
that basis. However, no resolution was reached. The SGCA held that there was
neither express nor implied statutory prohibition of the Option (and also not
prohibited as a common law head of illegal contract). Nevertheless, it was found
that the Option fell within the principles of common law illegality and was
subject to a proportionality analysis.

b. Ochroid v Chua Siok Lui – MLA – Appellant lent monies to the respondent
(wholesale food business) pursuant to many agreements for the purchase and
resale of specified foods and food related product overseas (740 of them, see at
[9]). Each agreement was supported by a tax invoice from the respondent stating
the type, quantity and price of the goods to which it related. The agreements

242
stated that the funds were to be repaid with a profit on a stipulated date. The
appellant sued to recover the sums in contract (with the profit – i.e. enforce the
contract), and in the alternative, unjust enrichment (without the profit) in respect
of about 70 agreements. The respondent relied on the defence of illegality in
that the sums were loans and that the plaintiff was not a licensed moneylender
under the MLA. The SGCA affirmed the SGHC's position that the agreements
were loans and that the appellant were unlicensed moneylenders.

c. The Enterprise Fund II v Jong Hee Sen – SFA – EFII claimed against Jong for
breach of obligation as warrantor under deed of undertaking in respect of a sale
of shares. Jong argued that transaction was void for illegality. Court held that
EFII was not carrying on business of “dealing in securities” under SFA because
of a lack of “system and continuity”; transactional was considered incidental to
EFII’s business, no requirement to obtain licence.

363. Under this category, statutory illegality generally takes no account of the
parties' subjective intentions or relative culpability and can render such contracts
unenforceable, even where the infraction was committed unwittingly (Ochroid at [28],
St John Shipping).

a. Re Mahmoud v Ispahani – Plaintiff sold to the defendant 150 tons of linseed oil.
There was an Order in force made prohibiting the buying and selling of linseed
oil without a license the plaintiff had a licence to sell the oil, but the defendant
did not. The defendant misrepresented that he had the licence to buy the oil, and
the plaintiff was induced into the contract. The defendant subsequently refused
to accept delivery of the oil on the ground that the contract was illegal and that
the defendant had no licence. Held that, as the defendant had no licence, the
contract of sale was prohibited by the Order and was illegal. No claim could be
made under the contract.

Statutory illegality: Implied prohibition

364. Where the statutory provision is not clear, the Court may, in interpreting the
statute, hold that there is an implied prohibition of the contract (and not merely the
illegal conduct). However, the Court should not find so, unless there is a "clear
implication" or necessary inference" that this was what the statute intended. One

243
example may be where the contract had its whole object the doing of the very act
which the statute prohibited (Ting Siew May at [110]). Judicial reticence is warranted
as statutory illegality generally takes no account of the parties' subjective intentions or
relative culpability, rendering contracts unenforceable even if infraction was unwitting
(Ochroid at [28]).

a. Implied statutory prohibition not found – St John Shipping v Joseph Rank -


Plaintiff was a shipowner who had a charterparty with charterers to transfer
goods. The ship, initially sailed from US (and was not overloaded), stopped at
a port inbetween the trip to UK and took on additional cargo, which caused it to
overload. The master was prosecuted for an offence for overloading the ship,
was convicted and fined 1200 pounds. The defendants and another cargo-owner
withheld payment equivalent to the freight of the overall additional cargo
carried by the ship by which it was found to be overloaded. They contended,
when sued for the balance of the freight, that the ship owners were not entitled
to recover any part of the contract as they had performed the charter in an illegal
manner. The Court held that the legislation was to penalise and deter
overloading. Contract did not have as its primary purpose the aim of
overloading. Moreover, this would result in the forfeiture of sums vastly in
excess of criminal penalties for violation, especially in situations where the law
was unwittingly broken (Fine for contravening the Act was £1,200; fees
withheld for carrying the excess cargo were £ 2,295)

i. Devlin J: “For example, a person is forbidden by statute from using an


unlicensed vehicle on the highway. If one asks oneself whether there is
in such an enactment an implied prohibition of all contracts for the use
of unlicensed vehicles, the answer may well be that there is, and that
contracts of hire would be unenforceable. But if one asks oneself
whether there is an implied prohibition of contracts for the carriage of
goods by unlicensed vehicles or for the repairing of unlicensed vehicles
or for the garaging of unlicensed vehicles, the answer may well be
different. The answer might be that collateral contracts of this sort are
not within the ambit of the statute.”

244
b. Implied statutory prohibition not found – Archbolds v S Spanglett – In this case,
D manufactured furniture and owned a number of vans with "C" licences which
allowed them to carry their own goods but not to carry for reward the goods of
others ("A" licences). P were carriers with A licensed vehicles. P was under the
impression (misrepresented by D) that D had A licenced vehicles and employed
D to carry part of a load for them. D's van was loaded with the whisky from P,
which was later stolen due to the driver's negligence. P sued D for damages, D
relied on its own illegality – that D's vans did not have a A licence. The Court
held that the contract was not prohibited by implication under the statute – It
noted that the object of the Road and Rail Traffic Act 1933 was not to interfere
with the owner of goods or his facilities for transport, but to control and promote
the efficiency of transport, by prohibiting various licence holders from
encroaching on one another’s territory, and prescribing penalties for breaching
these boundaries.

c. Implied statutory prohibition not found – Ting Siew May – In this case, the
relevant statutory instrument (a notice issued by the Monetary Authority of
Singapore pursuant to the Banking Act) was intended to target financial
institutions (issued to banks) rather than private transactions/public at large.
Although backdated option was meant to enable respondent to obtain larger loan
than was allowed, there were other objects of the contract (sale/purchase of the
property). No implied statutory prohibition found.

Common law illegality: Established head of public policy

365. For illegality at common law, the contract may fall into one of the existing heads
of public policy. This includes (Ochroid at [29]):

a. Contracts prejudicial to the administration of justice;

b. Contracts to deceive public authorities;

c. Contracts to oust the jurisdiction of courts;

d. Contracts to commit a crime, tort or fraud;

e. Contracts prejudicial to public safety;

245
f. Contracts prejudicial to the status of marriage;

g. Contracts liable to corrupt public life/contrary to public morality;

h. Contracts restricting personal liberty; and

i. Contracts in restraint of trade.

366. Although the categories are not closed, the Courts will not readily add new
categories. There is also always the issue as to whether, as society changes, the existing
categories themselves will need to be modified or even (in extreme cases) be done away
with. The entire legal enterprise in this particular sphere is exacerbated by the fact
that…the very nature of public policy is both fluid and problematic (Ochroid at [30]).

367. Contracts prejudicial to administration of justice – Contracts falling under


this head include: contracts to stifle prosecution; maintenance (i.e. assisting or
encouraging litigation by another without having any legitimate interest in the litigation
or other justification); and champerty.

a. Champerty agreements: These include conditional fee agreements by one


party to support litigation brought by another for a share of proceeds recovered
(Otech Pakistan v Clough Engineering). In this case, Clough engaged Otech to
assist in concluding a negotiated settlement with a third party in relation to
certain arbitration proceedings. One of the issues was, assuming that there was
a conditional fee agreement, whether the conditional fee agreement was
champertous. The court held it was, and extended the prohibition to arbitration.
The public policy was that the maintainer might, for his own gain, inflame the
damages, suppress evidence or suborn witnesses (the fears may however be
exaggerated), citing Re Trepca Mines.

b. Note – developments in the law – MinLaw is now looking at CFA for arbitration.
There are significant benefits to allowing such arrangements, including
allowing access to justice, developing SG as an international litigation hub –
https://www.straitstimes.com/singapore/courts-crime/law-ministry-seeks-
public-feedback-on-proposed-option-of-conditional-fee.

246
368. Pre-nuptial agreements – Certain pre-nuptial agreements may be challenged
under this head if the agreement enables them to negate or resile from the marriage
(such as requiring the couple to separate and live apart) (Kwong Sin Hua v Lua Lee
Yen).

a. In this case, there was an agreement between the husband and wife who had
been civilly married. They agreed that they would only cohabit and consummate
the marriage after they celebrated the traditional customary rites. Subsequently,
the wife refused to go through the rites, claiming she wanted to enter a nunnery
and live a religious life. The court gave effect to the agreement, and held the
party in default (refusing to proceed with the religious or customary ceremony)
as having in effect willfully refused to consummate the marriage, which was a
ground to declare the marriage void.

369. Public morality/corrupt public life

a. Against public morality – Pearce v Brooks – In this case, the defendant, a


prostitute, was sued by the plaintiffs, coachbuilders, for the hire of a brougham.
There was no evidence that the plaintiffs looked expressly to the proceeds of
the defendant’s prostitution for payment; but the jury found that they knew her
to be a prostitute, and supplied the brougham with a knowledge that it would be,
as in fact it was, used by her as part of her display to attract men. It was held
that the contract was unenforceable as the purpose of the contract (to build a
brougham) was contrary to good morals and infringed public policies. The
plaintiffs did know the purpose of the brougham.

b. Not against public morality – Armhouse v Lee Chappel – This case involved a
contract to advertise sex chat lines in magazines. Such advertisements was
considered by the court to be distasteful but not immoral to attract common law
public policy. The claimant could thus enforce the contract and claim payments
due to it.

Common law illegality: contracts tainted by illegality

370. Contracts which are tainted by illegality (entered into with the object of
committing an illegal act), but are not unlawful per se could be denied enforcement if

247
doing so was a proportionate response to the illegality (Ochroid at [36], Ting Siew
May at [66]). In adopting this proportionality approach the courts will consider (Ting
Siew May at [70]):

a. Whether allowing the claim would undermine the purpose of the prohibiting
rule;

b. The nature and gravity of the illegality;

c. The remoteness or centrality of the illegality to the contract;

d. The object, intent, and conduct of the parties; and

e. The consequences of denying the claim.

371. The list of factors are not closed (Ting Siew May at [71]).

372. The rationale is that this category of cases contains degrees of illegality. It may
not warrant that the entire contract, even if tainted in a minor fashion, is unenforceable
in a blanket fashion (Ting Siew May at [46]-[47]).

373. Illustrative cases

a. Ting Siew May – In this decision, the Court considered that (a) allowing the
claim may undermine the purpose of the prohibiting rule, which is to maintain
price stability in the real estate market; (b) the gravity was not trivial given the
need for stability in the property market; (c) there was sufficiently close
proximity between the contract and the illegality given the insertion of the false
date, which was an overt and integral step taken in the option; (d) the object and
intent from the outset was to use false data to get a higher loan, which was
prohibited under the notice; (e) the consequences of denying the claim was not
so great as to render non-enforcement disproportionate. No substantial expenses
incurred, no evidence of injustice to the buyers (loss of increase in value of
property/opportunity to buy another property). It was held that the contract was
unenforceable.

b. Parkingeye v Somerfield Stores – In this case, P brought a claim for damages


for repudiatory breach of the contract and D raised an illegality defence based

248
on false representations made in the demand letters sent by P to D's customers.
The demand letters were drafted by P and approved by D before the contract
was made. Under the contract, P is to supply D with an automated parking
system, P received no payment from D but instead was allowed to retain all
fines collected from D's customers who overstayed their free parking. P had
committed fraud by inserting falsehoods into some of the demand letters. The
EWCA held that the illegality defence was rejected: (a) P was found guilty for
fraud in separate proceedings; (b) not serious in general as apart from the
instances of illegality the contract was largely carried out lawfully; (c) no overt
step was taken in the contract, with the form of demand letters not stipulated in
the contract; (d) P did not have an intention of acting unlawfully but was merely
unaware of the legal implications of the letters; (e) to deny the claim would give
the defendant a windfall and left the plaintiff with no remedy for lost income.
It was held that the contract was enforceable.

c. 21st Century Logistical Solution v Madysen – This case Concerned a contract


for sale of goods, which was used for form of VAT fraud known as “missing
trader” or “carousel fraud”. The claimant was incorporated to buy goods from
abroad without VAT and to sell them within the UK with VAT added and
pocket the VAT without accounting to the tax authorities. When the claimant
(in liquidation) sought to enforce the debt against the defendant-purchaser, the
latter argued that the contract was unenforceable on account of illegality. The
Court held that the contractual claim was enforceable – not sufficiently
tainted by illegality: (a) allowing the claim not necessarily undermined the
purpose of the prohibiting rule as there were other regulatory means to punish;
(b) the gravity was fairly serious; (c) The contract was merely a straightforward
agreement and lawful, which provided opportunity to profit from the intended
fraud; (d) claimant did have the intention to perpetrate fax fraud, but there was
insufficient proximity between it's intention and the contract); (e) to deny the
claim would give the defendant a windfall.

249
Recovery of benefits conferred

374. A party who has transferred benefits pursuant to the illegal contract might be
able to recover those benefits on a restitutionary basis. This is the second stage of the
inquiry and there are three possible legal avenues for such recovery (Ochroid at [176]):

a. First, where the parties are not in pari delicto – which includes the more specific
categories of class protection statutes, situations where there has been fraud,
duress or oppression, and cases where the plaintiff entered into the illegal
transaction as a result of a mistake.

b. Second, where the doctrine of locus poenitentiae applies because there has been
timely repudiation by the plaintiff of the illegal contract. As presently advised,
there must be genuine and voluntary withdrawal by the claimant from the
illegal enterprise for the doctrine to apply and would not apply where the illegal
purpose was frustrated by circumstances beyond the plaintiff's control or is
simply no longer needed (no definite pronouncement).

c. Third, where the plaintiff brings an independent cause of action for the
recovery of the benefits conferred under the illegal contract, which does not
allow the plaintiff to enforce and profit from the illegal contract:

i. Unjust enrichment, subject to the defence of illegality and public policy


in the law of unjust enrichment, premised on the principle of
stultification which requires the court to determine whether to allow the
claim would undermine the fundamental policy that rendered the
underlying contract void and unenforceable in the first place; or

ii. Tort/trust claims based on the claimant's property or title, subject to the
principle of stultification.

Not in pari delicto

375. The first avenue of restitutionary recovery, which is the corollary of the in pari
delicto maxim, applies where the parties are not in pari delicto (ie, where the plaintiff
is less blameworthy than the defendant). The maxim has, as its underlying premise, the
idea that the party who is seeking restitutionary recovery is not (or at least is not legally

250
deemed to be) equally at fault vis-à-vis the other party. It should be emphasised that
this principle does not entail a broad examination of the relative blameworthiness of
each party. Instead, the maxim applies only in established situations, consisting of the
following three categories (Ochroid at [43]):

a. Class protection statute: where the relevant legislation which prohibited the
contract was a “class protection statute” that was intended to protect the class
of persons to whom the plaintiff belonged (Tokyo Investment v Tan Chor Thing)

i. In Tokyo Investment, Tan sought to recover shares from Tokyo (futures


brokers) which had been furnished as security for his brother’s trading
activities (Note: not stated in the case, but it was stated shares were
pledged – probably the share certificates). Tokyo was unlicensed – a
breach of legislation (Security and Futures Act). Tan could however
recover his shares as he did not know that the brokers were unlicensed
and less blameworthy. To allow recovery was consistent with the policy
of the act, which is to deter unlicensed broking.

1. Note: existing restitutionary claim was made in property law (i.e.


I am the owner to the shares). Illegality was raised as a defence,
with the pari-delicto exception being an exception to the
illegality defence.

b. Fraud, duress or oppression on the claimant – where the plaintiff entered into
the contract on the basis of fraud, duress or oppression (Shelly v Paddock).

i. In Shelly v Paddock, P lived outside of the UK but moved to England. P


wanted to buy a house in Spain which she saw on an advertisement. The
Ds represented that they were agents to sell the house for the English
owner H, but in fact they had no such authority and were themselves
under contract to buy the house from H by instalments. P agreed with
Ds to buy the house for £9,500, paying as deposit £80 cash in Spain,
paying the remainder later. This was contrary with the Exchange Control
Act which prohibits the payment of money out of England. Ds had
orchestrated a fraud – they took P's money and bought a house in
England instead. H was ready to take possession of the house from Ds

251
who had failed to pay those instalments. Ds were prosecuted, but for
some reason were not convicted for criminal law. P did not know about
the exchange controls. In this case, the loan was obtained by fraud, and
as such P could recover the sums.

1. Note: this is a case of tort of deceit claimed. Illegality would thus


be raised as a defence, and not in pari delicto was thus an
exception.

c. Mistake – where the plaintiff entered into the illegal transaction as a result of a
mistake as to the facts constituting the illegality (Aqua Art v Goodman).

i. In this case, there was a contract for purchase of property, but was illegal
as the purchaser was a foreigner who had not obtained approval to buy
it. The purchaser sought to recover the option fee and 5% deposit that
he had paid earlier on. The Court held that recovery was allowed as the
purchaser was mistaken about the need for approval.

1. Note: presumably, the position here was in unjust enrichment –


unjust factor would be mistake of law/facts or total failure of
consideration/performance. Illegality is raised as a defence, and
not in pari delicto was thus an exception.

376. We therefore see no need to revisit them, except to make the point, alluded to
earlier, that there will be some overlap between the not in pari delicto doctrine and the
cause of action in unjust enrichment in so far as the circumstances which justify the
application of the not in pari delicto doctrine may also give rise to “unjust factors” such
as mistake and duress. In such situations, the stultification principle would not apply
because it cannot be said that the integrity of the courts would be undermined or the
underlying policy of the law stultified if the claim by the plaintiff, who is deemed to be
less blameworthy in the eyes of the law, is allowed. Indeed, in some instances falling
within the not in pari delicto principle, awarding restitution will positively further
rather than stultify the policy of the rule that rendered the contract void and
unenforceable, such as where the very reason for the rule is to protect parties in the
plaintiff’s position (see Goff & Jones 2016 at para 35-50) (Ochroid at [170]).

252
Locus poenitentiae

377. Singapore has preferred a narrow doctrine of timeous repudiation, requiring that
the plaintiff needs to have genuinely repented of his or her illegality and voluntary
withdrawal, as this would encourage timely withdrawal from the illegal enterprise. A
broader doctrine may allow a plaintiff to circumvent the restraining principle of
stultification – this undermines the need for the plaintiff to establish the ordinary
requirements of a claim in unjust enrichment (i.e. unjust factor), and may even
encourage those who contemplate entry into an illegal transaction by providing them a
safety net (Ochroid at [171]-[173]).

a. Bigos v Boustead – In Bigos, Defendant failed in claim to recover share


certificate given as security for loan pursuant to agreement illegal under
exchange control regulations. Performance had been frustrated by claimant’s
failure to provide loan, not defendant’s voluntary withdrawal. The Court
disallowed restitution as the illegal purpose was frustrated by external
circumstances. The Court took a narrow view, requiring genuine repentance.

b. Aqua Art has suggested that genuineness, in the form of a subjective feeling of
remorse, may not be necessary (at [31])

c. Tribe v Tribe – A father transferred shares to his son for an illegal purpose – to
defeat claims by creditors. Father settled with his creditors and asked son to
return the shares; son refused, father sued him. The father was permitted to
recover shares, extending recovery as long as the withdrawal was voluntary.
Genuine repentance was suggested as not being necessary.

i. This was doubted in Ochroid at [175], and should preferably not be


followed.

378. Note: Birks always considered that locus poenitentiae as being both an unjust
factor of illegality (policy based), and an exception to illegality. This distinction
however, was not fully appreciated in Ochroid (at [171]-[175]).

253
Independent cause of action

Unjust enrichment

379. For an unjust enrichment claim to be satisfied, the following must be established
(Ochroid at [213]):

a. The defendant must have been enriched;

b. The enrichment was at the expense of the claimant;

c. The enrichment must be unjust; and

d. There are no defences to the claim.

380. Illegality however can again operate here to defeat the independent claim in
unjust enrichment: whether permitting recovery would undermine the fundamental
policy underlying the prohibition and make nonsense of the prohibition (Ochroid at
[215]).

a. Ochroid – In this case, the respondents were enriched from the principal sums
loaned to them from the appellants; the enrichment was at the expense of the
appellants; the unjust factor is the total failure of performance (i.e. counter-
performance of repaying the loan). The Court held that the defence of illegality
will apply:

i. The fundamental social and public policy is to deter unlicensed


moneylending (at [220]);

ii. The efficacy in deterring illegal moneylending would be severely


undermined if the courts could allow the unlicensed moneylender to
recover the principal sums. Although there is no identical yield (i.e. no
profit claimed), this may allow the illegal moneylenders leverage to
compel their debtors to make full repayments, as well as provide them
with a safety net to recover their principal sums (thereby allowing them
to grant further illegal loans) (at [225]).

254
Independent tort/trust claims

381. Historically, the approach towards recovery is whether the claimant had to
formally plead his or her illegally in their claim (Tinsley v Milligan). However, this rule
was prone to unfair outcomes, as it dependent on whether there was a separate
presumptive rule, which would arbitrarily shift the burden of proving a cause of
action from one party to another, without regard whether allowing the claim would
stultify the policy underpinning the illegality.

a. For example in Tinsley v Milligan, the parties had brought a house with the help
of a mortgage. The balance of the money was provided by them, but the legal
title to the house was transferred into the sole name of Tinsley, in order for
Milligan could obtain social security benefits by misrepresenting to the
Department of Social Security. Tinsley and Milligan subsequently fell out, and
Tinsley claimed possession of the property. Milligan had counterclaimed for a
declaration that the property was held on trust for both of them in equal shares.
The majority granted a declaration, and Milligan succeeded, relying on the
presumption of a resulting trust. Lord Goff, as the minority, criticised that the
present rules were indiscriminate in their effect and was capable of producing
injustice.

b. Note: it was in this context that the two exceptions to illegality/reliance rule
were developed – timeous repudiation and non in pari delicto.

382. The SGCA in Ochroid has suggested that the claims should generally be
allowed under tort or law of trusts, subject to whether the claim would stultify or
undermine the fundamental policy which has led to the illegality in the first place
(Ochroid at [168]).

Other claims?

383. It has been suggested that there may be extra contractual claims available
notwithstanding the contract which has been denied enforcement. For example:

a. Collateral contract – Strongman v Sincock – An architect contracted with


builders to supply materials and carry out work at his premises, and promised
orally that he would obtain all the licences necessary at that date. Some work

255
was carried out without the licenses necessary. Plaintiff sues architect for the
balance of the price over licensed amount, or alternatively damages for a similar
amount of breach of a collateral contract to obtain licences. Held that the
builders could not recover the price under the main contract as the contract was
illegal for being prohibited by the regulations. However, the assurance given
amounted to a collateral contract that he would obtain the licences or stop work
if he could not obtain them (in return for the workers to enter the main contract).
As the builders were not morally blameworthy or negligent, they could recover
damages for a breach of a collateral contract or even in fraud since they had
been led to commit the criminal offence, which was prohibited by the promise
of the architect.

i. Note: illegality was still considered here as a concern.

b. Fraudulent misrepresentation – Shelly v Paddock – see above for the facts.


The court also held that the illegality did not bar an action in the tort of deceit
as well, and the claimant was entitled to recover her money and an additional
sum for distress.

i. Note: illegality was again, concerned here – presumably as an exception


to illegality via non-in-pari delicto.

c. Negligent misrepresentation – Mohamed v Alaga – Somali translator entered


into illegal fee sharing agreement for professional services but was ignorant of
Solicitors’ Practice Rules. Given leave to amend case to claim damages in tort
for breach of duty to disclose to him that the agreement was prohibited.

i. Note: court did consider non in pari delicto and suggested quantum
meruit claim (really UE claim) allowed, or tortious claim could be
potentially arguable.

Restraint of trade

What is a restraint of trade and when does it apply?

384. Contracts in restraint of trade are contracts where one of the parties restricts her
future freedom to carry on her trade or business with others, usually by way of a

256
covenant. The doctrine is well recognised to apply in contracts for the sale of a business
and employment contracts (Man Financial v Wong Bark Chuan at [52]). There is some
uncertainty as to where else would the doctrine apply.

a. There is authority which suggests that, in the context of exclusive dealing


agreements, the covenantor must have given up some freedom or right which
they otherwise would have had prior to entry into the solus agreements (Esso
Petroleum).

b. In contrast, there is some authority which preferred a broader approach


advocated by Lord Wilberforce in Esso Petroleum, which would exempt some
contracts from the doctrine if they were prima facie molded under the pressures
of negotiation, competition and public opinion, and have assumed a form
which satisfies the test of public policy as understood by the courts at the time
(Shell Eastern Petroleum v Chuan Hong Auto).

c. The Court of appeal in Singapore has left this issue open (Man Financial v
Wong Bark Chuan at [58]).

385. A specific analysis of the recognised classes will be considered below.

General approach

386. Contracts in restraint of trade are contrary to public policy and are therefore
prima facie illegal. However, they are enforceable if they protect a legitimate
(proprietary) interest and are reasonable (1) in the interests of the parties, and (2) in the
interests of the public (Man Financial at [69]-[71]).

387. The conflicting policy considerations here lie between the parties' freedom of
contract and the covenantor's freedom to work (Tillman v Egon at [22]).

Legitimate (proprietary) interests

388. There cannot be a bare and blatant restriction of the freedom to trade. There
must always be a legitimate proprietary interest which the Court will then seek to
protect by way of the doctrine of restraint of trade (Man Financial at [79], CLAAS at
[44]). The requirement that the information be related to property is not necessary

257
however, and legitimate interest itself is sufficient (even though not property) (Lek
Gwee Noi at [57]).

a. Sale of a business – In this context, the main interest is that of the business'
goodwill (Man Financial at [80]);

b. Employment – The Courts generally adopt a stricter approach towards


restrictive covenants in this context. There are generally two main interests (and
the list is not closed, namely:

i. Trade secrets and highly confidential information belonging to the


employer (Man Financial at [83]-[84]). This however does not include
the employee's own talent or skill (Man Financial at [87]).

ii. Trade connections which refers to the personal knowledge of (and


influence over) the customers of the employer (Man Financial at [93]).

iii. The list is not closed – The courts have also recognised other legitimate
interests such as the maintenance of a stable and well trained
workforce in the context of non-solicitation restraints(Man Financial at
[121]-[122]).

Reasonable in the interests of the parties

389. A valid restraint of trade must be reasonable to the parties' interests and is
ascertained at the time when the restraint was entered into (CLAAS Medical v Ng Boon
Ching at [61]). The reasonableness analysis is highly fact-dependent, as such
precedents may be of little value (Chua Chian Ya at [31]). The factors include:

a. Area of the restraint (CLAAS at [59]);

b. Time of the restraint (CLAAS at [60]);

c. Consideration obtained (Man Financial at [139]); and

d. Activities (CLAAS at [50] – [57]).

258
Reasonable in the public interest

390. The restraint of trade must be reasonable to the public interests, which considers
the impact of local circumstances (Man Financial at [77]). The key interest here is
typically to prevent the establishment of monopolies (Man Financial at [75]), although
an argument could be that the monopoly is necessary for economies of scale/there are
consumer welfare effects (see the arguments raised in Pilkadaris v Asian Tour at [107]
– [109]).

Case studies

Sale of business context

Nordenfelt v Maxim Nordenfelt Guns

391. In this case, N sold his armaments business and agreed not to engage in the same
business worldwide for 25 years. He later claimed that the restraints was unenforceable.
The court held that the restraints were enforceable (see Lord Macnaghten):

a. Legitimate interest: Goodwill of the business.

b. Reasonable between parties: The restraints enabled N to obtain the full value
of what he had to sell (i.e. the goodwill, trade with the world for arms) and
without it the purchasers could not have been protected in the possession of
what they wished to buy;

c. Reasonable for public: It was not injurious to the British public to prevent a
person carrying on a trade in weapons of war abroad – how can the public be
injured by such a transfer of business anyway? The argument of "industrial
suicide" was given little weight – concerns about N being a burden to public
and no longer earn his trade is far-fetched, in light of the fact that he was well
remunerated for the sale.

CLAAS Medical v Ng Boon Ching

392. In this case, NBC ran a clinic practicing aesthetic medicine and a distributorship
business of aesthetic laser/light machines and skincare products (through a company he
incorporated, BCNG). 6 doctors wished to enter into a joint venture with NBC, and
they incorporated CLAAS. NBC became a shareholder in CLAAS, selling his 60%

259
share in BCNG to CLAAS (for $1.92m), and subsequently the remaining 40%. Under
the latest shareholders agreement, there was a restraint of trade provision which
prohibited all parties to being engaged in competition with the business of CLAAS
and/or the practice of Aesthetic Medicine for as long as they remained a shareholder,
and for a period of 3 years following the date when the party ceased to hold shares in
CLAAS. If NBC defaulted, he had to pay $1m in damages to CLAAS. NBC
subsequently sold all his shares in CLAAS, tendered his resignation as director of
CLAAS and BCNG, setup his own general and aesthetic medical practice.

a. The restraint: Clause 11(a) prohibited inter alia (at [15]:

i. Non-compete – being engaged and/or interested in any trade and/or


business carried on within Singapore which is similar to or in
competition and/or conflict (whether directly or indirectly) with the
Business of the [Appellant] and/or the practice of Aesthetic Medicine;

ii. Non-solicit employees – employ, solicit and/or entice away and/or


endeavour to employ, solicit and/or entice away any person(s) who is
employed by the [Appellant], and/or induce or seek to induce any such
person(s) to leave his/her employment with the [Appellant] for any
reasons whatsoever;

iii. Non-solicit customers – solicit the custom of any person and/or body
who is a customer of the [Appellant] and/or divert or seek to divert any
customer of the [Appellant] away from it; and/or

iv. No abetment – cause and/or permit any person directly or indirectly


under his/her control to do any of the foregoing acts or things.

b. Legitimate interest – Goodwill of the business (at [46]-[49]).

c. Reasonableness in the interest of the parties

i. Non-compete clause was too broad, as the activities restrained (by the
definition of Aesthetic Medicine – last part included "all procedures and
treatment as understood by aesthetic medicine") were too wide. The

260
business was NBC's existing clientele and there was no justification to
prevent NBC from engaging in other forms of aesthetic medicine.

1. The latter portion was however severed, see below.

ii. Non-solicitation clause was reasonable when read in the context –


every doctor was already bound by the non-compete clause. It will only
kick in if such a doctor left CLAAS and breached the non-compete
clause. Such a restriction was reasonable (subject to the non-compete
clause) (at [58]).

iii. For the non-compete restraint

1. Area: The extension to the whole of Singapore was reasonable


as NBC had a loyal following of patients wherever he was
located. Such a general restraint was warranted, otherwise the
goodwill will be severely undermined (at [59]).

2. Time: The time was reasonable. In fact, it was NBC who had
proposed the timeline and NBC had a much stronger bargaining
position – "they persuaded me to sell to them" (at [61]).

d. Reasonableness in the interest of the public – it is to the advantage of the


public to allow a trader who has established a lucrative business to dispose of
it to a successor by whom it may be efficiently carried on. That object could
not be accomplished if the law reserved to the seller an absolute and indefeasible
right to start a rival concern the day after he sold (at [49]).

Employment context

Man Financial v Wong Bark Chuan David

393. The respondent was the MD and CEO of the appellant company. The
respondent was told to resign with immediate effect, and was placed on “garden leave”
while he served out a three-month notice period. Respondent was also handed a
proposed termination agreement, containing restrictive covenants on non-solicitation
and non-competition for a period of 1 year. The respondent did not agree to the
provisions and did not sign the Agreement immediately. After negotiation, an

261
agreement was executed, and the respondent was prohibited, inter alia, soliciting the
employment of certain employees of the appellant for a period of seven months from
the termination date, or rendering advice/participate in/have an investment in a
competitor for a period of 7 months. The respondent was to receive compensation from
the appellant provided he did not breach the terms of the termination agreement.
However, before respondent was due to be paid, the appellant was informed that the
respondent had breached the agreement (non-solicitation and non-competition clauses)
and declined to provide compensation. Respondent sued appellant for compensation.

a. The restraint:

i. Clause C.1 – Non-solicit of employees – WBC was restricted from


soliciting the employment of certain employees of the appellant for a
period of 7 months from the termination cade.

ii. Clause C.3 – Non-compete – WBC is restricted from participating in


or rendering advice to a competitor for a period of 7 months from the
termination date.

b. The legitimate interest:

i. Non-solicit – There was a legitimate proprietary interest in MF


maintaining a stable, trained workforce (at [136]).

ii. Non-compete – there was no evidence to demonstrate an underling


proprietary interest which Cl C3 was intended to protect (at [15]). WBC
was alleged to have access to confidential information, but there was no
evidence to prove said access as well as nature of confidential
information.

c. Reasonableness in the interest of the parties

i. Non-compete not reasonable – there was no evidence to demonstrate


an underling proprietary interest which Cl C3 was intended to protect
(at [15]). Area is far too wide (presumably area is not specified and thus
global?).

262
ii. Non-solicit was reasonable – the ROT was negotiated, there was
significant consideration for the restraint, and on true construction it
only applied to senior staff who had contact with WBC during his
employment (at [134]-[142]).

d. Reasonableness in the interest of the public

i. Non-solicit was reasonable – assuming that the agreement was a


settlement, then there was an interest to uphold it in settlement of a
dispute (at [136]).

Smile Dental v Lui Andrew

394. AL was employed by SD in 2003. SD was under the impression that AL would
not stay in SG permanently and leave within 6 years. AL however in 2009 resigned
from SD, setup his own dental company located at a 5 minutes' walk away from SD's
clinic. AL had also engaged a former colleague from SD to join him as a shareholder
and dentist in the new company. SD sued AL for the breach of certain restrictive
covenants.

a. Legitimate proprietary interest: in the context of medical practitioners, their


special and intimate knowledge of the patients of the business (at [22]-[24]).

b. Restrictive covenants –

i. Non-solicit – Upon leaving The Practice, Dr. Lui will not seek to
damage or injure The Practice’s reputation or to canvass, solicit or
procure any of The Practice’s patients for himself or any other persons.

1. Note: no evidence of breach in this case [16]-[18].

ii. Non-compete – In the event that Dr. Lui leaves (whether resignation or
dismissal) The Practice, Dr. Lui shall not practice within a 3 kilometre
radius distance from the Smile Inc. Dental Surgeons practices at Suntec
City Mall and from Forum The Shopping Mall, and a 3 kilometre radius
from any other new Smile Inc. Dental Surgeons practices that have been
set up before and during his cessation of work at The Practice.

263
iii. Non-removal of information – In the event that Dr. Lui leaves (whether
resignation or dismissal) The Practice, existing and new corporate and
non-corporate contracts, as well as existing and new patients, shall
remain with The Practice. Patient data and records, office data and
records and computer software programmes and data shall remain the
property of The Practice, and such records, in full or in part, shall not be
copied manually, electronically or otherwise be removed from the
Practice.

c. Reasonable in the interests of the parties

i. Time – the restrictive covenants did not provide for a fixed duration of
operation and is thus automatically unreasonable (at [29]).

ii. No further discussion on the rest given the clear finding of no time
duration.

Lek Gwee Noi v Humming Flowers

395. In this case, LGN was employed as a sales manager in a company whom her
brothers were shareholders. The business was later acquired by its main competitor
later, with HFG being the wholly owned subsidiary of the competitor (Noel) buying
over the business. After the acquisition, the plaintiff continued as the sales manager of
the business but with the defendant as her new employer. Her employment agreement
contained express post-termination non-competition and non-solicitation covenants
(together, “the restrictive covenants”). Both these covenants had (a) a geographical
restriction, (b) an activity restriction and (c) a temporal scope.

a. Legitimate proprietary interest: Pre-existing trade connections and goodwill


(at [73]-[75]).

b. Restraints (at [25]):

i. Non-compete – First limb: for two years after 31 December 2011, the
plaintiff shall not undertake or be employed in the same or similar
business as the relevant company, or in any other business carried on

264
by the relevant company in Singapore, Malaysia or any country in
which the “relevant company” had offices on 31 December 2011.

ii. Non-solicit – Second limb: for the same period and within the same
areas, the plaintiff shall not canvass or solicit orders from any person
who was a customer of the relevant company while she was employed
by the defendant.

iii. Non-solicit – Third limb: the plaintiff shall not 265ndeavor to take away
from the relevant company any of its customers.

c. Reasonableness between the parties

i. Non-compete was unreasonable –

1. Geographical restriction was unreasonable: The defendant


company was merely a subsidiary with no plans to expand into
Malaysia. The geographical restriction was intended to protect
Noel’s interests rather than defendant’s interest.
2. Activity restriction was unreasonable: Blanket ban preventing
the plaintiff from competing with any business carried on by
any business carried on by Noel Gifts

ii. Non-compete was unreasonable

1. Activity restriction was unreasonable: definition of relevant


company is too wide. The clause further covered customers of
the defendant after the plaintiff’s employment had ended. No
conceivable trade connection to be protected.

2. Geographical restriction was unreasonable: same as non-


competition clause

3. Temporal restriction was unreasonable: connections could take


1 year to rebuild. 2 years was too long

265
Exclusive dealing agreements

396. Historically, such agreements were generally valid, but the restraint of trade
doctrine has subsequently been recognised to apply to sole supply agreements.

Esso Petroleum Co v Harper's Garage

397. In this case, Harper’s Garage agreed to accept and only sell petrol for its two
stations from Esso for a long period of time, exclusive dealing agreement. It agreed to
keep the garage open at all reasonable hours and not to sell the garage without ensuring
that the buyer entered a similar agreement. One agreement was for 4.5 years, the other
for 21 years.

a. Note – previously the owners had owned the garage. The majority seem to
consider where if a person buying land had no previous right to be on the land,
then when he takes possession of the land subject to a negative covenant, the
doctrine would not apply as there is no restraint on the trade.

b. Legitimate interest – Esso's legitimate interest was in maintaining a stable


distribution system (running economically and efficiently).

c. Reasonableness between the parties:

i. The tie for the agreement of 4.5 years was reasonable.

ii. The tie for 21 years however, stretches far beyond any period which
developments are reasonably foreseeable are not reasonable. There was
nothing proven by Esso that there was some clearly established
advantage to the producing company or the garage owner.

National Aerated Water v Monarch Co

398. NAW had a licence to produce/distribute/sell ‘Kickapoo Joy Juice’. The licence
prohibited them from handling any other product with the same name or syllable.

a. Legitimate interest – Monarch's proprietary interest in its trademark (at [44]-


[45]).

b. Reasonable between the parties

266
i. Not reasonable as the restraint limited the use of common English words
such as joy and juice (at [32]-[35]).

c. However, the court allowed severance of the words "joy juice" (at [44]-[45]).

d. The "kick" beverage consisted a part of or a syllable of "Kickapoo", and thus


the licence was breached.

Exclusive promotion agreements

399. These are cases which exclusive rights are given to publishers/promotion agents
by writers/sports stars/performers. The courts have usually no issues applying the
restraint of trade doctrine here (Shroeder Music v Macaulay, Chua Chian Ya v Music
& Movements).

a. Shroeder Music v Macaulay – In this case, M (21 years of age) entered a


standard form agreement with Schroeder Music, whereby they would have the
exclusive benefit of his compositions. The global copyright was assigned to
another party in return for a fixed percentage of any royalties. This was to last
five years and could be automatically extended for five years if the royalties
went above £5000. Schroeder Music could terminate or assign the contract, but
Macaulay could not, and Schroeder was under no obligation to publish or
promote anything. Macaulay claimed the agreement was contrary to public
policy.

i. The clauses were so one-sided given the duration, the fact that the
publisher may not even use the work, the fact that the writer's talents
could be sterilized, the fact that the writer had no right to terminate
meant that the clauses were unreasonable restraint of trade.

b. Chua Chian Ya v Music & Movements – The appellant, Tanya Chua, was a
local singer-songwriter who entered into a music publishing agreement with the
respondent for a period of three years . At the end of that period, Chua extended
the agreement for another three years (collectively known as “the arrangement”).
Under the terms of this arrangement, Chua “irrevocably and absolutely”
assigned the rights to her works composed during this period to M&M. In return,
M&M remained responsible for promoting Chua’s works and for collecting

267
royalties on Chua’s behalf. It was also contractually obliged to provide Chua
with a statement of all accounts. Chua noticed a discrepancy between the W/C
accounts and M&M’s summary and sought an explanation from M&M. M&M
acknowledged the discrepancies and gave various explanations as to how they
arose. Certain documents were also provided to Chua in explanation of these
discrepancies. However, Chua remained dissatisfied and eventually sought a
declaration that all rights to her compositions had reverted to her upon the
expiry of her contractual arrangement with M&M. She also argued that the
contractual provision which assigned the rights in her compositions to M&M
was unenforceable because it was in restraint of trade.

i. The SGCA considered that this was not a restraint of trade – there is
no restraint on an artiste's ability to make a living, but only a restraint
on the sale of her proprietary interest in her musical composition at [31].

ii. There was no right of automatic extension, nor was Chua bound for an
inordinate amount of time. M&M did not have an absolute and
unfettered discretion whether or not to exploit the compositions – in fact
it was under an obligation to exploit. Taken as a whole, the provisions
were reasonable (at [30]).

c. Proactive Sports Management v Wayne Rooney – Wayne Rooney set up a


company to which he assigned his image rights, in other words his rights to
exploit his image by, for example, his indorsement of sports products and by
other forms of promotion and sponsorship. His company contracted with the
claimant to act as its agent in negotiating contracts for the exploitation of his
image. His company breached the contract and, when sued by the claimant, it
contended that the contract fell within the doctrine. The claimant disputed that
it did so but accepted the trial judge's conclusion that, if it did so, its terms were
in unreasonable restraint of trade and unenforceable. The EWCA held that
although Rooney's trade was as a footballer, the business of exploiting such
image rights was dependent on his primary occupation of playing football – it
was part of a single trade.

268
i. The Court held that the contract was unreasonable as a restraint of trade
– public policy is concerned with the manner in which a person may
properly realise his potential not only for the good of the individual but
for the economic benefit of society generally.

ii. The contract was for a relatively long duration (8 years), there was a
lack of independent advice for Rooney and his youth (unreasonable
between parties).

Nuanced employment provisions

Forfeiture of benefit clauses – Mano Vikrant Singh v Cargill

400. In this case, M was an employee of C. In addition to his monthly pay, M


received an annual discretionary incentive award, with 50% being paid out in cash and
the remainder paid out in stages. There was a forfeiture provision which if M left C's
employment and competed within a period of 2 years from the date of termination, the
deferred award may be forfeited. M competed and C sought to forfeit the award. In this
case, the High Court considered that if this was a ROT clause, it would have been
unreasonable as it covered too wide a geographic area and the non-compete period (of
2 yaers) is too long. The only issue on appeal was whether the provision fell within
the scope of the restraint of trade doctrine – other issues not appealed (counsel is an
idiot).

a. The Court held that the forfeiture provision was subject to the doctrine – it
restrained M from leaving the employment of R to join a competitor by a threat
to forfeit a significant financial reward which was already vested in him (at
[35]-[41]).

b. A payment-for-loyalty clause, which allowed an additional payment if an


employee continued working for the company, was not a restraint (at [43]-[45].

c. However, the courts may consider applying the restraint of trade doctrine to
non-vesting cases, if the facts resulted in a reasonable expectation on the part
of the employee that she would be entitled to the benefit concerned, that the
clause which sought to deny the benefit may still come within the scope of the
doctrine. This is only applicable in exceptional circumstances where the

269
employee had been induced to enter or continue in employment in reliance of
the employer's representation, and this reliance resulted in some inequity (at
[62] – [68]).

Prohibition on owning shares in a competing business – Tillman v Egon

401. In this case, D was an employee of C, which was in the business of specialist
executive recruitment. After her employment ended, she wished to take up employment
with another company. C sought an injunction to restrain D under a covenant which
provided that D could not "directly or indirectly engage or be concerned or interested
in any business carried on in competition with any of the businesses". D argued that the
covenant was illegal, because it prevented her from even becoming a shareholder in a
competitor.

a. The court considered that the phrase "interested in" included a prohibition on
owning shares in a competing business (at [53]);

b. The covenant fell within the restraint of trade doctrine – the employment of top
executives is usually subject to conditions that she should hold shares in her
employer or be remunerated partly inn its shares or options to purchase them
(to align their interests). Such a restraint is thus a restraint on D's ability to work
(at [33]).

c. The employer had conceded that if the above questions were yes, the restraint
was unreasonable.

Trade association rules

Pilkadaris v Asian Tour

402. In this case, Ps were professional golfers, while D1 was a private company,
wholly owned by D2. Ds operated a professional golf tournament (Asian Tour) and Ps
were player-members of D2. Ps were penalised for breaches of a regulation which
required members of the D2 to apply for a release to play in certain golf tournaments
(i.e. conflicting golf tournaments (same day or near Asian Tour), or a designated
competing golf tournament). Participation without such release would result in a
member being fined up to US$5,000 and/or suspended. Ps sought to declare that the
regulation was an unreasonable restraint of trade.

270
a. The court held that the regulation was a restraint of trade – it governed the
actions of its members in their professional lives and how they earned a living
(at [69]-[70]).

b. Legitimate interest – D argued that it was to protect professional golfers and


their livelihood – to achieve this aim it was necessary to protect Asian Tour (at
[91]).

c. The regulation was unreasonable as between the parties – the power to declare
a golf-tournament as competing was far too wide and arbitrary; the 7 day time
period was much wider than the definition of conflicting golf tournaments used
by other tours; D2 had failed to establish why Asian Tour needed such a lengthy
period to protect itself.

d. The regulation was unreasonable in the interests of the public. While it was
good for Singapore to have a strong, healthy and viable Asian Tour, it was also
good for Singapore to have more world ranking tournaments played here.

Severance

403. There are two types of severance (Man Financial at [126]).:

a. first, there can be term severance (i.e. the severance of whole or entire
covenants). This is to excise the illegal terms of the contract entirely, thereby
saving the rest of the contract.

b. Second, there can be textual severance (i.e. cutting down the scope of a clause).
This is used typically to render a restraint of trade clause narrower, such that it
is no longer offensive.

Term severance

404. Under term severance, the basic test is whether the objectionable promise is
substantially the whole or the main consideration for the promise sought to be
enforced. If it is, the doctrine of severance will not apply (Man Financial at [128]).

a. In Man Financial, the Court characterised Clause C.1 (the non-solicitation


clause) as a condition, which thus was intended by the parties to constitute the

271
main consideration for the agreement. As such, term severance was inapplicable
– nevertheless Clause C.1 was construed to be reasonable.

Textual severance

405. To apply the doctrine of textual serverance, the Court will apply the blue-pencil
test – the court must be able to run a "blue pencil" through the offending words in that
clause, without altering the meaning of the provision and without rendering the
clause senseless (Man Financial at [127]). The remaining contractual terms must
continue to be supported by adequate consideration and must not change the
fundamental character of the contract between the parties (Lek Gwee Noi at [155]). The
severance could not arrive at a result which was inconsistent with the principles which
made the excised words void (at [172]).

Illustrative cases:

406. CLAAS – In this case, the non-compete clause was viewed as unreasonable, but
could be saved by textual severance.

a. "Aesthetic Medicine" means "moderately invasive medicine in the process of


laser and permanent hair reduction, photorejuvenation by lasers and intense
pulsed light machines and such, pigment management by lasers, acne control
via special skincare specific to [the company] and [BCNG’s] range and lasers,
mesotherapy of body and face and all procedures and treatment as understood
by aesthetic medicine".

407. Smile – In this case, there was no written limitation as to the time limit of the
covenants. It was conceded that as such there was nothing to sever (at [33]).

Notional severance

408. Notional severance refers to the Court's power to, beyond deleting the parties'
words, but to also alter their words or even insert entirely new words to cure the
illegality, while remaining as close as possible to the intentions of the parties expressed
in the agreement (Lek Gwee Noi at [175]).

a. The argument for notional severance is that all severance in effect is already a
rewriting of the parties' bargain – it was a fiction that the blue-pencil test did

272
not do so. The effects may be arbitrary as it may depend on accidents of drafting
and the form of expression, rather than the substance of the agreement (Lek
Gwee Noi at [177]).

b. The argument against is that notional severance is actually a unilateral variation


of the parties' obligations, imposed by the court with the benefit of hindsight –
it is not the actual parties' intention (unlike rectification).

c. Although in Smile Dental, SGCA did not express any concluded view on
whether notional severance is available, even if it was, this doctrine would not
apply in the employment context – creating uncertainty, without imposing
sanctions on the employer (apart from enforcing it to the extent of what might
be valid), and changes the risks between the parties (employers have no
inducement to ensure reasonableness of the covenant, imappropriately
increasing the risk that employee is forced to abide by an unreasonable
covenant) (Lek Gwee Noi at [182], [185]).

d. On the balance, the doctrine of notional severance is not accepted as part of


Singapore law (Lek Gwee Noi at [186]).

409. Notional severance has not been accepted in the English Courts as well, and to
do so would require legislation (Tillman v Egon at [85]).

273
Chapter 10: Duress

Judicial basis of duress

Inadequacy of the overborne will theory

410. Historically, duress was originally explained in terms of the victim's will being
overborne, so that he was incapable of making a free choice or even of acting
voluntarily.

a. The "Siboen" and the "Sibotre" – "… the Court must in every case at least be
satisfied that the consent of the other party was overborne by compulsion so as
to deprive him of any animus contrahendi".

b. Pao On v Lau Yiu Long – "In determining whether there was a coercion of will
such that there was no true consent, it is material to inquire whether the person
alleged to have been coerced did or did not protest… All these matters are…
relevant in determining whether he acted voluntarily or not".

411. Nevertheless, this approach is inadequate. A victim of duress submits


knowingly and intentionally. Duress thus does not "overbear" the will, nor destroy it –
it "deflects it".

a. DPP v Lynch –

i. Per Lord Wilberforce: "Duress does not destroy the will, for example,
to enter into a contract, but prevents the law from accepting what has
happened as a contract valid in law."

ii. Per Lord Simon: " Duress again deflects, without destroying, the will
of one of the contracting parties. There is still an intention on his part
to contract in the apparently consensual terms; but there is coactus volui
on his side. The contrast is with non est factum. The contract procured
by duress is therefore not void: it is voidable - at the discretion of the
party subject to duress."

b. Universe Sentinel –

274
i. Per Lord Diplock: "The rationale is that his apparent consent was
induced by pressure exercised upon him by that other party which the
law does not regard as legitimate, with the consequence that the consent
is treated in law as revocable unless approbated either expressly or by
implication after the illegitimate pressure has ceased to operate on his
mind".

ii. Per Lord Scarman (dissenting on result only): "The classic case of duress
is, however, not the lack of will to submit but the victim's intentional
submission arising from the realisation that there is no other practical
choice open to him".

Illegitimate pressure theory

412. The real objection is not that the victim did not consent, but that the consent
was induced by illegitimate pressure (Universe Sentinel). Subsequent decisions have
thus moved away from considering whether the will of an individual was "overborne",
preferring a test of whether the pressure applied was illegitimate.

413. This has been adopted locally, see EC Ridout SGHC at [47]:

a. "In [Wu Yang Construction] … a case on the tort of conspiracy and use of
unlawful means, Andrew Phang Boon Leong J (as he then was), rejected the
older analysis of the overborne will theory and adopted, obiter, at [78], Prof
Atiyah’s view of "coercion" or "vitiation of consent" from the "… perspective
of pressure that so distorts the voluntariness of the consent of the party that is
the alleged victim of economic duress that the law regards such pressure as
illegitimate. And what the law regards as illegitimate becomes, from that
particular perspective, situations where there has in effect been no consent at
all"".

414. Importance of the basis – why does this matter? (Chitty at [8-006]) If the
doctrine of duress is treated as resting on the absence of consent or of a voluntary act,
it would seem immaterial what has caused the absence of consent, or the act to be
involuntary.

275
a. Under a consent approach, duress is a question of fact, not of law. Furthermore,
absence of consent would logically render a contract void not voidable.

b. In contrast, because duress does not destroy the will or consent, it is:

i. Not possible to treat the issue as one of pure fact.

ii. What caused the will to be deflected or consent to be distorted is now


material.

iii. The inquiry is not merely whether a party lacks all choice. The inquiry
must also necessarily be as to the nature of the choices he was presented
with.

iv. The illegitimate pressure analysis may also affect questions of causation
– it may not be necessary to show that the threat was an overwhelming
cause of the victim entering into the contract.

Elements of duress

415. There are two main elements for the vitiating factor of duress (Universe Sentinel
at pg 400 per Lord Scarman, Tam Tak Chuen v Khairul bin Abdul (at [22]), EC Ridout
SGHC at [48]):

a. There must be illegitimate pressure exerted; and

b. The pressure must amount to compulsion of the will of the victim (i.e.
causation).

416. It bears noting that the above analysis depends on the type of duress in question.
The type of duress may:

a. Affect the illegitimate pressure analysis: The illegitimacy depends on the


nature of the threat and of the demand (Universe Sentinel at pg 401).

i. If the conduct threatened is independently lawful, the threat is generally


regarded as illegitimate.

276
ii. Threats of lawful conduct are generally treated as legitimate, unless they
are "immoral or unconscionable" when coupled with an illegitimate
demand.

b. Causation: The precise degree of causation required varies with the type of
duress in question, with economic duress posing a particularly difficult category.

Classes of duress and illegitimate pressure

Duress to person – unlawful acts

417. Generally, threats involving unlawful acts such as doing violence to a person
amounts to illegitimate pressure (Universe Sentinel at pg 401, Barton v Armstrong).

a. Barton v Armstrong – In this case, the defendant was the chairman of the
company, and the plaintiff was the managing director. The plaintiff had
contracted to buy out the defendant’s interest in the company, after it was
threated by the defendant with murder. The UKPC accepted that this was an
unlawful pressure (at pg 120).

Duress to goods – unlawful acts

418. Generally, threats involving unlawful acts such as doing violence to property
amounts to illegitimate pressure (Universe Sentinel at pg 401, The Siboen and the
Sibotre, Maskell v Horner). Note: historically, in Skeate v Beale, this was not
recognised as a form of valid duress.

a. Seizure of goods – Maskell v Horner – In this case, tolls were levied on the
plaintiff under a threat of seizure of goods. The tolls were in fact unlawfully
demanded. The court considered this to justify recovery of the tolls (this should
be a UE claim).

b. Threat to damage goods – The Siboen and the Sibotre – suggested in this case
that "if I should be compelled to sign a lease or some other contract… under an
imminent threat of having my house burnt down or a valuable picture slashed…
I do not think that the law would uphold the agreement" (at pg 335).

277
c. Evia Luck – "It was at one time thought that, at common law, the only form of
duress which would entitle a party to avoid a contract on that ground was duress
of the person… that limitation has been discarded" – (at pg 165).

Economic duress – threats to breach contracts – special unlawful act

419. Note – the acceptance of this only came after The Atlantic Baron and The
Universe Sentinel. As such, cases pre 1979/1983 should be regarded cautiously.

420. A threat to breach a contract, or a threat to cause a breach may amount to


economic duress (Sharon Global Solutions). Notwithstanding that a threat to breach a
contract is unlawful, the Courts may consider such threats in the circumstances to
amount to legitimate pressure. In this regard, the court will consider all circumstances,
including the states of mind of the parties before coming to a conclusion on whether
illegitimate pressure is made out (Sharon Global Solutions at [33]).

Statement of inevitable

421. Where one party simply stated that he would not be able to perform his
contractual obligations, which induces the other party to offer to pay more, such
explanation of difficulties may not amount to an illegitimate pressure as it is arguable
that no threat was made (Williams v Roffery Bros, Sharon Global at [32]).

a. Williams v Roffery Bros – In this case, the claimant sub-contractor had under-
priced carpentry work he was obliged to perform under a contract with the
defendants, and was in danger of being unable to finish the job. The claimant
explained his difficulties to the defendants, who on their own initiative
promised additional payment in exchange for the claimant’s completing the
work. The defendants subsequently refused to pay the renegotiated amount.
Although duress was not pleaded, the EWCA considered that since the initiative
for making the payment had come from the defendant, there could be no
economic duress on the facts.

b. EC Ridout SGHC – In this case, Quentin J suggested that "why should it depend
on who called the meeting if the message passed was the same – "if you do not
pay me more, I will be in delay or may not be able to complete my work at all?"

278
i. However, context does matter for a few reasons – motives of the parties
are relevant, including who initiated the conversation. We want to
encourage parties to cooperate and mutually accommodate each other,
especially if they had been responsible in the original contracting.

ii. Construing whether there is a threat is thus relevant – always open for
the courts to find an "implied threat", see B&S Contracts v Victor Green.

Good faith/Bad faith

422. The fact that the party making the threat has acted in good faith is in favour of
finding that the pressure applied is legitimate (Huyton SA at pg 637; DSND Subsea at
[134], Sharon Global at [37],[39]). One suggestion was to ask whether the threat was
made in bad faith – that it was made to exploit the plaintiff’s weakness rather than to
solve financial or other problems of the defendant (Burrows, The Law of Restitution).

a. Huyton SA v Peter Cremer – "[Professor Birks's] own preference, he indicated,


was for "the simplest and more open course... to restrict the right to restitution
to cases in which one party sought, mala fide to exploit the weakness of the
other""

b. DSND Subsea – " The Contract did not contain a provision which entitled
DSND to suspend work. The Contract simply did not make provision for a
situation such as occurred. If it were necessary so to hold, I would say that the
suspension of work on the RTIAs pending resolution of the insurance/indemnity
question, even if it was a breach of contract, and even if it amounted to pressure,
did not amount to illegitimate pressure. It was reasonable behaviour by a
contractor acting bona fide in a very difficult situation."

c. Sharon Global – "Against this background it cannot be said that the plaintiff
was seeking to exploit the situation to increase its profits when it informed the
defendant that it would not charter the vessel unless the defendant agreed to
share the additional costs."

423. Nevertheless, this factor may not carry much weight on the circumstances.

279
a. The Siboen and the Sibotre – "For good measure, though again I do not think
that this makes any difference, [Goff QC] added that duress must a fortiori be a
defence when the party threatening to break the contract is putting forward some
justification for doing so without any bona fides".

Reasonableness of demand

424. The reasonableness of the demand may be a relevant factor in the analysis
(DSND Subsea, B&S Contracts and Design).

a. DSND Subsea – " It was reasonable behaviour by a contractor acting bona fide
in a very difficult situation."

b. B & S Contracts & Design v Victor Green – " the plaintiffs were perfectly
prepared to pay what the men were demanding save for the fact, they said, they
did not have the money available. Well, then there came the offer of the
defendants to make the money available by giving them an advance. In those
circumstances I can see no reason why they should not have accepted the money
and paid the workforce save their own immediate economic interests, sand they
chose not to do that but to put pressure on the defendants by refusing the offer
and indicating that the only way out was for the defendants to hand over the
£4500 as a gift rather than as an advance", at pg 427.

Statutory policy

425. Considerations of statutory policy is relevant in considering whether a conduct


is lawful or unlawful (The Universe Sentinel).

a. In this case, the Universe Sentinel was "blacked" by the defendant trade union.
The blacking involved the union procuring tugmen to refuse to operate their
tugs in breach of their contracts of employment with the Harbour Authority. To
lift the blacking, the claimants, inter alia, agreed to pay and paid $6,480 into
the defendant's seamen's welfare fund. Having sailed from Milford Haven, the
claimants sought to recover the money paid inter alia under economic duress.
The defendant admitted that, subject to a trade dispute immunity, it had been
paid under economic duress. The statute in question conferred immunity against
specific torts (for e.g. allowing certain strikes). The UKHL by a majority held

280
that the defendant did not have the immunity asserted. The Courts also
considered that the underlying policy did not legitimize the conduct in question.

Illustrative cases

The Atlantic Baron – Duress made out

426. North Ocean Shipping v Hyundai Construction (The Atlantic Baron): In this
case, the shipbuilders were to be paid under a contract in US dollars. However, the
builders threatened not to honour the contract, unless they were paid an extra 10%
payment when the US dollar was devalued by 10%. It was held that this amounted to
economic duress.

a. There was no legal justification for insisting the increased price.

b. The owners realised that the builders would not accept anything other than an
unqualified agreement to the increase.

c. The owners had suggested arbitration between the parties, as to whether the
builders were entitled the additional 10% (including security for costs), but this
was refused by the shipbuilders.

B&S Contracts v Victor Green – Duress made out

427. B & S Contracts v Victor Green – In this case, P agreed to erect exhibition
stands for D under a force majeure clause. P agreed to erect stands for D at Olympia
under a contract with a force majeure clause, which provided that P would make every
effort to carry out the contract but that it was subject to variation or cancellation in the
event of a strike. P intended to use staff from their insolvent subsidiary in Wales, but
the men refused to work until their demand for £9,000 severance pay, to which they
were not entitled, was met. They rejected £ 4,500 offered by P who then told D that the
contract would be cancelled unless D paid the remaining GBP 4,500, not as an advance
(as was initially offered by D) on the contract price but as an additional sum to meet
the workers' demands. D paid the sum and the contract was performed. D deducted
GBP 4,500 from the contract price before it was paid. It was held that this amounted
to economic duress.

281
a. Note that in this case, there was a force majeure clause which obliged P to take
reasonable steps to avoid the strike. The focus in this question was thus whether
the plaintiffs were obliged to pay the full sum, which the Courts considered yes.

b. The court found as a fact that the plaintiffs were perfectly prepared to pay what
the workers were demanding, save for the fact they did not have the cash
available.

c. The court considered that the offer of the defendants to make money available
in advance was reasonable – they should have accepted the money and paid the
workforce.

DSND Subsea v Petroleum Geo-Services – Duress not made out

428. In this case, DSND was carrying out construction work for PGS. DSND
suspended its work pending the signing of a contractual variation (MOU) with more
favourable terms for DSND. Subsequently, in an action by DSND, PGS sought to
escape from the MOU on the ground inter alia that it had been induced by duress. This
was not made out.

a. Even if DSND was in breach of contract, DSND was acting reasonably in good
faith insisting on the new terms at [134].

b. The negotiated terms of the MOU was a "give and take on both sides". PGS had
recognised that DSND's concerns over insurance and indemnity as genuine
given the risks involved in the project at [85].

c. PGS had not entered into the agreement because of the duress and had realistic
practical alternatives [136]-[142].

d. PGS affirmed the contract [143]-[148].

Huyton SA v Peter Cremer – Duress not made out

429. In this case, Cremer contracted to sell to Huyton 30,000 tonnes of wheat,
payment by cash against the presentation by the seller of certain documents. Huyton
rejected the documents and refused to pay, alleging that the documents presented did
not comply with the contract (although it had received the goods). Huyton refused to

282
pay until Cremer agreed to compromise, whereby it gave up any right to arbitration (to
try to recover all its loss) in return for Huyton paying the contract price. After Huyton
had paid, Cremer sought arbitration and alleged that the compromise was entered under
duress.

a. The Court held that there was no illegitimate pressure, as Huyton was not in
breach of contract by refusing to pay. This was a special type of contract
whereby the risk of the goods/property in them did not pass until payment was
made, in accordance with the documentation.

Sharon Global v LG – Duress not made out

430. Sharon Global Solutions v LG International – In this case, P contracted to sell


steel products to D, who in turn entered into a back-to-back agreement to sell the same
to POSCO (important customer of LG group, which D is part of). Due to P's
inexperience, P had difficulty securing a vessel to ship the products. P eventually found
a vessel at a higher cost freight. P was unwilling to charter the vessel, unless D agreed
to share the additional costs. P was prepared to breach the contract and allow D to call
on the performance bond P had issued. The defence of economic duress was not made
out

a. Both parties intended to co-operate to gain entry into the HBI (some iron)
business – they had consultations over the chartering of the vessel and they had
collectively supervised the loading fo the HBI.

b. P, through its inexperience, misjudged the freight costs badly – it was prepared
to forfeit the performance bond.

c. P bore a greater burden under the agreement by paying its share of the
additional freight than it would have under the performance bond.

d. P had also taken steps to borrow money from other people to meet its share of
the freight. P was not seeking to exploit the situation to increase its profits.

e. It would seem also that causation is not made out – no explanation by D on the
commercial consequences of their breach with POSCO.

283
Lawful act duress

431. Generally, where the acts are lawful in themselves, it would be "extremely
difficult" to prove economic duress simply because the doctrine of economic duress
requires proof of illegitimate pressure, as opposed to mere commercial pressure (EC
Ridout SGHC at [47]). Some factors may include:

a. Whether the threat involves some collateral, improper motive (such as an abuse
of process) (Tam Tak Chuen at [50]);

b. Whether the demand is made in good faith (CTN Cash and Carry, Tam Tak
Chuen at [50]);

c. Whether the demand is justified/reasonable (R v AG for England and Wales,


Tam Tak Chuen at [50]); and

d. Whether the threat is considered unconscionable in light of all the circumstances


(Tam Tak Chuen at [50]).

i. This concept of unconscionability has been criticised in EC Ridout at


[49], as Quentin J cautioned against re-writing of disadvantageous
contracts, which may have the effect of upsetting contractual certainty.

Illustrative cases

Refusal to contract – CTN Cash and Carry – Duress not made out

432. In CTN Cash and Carry, D delivered cigarettes to P. However, they were
mistakenly delivered to B's Burnley warehouse rather than their Preston warehouse.
The cigarettes were then stolen. Believing mistakenly that the risk had passed to P, D
demanded the £17,000 contract price and made clear that they would withdraw the
claimants' credit facilities on future contracts if they failed to pay. The claimants paid
the sum, but later sought repayment on the ground they had paid under duress. The
EWCA held that the threat was not illegitimate.

a. The EWCA gave weight onto the claimant's bona fide belief that it was owed
the sum in question and that risk had passed.

284
b. There was also no evidence that the defendant's belief was unreasonable – D
took advice that it had good grounds for making the demand.

c. Note however, that D later in a late stage of the trial accepted that the risk in
the goods had not in law passed to P.

Exercise of a legal right or discretion – R v AG for England and Wales – Duress not made out

433. In this case, R was was a soldier with the SAS, and was allocated to the
Parachute Regiment as his parent unit. R fought in the Gulf War as part of the Bravo
Two Zero patrol. Several personnel of the team wrote books recounting their
experiences, which remaining members did not think accurately represented the events.
The MOD, attempting to prevent similar publications, required all new and existing
special forces personnel to sign a confidentiality agreement, failing to sign would lead
to soldier being returned to parent unit (in effect a demotion). R signed the agreement
but later left the service to write his own book on his events in the patrol. He pleaded,
amongst other things, that the contract should be vitiated for duress, but failed.

a. The court held that the threat was lawful – the Crown was entitled at its
discretion to transfer any member of the SAS to another unit.

b. The demand was reasonable as well, as it was feared that the disclosures would
threaten the security of the operation and personnel and were undermining the
effectiveness and employability of the UKSF.

Exercise of legal right to sue – Lee Kuan Yew v Chee Soon Juan – Duress not made out

434. In this case, Chee spoke and published certain words which allegedly defamed
Lee Kuan Yew. Pursuant to a letter of demand by Lee's lawyers, Chee apologized and
agreed to compensate Lee with damages and indemnify him for all costs and expenses
incurred. Lee sought to enforce the "contract" by way of interlocutory judgment. The
issue was whether the compromise could be set aside on the ground of duress.

a. The Court held that a threat made bona fide, not manifestly frivolous or
vexatious would not amount to duress.

i. Lee's lawyer suggested that legal proceedings which is used to extort


money may amount to illegitimate pressure.

285
b. On the evidence, the allegation of mala fides was an afterthought.

Exercise of legal right to sue – Tam Tak Chuen – Duress made out

435. In this case, the plaintiff and the defendant were doctors in partnership. They
were equal shareholders of Eden Aesthetics Pte Ltd and Eden Healthcare Pte Ltd.
Defendant suspected plaintiff of having an illicit affair with one of their nurses.
Defendant installed CCTV in clinic and obtained evidence of plaintiff’s activities.
Defendant confronted plaintiff with the footage and threatened to apply to court for
companies to be wound up and the incriminating video would be tendered to court as
evidence. He demanded plaintiff to sell him the shares at a gross undervalue (which
was accepted), or the plaintiff to buy his shares. At the same night, the defendant’s wife
visited the plaintiff’s wife and showed her the footage. The court held that there was
illegitimate pressure.

a. The threat to go to court for winding up was a lawful action. However, the
defendant was acting with a collateral motive to use the winding-up proceedings
as a cover to release the evidence, so as to threaten the plaintiff to sell the shares
at a gross undervalue.

b. The demands were unreasonable: the value of the shares was far more than the
amount defendant offered to pay (offered amount not even 25% of the value of
shares) and the requirement for the plaintiff to take over half the value of the
share worth of liabilities as unreasonable.

c. The circumstances in which the demand was made was unconscionable. There
was 3 months of planning, meeting was arranged late at night, sprung evidence
on the plaintiff in a way that was calculated to unnerve the plaintiff. Underlined
the point that plaintiff had more to lose going to court and wanted to buy the
shares at a price that was manifestly disadvantageous to plaintiff.

Taking advantage of weaker bargaining power – EC Ridout SGHC – Duress not made out

436. In this case, AA had sought to set aside an option contract on the ground that it
was procured under duress. The Court held that there was legitimate commercial
advantage and not pressure.

286
a. AA been unsuccessful in raising a loan and was coming under increasing
pressure to reduce his loan with HLF.

b. AA was a seasoned businessman who has been through much in his business
career.

c. AA had the benefit of advice from his lawyer and his CFO.

d. EC was not obliged to lend AA money.

e. However, if EC had promised to lend upfront, delayed matters to the last minute,
then presented a smaller sum agreement or the option to sell such that it was too
late for AA to refuse, there may have been sufficient illegitimate pressure.

Threat to sue – Goh Bee Lan v Yap Soon Guan – Duress not made out

437. In this case, P had lent money to D. D was unable to repay and the parties had
on several occasions agreed to restructure the loan. In the final iteration, the loan
amount claimed was a sum of $3.25m, lower than the original sum. The Court held that
there was no exertion of illegitimate pressure.

a. The demand was not extravagant or unconscionable. It was the defendants who
calculated and volunteered the sums, which was lower than the original loan.

b. The defendants had benefit of independent legal advice.

c. P had acted in good faith – he had stood as guarantor for a separate loan and
thus needed the money to pay his creditor. The Taiwanese lender wanted
immediate repayment from the defendants (such that P would have funds to pay
the Taiwanese Lender), but P had interceded. P had sought to help D.

Classes of duress and causation

Duress to persons

438. Given the illegitimacy of the pressure, the Courts only require that the pressure
to be “a cause” of the victim’s decision to contract. The burden of proof is reversed,
and the person exercising duress must show that the threat had no influence on the
victim at all (Barton v Armstrong).

287
a. In this case, the trial judge had denied relief because the chairman’s main reason
for buying out the defendant was to ensure the company’s survival. The Privy
Council disagreed, holding that the chairman could succeed if the threats were
merely a reason for him to enter into the contract.

Duress to goods, economic duress, lawful act duress

439. In these cases, the complainant must show that the threat was a "significant
cause" (Evia Luck). This requires a "but for" test as a minimum (Huyton SA v Peter
Cremer at pg 636).

440. There is however some uncertainty as to whether there is an additional


normative requirement of "no reasonable alternative".

a. In Huyton SA, the EWHC was unwilling to lay down a rule that there is a third
essential ingredient of no practical alternative.

b. In contrast, in DSND Subsea, the EWHC took the view that no practical
alternative was a separate element.

441. Locally, there is some uncertainty as to whether the Barton v Armstrong


approach would apply to all cases of duress. The better view is that it should not:

a. In Tjong Very Sumito v Chan Sing En SGHC, the court endorsed the sliding
scale approach, preferring the but-for test to apply as a minimum (at [247],
citing EC v Ridout SGHC).

b. In EC Ridout SGHC, the court endorsed the Barton v Armstrong approach at


[48], citing an earlier decision in Tam Tak Chuen. However, contradictorily, the
Court then went to cite Treitel for the proposition that the lower threshold
should not apply, with emphasis added.

c. In Tam Tak Chuen, the Court applied the Barton v Armstrong approach in the
context of a lawful act duress (at [62]).

288
General factors in ascertaining causation

442. Generally, the Courts will consider the following factors in ascertaining whether
the illegitimate pressure led to compliance with the demand (Pao On at pg 635, Tam
Tak Chuen at [62], EC Investment at [44]):

a. Did the victim protest before complying with the demand?

i. The fact that there is protest tends to point in favour of causation and its
absence otherwise (Tjong Very Sumito at [261], Lee Kuan Yew at [38]-
[39]).

ii. However, the failure to protest is not fatal, especially if there is a lack of
any practicable choice but to submit (The Universe Sentinel at [400]).
The victim may just want to avoid antagonising the bully.

b. Were there alternative courses open to the victim?

i. If there were such alternative, the plaintiff will seldom succeed, see
DSND at [136].

1. In DSND Subsea, the Court considered that the PGS


representative had reasonable alternatives – he could have
considered if there were alternative vessels available (PGS did
not), could have terminated the contract, sued DSND for breach,
sued DSND's parent company under the guarantee. There was
no suggestion that PGS would not have been able to recover
these costs from termination.

ii. The absence of such alternatives is a factor in favour (Tam Tak Chuen
at [65]).

1. In Tam Tak Chuen, the defendant gave the plaintiff 3 options –


One was that the plaintiff could purchase the shares (buy him
out) from the defendant, but evidence showed that the defendant
was not really interested in selling; two was that the plaintiff
could buy the shares; three was that the company be wound up

289
with the evidence of adultery will be presented. The Court held
that there was no reasonable alternative.

c. Whether the victim had access to independent legal advice?

i. In EC Investment, AA had advice from a solicitor (at [58]-[59]), which


was a factor against finding causation.

d. What steps did the victim take to avoid the contract once the pressure had
abated?

i. If no steps were taken, and the victim had continued to perform its
obligations, the Court may find that contract was affirmed (The Atlantic
Baron).

1. In this case, the court found that the owners in paying over the
additional sums demanded, after the threat had abated (of
refusing to build the tanker by the shipbuilders), had affirmed
the contract variation.

Remedies for duress

Recission of contract

443. Where an existing contract has been found to be induced by duress, such a
contract is voidable by the innocent party (The Evia Luck at pg 168, DPP v Lynch at pg
695). As such, the usual bars to recission (such as affirmation) will apply.

a. Note however, where an existing contract which is varied but is found to lack
consideration, the variation is ineffective at law, with the variation contract
being void (i.e. Foakes v Beers).

Restitution for unjust enrichment

444. If there is an extra-contractual payment, there is no contract to be rescinded –


the victim may sue for restitution for unjust enrichment on the basis that the payment
was made under duress – outside of syllabus.

290
Damages

445. There is some uncertainty as to whether duress may amount to a tort (The
Universe Sentinel). Generally, the tort of intimidation is understood to apply only to
unlawful threats (see Lee Kuan Yew at [48]-[49]).

291
Chapter 11: Undue Influence

General rule

446. A contract is voidable if it was entered into while a party to the contract was
under the undue influence of the other contracting party, or in certain circumstances, of
a third party. There are two classes of undue influence. (BOM v BOK at [101]).

a. Class 1 is also known as actual undue influence. Here the plaintiff has to
establish that (i) the defendant had the capacity to influence him; (ii) the
influence was exercised; (iii) the exercise was undue; and (iv) its exercise
brought about the transaction.

b. Class 2 undue influence is known as presumed undue influence. Under this class
of undue influence, the plaintiff is not required to prove actual undue influence.
It suffices for the plaintiff to demonstrate (i) that there was a relationship of trust
and confidence between him and the defendant; (ii) that the relationship was
such that it could be presumed that the defendant abused the plaintiff's trust and
confidence in influencing the plaintiff to enter into the impugned transaction;
and (iii) that the transaction was one that calls for an explanation.

i. Under Class 2A, there are relationships which the law irrebuttably
presumes to give rise to a relationship of trust and confidence. Once the
plaintiff shows his relationship with the wrongdoer triggers the
presumption and that the impugned transaction calls for an explanation,
there is a rebuttable presumption that the wrongdoer has exerted undue
influence.

ii. Under class 2B undue influence, the plaintiff must prove that there is a
relationship of trust and confidence. If it is shown that there was such a
relationship and that the transaction calls for an explanation, then there
is a rebuttable presumption of undue influence.

Class 1: Actual undue influence

292
447. Here the plaintiff has to establish that (i) the defendant had the capacity to
influence him; (ii) the influence was exercised; (iii) the exercise was undue; and (iv) its
exercise brought about the transaction (BOM v BOK SGCA at [101], BCCI v Aboody
at pg 967).

a. The heart of the inquiry is whether the person exercising the undue influence
has exercised such domination over the plaintiff victim's mind that his
independence of decision was substantially, or even totally, undermined (BOM
v BOK at [103]).

448. Illustrative cases:

a. BCCI v Aboody – In this case, the defendants were husband and wife, and
directors and shareholders of a family company. The company constantly
exceeded the prescribed limits of its overdraft at the plaintiff bank. The
company's liabilities were secured by 3 guarantees from the defendants, and 3
charges of the wife's house in favour of the bank. The company collapsed and
the bank enforced the security. The wife challenged the validity of the
guarantees on the ground that they had been obtained by actual undue influence.
The court upheld the trial judge's decision to uphold the transaction.

i. In this case, the EWCA laid down the test for actual undue influence at
pg 967.

ii. Wife was an Iraqi Jew, born and educated in Baghdad. Family moved
exclusively within the local Iraqi Jewish community and observed its
customs. She has never gone out to work (as she was a girl). Business
was a man's exclusive province. Marriage was arranged by her father –
to the husband of 20 years older. Wife was engaged to him on the first
day of meeting him, married within the month. Wife was provided with
a dowry, which was later used to buy a house in her name.

iii. Wife was made a co-director and secretary of the company (without any
training or experience). Husband assured her she would not have to do
anything. Wife never received any remuneration or salary. All business
decisions were made by the husband.

293
iv. Wife would merely sign any document when husband asked her to sign.
She had blindly signed many company documents over the years –
board meetings which did not take place, EGM which did not take place.
Husband ran the company as if it was a one-man business.

v. During the execution of the security, lawyer had tried to explain the
security documents which she was signing, but she was indifferent to
his suggestions to limit liability/she wanted to go through the signing.
Husband had suddenly burst into the room uninvited, yelling at the
lawyer (from the bank) – "why the hell don't you get on with what you
are paid to do and witness her signature?". This was followed by a scene,
wife was reduced to tears. Eventually signed.

vi. Capacity to influence and exercise of influence: The court considered


that as the wife was, at the invitation of her husband, habitually prepared
to sign documents relating to the affairs of the company, without
considering their contents because she trusted him. The Court also
considered that the husband did invite her to enter into the transactions
– these two facts satisfied that the husband had capacity to influence,
and the influence was exercised.

vii. Influence was undue: Judge did not find that husband had bullied,
coerced or placed any pressure on wife. There was no intention to injure
the wife as well. However, husband had omitted to tell wife the risks
involved in the transaction, and used his influence to induce her to sign
security documents, thereby depriving her of free use of any
independent and informed judgment.

viii. Causation: However, the court seemed to accept that even without the
influence, the wife would have been willing to continue to enter into the
transaction in any event, on a balance of probabilities (at pg 971). As
such, causation was not made out.

b. BOM v BOK – In this case, BOK was a bereaved son whose mother had been
recently killed. He signed a declaration of trust, prepared by his legally-trained
wife, 3 days after his mother's funeral. The trust purported to constitute himself

294
and his wife as joint trustees for the sole benefit of their infant son, BOL. BOK
brought an action to set aside the deed of trust on the grounds of inter alia,
undue influence. The High Court set aside the transaction on the ground of
undue influence (Class 1 and 2), but the SGCA only affirmed it on the basis
of Class 1.

i. Background facts – husband was a trader, who shared a close


relationship with his mother. Wife is 4 years older (37), was a practicing
lawyer previously. Mother and wife's family was estranged, as mother
was against the marriage. Mother was killed. The wife had, after the
funeral, suddenly asked the husband to sign a deed of trust. The husband
had initially refused to sign it.

ii. Relationship between the parties – the parties did not have a close or
trusting relationship. Husband did not want to initially tell the wife what
assets the mother left him. Husband had secretly recorded conversations
between himself and the wife. Wife had secretly approached the
husband's lawyer to find out what assets the mother had left, asked her
father (who was a senior lawyer) to pressure husband.

iii. The wife had capacity to influence the husband: (1) she was the wife.
Although the husband did not trust her enough to disclose the contents
of his mother's will, she had previously advised him on matters of law.
(2) he was in a state of grief and isolation and was susceptible to
influence – wife was aware and acted with great urgency to take
advantage. (3) her attempt to persuade was legitimized by her father – a
senior lawyer respected by the husband.

iv. The wife had exercised the influence: she had persistently asked him to
sign the DOT and by misrepresenting to him its true legal effect (i.e. it
will divest him all of his assets immediately, and not only on death).

v. The exercise was undue: this was based on a lie that he would remain
free to deal with his assets – undue influence may arise from
misrepresentation. The husband did not have independent legal advice.

295
The wife had "twisted the mind" of the plaintiff to have him sign the
DOT.

vi. The influence did cause the husband to sign the DOT.

Class 2: Presumed undue influence

449. . Under this class of undue influence, the plaintiff is not required to prove actual
undue influence. It suffices for the plaintiff to demonstrate (i) that there was a
relationship of trust and confidence between him and the defendant; (ii) that the
relationship was such that it could be presumed that the defendant abused the plaintiff's
trust and confidence in influencing the plaintiff to enter into the impugned transaction;
and (iii) that the transaction was one that calls for an explanation (BOM v BOK at [101]).

450. What types of conduct fall within this class? –

a. The influence one person has over another provides scope for misuse without
specific overt acts of persuasion. The relationship between two individuals may
be such that, without more, one of them is disposed to agree to a course of
action proposed by the other. Typically, this occurs when one person places
trust in another to look after his affairs and interests, and the latter betrays this
trust by preferring his own interests (Etridge (No.2) at [9]).

b. Even this test is not comprehensive. The principle is not confined to cases of
abuse of trust and confidence. It also includes… cases where a vulnerable
person has been exploited. Indeed, there is no single touchstone for determining
whether the principle is applicable. Several expressions have been used in an
endeavour to encapsulate the essence: trust and confidence, reliance,
dependence or vulnerability on the one hand and ascendancy on the other.
None of these descriptions is perfect. None is all embracing. Each has its proper
place (Etridge (No.2) at [11]).

Class 2A: Irrebuttable presumption that one party had influence over the other

451. Under Class 2A, there are relationships which the law irrebuttably presumes to
give rise to a relationship of trust and confidence (BOM v BOK at [101]).

a. Such relationships include (Etridge (No.2) at [18]):

296
i. Parent over the child (Chitty at [8-080]);

ii. Religious advisor over disciple (BCCI v Aboody at pg 953);

iii. Doctor and patient (BCCI v Aboody at pg 953);

iv. Solicitor and client (BOM v BOK at [108]);

v. Other fiduciary relationships.

b. These relationships do not include:

i. Husband-wife (Etridge (No.2) at [157]);

ii. Banker/customer (Etridge (No.2) at [157]);

iii. Child-parent;

iv. Employer-employee;

v. Sibling-sibling.

452. In these cases, the complainant need not prove he actually reposed trust and
confidence in the other party. It is sufficient for him to prove the existence of the type
of relationship (Etridge (No.2) at [18]).

453. However, whilst there is a presumption of undue influence arising out of the
parent/child relationship, there should be a limit to the application of the presumption,
depending on the respective ages and positions in life of the parent and child. It does
not make sense to presume that a father of 90 can exercise undue influence over his
sons who are all over 60 and for decades conducted their lives as independent adults
(Rajabali Jumabhoy v Ameerali R Jumabhoy).

a. In this case, the board of SIS (including A, Y, M) granted R (MD of SHL) an


option to purchase all SHL shares SIS held at the price of $0.85 per share
(market price was $0.59). R was A's son and Y/M's nephew. R drafted the option
(under the control of his grandfather (ostensibly), but the grandfather was not
well versed in financial instruments while R was. It was argued that Y and M
exercised undue influence on R in procuring the option (by the plaintiffs – the

297
grandfather). The defendants however said that it was the grandfather who
procured A/Y (sons) to approve the option, and R had notice of the exercise of
the undue influence.

b. The Court considered that R and A/Y did not have a presumption of undue
influence (at [185]).

c. The Curt considered that the grandfather was not presumed to exercise undue
influence over A/Y (who are over 60 years old).

d. The court also found that no actual undue influence was exercised.

Class 2B: Factually established relationships of trust and confidence

454. Under class 2B undue influence, the plaintiff must prove that there is a
relationship of trust and confidence (BOM v BOK at [101]). This is an evidential
presumption – the legal burden of proof lies with the complainant, but showing
evidence of this would allow the court to draw an inference that there was capacity to
influence, which the court will do so in the absence of satisfactory explanation (Etridge
(No.2) at [14]).

455. Illustrative cases:

a. Credit Lyonnais Bank Nederland v Burch – In this case, Burch was a junior
employee of a company (API) which she had no financial interest. She was
asked by her boss, Mr Pelosi, the major shareholder in that company, to
mortgage her flat as security for an increase in the company's overdraft. She
therefore entered into a transaction with the claimant bank under which she gave
them an unlimited guarantee of the company's debts, secured by a second
charge over her flat. The bank did not explain to her the nature of the transaction
into which she was entering and, although it advised her to obtain independent
advice, she did not do so. The company went into liquidation and the bank
sought possession of the defendant's flat. In dismissing the claimant bank's
appeal, the EWCA held that the transaction should be set aside for presumed
undue influence.

298
i. The Court (Millet LJ in particular) placed great weight on the fact that
the transaction was so disadvantageous, coupled with the fact that a
relationship of employer and employee had the potential of developing
into a relationship of trust and confidence, allowed an inference that the
normal relationship had indeed developed into a relationship of trust
and confidence. There was no direct evidence on the relationship, no
suggestion that there was a sexual or emotional tie between Burch and
her employer.

ii. Millet LJ considered that if there was such a sexual or emotional tie,
such as a wife, the transaction might at least become explicable – if a
suitably limited guarantee of her husband's interest, secured over her
matrimonial home could be justified if (a) livelihood of her and family
depended on the success of the business and (b) not doing so may put at
risk of the marital relationship as well as the the continued prosperity of
herself and her family.

b. Lloyds Bank Ltd v Bundy – In this case, the father gave a personal guarantee
and a charge on his home (his only asset) for his son's company's overdraft. The
business of the company was precarious. The father, an elderly farmer, was a
customer of the bank for many years. The court held that there was a
relationship of trust and confidence between the father and the bank
representative, and that the transaction was set aside on the grounds of undue
influence.

i. Sir Eric – "It not infrequently occurs in provincial and country branches
of great banks that a relationship is built up over the years, and in due
course the senior officials may become trusted councilors of customers
of whose affairs they have an intimate knowledge. Confidential trust
is placed in them because of a combination of status, goodwill and
knowledge… [the father] said "I always trusted him"".

ii. Denning of course suggested an even broader principle – "English law


gives relief to one who, without independent advice, enters into a
contract upon terms which are very unfair or transfers property for a

299
consideration which is grossly inadequate, when his bargaining power
is grievously impaired by reason of his own needs or desires or by his
own ignorance or infirmity, coupled with undue influences or pressures
brought to bear on him by or for the benefit of the other".

Transaction which calls for an explanation

456. Under this class of undue influence, the plaintiff must establish that the
transaction was one that calls for an explanation (apart from the relationship of trust
and confidence) (BOM v BOK at [101]).

a. It is not necessary (or sufficient) for the transaction to be manifestly


disadvantageous to the complainant. In most cases, it is acceptable for the
complainant to enter into transactions which may be disadvantageous to her (e.g.
Wife's affection and self-interest runs hand-in-hand in inclining her to join with
her husband in charging the matrimonial home for the husband's business,
which is a source of the family income) – Etridge (No.2) at [28].

b. The question is whether the transaction is one which cannot be reasonably


accounted for on the ground of friendship, relationship, charity, or other
ordinary motives on which ordinary men act (Allcard v Skinner, Etridge
(No.2)).

c. Factors may include: Size of the transaction and how onerous it is to the weaker
party; Motives and history behind the transaction; The nature of the relationship
between the parties.

457. Illustrative cases:

a. Allcard v Skinner – In this case, the claimant inherited her father's wealth. She
became a nun in a covent run by the Mother Superior. According to the rules of
the convent, the claimant had to give up all her property. She made various
substantial gifts (e.g. shares) to the Mother Superior for the purposes of the
sisterhood. In accordance with the convent rules, she did so without seeking
independent outside advice. The claimant left the convent after 8 years, but
made no demand for the gifts until 1885. The EWCA held that the claimant

300
would have been entitled to the return of the gifts for undue influence, but her
claim was barred by laches and acquiescence.

i. None of the judges found bad behavior on part of the Mother Superior
– Cotton LJ said that the Mother Superior did not act with any selfish
motive; Lindley LJ said that there was no pressure (except the vows and
rules), no deception was practiced, no unfair advantage was taken of her,
monies taken were applied to legitimate objects of the sisterhood.

ii. The gift was set aside, on the basis that there was presumed undue
influence (Class 2A), and the gift was not reasonably accounted for.

Rebutting the evidential presumption of undue influence

458. Once a relationship under Class 2A or 2B has been established along with a
transaction which calls for an explanation, the dominant party must adduce some
sufficient evidence to rebut the evidential presumption (Etridge (No.2) at [219]).

a. The type and weight of evidence needed to rebut the presumption will depend
on the weight of the presumption itself (Etridge (No.2) at [153]).

b. This can be discharged by showing (1) there was no use/misuse/abuse of the


dominant person, or (2) the subservient party was not acting under the influence
but was in a position to exercise free and independent judgment.

c. Some factors include:

i. Background, qualifications and personality of the weaker party (see e.g.


Rajabali Jumabhoy in the context of Class 1 undue influence);

ii. Complexity of the transaction;

iii. Legal advice (Inche Noriah v Shaik Allie Bin Omar, Etridge (No.2));

iv. Whether the transaction has been explained to the weaker party – not
necessarily by lawyer (Inche Noriah v Shaik Allie Bin Omar);

v. Alternative benign explanations for the transaction other than undue


influence?

301
Third party undue influence

Put on inquiry

459. Undue influence exerted by a third party on a party to the contract may allow
that party to avoid the contract, if the other contracting party had actual or constructive
notice of the undue influence (Etridge (No.2)). The question is when a party is put on
inquiry?

a. Previously, in Barclays Bank v O'Brien, a lender is put on inquiry when (a) a


transaction is on its face not to the financial advantage of the other party, and
(b) there is a substantial risk that the husband has committed a legal or equitable
wrong that entitles the wife to set aside the transaction (at pg 196).

b. In Etridge No.2, the UKHL took the view that it would be too uncertain if
lenders are expected to probe into the emotional relationships between the
debtors and the guarantors. The test should be clear and easy to apply – a lender
would be put on inquiry whenever a person offers to stand as surety for the debts
of another and the relationship between the parties are non-commercial (at [87])
– This approach is rather broad.

c. Singapore seems to take a look at the substantive character of the transaction


concerned, requiring the Court to consider whether there was any red flags
which the counterparty would know of (Hsu Ann Mei Amy v OCBC at [29], in
the context of elderly financial abuse).

460. Illustrative case

a. Bank of East Asia v Mody Sonal – This case involved an action by the bank
against 3 family members for a guarantee given by them to the bank, to secure
overdraft facilities extended by the bank. The family members were all directors
of the company – the first and third defendants were daughter and wife of the
second defendant, and were also shareholders in the company. One of the issues
which arose was whether or not the guarantee was procured from the wife and
daughter by undue influence, and if so, whether the bank should be fixed with
constructive notice of such undue influence. The Court held that the plaintiff
bank was not put on inquiry – The defendants were shareholders in the

302
company; the second defendant had no shares in the company itself, and the
defendants gave the guarantee as directors of the company – the wife and
daughter were the only persons who stood to gain if the company were to use
the facilities to the advantage. On the facts, there was also no actual or presumed
undue influence exercised.

461. Note: an argument has been made that the doctrine of "infection" could be
extended to other types of wrong, such as misrepresentation, see The Law of Contract
in Singapore at [12.183]). If this argument is correct, it may extend to duress and
unconscionability as well.

Curing the infection

462. Once the bank is put on inquiry, the bank is required to take reasonable steps to
ensure that the influenced party fully understands what she is doing (Etridge (No.2)).

a. The bank should communicate directly with the guarantor to tell him or her that
for the bank's protection, it will need him or her to take separate legal advice;
or

b. request that he or she attend a private meeting with a representative of the bank
at which that representative tells him or her the extent of his or her liability,
warns of the risks involved for him or her, and requests that he or she take
separate legal advice.

463. If the bank decides that the guarantor should receive separate legal advice, the
bank should:

a. inform the guarantor, directly, that for its own protection, the bank will need
written confirmation from a solicitor acting for him or her, to the effect that the
solicitor has fully explained to him or her the nature of the documents and the
practical implications they will have for him or her;

b. inform him or her that the purpose of the solicitor's written confirmation is that,
once the transaction has been entered into, he or she should not be able to
dispute that he or she is legally bound by the documents he or she has signed;

303
c. ask him or her to nominate a solicitor whom he or she is willing to instruct to
advise him or her, separately from the bank or any third party which the
guarantee guarantees the obligations of, and to act in giving the necessary
confirmation to the bank; and

d. tell him or her that if he or she wishes, the solicitor may be the same solicitor as
the one acting for the third party which the guarantee guarantees the obligations
of in connection with the transaction but that he or she may prefer to instruct a
different solicitor.

464. Once the guarantor has nominated and instructed a solicitor, the bank should
get the principal debtor's consent (as the bank's customer) to send all the necessary
information to the solicitor to enable the solicitor to advise properly. The information
to be sent to the solicitor will depend on the circumstances but ordinarily will be:

a. an explanation of the purpose for which the facility/new facility is required;

b. a note of the current amount of indebtedness of the principal debtor;

c. a note of the amount of the principal debtor's current overdraft facility;

d. a note of the amount and terms of the proposed new facility; and

e. a copy of any written application by the principal debtor for a facility.

465. Once the solicitor has advised the guarantor, the bank should ask the solicitor
to certify to the bank that it has acted for the guarantor, has explained the transaction,
the documents, the implications for the guarantor of signing the documents, the
potential liability that the guarantor is taking on and the risks. It is reasonable for the
bank to rely on a solicitor's confirmation that it has acted for the guarantor and has
advised him or her appropriately, but not if either of the following circumstances apply:

a. the bank knows that the solicitor has not properly advised the guarantor; and/or

b. the bank knows of facts from which it ought to have realised that the guarantor
had not received appropriate advice.

304
466. Where the bank believes or suspects that the guarantor has been misled by any
third party which the guarantee guarantees the obligations of or is not entering into the
transaction of his or her own free will, the bank should notify the guarantor's solicitors
of the facts.

467. Illustrative case: OCBC v Tan Teck Khong

a. In this case, the mother (who was not in good health having suffered previously
two strokes) had executed a mortgage under the undue influence of her third
son (first and second sons represented the committee of the mother). The
mother's committee sought to set aside the mortgage against the bank who was
enforcing it. However in this case, the solicitor advising the mother had acted
for both the bank and the mother.

i. The Court held that the solicitor should have had a private meeting with
the mother, explain the liabilities and risks she would be exposed to, and
advise her that she had the right not to proceed with the transaction.

ii. However, the manner in which the documents were explained were
unsatisfactory – the documents were handed to the mother's Hainanese
interpreter without any input from him.

iii. Nevertheless, the bank was entitled to rely on the solicitor in this case,
given that Singapore allows for a bank to act for both parties (at [68]).

iv. Nevertheless, because the committee had asserted that there was an
existing mortgage to be discharged, they had affirmed the mortgage.
As such the bank's mortgage was intact.

305
Chapter 12: Unconscionability

Pre-BOM v BOK

468. Prior to BOM v BOK, the doctrine of unconscionable bargains apply where (a)
the complainant was suffering from certain types of bargaining weakness; (b) the
defendant must have exploited the complainant's weakness; (c) the resulting transaction
is manifestly improvident to the complainant; and (d) the complainant lacked adequate
independent advice (Boustany v Piggott). Once this is established, the onus is on the
other party to show that the transaction was fair, just and reasonable (Earl of Aylesford
v Morris).

469. Historically, this doctrine was developed in situations were expectant heirs
were exploited by the other party and deprived of their respective inheritances. It was
to prevent improvident persons from spending or ruining their estates before they come
to them, though no proof of actual fraud or imposition is made out (BOM v BOK at
[127]).

a. Earl of Aylesford v Morris – In this case, the claimant, who was 21 years old
and stood to inherit his wealthy father's estates, had run up a large number of
debts. To pay these off, he entered into a contract of loan with Morris. The
claimant had no independent advice and the rate of interest set aside. Morris
was found to have taken advantage of the claimant's weakness of youth, so
that the contract should be set aside.

i. "Fraud does not here mean deceit or circumvention; it means an


unconscientious use of the power arising out of these circumstances and
conditions; and when the relative position of the parties is such as prima
facie to raise this presumption, the transaction cannot stand unless the
person claiming the benefit of it is able to repel the presumption by
contrary evidence, proving it to have been in point of fact fair, just, and
reasonable."

ii. The weakness here seems to be the youth's inexperience and moral
imbecility.

306
b. Fry v Lane – In this case, the claimants were two brothers, JBF and GF. They
had poorly paid jobs, one working for a plumber and the other as a laundryman.
They each had a reversionary interest in the estate of their uncle, subject to the
life tenancy of their aunt. In 1878, they sold their interests to the defendant,
Lane, for £170 and £270 respectively. In doing so, they were advised by an
inexperienced solicitor, who was also acting for Lane. When the aunt died in
1886, the interests were each worth £730 and an actuary stated that in 1878 JB
Fry’s reversionary interest would have been worth £475. The claimants brought
an action to set aside the transactions, and this succeeded on the basis that the
sales had been at an undervalue to men who were poor and ignorant and had not
been independently advised. The contract was set aside.

i. "The circumstances of poverty and ignorance of the vendor, and absence


of independent advice, throw upon the purchaser, when the transaction
is impeached, the onus of proving… [that the purchase was fair just and
reasonable.]".

ii. The Court concluded that (a) the price of the inheritance was
considerably below the real value; (b) the brothers were poor, ignorant
men; (c) they were not on equal terms with the purchaser; (d) they did
not had independent legal advice – the lawyer was junior and
inexperienced, and he did not properly protect the vendors despite acting
for both the seller and vendors. The contract was thus set aside.

iii. Andrew Burrows consider that the combination of mental inadequacy


(weakness) and substantive unfairness (undervalue) raised an
evidential presumption of exploitation of weakness, which will lead the
contract being set aside unless rebutted by the defendant (showing
independent advice) – Burrows, A Casebook on Contract at pg 856.

470. Initially, the requirement of infirmity was narrow. Fry v Lane required that the
infirmity be poverty and ignorance. However, in Cresswell v Potter, this requirement
was relaxed to encompass members of the lower income group and the less highly
educated. This was subsequently expanded in further cases such as Blomley v Ryan (to
poverty or need of any kind, sickness, age, sex, infirmity of body or mind,

307
drunkenness, illiteracy or lack of education, lack of assistance or explanation where
assistance or explanation is necessary) and Boustany v Pigott.

a. Cresswell v Potter – The claimant was a Post Office telephonist. On her divorce
from the defendant, she entered into a contract with him by which she conveyed
to him her interest in the matrimonial home (‘Slate Hall’) in return for being
released from liability under the mortgage. A couple of years later, the
defendant sold the property for some £3350 which, after deducting costs, was
estimated to have made the defendant a profit of £1400. The claimant
successfully sought to have the contract with the defendant set aside so that she
would be entitled to half of those profits. It was held that she was the modern
equivalent of a poor and ignorant person and that the Fry v Lane principle
applied.

i. "What has to be considered is, first, whether the plaintiff is poor and
ignorant; second, whether the sale was at a considerable undervalue; and
third, whether the vendor had independent advice. I am not, of course,
suggesting that these are the only circumstances which will suffice; thus
there may be circumstances of oppression or abuse of confidence which
will invoke the aid of equity. But in the present case only these three
requirements are in point. Abuse of confidence, though pleaded, is no
longer relied on; and no circumstances of oppression or other matters
are alleged.", Per Megarry J.

ii. The Court found that (a) she was poor – though not destitute; (b) she
was ignorant in the context of property transactions in general, even
though she was not ignorant in the general sense (telephonist required
considerable alertness and skill); (c) The sale was at aa considerable
undervalue – the value of the property was much more than the
underlying mortgage, and the value of the property had risen much
more; (d) there was no independent advice for the claimant. The
defendant had failed to show that the transaction was fair, just and
reasonable.

308
b. Blomley v Ryan – In this case, B bought a property from R. R was 78 years old
and had impaired faculties from prolonged and excessive consumption of
alcohol. The transaction was noted to be at a "gross undervalue" – nearly 30%
less than the true value of the property. There was no suggestion of kinship,
friendship or gratitude which might incline R to make a present of the discount
to B. The deposit for the contract of £25,000 was a mere £5. The payment was
to be over 4 years, at an interest rate lower than the bank rate. The parties were
represented by a common solicitor – who had failed to realise R's weakness (B
and R however had earlier discussions). The Court held that the contract was
unconscionable and set it aside.

i. In particular, it was suggested that poverty or need of any kind,


sickness, age, sex, infirmity of body or mind, drunkenness, illiteracy
or lack of education, lack of assistance or explanation where
assistance or explanation is necessary would suffice as the necessary
infirmity (Per Fullagar J).

ii. In particular, Fullagar J further suggested that it is not essential that the
weaker party suffered loss or detriment by the bargain.

c. Boustany v Pigott – The claimant, Miss Pigott, was an elderly landlady. As she
had become ‘quite slow’, her cousin, George Pigott, managed her affairs. One
of her tenants, Mrs Boustany, the defendant, persuaded the claimant, while
George Pigott was away, to agree to a new lease being drawn up by an
independent lawyer. The lease was for 10 years at a monthly rent of $1,000 (the
existing rent had been $833.35) renewable at the tenant’s option for another 10
years at the same rent. The lawyer, Mr Kendall, pointed out to Miss Pigott the
disadvantageous aspects of the transaction (that there was no provision for
review of the rent) but she insisted on going ahead with it. When George Pigott
discovered what had happened, he protested to Mrs Boustany who was
unmoved. Acting on Miss Pigott’s behalf, he therefore sought a declaration that
the lease was an unconscionable bargain, which should be declared rescinded.
The Privy Council upheld the decision of the Court of Appeal of the Eastern
Caribbean States that the lease was an unconscionable bargain and should be
set aside.

309
i. In this case, several key features were prominent: (a) Pigott was
suffering under some mental defect, such that her cousin was the one
who managed her affairs; (b) the tenants had previously negotiated with
the cousin and knew he was the representative; (c) the terms were
severely disadvantageous; (d) the lawyer had pointed out to the parties
that the terms were disadvantageous; (e) the lawyer did not know
anything about Pigott save that he had prepared the earlier lease; (f)
Pigott did not use her usual family solicitors – the Court inferred that the
tenants had exploited Pigott's weakness in the circumstances (the tenants
did not give evidence).

ii. Note: in this case, Pigott could not be said to be poor in the sense of
being destitute or from a low family background.

471. Mere weakness is insufficient. There must be existence of exploitative conduct


(Hart v O'Connor).

a. In this case, O'Connor was a trustee of a trust over a family farm. O'Connor's
solicitor recommended that he could lease out the farm, sell the property to his
nephews (who weren't interested) or sell it to a third party. Hart, O'Connor's
neighbor, was interested in buying the property. After negotiations with Jack
and his solicitor, they arranged for the farm to be leased to Hart, with an option
to purchase. Within a month, Hart decided to purchase the farm instead and
contacted O'Connor's solicitor to obtain an outright sale of the farm. However,
it turns out that O'Connor was suffering from senile dementia. The sale value
was about 10% lower than the subsequent valuation, payable over 2 years after
possession. O'Connor subsequently died. The surviving brothers of O'Connor
sued to set aside the sale.

i. The UKPC held that the contract was not an unconscionable bargain.
Hart had acted with complete innocence throughout, there was no
equitable fraud, no victimization, no taking advantage, no overreaching
or other description of unconscionable doings which might have
justified the intervention of equity.

310
ii. The contract can only be set aside due to incapacity, if the incapacity
was known to the other party.

BOM v BOK

472. To invoke the doctrine of unconscionability, the plaintiff must establish that (at
[141]-[142]):

a. She was suffering from an infirmity;

i. The infirmity must have been of sufficient gravity as to acutely affected


the plaintiff's ability to conserve his own interests. The infirmity must
have been, or ought to have been, evident to the other party procuring
the transaction (at [141]).

b. That the other party exploited in procuring the transaction.

i. Absence of independent advice and the characterization of a transaction


are factors which underscore and highlight the exploitation of an
infirmity that renders a transaction improvident – but not mandatory
elements (at [155]). – Note: my interpretation that the factors go to this
exploitation stage rather than the latter stage.

ii. Usually will require an overt act (at [152]).

iii. Likely to require the other party to know (at [154]).

c. Upon satisfaction of this requirement, the burden is on the defendant to


demonstrate that the transaction was fair, just and reasonable.

473. It is not necessary (unlike Fry v Lane) to show that there was a transaction at
considerable undervalue and that the weaker party did not have independent advice.
These are not mandatory, but would be very important factors which the Court would
take into account (BOM v BOK at [141]).

474. In BOM v BOK, the Court affirmed the SGHC decision to set aside the deed of
trust on the basis of unconscionability.

311
a. Infirmity – The husband suffered from acute grief that impaired his ability to
make decisions and made him susceptible to influence.

b. Exploitation: The wife knew about it and took advantage of it by leveraging on


his sense of isolation (at [154]).

c. Fair, just and reasonable: The deed was clearly a transaction at an undervalue
and the husband had no independent legal advice. The DOT was by no means
fair just and reasonable (at [155]).

Reversal of burden of proof – is the transaction fair, just and reasonable?

475. This is illustrated by the case of Fong Whye Kon v Chan Ah Thong:

a. In this case, F entered into a contract to buy a flat from C. C was 84 years old,
was illiterate and in poor health. C wished to sell her flat and live with relatives
in China. A friend told her that the house was worth $245,000 and asked for
$5,000 for selling the property, which C agreed to pay. The friend brought F to
view the property, and agreed to buy without bargaining at the price. C granted
F an option, which F exercised after paying 10% of the purchase price as the
deposit. Completion was however delayed. C's solicitors wrote to F's solicitors
telling them she did not intend to proceed with the sale – she had doubts about
returning to China and had been unable to find alternative accommodation. F
refused, seeking to sue on the contract and for specific performance. C then
discovered that the price was at a gross undervalue – the flat's true market value
was $390,000. The Court found that there was unconscionability presumed
from the circumstances, given the weakness of C, coupled with the fact that the
transaction was at a gross undervalue. The onus was on F to prove that the
transaction was fair, just and reasonable.

i. F chose not to adduce any evidence in rebuttal – his counsel rested the
case on the terms of the contract and to have the case decided on the
evidence available and submissions made.

ii. The court held that the inadequacy of consideration is so great that it
was a strong factor in deciding that there was unfair conduct (at [14]);

312
iii. Although solicitors for P were involved, there is no evidence that they
were concerned with the price – this was decided before any solicitors
were involved (at [15]);

iv. There was no affirmation (Court said election/estoppel), from the


conduct of C in seeking extensions for completion and not raising any
objection to the sale going on (at [18]);

v. The plaintiff has chosen to keep a stony silence about the


circumstances of the transaction, including his knowledge of the true
value of the property and of the defendant's condition… He probably
knew he was getting a deal that was too good to be true, and that C
through her ignorance and infirmity was not dealing with him on an
equal footing.

vi. In these circumstances, the prima facie case is unrebutted.

313
Biblography

Uncategorized References

Smith v Hughes [1871] LR 6 QB 597 (EWCA) ("Smith v Hughes")


Tribune Investment Trust Inv v Soosan Trading Co Ltd [2000] 2 SLR(R) 407 (SGCA)
("Tribune Investment Trust Inv v Soosan Trading Co Ltd")
Mindy Chen-Wishart, Contract Law (OUP, 6th Ed, 2018)
Gibson v Manchester CC [1979] 1 WLR 294 (UKHL) ("Gibson v Manchester CC")
Aircharter World Pte Ltd v Kotena Nasional Bhd [1999] 2 SLR(R) 440 (SGCA)
("Aircharter World Pte Ltd v Kotena Nasional Bhd")
Harvey v Facey [1893] AC 552 (UKPC) ("Harvey v Facey")
Storer v Manchester CC [1974] 1 WLR 1403 (EWCA) ("Storer v Manchester CC")
R v Warwickshire CC [1993] AC 583 (UKHL) ("R v Warwickshire CC")
Pharmaceutical Society of Great Britan v Boots Cash Chemists [1953] 1 QB 401
(EWCA) ("Pharmaceutical Society of Great Britan v Boots Cash Chemists")
Patridge v Cittenden [1968] 1 WLR 1204 (EWCA) ("Patridge v Cittenden")
Workspace Consultants Pte Ltd v Teo Seng Siew and Another [1998`] SGHC 372
(SGHC) ("Workspace Consultants Pte Ltd v Teo Seng Siew and Another")
Carlill v Carbolic Smoke Ball [1893] 1 QB 256 (EWCA) ("Carlill v Carbolic Smoke
Ball")
Barry v Davies [2000] 1 WLR 1962 (EWCA) ("Barry v Davies")
Blackpool and Fylde Aero Club Ltd v Blackpool Borough Council [1990] 1 WLR
1195 (EWCA) ("Blackpool and Fylde Aero Club Ltd v Blackpool Borough
Council")
Thornton v Shoe Lane Parking Ltd [1971] 2 QB 163 (EWCA) ("Thornton v Shoe
Lane Parking Ltd")
Errington v Errington [1952] 1 KB 290 (EWCA) ("Errington v Errington")
Daulia v Four Millbank [1978] Ch 231 (EWCA) ("Daulia v Four Millbank")
Mobil Oil v Lyndel Nominees [1998] 153 ALR 198 (FCA) ("Mobil Oil v Lyndel
Nominees")
Entores v Miles Far East Corp [1953] 2 QB 327 (EWCA) ("Entores v Miles Far East
Corp")
1L30G Pte Ltd v EQ Insurance Co Ltd [2017] SGHC 242 (SGHC) ("1L30G Pte Ltd v
EQ Insurance Co Ltd")
Household Fire and Carriage Accident Insurance Co Ltd v Grant [1879] 4 Ex D 216
(EWCA) ("Household Fire and Carriage Accident Insurance Co Ltd v Grant")
Brinkibon v Staghag Stahl [1982] 2 WLR 264 (UKHL) ("Brinkibon v Staghag Stahl")
Manchester Diocesan Council for Education v Commercial and General Investments
Ltd [1970] 1 WLR 241 (EWHC) ("Manchester Diocesan Council for Education v
Commercial and General Investments Ltd")
Midlink Development Pte Ltd v The Standsfield Group Pte Ltd [2004] 4 SLR(R) 258
(SGHC) ("Midlink Development Pte Ltd v The Standsfield Group Pte Ltd")
R v Clarke [1927] 40 CLR 227 (HCA) ("R v Clarke")
Pao On v Lau Yiu Long [1980] AC 614 (UKPC) ("Pao On v Lau Yiu Long")
Sim Tony v Lim Ah Ghee [1995] 1 SLR(R) 886 (SGCA) ("Sim Tony v Lim Ah
Ghee")
Chappel & Co v Nestle [1960] AC (UKHL) ("Chappel & Co v Nestle")

314
White v Bluett [1853] 23 LJ Ex 36 (ExC) ("White v Bluett")
Arrale v Costain Civil Engineering [1976] 1 Lloyd's LR 98 (EWCA) ("Arrale v
Costain Civil Engineering")
Glasbrook Bros Ltd v Glamorgan CC [1952] AC 270 (UKHL) ("Glasbrook Bros Ltd
v Glamorgan CC")
Ward v Byham [1956] 1 WLR 496 (EWCA) ("Ward v Byham")
The law of contract in Singapore (Academy Publishing, 2012)
Re Selectmove [1995] 1 WLR 474 (EWCA) ("Re Selectmove")
MWB v Rock Advertising [2016] 3 WLR 1519 (EWCA) ("MWB v Rock
Advertising")
Woodhouse v Nigerian Produce [1972] AC 741 (UKHL) ("Woodhouse v Nigerian
Produce")
Al Shams Global v BNP [2019] 3 SLR 1189 (SGHC) ("Al Shams Global v BNP")
Lam Chi Kin David v Deutsche Bank AG [2010] 2 SLR(R) 896 (SGHC) ("Lam Chi
Kin David v Deutsche Bank AG")
The Post Chaser [1982] 1 All ER 19 (EWHC) ("The Post Chaser")
Central London Property Trust v High Trees [1947] 1 KB 130 (EWHC) ("Central
London Property Trust v High Trees")
Abdul Jalil bin Ahmad bin Talib v A Formation Construction Pte Ltd [2006] 4 SLR(R)
778 (SGHC) ("Abdul Jalil bin Ahmad bin Talib v A Formation Construction Pte
Ltd")
D&C Builders v Rees [1966] 2 QB 617 (EWCA) ("D&C Builders v Rees")
Williams v Stern [1879] 5 QBD 409 (EWCA) ("Williams v Stern")
Collier v Wright [2008] 1 W:R 643 (EWCA) ("Collier v Wright")
Ajayi v Briscoe [1964] 1 WLR 1326 (UKPC) ("Ajayi v Briscoe")
Hughes v Metropolitan Railway Company [1877] 2 AC 439 (UKHL) ("Hughes v
Metropolitan Railway Company")
Gay Choon Ing v Loh Sze Ti Terence Peter [2009] 2 SLR(R) 332 (SGCA) ("Gay
Choon Ing v Loh Sze Ti Terence Peter")
Balfour v Balfour [1919] 2 KB 571 (EWCA) ("Balfour v Balfour")
De Cruz Andrea Heidi v Guangzhou Yuzhitand Health Products Co Ltd [2003] 4
SLR(R) 682 (SGHC) ("De Cruz Andrea Heidi v Guangzhou Yuzhitand Health
Products Co Ltd")
Parker v Clarke [1960] 1 WLR 286 (EWHC) ("Parker v Clarke")
Rose and Frank Co v Crompton and Bros Ltd [1925] AC 445 (UKHL) ("Rose and
Frank Co v Crompton and Bros Ltd")
Heilbut, Symons & Co v Buckleton [1913] AC 30 (UKHL) ("Heilbut, Symons & Co
v Buckleton")
Bannerman v White [1861] 142 ER 685 ("Bannerman v White")
Oscar Chess v Williams [1957] 1 WLR 370 (EWCA) ("Oscar Chess v Williams")
Ecay v Godfrey [1947] 80 Lloyd's LR 286 (EWHC) ("Ecay v Godfrey")
Routledge v Mckay [1954] 1 WLR 615 (EWCA) ("Routledge v Mckay")
Zurich Insurance v B-Gold [2008] 3 SLR(R) 1029 (SGCA) ("Zurich Insurance v B-
Gold")
Mendelssohn v Normand Ltd [1976] 1 WLR 1078 (EWCA) ("Mendelssohn v
Normand Ltd")
Ang Sin Hock v Khoo Eng Lim [2010] 3 SLR 179 (SGCA) ("Ang Sin Hock v Khoo
Eng Lim")
Wen Wen Food Trading v Food Republic [2019] SGHC 60 (SGHC) ("Wen Wen Food
Trading v Food Republic")

315
L'Estrange v Graucob [1934] 2 KB 394 (EWCA) ("L'Estrange v Graucob")
Consmat Singapore v Bank of America National Trust [1992] 2 SLR(R) 195 (SGHC)
("Consmat Singapore v Bank of America National Trust")
Press Automation Technology v Trans-link Exibition Forwarding [2003] 1 SLR(R)
712 (SGHC) ("Press Automation Technology v Trans-link Exibition Forwarding")
Grogan v Robin Meredith Plant [1996] CLC 1127 (EWCA) ("Grogan v Robin
Meredith Plant")
Parker v South Eastern Railway Co [1877] 2 CPD 416 (EWCA) ("Parker v South
Eastern Railway Co")
Interfoto Picture Library v Stiletto Visual [1989] 1 QB 433 (EWCA) ("Interfoto
Picture Library v Stiletto Visual")
Hakko Products v Danzas [1999] 1 SLR(R) 651 (SGHC) ("Hakko Products v
Danzas")
R1 International v Lonstroff [2015] 1 SLR 521 (SGCA) ("R1 International v
Lonstroff")
Hollier v Rambler Motors [1972] 2 QB 71 (EWCA) ("Hollier v Rambler Motors")
MGA International v Wajilam Exports [2010] SGHC 319 (SGHC) ("MGA
International v Wajilam Exports")
British Crane Hire v Ipswich Plant [1975] 1 QB 303 (EWCA) ("British Crane Hire v
Ipswich Plant")
Sembcorp Marine v PPL Holdings [2013] 4 SLR 193 (SGCA) ("Sembcorp Marine v
PPL Holdings")
Xia Zhengyan v Geng Changqing [2015] 3 SLR 732 (SGCA) ("Xia Zhengyan v Geng
Changqing")
Ding Pei Zhen v Yap Son On [2015] 5 SLR 911 (SGHC) ("Ding Pei Zhen v Yap Son
On")
Goh Yihan, "Towards a consistent use of subsequent conduct in Singapore contract
law" (2017) 5 JBL 387
Master Marine AS v Labroy Offshore [2012] 3 SLR 125 (SGCA) ("Master Marine AS
v Labroy Offshore")
The Moorcock [1889] 14 PD 64 (EWCA) ("The Moorcock")
Shirlaw v Southern Foundries [1939] 2 KB 206 (EWCA) ("Shirlaw v Southern
Foundries")
Jet Holding v Cooper [2006] 3 SLR(R) 769 (SGCA) ("Jet Holding v Cooper")
Dunlop Pneumatic Tyres v Selfridge [1915] 1 AC 847 (UKHL) ("Dunlop Pneumatic
Tyres v Selfridge")
Tweddle v Atkinson [1861] 1 B&S 393 (Court of Queen's Bench) ("Tweddle v
Atkinson")

316

You might also like